www.vet4arab.co.cc
www.vet4arab.co.cc
Mosby's
ReView Questions &Answers for
Veterinary
Boards
Clinical Sciences
...
648 downloads
3280 Views
46MB Size
Report
This content was uploaded by our users and we assume good faith they have the permission to share this book. If you own the copyright to this book and it is wrongfully on our website, we offer a simple DMCA procedure to remove your content from our site. Start by pressing the button below!
Report copyright / DMCA form
www.vet4arab.co.cc
www.vet4arab.co.cc
Mosby's
ReView Questions &Answers for
Veterinary
Boards
Clinical Sciences
"---""
www.vet4arab.co.cc Dedicated to Publishing Excel'ence
~
A Times Mirror .. Company
Contributors
Publisher: John A. Schrefer Executive Editor: Linda L. Duncan Senior Developmental Editor: Teri Merchant Project Manager: Linda McKinley Production Editor: Julie Zipfel
Editing and Production: Top Graphics Design: Renee Duenow Manufacturing Manager: Unda Ierardi Cover desigrl: Iennifer Marmarinos
Introduction Paul W. Pratt, VMD Executive Editor, Mosby-Year Book, St. Louis, Missouri jeffrey L. Rothstein, DVM, MBA Hospital Director, Elm Animal Hospital, Roseville, Michigan
SECOND EDmON Copyright © 1998 by Mosby-Year Book, Inc.
Anesthesiology
Previous edition copyrighted 1993 All rights reserved. No part of this publication may be reproduced, stored in a retrieval system, or transmitted, in any form or by any means, electronic, mechanical, photocopying, recording. or otherwise, without prior written permission from the publisher. Permission to photocopy or reproduce solely for internal or personal use is permitted for libraries or other users registered with the Copyright Clearance Center. provided that the base fee of $4.00 per chapter plus $.10 per page is paid directly to the Copyright Clearance Center, 222 Rosewood Dr., Danvers, MA 01923. This consent does not extend to other kinds of copying. such as copying for general distribution, for advertising or promotional purposes, for creating new collected works, or for resalc. Printed in the United States of America Composition by Ibp Graphics
Printing/binding by R.R. DonneDey & Sons Company
Etta M . Wertz, DVM, MS, Dipl ACVA Staff Anesthesiologist, Alameda East Veterinary Hospital, Denver, Colorado
Clinical Pathology Carol B. Grindem, DVM, PhD, Dipl ACVP Associate Professor of Clinical Pathology, College ofVeterinary Medicine, North Carolina State University, Raleigh, North Carolina Barry Thomas Mitzner, DVM President, Southeast Vetlab, Inc., Miami, Florida
Mosby-Year Book, Inc. 11630 Westline Industrial Drive St. Lottis, Missouri 63146
Rose E. Raskin, DVM, PhD, Dipl ACVP Associate Professor and Service Chief of Clinical Pathology Laboratory, Department of Physiological Sciences, University of Florida, Gainesville, Florida
Ubrary of Congress Cataloging In Publication Data
Clinical sciences / edited by Paul W. Pratt.-2nd ed. p.
Colin I. Dunlop, BVSc, Dipl ACVA Advanced Anesthesia Specialists, Hunters Hill, New South Wales, Australia
Includes bibliographical refcrences.
ISBN 0·61 51-7462·4 1. Veterinary medicine-United States-Examinations. questions,
etc. l. Pratt, Paul W. 1I. Series. SF759.C56 1997 636.069'076-<1c21 97 98 99 00 01
97-37128 CIP
judith A. Hudson, DVM, PhD, Dipl ACVR Associate Professor, Department of Radiology, Auburn University, Auburn, Alabama William R. Klemm, DVM, PhD Professor, College of Veterinary Medicine, Texas A & M University, College Station, Texas Martha L. Moon, DVM, MS, Dipl ACVR Associate Professor, Virginia· Maryland Regional College of Veterinary Medicine, Virginia Polytechnic Institute, Blacksburg, Virginia Paul W. Pratt, VMD Executive Editor, Mosby- Year Book, St. Louis, Missouri Donald E. Thrall, DVM, PhD, Dipl ACVR Professor of Radiology, College of Veterinary Medicine, North Carolina State University, Raleigh, North Carolina Larry Patrick Tilley, DVM, Dipl ACVIM President, Vetmedfax Consultation Services, Santa Fe, New Mexico Erik R. Wisner, DVM, Dipl ACVR Assistant Professor, Department of Radiology, School of Medicine, and Department of Surgicai and Radiological Sciences, School of Veterinary Medicine, University of California, Davis, California
Hematology and Cytology
cm.-(Mosby's review questions & answers for veterinary
boards)
S.T. Finn-Bodner, DVM, MS, Dipl ACVR Associate Professor, Department of Radiology, Auburn Uni· versity, Auburn, Alabama
Diagnostic Imaging and Recordings john M. Bowen, DVM, PhD Professor of Pharmacology and 1bxicology, Associate Dean for Research and Graduate Affairs, College of Veterinary Medicine, University of Georgia, Athens, Georgia
Rick L. Cowell, DVM, MS, Dipl ACVP Professor, Department of Anatomy, Pathology, and Pharmacology, College of Veterinary Medicine, Oklahoma State University, Stillwater. Oklahoma
W. jean Dodds, DVM President, HEMOPET, Irvine, California
/ 9 6 7 6 5 4 3 2 1
v
vi
www.vet4arab.co.cc
CONTRIBUTORS
Brad L. Hines, DVM
Thomas J. Burke, DVM, MS
Internist, Metroplex Veterinary Centre, Irving, Texas
Professor of Medicine, College of Veterinary Merlicine, University of Illinois, Urbana, Illinois; Consultant, Capitol Illini Veterinary Services, Ltd., and lincoln Park Zoo
Immunology Laurel J. Gershwin, DVM, PhD, Dipl ACVM Professor ofImmunology, VMTH Service Chief, Clinical Immunology and Virology, School of Veterinary Medicine, University of California, Davis, California
Nutrition Linda D. Baker, VMD, MS, Dipl ACVN Veterinarian, Dairy Production Systems Consultation, Mertztown, Pennsylvania
Edward A. Moser, VMD, Dipl ACVN Veterinary Nutritionist, Veterinary Nutrition Specialists.
Selinsgrove, Pennsylvania
Pharmacology Terrence P. Clark, DVM, PhD Assistant Professor, Director, Clinical Pharmacology Service, Department of Physiology and Pharmacology, Auburn University, Auburn, Alabama
Lloyd E. Davis, DVM, PhD Professor Emeritus of Clinical Pharmacology, College of Veterinary Medicine, University of Illinois, Urbana, illinois
Sue Hudson Duran, RPh, MS Assistant Professor and Director of Pharmacy, Department of Large Animal Surgery and Medicine, Auburn University, Auburn, Alabama
Jim E. Riviere, DVM, PhD Burroughs Wellcome Distinguished Professor of Veterinary Pharmacology, College of Veterinary Medicine, North Carolina State University, Raleigh, North Carolina
Principles of Surgery Thomas P. Colville, DVM, MSc Associate Professor, Director, Veterinary Technology Program, Department of Veterinary and Microbiological Science, North Dakota State University, Fargo, North Dakota
Leslie A. Dierhauf, VMD Wildlife and Conservation BiolOgist, United States Fish and Wildlife Service, Albuquerque, New Mexico
Preface
Maarten Drost, DVM Professor of Reproduction, College of Veterinary Medicine, University of Florida, Gainesville, Florida
Bruce E. Eilts, DVM, MS, Dipl ACT Professor of Theriogenology, Department of Veterinary Clinical Sciences, School of Veterinary Medicine, Louisiana State University, Baton Rouge, Louisiana
Fredric L. Frye, DVM, MSc, CBiol, FIBiol Visiting Professor at the University of London, University of Bristol, University of Edinburgh; Former Clinical Professor of Medicine, University of California, Davis, California
Tina Gemeinhardt, DVM Huff Animal Hospital, Delta, British Columbia
Stormy Hudelson, DVM, Dipl ABVP Tucson, Arizona
David G. Huff, DVM Consulting Veterinarian, Vancouver Aquarium, Delta, British Columbia
James D. Letcher, DVM
This series of five review books was developed to help candidates prepare for scholastic, licensure, and certification examinations. Although the books are not definitive texts, they can help candidates organize their study preparations and detect areas in which more study is required. The five-volume series contains over 8,100 questions, each with an accompanying answer. A short explanation or rationale is provided for every answer. New to this edition are additional sections on specialties, thousands of new questions, and updating of all questions according to current medical and surgical practices. I am indebted to our group of 150 eminently qualified contributors, who have taken the time from their busy professional and personal lives to carefully craft questions on their respective subject areas. Their en-
thusiasm and ingenuity in developing challenging questions are evident throughout the five volumes. Although I had considered myself fairly well read in our field, I was humbled by the depth and breadth of knowledge illustrated in their questions. We have gone to great effort to root out all errors and ambiguous statements. Despite these precautions, however, a number of flaws undoubtedly have escaped notice. We would be grateful if readers would notify us of any errors, ambiguities, or questionable statements in these books. We also encourage readers to send their comments or criticism on any aspect of these books. In this way we can improve the quality of future editions.
Paul W. Pratt, l-MD Santa Barbara, California
Okeechobee, Florida
Seyedmehdi Mobini, DVM, MS, Dipl ACT Professor of Veterinary Science, Research/Extension Veterinarian, Fort Valley State University, Fort Valley, Georgia
Daniel H. Nielsen, DVM New Berlin Veterinary Hospital, New Berlin, New York
Michael B. Paster, DVM Avalon Animal Hospital and Bird Clinic, Carson, California
Dorcas O. Schaeffer, DVM, MS, Dipl ACLAM Assistant Professor, Department of Comparative Medicine, Facility Director, Office of Laboratory Animal Care, College of Veterinary Medicine, University of Tennessee, Knoxville, Tennessee
Roderick C. Tubbs, DVM, MS, MBA, Dipl ACT, ABVP Swine Veterinarian, Bowling Green, Kentucky
Theriogenology William F. Braun, Jr., DVM, Dipl ACT Veterinarian, Southwest Veterinary Services, Bloomington, Wisconsin; Fonnerly Associate Professor, College of VeterinaryMedicine, University of Missouri, Columbia, Missouri
Toxicology Gary D. Osweiler, DVM, PhD, Dipl ABVT Professor of Veterinary Toxicology, Director, Veterinary Diagnostic Laboratory, Iowa State University, Ames, Iowa
vii
'-
www.vet4arab.co.cc
Contents SECTION 1
SECTION 6
Aulesdrresiology,
Nutrition,
1
143
CI. Dunlop, E.M. Wertz
L.D. Baker, E.A. Moser
SECTION 2
SECTION 7
Clinical Pathology,
35
Pharmacology,
155
CB. Grindem, B.T. Mitzner, R.E. Raskin
T.P. Clark, L.E. Davis, S.H. Duran, J.E. Riviere
SECTION 3
SECTION 8
Diagnostic Imaging and Recordings, 63 J.M . Bowen, S.T. Finn-Bodner, JA Hudson, w'R. Klemm, M.L. Moon, P.W. Pratt, D.E. Thrall, L.P. Tilley, E.R. Wisner
SECTION 4
Hematology and Cytology, 101 R.L. Cowell, W.J. Dodds, B.L. Hines
185
T.P. Colville
SECTION 9
Theriogenology,
199
W.F. Braun, Jr., T.J. Burke, L.A. Dierauf, M. Drost, B.E. Eilts, F.L. Frye, T. Gemeinhardt, S. Hudelson, D.G. Huff, J.D. Letcher, S. Mobini, D.H. Nielsen, M .B. Paster, D.O. Schaeffer, R.C Tubbs
SECTION 10
Toxicology,
SECTION 5
Immunology,
Principles of Surgery,
245
G.D. Osweiler
131
L.J. Gershwin
Practice Aulswer Sheets,
255
Ix
-
- -----------.....-:.............
~------
www.vet4arab.co.cc
SECTION
1 Anesthesiology c.1.
Dunlop, E.M. Wertz
Recommended Reading Hall LW; Clarke KW: Veterinary anaesthesia, ed 9, London, 1992, Bailliere Tindall. MuirWW, Hubbell IA: Equine anesthesia, St Louis, 1991, Mosby. Muir WW; Hubbell 1A: Handbook of veterinary anesthesia, ed 2, St Louis, 1996, Mosby. Short CEo Principles and practice of veterinary anesthesia, Baltimore, 1987, Williams & Wilkins. Thurman IC et al: Lumb and Jones' veterinary anesthesia, ed 3, Baltimore, 1996, Williams & Wilkins. ',' Practice answer sheets are on pages .255-256.
Questions c.1. 1. Which of the following is 1Wt an objective of preanesthetic medication?
a. alleviate or minimize pain b. facilitate handling c. minimize undesirable postanesthetic recovery complications d. increase reflex autonomic activity e. decrease the dose of parenteral anesthetic induction drugs 2. Atropine acts on the parasympathetic nervous sYstem by:
a. depressing formation of acetylcholine b. enhancing formation of cholinesterase c. directly inhibiting neurochemical reactions at the myoneural junction
Dunlop d. competitive antagonism of acetylcholine e. stimulating adrenergic activity 3. Bradycardia during general anesthesia of dogs can be treated with intravenous atropine, usually at 0.01 to 0.02 mglkg Iv. If the bradycardia worsens after atropine administration, the most appropriate course of action is to:
a. repeat the dose after 1 or 2 minutes b. wait for atropine to cause its sympathetic effect c. cease administration of anesthetic and insufflate with oxygen d. administer epinephrine intravenously e. insert a transvenous pacemaker because the dog must have sick sinus syndrome
Correct answers are on pages 25-33.
1
2
www.vet4arab.co.cc
SECTION 1
4. One of the differences between atropine and glycopyrrolate is that: a. glycopyrrolate causes a greater increase in the heart rate than atropine b. glycopyrrolate causes a smaller increase in the heart rate than atropine c. atropine is longer acting than glycopyrrolate d. atropine can be antagonized by epinephrine but glycopyrrolate cannot e. glycopyrrolate has a different site of action than atropine
5. One milliliter of a 10% solution of a drug contains how many milligrams of that drug? a . 10mg/ml b.20mg/ml c. 1.0 mg/ml d. 0.1 mg/ml e. 100mg/ml
a. b. c. d.
acts at the myoneural junction causes profound cardiac depression does not provide analgesia causes respiratory paralysis because of its muscle relaxant properties e. can be very effective in small doses
10. A drug with a similar mechanism of action to guaifenesin is: a. b. c. d. e.
diazepam glycopyrrolate butorphanol atracurium succinylcholine
11. In horses the lethal effects ofguaifenesin:
6. Of the following, which is rwt an action of phenothiazine tranquilizers on the nervous system? a. b. c. d.
antiemetic sympathomimetic potentiate seizures affect the reticular formation and hypothalamus e. sedation
7. Acepromazine is classified as: a. b. c. d. e.
9. Guaifenesin, a muscle relaxant commonly used for large animal anesthesia,:
a phenothiazine tranquilizer an anticholinergic an a2-adrenergic agonist an opioid agonist a benzodiazepine
B. In horses acepromazine should be used with caution in: a. pregnant mares b. horses with preexisting second-degree atrioventricular block c. working stallions d. adult draft horses e. arthroscopic surgical patients positioned in dorsal recumbency
a. are primarily manifested by respiratory center depression b. are not seen because such large volumes must be administered c. are primarily manifested by skeletal muscle relaxation resulting in respiratory muscle paralysis d. are seen at 3 to 4 times the therapeutic dose e. do not include cardiac depression 12. If you administer 350 ml ofa 5% solution of
guaifenesin to a 250-kg pony, how many mg/kg would be administered? a. b. c. d. e.
35 250 70 10 100
13. Atracurium: a. reduces the requirement for inhalation anesthesia because of its analgesic properties b. is a central-acting muscle relaxant c. crosses the placental barrier d. causes bradycardia following intravenous administration e. should not be used unless patients are receiving intermittent positive-pressure ventilation
© 1998 Mosby-Year Book, Inc. Photocopying is prohibited by law.
Anesthesiology
14. Elimination of atracurium from the body: a. produces the metabolite laudanosine, which depresses central nervous system function b. is enhanced by its inherent instability at body temperature and pH c. is prolonged in patients with renal disease only d. is prolonged in patients with both renal and liver disease e. is prolonged following intravenous infusion
15. Atracurium has advantages over succinylcholine in that: a. b. c. d.
it has a shorter duration of action it does not cause ganglionic blockade it does not cause muscle rigidity reversal of succinylcholine requires administration of neostigmine e. it paralyzes all skeletal muscles to the same degree
16. In an anesthetized patient with profound bradycardia, injection of neostigmine: a. b. c. d.
exacerbates the bradycardia rapidly resolves the bradycardia does not change the heart rate causes sudden arousal of the anesthetized animal e. prolongs muscle relaxation
17. In the central nervous system, what is the action ofxylazine or detomidine? a. b. c. d.
antagonism of internuncial neurons agonism of fl. -opiate receptors antagonism of fl.-opiate receptors agonism of presynaptic a2 adrenergic receptors e. antagonism of presynaptic a2 adrenergic receptors
lB. In 'horses and cattle xylazine premedication causes: a. increased cardiac output and heart rate b. second -degree atrioventricular block and bradycardia c. hypoglycemia
3
d. decreased uterine muscle tone in pregnant animals e. increased ventilation
19. In dogs appropriate treatment for bradycardia associated with xylazine administration is: a. an intravenous bolus dose of lactated Ringer's solution b. intravenous neostigmine c. intravenous atropine d. intravenous yohimbine e. intravenous doxapram
20. Morphine, oxymorphone, and fentanyl produce analgesia and somnolence primarily by their action on: a. b. c. d. e.
a 2 adrenergic receptors K-opiate receptors fl.-opiate receptors internuncial neurons in the spinal cord a-opiate receptors
21. Which of the following is the most potent opioid agent? a. b. c. d. e.
meperidine (Demerol) morphine oxymorphone (Numorphan) fentanyl (Sublimaze) butorphanol (Torbugesic)
22. Which drug does rwt have a narcotic effect? a. b. c. d. e.
levallorphan naloxone butorphanol oxymorphone meperidine
23. Concerning opioids, such as oxymorphone, which statement is least accurate? a. b. c. d. e.
They are good analgesics. They cause bradycardia. They improve ventilation. They depress laryngeal function. They cause minimal myocardial depression.
Correct answers are on pages 25-33.
www.vet4arab.co.cc
SECTION 1
4
24. Parenterally administered opiate analgesics, such as morphine, relieve pain by:
a. b. c. d.
inhibiting transmission of afferent impulses interfering with perception of pain interfering with the response to pain inhibiting initiation or transmission of impulses and disrupting perception of such impulses e. eliminating the source of pain impulses 25. Oxymorphone:
a. provides rapid induction of anesthesia and is suitable for patients with upper airway obstruction b. can be administered intravenously in combination with diazepam for chemical restraint suitable to achieve endotracheal intubation c. does not cause significant respiratory depression d. increases vagal tone and heart rate e. is likely to cause laryngospasm during endotracheal intubation 26. In dogs intravenous administration offentanyl causes:
a. b. c. d. e.
hypotension tachycardia atrioventricular block bradycardia decreased myocardial contractility
27. Following intravenous administration offentanyl to a dog, you observe marked bradycardia. The bradycardia could be appropriately treated with any of the following agents except:
a. b. c. d. e.
lidocaine atropine nalorphine naloxone butorphanol
c. does not depress respiration as much as pure fL-opioid agonists (e.g., morphine) d. is subject to the limitations of the Harrison Narcotic Act e. provides better visceral analgesia in dogs than does oxymorphone 29. Narcotic analgesics for horses include morphine, pentazocine, and butorphanol. Concerning these three drugs, which statement is most accurate?
a. They are all semisynthetic agents with opioid agonistic and antagonistic effects. b. They have no potential for abuse by people, so their use is not restricted. c. They may cause central nervous system excitement if administered to an otherwise untranquilized horse. d. Their effects can be antagonized with yohimbine. e. They increase gastrointestinal motility. 30. Which drug can antagonize opiate-induced respiratory depression?
a. b. c. d. e.
yohimbine naloxone ephedrine etorphine (M 99) dopamine
31 . Thiobarbiturates:
a. are characterized by slow induction of anesthesia b. should never be used in cats c. may produce cardiac arrhythmias following intravenous administration d. are not bound to plasma proteins e. produce effects that can be reversed with naloxone 32. Which property of thiobarbiturates accounts for their short duration ofaction?
28. As an analgesic, butorphanol:
a. has a longer duration of action than morphine b. is completely antagonized by naloxone
a. b. c. d. e.
rapidly metabolized quickly deposited in muscle and fat depots distributed in extracellular tissues rapid renal clearance Hoffman elimination
© 1998 Mosby-Year Book, Inc. Photocopying is prohibited by law.
Anesthesiology
33. After thiobarbiturate anesthesia, prolonged recovery:
a. b. c. d. e.
may occur in thin, emaciated patients should be treated with mechanical ventilation may be due to hyperproteinemia should be treated with neuroleptics may be due to hyperthermia
34. Thiobarbiturates cause:
a. b. c. d. e.
increased intracranial pressure renal toxicity bradycardia postinduction apnea increased intraocular pressure
35. Following anesthetic induction with thiopental in a dog, you intubate the animal and administer halothane and oxygen via a circle system. The dog immediately becomes apneic. The most appropriate course of action is to:
a. institute intermittent positive-pressure ventilation at 12 breaths/min, at a volume of 100 rnl/kg b. turn off the flow of halothane because the dog is deeply anesthetized c. stimulate upper airway reflexes by gently moving the endotracheal tube backward and forward d. institute intermittent positive-pressure ventilation at 2 or 3 breathS/min, at a volume of 10 rnl/kg e. administer another one-quarter dose of thiopental because the dog's apnea is caused by light anesthesia 36. Concerning use of thiopental in cats, which statement is most accurate?
a. Thiopental should not be used in cats because they are more sensitive to the drug and more prone to toxicity than are dogs. b., It can be given intramuscularly or subcutaneously to cats. c. It is an appropriate choice for uremic, dehydrated cats.
5
d. It is associated with excessive muscle tone, so diazepam should also be administered. e. It can cause increased laryngeal reflexes and coughing, especially at light anesthetic levels. 37. Propofol:
a. b. c. d. e.
lowers the seizure threshold increases muscle tone causes renal toxiCity causes postinduction apnea increases intraocular pressure
38. Anesthetic induction with propofol causes:
a. b. c. d. e.
bradycardia increased intracranial pressure hypotension in hypovolemic patients hypertension in normovolemic patients decreased cardiac output from myocardial depreSSion
39. In dogs recovery from propofol anesthesia is characteristically:
a. smooth and rapid b. rapid and accompanied by excitement c. rapid but rough because redistribution sites become saturated d. rapid because propofol is highly soluble in water and rapidly excreted in urine e. rapid and accompanied by marked ataxia on standing 40. Propofol:
a. is very irritating when administered perivascularly b. should be administered intramuscularly because it is a colloidal solution c. is dissolved in a soybean oil/egg lecithin emulsion d. should be refrigerated because it is unstable at room temperature e. can be used in multidose vials because it contains a bacteriostatic agent
Correct answers are on pages 25-33.
www.vet4arab.co.cc
SECTION 1
6
41. Concerning ketamine, which statement is /east accurate?
a. It is preferable to thiamylal as an induction agent in patients with impaired cardiovascular function. b. It can be combined with acepromazine, diazepam, or xylazine. c. It can cause hallucinations in human beings. d. It should never be administered intramuscularly. e. It may be associated with rough, hyperreflexive recoveries. 42. Ketamine:
a. causes tachycardia and hypertension via sympathetic stimulation b. causes circulatory depression c. decreases intraocular pressure d. decreases cerebral blood flow and intracranial pressure e. decreases myocardial oxygen consumption 43. Ketamine is a good choice for anesthetic induction in a:
a. cat with an elevated blood urea nitrogen level and a history of chronic renal failure b. dog that has been hit by a car and has evidence of head trauma c. cat with hyperthyroidism and a resting heart rate of 200 beats/min d. brachycephalic dog with upper airway obstruction e. male cat with a history of urethral obstruction 44. Concerning preanesthetic agents, which statement is /east accurate?
a. Meperidine is used for premedication and pain relief. b. Glycopyrrolate may be used for premedication to reduce the incidence of vagus-mediated bradycardia. c. Ketamine is an excellent example of a hypnotic premedicant. d. Some premedicants may prolong recovery from general anesthesia. e. An advantage of many premedicants is that they reduce the amount of anesthetic agent required to induce and maintain general anesthesia.
45. Major classes of drugs usually considered preanesthetic medications include all the following except:
a. b. c. d. e.
dissocia tives opioids hypnotics sympathomimetics neuroleptanalgesics
46. Concerning injectable anesthetics, which statement is most accurate?
a. b. c. d. e.
TIletamine is an opioid. Thiamylal is a hypnotic. Detomidine is a phenothiazine tranquilizer. Propofol is a dissociative. Meperidine is a benzodiazepine.
47. Dogs are given neuroleptanalgesic combinations to achieve profound sedation and analgesia. An example of this combination is:
a. b. c. d. e.
atropine and acepromazine droperidol and glycopyrrolate oxymorphone and thiopental butorphanol and ketamine fentanyl and acepromazine
48. Parenteral administration of which drug causes some visceral analgesia?
a. b. c. d. e.
ketamine acepromazine glycopyrrolate pentobarbital atracurium
49. All the following drugs are almost completely metabolized by the liver before renal excretion except:
a. b. c. d. e.
diazepam thiopental fentanyl glycopyrrolate ketamine
© 1998 Mosby-Year Book, Inc. Photocopying is prohibited by law.
50. To produce epidural anesthesia in a dog, a local anesthetic drug should be injected at the:
a. sacrococcygeal junction to avoid recumbency b. junction of the fifth and sixth lumbar vertebrae to ensure mixing with cerebrospinal fluid c. lumbosacral space, between the periosteum and dura mater d. lumbosacral space, between the pia mater and arachnoid e. lumbosacral junction, between the pia mater and arachnoid 51. If2% lidocaine is appropriately administered epidurally to a dog at 1 mll3 to 4.5 kg, the dog is not likely to:
a. become recumbent and have hind limb analgesia b. remain ambulatory and have only partial hind limb analgesia c. maintain function of the diaphragm and intercostal muscles d. have analgesia of the ventral midline caudal to the umbilicus e. remain alert 52. Which of the following provides surgical analgesia of more than 2 hours'duration when administered epidurally in an appropriate dose?
a. b. c. d. e.
7
Anesthesiology
bupivacaine 0.5% (Marcaine) lidocaine 2% (Xylocaine) morphine at 0.5 mg/rnl (Duramorph) mepivacaine 2% (Carbocaine) meperidine at 50 mg/rnl (Demerol)
53. An epidural local anesthetic block in dogs should be used with caution or is contraindicated in:
a. a pregnant bitch presented for cesarean section b. a dog with anal fistulae c. an old dog requiring repair of a ruptured cranial cruciate ligament d. a dog with sacral fractures following automobile trauma e. a dog with an infected phalangeal joint requiring toe amputation
54. Hypotension occurring during spinal anesthesia may be due to:
a. b. c. d. e.
preganglionic sympathetic blockade blockade of nerves to the adrenal medulla muscular paralysis loss of spinal fluid venous constriction
55. Epidural local anesthesia may cause any of the following except:
a. central nervous system depression and seizures b. hypotension c. respiratory muscle paralysis d. local tissue inflammation e. analgesia of 12 hours' duration 56. An advantage of bupivacaine over lidocaine for epidural anesthesia for cesarean section in dogs is that bupivacaine:
a. has a short duration of action b. has minimal placental transfer and fetal uptake c. does not cause vasodilation and hypotension d. is more concentrated than lidocaine e. has fewer cardiac effects than lidocaine when inadvertently given intravenously 57. When attempting to administer an epidural local anesthetic to a dog at the lumbosacral junction, you aspirate cerebrospinal fluid. The most appropriate course of action is to:
a. not administer any epidural anesthetic but instead use general anesthesia for the procedure b. administer the local anesthetic as planned c. substitute morphine for lidocaine as the local anesthetic d. administer the local anesthetic but at one third less than the calculated dose e. keep trying until you correctly place the needle in the epidural space
Correct answers are on pages 25-33.
www.vet4arab.co.cc
SECTION 1
8
58. To produce spinal anesthesia, a local anesthetic should be injected:
a. b. c. d. e.
between the pia mater and arachnoid between the arachnoid and dura mater into the epidural space between the pia mater and spinal cord in the intramedullary area
59. When attempting to administer epidural morphine to a dog at the lumbosacral junction, you obtain cerebrospinal fluid. The most appropriate course ofaction is to:
a. not administer any morphine but rather use parenteral analgesia b. administer the epidural dose of morphine as planned c. use fentanyl instead of morphine d. administer morphine but at one third less than the calculated dose e. use lidocaine instead of morphine 60. Morphine for epidural/spinal use (Duramorph) is sold in 1 O-ml glass vials at a concentration of 0.5 mglml. Morphine for parenteral administration is sold in multiple-dose rubber-topped vials at a concentration of 15 mglml. Morphine for epidural/spinal use is packaged in smaller vials and at a lower concentration because:
a. drugs at concentrations greater than 1 mg/rnl should not be administered epidurally b. solutions in multiple-dose vials are less convenient to use than solutions in glass vials c. the volume of morphine used in epidural anesthesia is small d. the morphine in the two solutions is different e. the morphine solution supplied in glass vials is preservative free and intended for singlepatient use 61. In dogs presented for hind limb surgery. epidural injection ofmorphine:
a. increases the requirement for other anesthetic drugs b. decreases the requirement for other anesthetic drugs c. does not change the requirement for other anesthetic drugs
d. as the sole anesthetic agent provides sufficient analgesia for surgery e. does not produce forelimb analgesia
66. Which of the following is lWt a characteristic of an ideal inhalation agent?
a. nonflammable and nonexplosive at clinical concentrations b. low solubility in blood and tissues so that induction and recovery are rapid c. low fat solubility d. high solubility in blood and tissues so that saturation is more rapid and induction is hastened e. only minimally metabolized and excreted largely unchanged
62. As compared with halothane or methoxyflurane, isoflurane has:
a. lower solubility, resulting in rapid changes in anesthetic depth b. less effect on cardiac and pulmonary function c. greater potential to produce catecholamineinduced arrhythmias d. greater potency e. a more rapid rate of metabolism 63. The minimum alveolar concentration (MAC) ofan inhalation anesthetic preventing response to a standard noxious stimulus is a measure of potency. Of the following, which does lWt affect MAC?
a. age b. hypothermia c. concurrent administration of sedatives and tranquilizers d. duration of anesthesia e. pregnancy 64. Which inhalation agent is most potent?
a. b. c. d. e.
halothane nitrous oxide isoflurane methoxyflurane propofol
65. Mask induction of anesthesia using inhalation agents is:
a. the best way to induce anesthesia in a patient with an upper airway obstruction b. best accomplished using a volatile anesthetic that is highly soluble in blood, such as methoxyflurane c. best accomplished using a volatile anesthetic that is relatively insoluble in blood, such as halothane d. not influenced by the adequacy of ventilation e. less expensive than routine parenteral anesthetic inductions
© 1998 Mosby-Year Book, Inc. Photocopying is prohibited by law.
Anesthesiology
67. Which inhalation anesthetic is subject to the greatest degree of metabolism?
a. b. c. d. e.
nitrous oxide halothane methoxyflurane desflurane isoflurane
a. b. c. d. e.
isoflurane nitrous oxide methoxyflurane halothane desflurane
71. Concerning nitrous oxide, which statement is least accurate?
a. It should not be used in patients with conditions involving gas pockets, such as a pneumothorax. b. It is the most soluble inhalation anesthetic and rapidly moves into and out of the body. c. It can cause diffusion of hypoxemia at the end of inhalation anesthesia. d. It requires high fresh gas flows, increasing the cost of anesthesia. e. It is not scavenged by activated charcoal.
a. is very potent (MAC 0.23 vol%) and so should not be administered in simple in-circuit vaporizers b. has a low vapor pressure and can easily produce dangerously high concentrations c. is flammable in clinical concentrations d. is effective in low concentrations e. produces poor muscle relaxation 73. Renal tubular damage can occur in dogs subjected to prolonged methoxyflurane administration and is secondary to:
69. Increasing depth of inhalation anesthesia is accompanied by:
a. b. c. d. e.
d. should only be used with halothane e. should be used in closed-circuit anesthesia
72. Methoxyflurane:
68. Which inhalation anesthetic agent is most likely to induce cardiac arrhythmias in the presence of elevated blood levels of catecholamines?
a marked increase in cardiac output an increase in total ventilation per minute an increase in arterial carbon dioxide tension a decrease in dead space ventilation an increase in arterial blood pH and a decrease in respiratory frequency
70. Nitrous oxide:
a. has no analgesic activity b. should not be used in concentrations less than 80% c. is not sufficiently potent to produce anesthesia in dogs
9
a. b. c. d.
myocardial depression hepatic necrosis toxic effects of fluoride ions hypoxia that occurs at 1.5 times the maximum alveolar concentration e. hypotension 74. The term second-gas effect refers to:
a. increased ventilation caused by nitrous oxide b. displacement of oxygen from the alveoli by rapid escape of nitrous oxide from the blood during recovery from anesthesia c. the additive effect of inhalation anesthetics when given in combination with nitrous oxide d. distention of the bowel during inhalation of nitrous oxide e. low arterial oxygen levels caused by fresh gas mixtures that are less than 95% oxygen
Correct answers are on pages 25-33.
www.vet4arab.co.cc
SECTION 1
10
75. Uptake of inhalation anesthetics from the alveoli into the blood depends on the: a. MAC (potency) of the agent b. patient's packed cell volume (hematocrit) c. the agent's alveolar concentration and blood solubility and the patient's cardiac output d. inspired oxygen concentration and MAC e. tissue solubility of the agent
80. You anesthetize a healthy dog with halothane. During anesthesia the dog develops a ventricular arrhythmia. The most appropriate course of action is to:
76. Other constants being equal, the speed of induction with an inhalation anesthetic varies inversely with that agent's: a. b. c. d. e.
a. increase the anesthetic depth b. attempt defibrillation c. infuse lidocaine intravenously at 20 to 40 flog/kg/min d. increase the flow of oxygen e. institute intermittent positive-pressure ventilation
rubber/gas solubility ratio molecular weight blood/gas solubility coefficient MAC saturated vapor pressure
77. Which of the following accounts for a patient's rapid awakening after halothane or isoflurane anesthesia, as compared with recovery from methoxyflurane anesthesia? a. Liver metabolism and renal excretion occurs rapidly. b. They are relatively insoluble in blood and clear rapidly from the lungs. c. They have no analgesic properties, so ventilation is stimulated by pain. d. They depress ventilation less than methoxyflurane, and more normal ventilation does not slow recovery. e. They are more potent than methoxyflurane, so there is less anesthetic to eliminate.
81 . Delivery of inhalation anesthetics from out-ofcircuit precision vaporizers depends on the fresh gas flow rate. A 30-kg dog requires 5 ml of halothane per minute to stay anesthetized. If the fresh gas (O~ flow rate is either 500 or 120 ml! min, the appropriate vaporizer setting is: a. b. c. d. e.
1.5% for both flow rates 1% and 4%, respectively 5% for both flow rates 0.5% and 5%, respectively 1% and 2.5%, respectively
82. Why are precision halothane vaporizers installed outside the circle system? a. Vaporizers require complicated pressure valves for installation within the circle system. b. They use less halothane than in-circuit vaporizers. c. They increase absorption of carbon dioxide. d. They are less affected by inspiration and expiration than in-circuit vaporizers. e. High concentrations of halothane can be delivered.
78. All the following influence the speed of recovery from inhalation anesthesia except: a. b. c d. e.
c. is the only inhalation agent that lowers the threshold to catecholamine-induced ventricular arrhythmias d. depresses cardiac function by decreasing myocardial contractility, causing vasodilation and depressing the sinoatrial node e. is more expensive to use than methoxyflurane
duration of anesthesia potency of the agent pulmonary blood flow alveolar ventilation solubility of the agent in body tissues
Anesthesiology
83. Inhalation anesthesia can be induced more rapidly in neonatal animals than adults because neonates: a. are relatively smaller but have a proportionately larger surface area/body weight ratio b. contain less body water c. become hypothermic, so less anesthetic is required d. have relatively greater alveolar ventilation and cardiac output e. have a relatively larger larynx 84. Which of the following is not an appropriate action to combat hypotension in a dog anesthetized with halothane? a. decrease the anesthetic concentration b. increase the rate of intravenous fluid administration c. treat the underlying cause (e.g., hypovolemia) d. administer an intravenous bolus of epinephrine e. infuse dopamine or dobutamine intravenously 85. For which agent does the reading on the pressure gauge give no indication of quantity ofgas remaining in the cylinder? a. b. c. d. e.
nitrogen oxygen helium halothane nitrous oxide
a. is eliminated completely unchanged via the lungs b. is the most potent respiratory depressant of the commonly used inhalation anesthetics
86. The pressure in a nitrous oxide cylinder decreases when: a. the cylinder is one half full b. the cylinder is one fourth full c. all the liquid nitrous oxide has vaporized, which occurs when the tank is nearly empty d. the cylinder is heated e. the cylinder is initially opened, then the pressure gradually decreases until the tank is empty
© 1998 Mosby-Year Book,
Inc. Photocopying is prohibited by law.
a. oxygen b. carbon dioxide
c. nitrous oxide d. cyclopropane e. propane 88. Cylinders full of compressed gas: a. are safe even if they are knocked over and the valve fractures b. are marked in different colors to identify the manufacturer c. have connecting thread sizes and diameters that are unique for each gas d. do not need to be fixed to a solid object once inside a building e. are lighter than empty cylinders 89. The oxygen flush valve: a. allows oxygen to flow into the breathing system without going through the vaporizer b. is part of the vaporizer c. is used to rapidly increase the concentration of anesthetic within the breathing system d. should only be used in emergencies e. allows excess pressure to escape from the breathing system 90. With a vaporizer located within the circle breathing system, which of the following will increase the anesthetic concentration within the breathing circuit?
87. Which gas is not liquefied by pressure applied at room temperature?
79. Halothane:
11
a. b. c. d. e.
increased tidal volume increased body temperature increased inspired oxygen concentration decreased humidity in the patient circuit decreased rubber to gas solubility of the anesthetic
91. Concerning vaporizers lacated within the breathing circuit, which statement is most accurate? a. They can only be used for anesthetics with low vapor pressure (e.g., methoxyflurane) . b. They can only be used for anesthetics with high vapor pressure (e.g., halothane) . c. They do not alter resistance to gas flow in the circuit. d. They must be high-efficiency vaporizers. e. As fresh gas flow to the circuit increases, the circuit anesthetic concentration decreases.
Correct answers are on pages 25-33.
SECTION 1
12
92. Which of the following ensures unidirectional gas flow in a circle system?
a. unidirectional flow of fresh gas into the circuit b. pop-off valve that allows gas flow out of the circuit c. resistance by soda lime d. reduced pressure in the patient's lungs e. inspiratory and expiratory one-way valves
96. Concerning soda lime and the carbon dioxide absorption canister, which statement is kast accurate?
a. Unused soda lime is white before the patient is connected to the circuit. b. Soda lime becomes blue or pink after prolonged service. c. The soda lime canister becomes warm when in service. d. The soda lime granules are easily crushed between one's fingers when the granules are exhausted. e. The soda lime granules are easily crushed between one's fingers when the granules are unused.
93. Which item is not a component of the breathing circuit in a circle anesthetic system?
a. b. c. d. e.
rebreathing bag exhaust (pop-off) valve oxygen flush valve Ypiece inspiratory and expiratory one-way valves
94. If the inspiratory unidirectional valve in a circle system patient breathing circuit is assembled without the valve leaflet in place, what would happen if the machine were put into service?
a. no untoward results because the other unidirectional valve would function properly b. anesthetic overdose c. increased concentration of carbon dioxide in the inspired gas d. increased resistance to breathing e. no untoward results because the rebreathing hoses would still be in place 95. The machine dead space ofa circle system (the gas volume that must be rebreathed at each breath containing carbon dioxide from the previous expired breath) is comprised of the:
a. area from the inspiratory valve to the endotracheal tube connection b. area from the Ypiece connection to the endotracheal tube c. entire patient breathing circuit d. area from the soda lime canister to the endotracheal tube connection e. area from the expiratory valve to the endotracheal tube connection
www.vet4arab.co.cc
97.
If the soda lime in the breathing circuit becomes exhausted during an anesthetic procedure, you would expect to observe: a. only an increased respiratory rate b. only an increased depth of breathing c. an increased heart rate and decreased blood pressure d. an increased heart rate and increased blood pressure e. no physiologic changes
98. A problem within an anesthetic waste gas scavenging system that could be fatal to the patient is:
a. exhaustion of the activated charcoal canister b. occlusion of the tubing from the pop-off valve to the scavenger c. overfilling of the reservoir container with spillage out of the positive-pressure relief valve d. disconnection of the tubing from the pop-off valve e. occlusion of the vacuum hose from the reservoir container
© 1998 Mosby-Year Book, Inc. Photocopying is prohibited by law.
Anesthesiology
99. To check for gas leaks within the breathing circuit ofa circle rebreathing system:
a. the circuit must be immersed in a container of water b. the gas flow into the circuit must be measured c. the rebreathing hoses and bag must be disconnected d. a functional in-circuit pressure gauge is necessary e. resistance to collapse of the reservoir bag can be assessed manually 100. A rebreathing circle anesthetic system is described as semiclosed when:
a. b. c. d. e.
the oxygen flowmeter is partly closed gas flows out through the pop-off valve the vaporizer is at a low setting the pop-off valve is closed the oxygen flowmeter is completely closed
101. The efficiency of carbon dioxide absorption by a soda lime canister depends on the:
a. b. c. d. e.
length of the canister volume of soda lime used amount of water vapor in the expired gases resistance the soda lime offers to gas flow volume and distribution of gas flOwing through the canister
103. Nonrebreathing anesthetic circuits:
a. are generally used in animals weighing less than 5 kg b. have more resistance than circle systems c. require lower fresh gas flows than circle systems d. produce less waste anesthetic gas pollution than circle systems e. cannot be used for patients requiring positive-pressure ventilation 104. A characteristic of the T piece or Norman elbow anesthetic delivery system is that it:
a. allows economical use of anesthetic gases and vapors b. does not require a soda lime canister c. requires low fresh oxygen flows d. results in minimal environmental pollution e. can be used for closed -circuit anesthesia 105. In a Bain's nonrebreathing circuit, the flow of fresh oxygen should be:
a. only sufficient to meet the animal's oxygen requirement b. approximately 30 mllkg/min c. sufficient to prevent rebreathing of expired gases d. able to fill the reservoir bag e. low enough to prevent gas flow out through the pop-off valve
·102. In a functional anesthetic circuit:
a. unidirectional in-circuit valves ensure movement of oxygen from the flowmeter to die preciSion vaporizer b. the reservoir bag stores fresh anesthetic gas and must accommodate the patient's tidal volume c. soda lime canisters help cool the warm gas expired by the patient d. precision out-of-circuit vaporizers receive gas flow from the reservoir bag e. oxygen tanks and flow meters are in the patient circuit portion of a circle system
106. When using a nonrebreathing circuit and an out-of-circuit vaporizer, the inspired concentration of anesthetic:
a. changes slowly with vaporizer adjustments b. is about 20% to 30% lower than the vaporizer setting c. is about 20% to 30% higher than the vaporizer setting d. is the same as the vaporizer setting e. depends on the blood/gas solubility ratio of the anesthetic agent
Correct answers are on pages 25-33.
13
14
www.vet4arab.co.cc
SECTION 1
107. The minimum fresh oxygen flow required for a patient rebreathingfrom a circle anesthetic system is:
a. determined by the size of the reservoir bag b. dependent on the location of the vaporizer c. determined by the patient's respiratory rate and volume d. dependent on the volume of the anesthetic circuit e. determined by the patient's metabolic rate 108. A dog's metabolic requirement for oxygen is calculated as:
a. b. c. d. e.
twice the volume of carbon dioxide produced 30 mllkg/min x body surface area 200 mJ/kg/min x body surface area kg" 75 x 10 mJ/min kg"·5 x 5 mil min
109. Which breathing system could be used for closed-system/low-flowanesthesia?
a. b. c. d. e.
circle rebreathing system McGill breathing system Ayres T piece Norman elbow Bain's breathing circuit
110. Which of the following is not an advantage ofa closed anesthetic system?
a. b. c. d. e.
economical use of anesthetic agents high humidity in the inspired gas slow changes in anesthetic concentration conservation of body heat reduced anesthetic pollution
111. Which of the following is a disadvantage of tracheal intubation?
112. Which of the following is nota reliable indication ofsuccessful intubation?
a. condensation inside the tube with expiration b. tube placement confirmed with a laryngoscope c. no tube evident on palpation of the esophagus at the level of the larynx d. movement of a piece of lint held at the tube end with expiration or inspiration e. chest wall movement on manual compression of the reservoir bag 113. Ifyou were to measure halothane concentration in a circle system during the first 5 minutes of anesthesia, which of the following would be true?
a. Concentrations of expired halothane would be lower than concentrations of inspired halothane. b. Concentrations of expired halothane would be higher than concentrations of inspired halothane. c. Concentrations of inspired halothane would be higher than concentrations of halothane in the vaporizer. d. Concentrations of expired halothane would be equal to concentrations of inspired halothane. e. Concentrations of inspired halothane would be lower than concentrations of expired halothane. 114. Which of the following values is least likely to change when the depth ofanesthesia is changed?
a. b. c. d.
blood pressure expired concentration of carbon dioxide jaw muscle tone oxygen saturation of lingual tissues (pulse oximetry) e. minute ventilation
a. Connects the patient's airway with the anesthetic circuit. b. Prevents dilution of inspired anesthetic gases with room air. c. May decrease anatomic dead space. d. May increase mechanical dead space. e. Facilitates positive-pressure ventilation.
© 1998 Mosby-Year Book, Inc. Photocopying is prohibited by law.
Anesthesiology
115. In horses, which of the following indicates a deep plane of anesthesia?
a. mean arterial pressure of 60 mm Hg, respiratory rate of6/min, corneal reflex present b. mean arterial pressure of 75 mm Hg, respiratory rate of 4/min, spontaneously closes eye c. mean arterial pressure of 60 mm Hg, respiratory rate of 3/min, periodic nystagmus d. mean arterial pressure of 76 mm Hg, respiratory rate of 6/min, brisk palpebral reflex e. mean arterial pressure of70 mm Hg, respiratory rate of 4/min, spontaneously closes eye, lacrimation 116. An esophageal stethoscope:
a. should be placed just cranial to the thoracic inlet b. is useful for assessing cardiac function c. should be positioned to auscultate the loudest heart and lung sounds d. requires considerable technical training for competent use e. should be placed just cranial to the esophageal sphincter .117. During ovariectomy in a dog anesthetized with halothane, the heart rate drops from 120 to 50 beats/min. Which of the following is most appropriate for treatment ofbradycardia?
a. b. c. d. e.
ketamine intravenously at 6 to 10 mg/kg atropine intravenously at 0.01 to 0.02 mg/kg epinephrine intravenously at 0.5 to 1.0 mg/kg dopamine intravenously at 5 to 10 fJ.g/kg/ min isoproterenol intravenously at 0.5 to 1.0 mg/kg
118. In cats a heart rate of30 beats/min is considered dangerously low. When used at recommended dosages, the drug most likely to cause bradycardia is:
15
c. xylazine d. tiletamine-zolazepam e. propofol 119. Tachycardia during anesthesia:
a. b. c. d. e.
may be due to hyperkalemia may be due to hypercapnia should be treated with lidocaine should be treated with isoproterenol is a sign of deep anesthesia
120. Palpation of the arterial pulse provides an indication of
a. b. c. d.
systolic arterial pressure mean arterial pressure systolic and diastolic pressures the difference between systolic and diastolic pressures e. the difference between systolic pressure and no blood pressure 121. Arterial blood pressure:
a. reflects only the amount of vasoconstriction present b. may appear spuriously high if an overly wide cuff is used for indirect measurement c. is more accurately measured using the Doppler indirect method than using an arterial catheter and pressure transducer d. depends on cardiac output, total peripheral resistance, and circulating blood volume e. indicates the adequacy of tissue perfusion 122. Which of the following is not used to detect the arterial pulse for indirect determination of blood pressure?
a. b. c. d. e.
pulse oximetry Doppler ultrasound oscillometry palpation auscultation
a. thiopental b. ketamine
Correct answers are on pages 25-33.
www.vet4arab.co.cc
SECTION 1
16
123. Indirect blood pressure measurement techniques use a pneumatic cuffand a method to detect pressure changes, sound, or blood flow. Compared to direct measurements of blood pressure via an arterial catheter, indirect methods:
a. are just as reliable and automated b. accurately measure both systolic and diastolic pressures c. accurately measure mean arterial pressure d. are not influenced by the size of the cuff or the animal's limb e. are influenced by the animal's position 124. When indirectly measuring blood pressure:
a. a loose cuff yields more accurate readings than a snugly applied cuff b. the cuff should be placed distal to the site of pulse palpation or auscultation c. the cuff width should be 30% to 40% of limb circumference d. a loose cuff yields spuriously low readings e. an overly narrow cuff yields spuriously low readings 125. In a laterally recumbent anesthetized dog, systolic arterial pressure is highest in the:
a. b. c. d. e.
aorta carotid artery renal artery femoral artery cranial tibial artery
126. If a transducer used to measure the blood pressure (e.g., Doppler crystal, dir~ct pre~sure transducer, aneroid manometer) IS applied to an animal and located lower than the level of the heart:
a. the systolic and diastolic pressure readings will be spuriously high b. the systolic and diastolic pressure readings will be spuriously low c. the systolic and diastolic pressure readings will be accurate d. only the diastolic pressure reading will be spuriously high e. only the diastolic pressure reading will be spuriously low
127. Mean arterial pressure is:
a. equal to the pulse pressure b. equal to diastolic pressure plus one third of the difference between systolic and diastolic pressures c. the difference between the average systolic and diastolic pressures d. not as important as systolic pressure e. a measure of perfusion 128. An anesthetized animal would be considered hypotensive if the:
a. systolic arterial pressure were below 120 mm Hg b. systolic arterial pressure were below 100 mm Hg c. diastolic arterial pressure were below 80 mm Hg d. diastolic arterial pressure were below 60 mm Hg e. mean arterial pressure were below 60 mm Hg 129. Appropriate treatment for hypotension during inhalation anesthesia includes any of the following except:
a. administration of a vasopressor (e.g., dobutarnine) b. decreasing the concentration of inspired anesthetic c. intravenous administration of boluses of fluid at 10 to 20 mllkg d. administration of a stimulant (e.g., doxapram) e. administration of a j3-adrenergic agonist (e.g., dobutarnine) 130. Cardiac output:
a. is the product of heart rate X arterial blood pressure b. is increased by most general anesthetics c. is increased by most inhalation anesthetics d. may be monitored using the Doppler unit placed on a front or hind limb e. may decrease with bradycardia
© 1998 Mosby-Year Boo/;;, Inc. Photocopying is prohibited by law.
131. A vasoconstrictor drug that exerts little or no inotropic effect is:
a. b. c. d. e.
epinephrine ephedrine isoproterenol phenylephrine dopamine
a. b. c. d. e.
dilates renal arteries stimulates a-receptors increases the heart rate increases the respiratory rate causes bradycardia
epinephrine phenylephrine dopamine dobutamine ephedrine
increase the blood pressure increase the heart rate increase tissue perfusion decrease hemoglobin saturation with oxygen decrease production of carbon dioxide
135. Anatomic dead space:
a. b. c. d. e. _.
a. b. c. d. e.
respiratory rate mucous membrane color Paco2 pH of arterial blood tidal volume
139. The effect of increased arterial carbon dioxide levels in an otherwise normal anesthetized animal is:
134. The objective ofadministering a positive inotrope to a hypotensive anesthetized animal is to:
a. b. c. d. e.
200 mllkglbreath 12 mllkglbreath 30 mllkglbreath kg"·75 x 10 mil breath 5 mllkglbreath
138. The best indicator of adequacy of ventilation in anesthetized animals is:
133. When administered for hemostasis in an animal anesthetized with halothane, which drug is most likely to cause premature ventricular contractions?
a. b. c. d. e.
d. Vt 160 ml, respiratory rate of IOlmin e. Vt 250 mI, respiratory rate of 151min 137. An average tidal volume for a dog at a light anesthetic level is:
132. In small doses dopamine:
a. b. c. d. e.
17
Anesthesiology
cannot change during anesthesia usually is greater than the tidal volume can increase during anesthesia is the same as mechanical dead space decreases as the duration of anesthesia increases
136. Which tidal volume (VJ and respiratory rate produce the most ventilation?
a. Vt 160 mI, respiratory rate of 15/min b. Vt 250 mI, respiratory rate of 12/min c. Vt 400 mI, respiratory rate of 8/ min
a. b. c. d. e.
a decrease in respiratory rate a decrease in respiratory depth a sudden decrease in blood pressure a sudden decrease in heart rate injected mucous membranes
140. Lung compliance is a measure of:
a. the resistance to lung inflation by manual compression of the reservoir bag b. the amount of pressure required to inflate the lung c. the difference in pressure between the ventilator and the rebreathing circuit d. the change in lung volume for each unit of transpulmonary pressure change e. end-inspiratory pressure indicated on the pressure gauge located in the breathing circuit 141. Carbon dioxide acts on the heart by causing:
a. b. c. d. e.
decreased cardiac output increased cardiac output decreased ventricular filling in diastole increased venous return decreased afterload
Correct answers are on pages 25-33.
www.vet4arab.co.cc
SECTION 1
18
142. An appropriate treatment to combat hypercapnia in anesthetized small animals is:
a. b. c. d. e.
ephedrine to stimulate ventilation dobutamine to increase the heart rate increased flow rate of oxygen decreased anesthetic depth addition of nitrous oxide to the inspired gas
143. In a nonanesthetized animal ventilating normally. you would expect the Pacoz to be:
a. b. c. d. e.
approximately 25 mrn Hg approximately 40 mrn Hg approximately 55 mm Hg twice the value of Pao2 half the value of Pao 2
144. An increase in arterial carbon dioxide levels during anesthesia in an otherwise healthy animal:
a. decreases the plasma bicarbonate level below 20 mrnollL b. increases the base excess above 3 mEq / L c. has no effect on blood pH d. increases the blood pH e. decreases the blood pH 145. Oxygenation of tissues depends on the:
a. oxygen-carrying capacity of the blood b. saturation of hemoglobin c. oxygen content of arterial blood and blood flow d. body temperature e. arterial oxygen level 146. The hemoglobin-oxygen dissociation curve is shifted to the left by:
a. b. c. d. e.
decreased blood pH decreased body temperature increased blood 2.3-diphosphoglycerate levels increased arterial carbon dioxide levels increased hemoglobin levels
147. Which arterial oxygen tension (Paaz) would markedly alter the arterial oxygen content?
a.560mmHg b.300mmHg c. 200mrnHg d.140mmHg e.70mmHg 148. Which of the following is least likely to increase the tissue oxygen delivery in an anesthetized horse breathing 90% oxygen and with a Paaz of 65 mm Hg and a Pacaz of 45 mm Hg?
a. a blood transfusion to raise the hemoglobin concentration by 20% b. intravenous dobutamine infusion at 3 ",g/kg/min c. positive-pressure ventilation to reduce the Paco2 to 40 mrn Hg d: a reduction in the alveolar halothane level from 1.8 to 1.1 vol% e. intravenous administration of a bolus of fluid at 20 mllkg and ephedrine at 0.1 mg/kg 149. An anesthetized dog breathing 33% oxygen has a Pacoz of39 mm Hg. a Paaz of 43 mm Hg. and a blood pH of 7.39. What is the most appropfiate course of action?
a. increase blood carbon dioxide levels to create greater ventilatory drive b. administer sodium bicarbonate intravenously c. increase the flow rate of oxygen d. replace the soda lime in the canister e. continue with the procedure. since these values indicate no need for changes in the anesthetic protocol 150. Which of the following is notan effect of carbon dioxide on the brain?
a. b. c. d. e.
increases cerebral blood flow decreases neuronal activity increases cerebrospinal fluid pressure causes an inert gas-like narcotic effect decreases cerebral blood pressure
© 1998 Mosby-Year Book. Inc. Photocopying is prohibited by lLlw.
Anesthesiology
19
E.M. Wertz 151. In dogs anatomic landmarks used to identifY the epidural space are the:
a. dorsal spinous process of L7 and the dorsal aspect of the ilial wings b. dorsal spinous processes of the sacrum and the iliac crests c. dorsal depression on the midline cranial to the dorsal spinous process of L5 d. dorsal spinal processes of L5-6 and the proximal aspect of both femurs e. first obvious midline depression caudal to the sacrum 152. In an attempt to inject bupivacaine epidurally. the needle enters the subarachnoid space. What is the most appropriate course ofaction?
a. Only half of the calculated dose should be administered. b. The entire calculated dose can be administered without complication. c. A volume of cerebrospinal fluid equal to the volume ofbupivacaine to be injected should be withdrawn and then the bupivacaine injected. d. Two times the calculated dose should be administered. e. The needle should be withdrawn and no further attempt made to administer bupivacaine epidurally. 153. In an attempt to inject morphine epidurally. the needle enters the subarachnoid space. What is the most appropriate course ofaction?
a. Only half of the calculated dose should be administered. b. The entire calculated dose can be administered without complication. c. A volume of cerebrospinal fluid equal to the volume of morphine to be injected should be withdrawn and then the morphine injected. d. Two times the calculated dose should be administered. e. The needle should be withdrawn and no further attempt made to administer morphine epidurally.
154. Concerning nitrous oxide (NOz). which statem ent is most accurate?
a. N0 2 disassociates to produce O2 and N. eliminating the need for simultaneous O2 administration. b. N0 2 is a potent anesthetic (MAC = approximately 200%) in dogs. c. The high potency of N0 2 is related to its low solubility. d. The rapid onset of action of N02 is caused by its low blood:gas partition coefficient (0.47) . e. N0 2 is easily scavenged using an activated charcoal absorbent scavenging system. 155. Several scavenging methods are employed to minimize occupational exposure to inhalant anesthetic agents (halogenated hydrocarbons). Which statement most accurately describes an acceptable method ofexhaust ofanesthetic vapors?
a. The exhaust hose coming from the anesthetic machine is connected to the recirculating air flow system of the building. b. The exhaust hose coming from the anesthetic machine is connected to a canister containing activated charcoal. c. The exhaust hose coming from the anesthetic machine is positioned so it ends just above the level of the floor. dropping the vapors just at ground level. d. The exhaust hose coming from the anesthetic machine is connected to a floor drain. e. The pop-off valve is kept closed during anesthesia to prevent escape of anesthetic vapors into the room. 156. Propofol is classified as:
a. b. c. d. e.
a phenothiazine tranquilizer an <X2-agonist a sedative-hypnotic an anticholinergic an opioid agonist
Correct answers are on pages 25-33.
www.vet4arab.co.cc
SECTION 1
20
157. In dogs hypotension and apnea may occur on induction of general anesthesia with propofol. These effects can be minimized by: a. slow intravenous injection (over 30 to 60 seconds). given to effect b. administration of half of the calculated dose, rapid, followed by slow infusion of the remainder c. no sedation or tranquilization before propofol administration d. administration of the calculated dose intramuscularly and waiting 5 minutes for onset of the full effect e. heavy premedication with morphine before induction
158. The duration of analgesia produced by morphine administered epidurally is: a. b. c. d. e.
159. After epidural administration ofmorphine, the onset ofanalgesia becomes evident in approximately: . a. b. c. d. e.
15 to 30 seconds 5 to 10 minutes 15 to 30 minutes 1 hour 2 to 3 hours
Sevoflurane 160 Yes No(?) 47 0.68 2.6
Desflurane
Methoxyflurane
240 None Stable 91 1.4
243 Yes Decomposes 224 2.4 0.87
23 Yes Decomposes 970 13.0 0.23
664 None Yes 18.7 0.42 Approx.6.0
160. Which anesthetic has the most rapid onset of action? a. b. c. d. e.
Isoflurane
Halothane
halothane isoflurane sevoflurane desflurane methoxyflurane
1.5
flow compensated. With which vaporizer must you manually compensate for fresh gas flow and temperature? a. b. c. d. e.
desflurane sevoflurane isoflurane halothane methoxyflurane
Ohio#8 Vapor "Tec" types Copper kettle "Matic" types
164. When an anesthetized patient is ventilated by wind, the volume of each breath delivered is based on:
] 62. Which anesthetic can be safely administered
using a nonprecision vaporizer? a. b. c. d. e.
desflurane isoflurane methoxyflurane halothane sevoflurane
167. Which of the following is not an agonist: antagonist pair? morphine and naloxone butorphanol and yohimbine detomidine and tolazoline medetomidine and atipamezole diazepam and flumazenil
163. Most precision vaporizers are temperature and
161. Which anesthetic is the most potent? a. b. c. d. e.
166. During an anesthetic procedure, the pulse oximeter reads 91%. Which partial pressure of oxygen would not correlate with this reading?
a. b. c. d. e.
a. the size ofthe rebreathing bag attached to the anesthetic machine b. the patient's minute ventilation c. the patient's tidal volume d. a desired peak airway pressure of 40 to 50 cm of water e. expansion of the patient's chest wall on observing the patient
© 1998 Mosby-Year Book, Inc. Photocopying is prohibited by law.
168. Which drug is administered to partially antagonize the effects of morphine while maintaining some analgesia? a. b. c. d. e.
flumazenil yohimbine butorphanol naloxone acepromazine
cats rabbits horses swine cattle
171. In a dog breathing room air at sea level, the expected partial pressure of oxygen would be: a.55t060mmHg b.65t070mmHg c. 75 to 80 mm Hg d. 85t090mmHg e. 95 to 100 mm Hg 172. In a dog breathing 100% oxygen at sea level, the expected partial pressure of oxygen would be: a.60t065mmHg b.80t085mmHg c. >llOmmHg d. 100 to 105 mm Hg e.90t095mmHg 173. A packed cell volume of36% most closely approximates a hemoglobin content of a. b. c. d. e.
6 g/dl 9 g/dl
12 g/dl 18 g/dl 36 g/dl
174. Which packed cell volume indicates the lowest acceptable oxygen-carrying capacity?
169. Which species is least susceptible to the effects of xylazine? a. cats b. rabbits c. horses d. swine e. cattle
170. Which species is most susceptible to the effects of xylazine? a. b. c. d. e.
capnometer pulse oximeter Wright's respirometer Doppler ventilator
a. 91 mmHg b.150mmHg c.300mmHg d.52mmHg e.200mmHg
Questions 160 through 162 Vapor pressure (mm Hg) at 20' C Preservatives Stability in soda Hme Oil:gas partition coefficient Blood:gas partition coefficient MAC (dogs)
165. Which instrument is used to measure a patient's tidal volume? a. b. c. d. e.
20 to 40 minutes 45 to 60 minutes 4 to 6 hours 16 to 24 hours 48 to 72 hours
21
Anesthesiology
a. 12% b.21% c. 18% d. 30% e.35%
Correct answers are on pages 25-33.
22
www.vet4arab.co.cc
SECTION 1
175. Which of the following is the least appropriate treatment for a mean blood pressure of60 mm Hg?
a. Decrease the percent of inhalation anesthetic being administered. b. Administer fluids intravenously. c. Administer dobutamine. d. Administer ephedrine. e. Begin intermittent positive-pressure ventilation. 176. Which neuromuscular relaxant produces depolarization at the motor end-plate, causing muscle fasciculations?
a. b. c. d. e.
propofol pancuronium atracurium succinylcholine midazolam
180. Concerning use of acepromazine as a preanesthetic agent, which statement is least accurate?
a. Acepromazine decreases the amount of anesthetic required. b. Acepromazine decreases the incidence of vomiting. c. Acepromazine smoothes the recovery period, minimizing paddling and vocalization. d. Acepromazine increases the amount of agent necessary for anesthetic induction. e. Acepromazine decreases the incidence of epinephrine-induced arrhythmias. 181. Concerning propofol, which statement is least accurate?
177. Which neuromuscular relaxant cannot be effectively antagonized?
a. b. c. d. e.
d. determine appropriate preanesthetic medications for the patient e. choose an appropriately sized intramedullary pin for fracture fixation
gallamine pancuronium atracurium succinylcholine vecuronium
178. An E cylinder of oxygen contains approximately 700 L of oxygen when full. Approximately how many liters of oxygen is contained in an E cylinder when the pressure is 1100 psi?
a. b. c. d.
350L 700L 175 L More information is necessary to determine the volume of gas remaining. e. 525 L 179. The pin index safety system is used to:
a. prevent overfilling of the anesthetic vaporizer b. choose an appropriately sized endotracheal tube for the patient c. prevent interchange of oxygen and nitrous oxide E cylinders at the yoke hookup
a. It is administered intravenously or intramuscularly to induce anesthesia. b. It induces apnea when administered intravenously. c. It induces hypotension when rapidly administered intravenously. d. Because the vial contains no preservatives, any unused portion should be discarded. e. It does not cause tissue sloughing following inadvertent perivascular administration.
Anesthesiology
183. What is the best preanesthetic to manage this case?
a. b. c. d. e.
oxymorphone and atracurium oxymorphone and acepromazine ketamine and diazepam atropine and acepromazine No preanesthetic should be used.
184. Which anesthetic would be the best for maintenance ofanesthesia?
a. b. c. d. e.
halothane in oxygen methoxyflurane in oxygen intravenous bolus of pentobarbital isoflurane in oxygen constant intravenous infusion of thiopental
185. How would ventilation best be managed?
a. b. c. d. e.
intermittent positive-pressure ventilation continuous positive airway pressure positive end-expiratory pressure spontaneous ventilation assisted ventilation
182. Which of the following poses the greatest danger during anesthesia?
a. b. c. d. e.
hepatic dysfunction kidney dysfunction hypotension hypertension hypercapnia
© 1998 Mosby-Year Book, Inc. Photocopying is prohibited by law.
c. procaine and bupivacaine d. mepivacaine and chloroprocaine e. procaine and lidocaine 189. Which local anesthetic has the longest duration of effect?
a. b. c. d. e.
procaine lidocaine mepivacaine chloroprocaine bupivacaine
190. Which local anesthetic is ineffective when applied topically?
a. b. c. d. e.
procaine hexylcaine lidocaine dibucaine tetracaine
Questions 191 through 196 ~~ a 40-kg German shepherd with osteosarcoma ofthe
186. Which method of noninvasive monitoring is necessary in this patient?
a. b. c. d. e.
indirect blood pressure measurement pulse oximetry tidal volume measurement electrocardiography capnometry
Questions 182 through 186 A depressed, 15-year-old golden retriever has a head tilt to the right. The complete blood count and serum chemistry results are within normal limits. To confirm your suspicions of an intracranial mass, computerized tomography (CT scan) is necessary. The procedure requires general anesthesia.
23
187. Local anesthetics exert their effect by:
a. interruption of the Na-Kpump, inhibiting any ion exchange b. occupation and blockade of muscarinic cholinoreceptors c. prevention of rapid influx of Na into nerve axons, preventing the action potential d. inactivating slow Ca channels e. occupation and inhibition of nicotinic cholinoreceptors 188. Which pair of drugs are "amide" types of local anesthetics?
zllum, hemzpelvectomy is selected as the treatment. Preoperative blood studies are normal. Morphine and atropine are given as premedicants. Anesthesia is induced with intravenous thiopental to effect and maintained with isoflurane in oxygen. A morphine epidural is administered to help manage pain during and after surgery. Patient monitoring includes electrocardiography. capnometry. pulse oximetry. and direct blood pressure measurement. Systolic blood pressure is initially 100 mm Hg and diastolic pressure is 55 mm Hg. However, as surgery progresses, the systolic pressure drops to 70 mm Hg and diastolic pressure to 30 mm Hg; blood pressure continues to fall. A large pool ofbloody fluid on the floor under the surgery table is rapidly increasing in size. 191. What is the mean arterial blood pressure initially?
a. 70mmHg b.78mmHg c.85mmHg d. it cannot be determined from the information given e. lOOmmHg
a. procaine and mepivacaine b. lidocaine and bupivacaine
Correct answers are on pages 25-33.
www.vet4arab.co.cc
SECTION 1
24
192. At what mean arterial blood pressure would you begin to suspect inadequate perfusion of vital organs?
a. BOmmHg b.70mmHg c.60mmHg d. SOmmHg e. The mean arterial pressure does not indicate this problem. 193. What is the mean arterial blood pressure after the decrease in blood pressure is observed?
a. SOmmHg b.66mmHg c.70mmHg d.43mmHg e. It cannot be determined from the information given. 194. What is the estimated blood volume of this dog?
a. b. c. d. e.
2400 mI 3200ml 2BOO mI 1600ml 2000 mI
195. Intraoperatively, blood should be administered when the packed cell volume falls below:
a.4S% b. 40% c.3S% d.30% e.2S%
a. packed cell volume 30%, total plasma protein concentration 6.S g/dl b. packed cell volume 20%, total plasma protein concentration 7.0 g/ dl c. packed cell volume 40%, total plasma protein concentration 2.S g/dl d. packed cell volume 3S%, total plasma protein concentration 4.S g/dl e. packed cell volume 4S%, total plasma protein concentration 7.0 g/ dl 198. A 45-kg dog is seen after acute loss ofblood, and you decide to administer intravenous fluids as . treatment. The dog's packed cell volume is 35%, and total plasma protein concentration is 4.2 g/dl. Which of the following is most appropriate as the initial fluid therapy to prevent vascular colfapse?
a. 2BOO mI of 0.9% saline solution infused IV over 4 hours b. 4S0 mI of lactated Ringer's solution injected SC at multiple sites c. 1BO mI of 7% saline solution infused IV over 10 minutes d. 2S0 mI of plasma infused IV over 30 minutes e. 4S0 mI of whole blood infused IV over 1 hour 199. Midazolam is classified in the same category as:
a. b. c. d. e.
196. Approximately how many milliliters of red blood cells have been lost when this critical packed cell volume is reached?
a. 160 mI b.320ml c. 4BO mI d.640ml e. BOO mI
197. In a patient with which of the following blood values would it be appropriate to administer only a colloidal solution (e.g., plasma)?
diazepam atipamezole mepivacaine neostigmine propofol
200. Medetomidine is classified as:
a. b. c. d. e.
a hypnotic an amide type of local anesthetic a tranquilizer an "z agonist an opioid antagonist
© 1998 Mosby-Year Book, Inc. Photocopying is prohibited by law.
Anesthesiology
25
Answers 1. d Premedication agents, such as atropine, are used to minimize undesirable reflex autonomic activity. 2. d Atropine acts by competitive antagonism of acetylcholine at muscarinic receptors. This competition can be overcome by increasing the concentration of acetylcholine at receptor sites (as seen following administration of anticholinesterases). 3. a Atropine stimulates the vagus center in the brain, which can slow the heart rate initially in response to IV administration of small doses. 4. b Glycopyrrolate causes less of an increase in heart rate than atropine. S. e A 10% solution = 10 g/100 ml = 10,000 mgllOO mI = 100 mg/ml. 6. b Phenothiazine tranquilizers are sympatholytic (cause vasodilation). 7. a Acepromazine is one of the most commonly used phenothiazines. B. c Acepromazine can cause persistent paraphimosis or priapism in adult intact male horses. Working stallions are at higher risk for these problems. 9. c Guaifenesin causes muscle relaxation by its action on internuncial neurons in the spinal cord. It has no recognized analgesic properties and so should not be used as a sole agent for restraint for surgery. 10. a Diazepam is a centrally acting muscle relaxant. 11. d Overdosage of guaifenesin is associated with skeletal muscle rigidity and often cardiac arrest preceding respiratory arrest by several minutes. The therapeutic dosage is 100 mg/kg. 12. c 3S0 mI x SO mg/mI = 17,SOO mg/2S0 kg = 70 mg/kg. 13. e Atracurium blocks postsynaptic acetylcholine receptors at the neuromuscular junction and so causes paralysis of the intercostal muscles and diaphragm. 14. b Atracurium's molecular stability depends on an alkaline pH and low temperature. This method of limiting the drug's action is known as Hoffman elimination. IS. c Succinylcholine is a depolarizing neuromuscular blocking agent that initially causes rigidity, tachycardia, and hypertension. It has a short duration of action because it is metabolized by plasma cholinesterase.
Atracurium is a non depolarizing blocker and causes relaxation alone. Some muscle groups are more susceptible (e.g., extraocular and limb muscles) than others (e.g., facial muscles, diaphragm), so different doses are required to achieve equivalent relaxation. 16. a Neostigmine inhibits acetylcholine esterase, causing increased levels of acetylcholine at such receptors. This increases preganglionic muscarinic activity, slowing the heart rate. Atropine (0.04 mg/kg IV) should be given immediately before neostigmine. 17. d These drugs are classified as " z-adrenergic agonists. lB. b "z.agonists cause second -degree ~trioventricular block and bradycardia, reSUlting III decreased cardiac output. Hyperglycemia, increased urine output, and increased intrauterine pressure also occur. Respiratory function is depressed because of sedation. 19. c Intravenous atropine rapidly antagonizes the xylazine-induced bradycardia. An " z-adrenergic antagonist, such as yohimbine, would only partially antagonize the bradycardia and decrease the sedative-analgesic (desirable) effects of xylazine. 20. c Pure opioid agonists exert their effects via agonism of fJ.-opiate receptors. 21. d Assuming the potency of morphine (mg/kg basis) is 1, the potency offentanyl is 100 x, oxymorphone 10 x , butorphanol 2 x , and meperidine O.S X to O.2S x. 22. b Naloxone (Narcan), a pure opioid antagonist, has no narcotic/ agonistic effects of its own. 23. c Opioid fJ.-agonists generally cause respiratory depression, especially after intravenous or repeated intramuscular administration. 24. b Opiates alter the perception of pain by the central nervous system. Human patients may still be aware of the pain without being in pain. 2S. b In dogs either oxymorphone or fentanyl is commonly given to allow intubation. The drug is usually administered in combination with diazepam. The induction is slow (2 to S minutes) and causes respiratory depression. Therefore it should be preceded by S minutes of preoxygenation via face mask. 26. d Apart from bradycardia, there is minimal cardiac depression from intravenous oxymorphone and fentanyl.
26
SECTION 1
27. a Nalorphine and naloxone are opioid antagonists and butorphanol is a partial antagonist; therefore, all would combat the bradycardia. Atropine would also combat bradycardia without antagonizing the desirable sedative or analgesic/ properties of fentanyl. 28. c Butorphanol is a mixed (K) agonist and ("') antagonist and so is associated with less respiratory depression. 29. c Opioids can cause central nervous system stimulation in horses, especially if administered intravenously and without other tranquilizers/ sedatives. 30. b Naloxone is a ",-receptor antagonist that can reverse respiratory depression. 31. c In healthy dogs there is a high incidence (up to 40% of cases) of transient ventricular arrhythmias following anesthetic induction with intravenous thiobarbiturates. Thls does not usually cause clinical problems in healthy patients. 32. b Thiobarbiturates owe their short duration of action to almost immediate distribution from the plasma to well-perfused tissue, and then to fat depots. Therefore there is rapid recovery of consciousness after a single anesthetic dose. 33. a Thiobarbiturates can produce prolonged recovery because of saturation of muscle and fat storage depots and their subsequent release and slow metabolism and elimination. Hypoproteinemia, dehydration, acidosis and hypothermia all exacerbate this problem. 34. d Thiobarbiturates decrease intracranial and intraocular pressure and usually cause no change or an increased heart rate. 35. d Apnea is commonly seen following thiopental induction of anesthesia, especially with a hlgh inspired oxygen level. Giving 2 or 3 normal breaths per minute ensures that the patient remains oxygenated and receives some halothane; it also allows the arterial carbon dioxide to rise to a level sufficient to stimulate spontaneous ventilation. If no halothane were administered, this dog would be awake in 10 to 15 minutes. Propofol also causes postinduction apnea. 36. e Thiobarbiturates cause tissue irritation and sloughing after perivascular injection and are associated with prolonged recovery in dehydrated, emaciated, hypoproteinemic animals. They produce relatively good muscle relaxation at anesthetic doses.
www.vet4arab.co.cc
37. d Similar to thiobarbiturates, propofol depresses seizure activity, decreases intraocular pressure, and causes muscle relaxation. 38. c Hypotension can be observed following administration of propofol in healthy patients. This effect is exacerbated by dehydration. Propofol does not directly alter the heart rate or myocardial contractility. 39. a Typically, propofol does not cause excitement in recovery. Patients are quickly ambulatory and not ataxic. In part, recovery is due to rapid metaboHsm. 40. c In the commercially available product, propofol is dissolved in a nonirritant milky oil emulsion and can be given intramuscularly (requires large volume) as well as intravenously. It is stable at room temperature but contains no preservative agents. ' 41. d Ketamine can be administered intra- • muscularly, although this causes pain. 42. a Ketamine stimulates the cardiovascular system, increasing myocardial oxygen consumption, and also increases intraocular and intracranial pressures. 43. d Rapid induction of anesthesia and maintenance of laryngeal function are desirable in brachycephalic dogs. In addition, rapid recovery is desirable. Although not producing the fastest induction and smoothest recovery (thiopental or propofol would be better), ketamine would be acceptable. 44. c Ketamine is a dissociative agent. 45. d Sympathomimetics (e.g., dopamine, dobutamine) are usually administered to stimulate cardiovascular function. 46. b Hypnotic agents include pentobarbital, thiopental, thiamylal, propofol, and chloral hydrate. Tiletamine is the dissociative agent in Telazol (tiletamine-zolazepam). 47. e A neuroleptanalgesic combination usually consists of a sedative/tranquilizer (e.g., acepromazine) and an opioid analgesic (e.g., fentanyl). 48. a Ketamine produces mild visceral analgesia in people. 49. e The kidney is the major excretory organ for almost all sedative and hypnotic drugs. Because they have high lipid solubility and are poorly ionized, they are metabolized by the liver before excretion. Ketamine is excreted without metabolism via the kidney in dogs and cats.
© 1998 Mosby-Year Book, Inc. Photocopying is prohibited by law.
Anesthesiology
50. c The epidural space is located immediately outside the dura mater. 51. b An appropriate dose of epidural local anesthetic produces excellent analgesia and hind limb muscle paralysislrelaxation. 52. a Bupivacaine produces both sensory and motor blockade of 2 to 3 times longer duration than lidocaine or mepivacaine. 53. d Epidural anesthetics should not be administered when there is trauma, bleeding, or infection at the injection site. 54. a Preganglionic sympathetic blockade causes vasodilation in the anesthetized region. 55. e Local anesthetic analgesia persists approximately as long as muscle paralysis/relaxation. The longest duration of effect is from bupivacaine, whlch lasts 4 to 6 hours. 56. b Bupivacaine has the lowest placental transfer of any of the commonly used local anesthetic agents. This is because ofits high protein binding in maternal blood. 57. d If the same dose as required for epidural anesthesia is administered into the cerebrospinal fluid, the anesthetic moves far cranially and could cause respiratory muscle paralysis. 58. a In spinal anesthesia the anesthetic agent is administered into the cerebrospinal fluid, whlch circulates in the subarachnoid space. 59. b Giving the same dose as a spinal injection that was calculated for epidural administration would produce excellent analgesia and incur no more risk of adverse effects (as compared with epidural administration). This is not the situation for local anesthetics administered into the cerebrospinal fluid. 60. e Epidural drugs are also occasionally administered into the cerebrospinal fluid and so should be preservative free. Although the volume of drug administered epidurally is important (hence the different concentrations), this is not the only reason that the solution is packaged in a vial. 61. b Epidural morphine reduces the anesthetic requirement to produce analgesia in the hind limbs, and also to a lesser extent in the forelimbs. 62. a Isoflurane causes respiratory depression in dogs and horses, is relatively less arrhythmogenic, and is excreted virtually unchanged. It is one sixth as potent as methoxyflurane and two thirds as potent as halothane.
27
63. d The anesthetic requirement is reduced by increasing age, hypothermia, concurrent use of other central nervous system depressants and pregnancy. 64. d MAC values for these anesthetics in dogs are 0.9 vol % for halothane, 215 vol % for nitrous oxide, and 1.4 vol % for isoflurane, versus 0.23 vol % for methoxyflurane. Propofol is not an inhalation agent. 65. c Mask inductions are slow (undesirable if rapid airway control is desirable) , with speed of induction dependent on drug solubility (insoluble agents produce faster induction) and ventilation (hyperventilation produces faster induction). Mask induction can be expensive, especially with isoflurane (about $5 for induction in a dog) . 66. d Highly soluble agents (e.g., methoxyflurane) have slow equilibration times and therefore slow induction and recovery times. 67. c About 50% of an inhaled dose of methoxyflurane ' and 25% of a dose of halothane are metabolized. Other agents have minimal metaboHsm. 68. d In decreasing order of the likelihood of arrhythmogenesis: halothane> methoxyflurane > isoflurane, desflurane, and nitrous oxide. 69. c Dose-dependent ventilatory and cardiovascular depression occurs. Respiratory depression increases arterial carbon dioxide tension. 70. c The MAC of nitrous oxide in dogs is approximately 215 vol %, so under normal conditions (1 atmosphere of pressure), it cannot alone produce anesthesia. 71. b Nitrous oxide is the most insoluble of the commonly used inhalation agents. 72. d 1.3 to 1.5 times the MAC (0.23 vol %) produces a surgical plane of anesthesia. 73. c Fluoride ions produced by metabolism of methoxyflurane are nephrotoxic. 74. c When nitrous oxide is given with halothane, the peak blood concentration of halothane is attained more rapidly than if halothane were used alone. Concurrent use of nitrous oxide causes halothane to be taken up at an accelerated rate, known as the second-gas effect. Answer b describes diffusion hypoxia.
28
SECTION 1
75. c The greater an agent's alveolar concentration, the more rapid is its absorption. Uptake also depends on blood solubility. An anesthetic that is very soluble is rapidly taken up by the blood. Cardiac output determines the blood flow through the lungs and therefore the quantity absorbed. The correct ratio of inspired air to lung perfusion (ventilation/perfusion ratio) ensures optimal anesthetic uptake. 76. c The ideal agent, producing rapid induction and recovery, would have low blood solubility and low lipid solubility, rapidly achieving equilibrium (partial pressure) among gas, blood and tissue phases/compartments. Solubility is an index of the capacity of the blood or tissue to retain anesthetic. 77. b A small amount of vapor entering tissue in which the agent is not soluble causes a high vapor tension. Therefore during anesthetic recovery, arterial and brain concentrations drop to negligible levels within minutes and the patient awakens rapidly. 78. b The duration of anesthesia determines the saturation level of body tissues. Anesthetics that are highly soluble in fatty tissues take longer to be expelled from the tissues. If pulmonary blood flow or ventilation is inadequate, inhalation agents cannot be cleared through the lungs and recovery is delayed. Hthe agent is highly soluble in the blood, it is not removed quickly. An example is methoxyflurane, which is highly soluble in blood and has a long recovery period. 79. d Up to 20% of the inhaled dose of halothane is metabolized. Isoflurane causes more respiratory depression in dogs and horses. Methoxyflurane and isoflurane are arrhythmogenic, although significantly less so than halothane. Halothane is by far the cheapest commonly used inhalation agent. 80. a Painful stimulation of lightly anesthetized patients causes catecholamine release, which can lead to ventricular arrhythmias in halothane-anesthetized patients. Increasing the anesthetic depth can eliminate these arrhythmias. 81. b Halothane, 5 ml/500 ml O2 (per minute) =1% Vaporizer setting; 5 ml/120 ml O2 (per minute) = 4.2% Vaporizer setting. Each setting would result in a similar anesthetic depth. This illustrates the importance of fresh gas flow in determining anesthetic delivery to the circuit.
www.vet4arab.co.cc
82. d With in-circuit vaporizers, output depends on ventilation (increases with increasing ventilation). In-circuit vaporizers are of simple "flow over liquid" design and generally use less anesthetic than out -of- circuit (precision) vaporizers. The vapor pressure of halothane at 20· C is 243 mrn Hg. At sea level (760 mni Hg), 243/760 = 32%, which means a 32% concentration of halothane can be achieved. There is a similar situation for isoflurane. Out -of-circuit precision vaporizers only deliver a 5% maximum for these agents. 83. d Uptake of anesthetic is improved with increased ventilation. The "centralized" circulation of neonates ensures rapid delivery of anesthetic to the brain. 84. d Halothane markedly lowers the arrhythmogenic dose of epinephrine in dogs. 85. e Nitrous oxide is a liquid when stored' under pressure at room temperature. Halotl'lane is a vapor, not a gas, and therefore is not stored under pressure. Gases that can be liquefied at room temperature are purchased by weight. Pressure is an accurate index of quantity for nonliquefiable compressed gases. 86. c The pressure in the cylinder begins to drop when all the liquid nitrous oxide has been vaporized and the gas loses pressure. Cylinders must be watched thereafter for rapid emptying. Another cause of decreased pressure is chilling of the cylinder. 87. a All except oxygen are liquefied at room temperature under pressure. As such, they are safely handled as liquids in cylinders. 88. c Gas cylinders for various agents are identified by a unique color, thread size, and thread diameter. E cylinders also have a pin index combination. 89. a The emergency oxygen flush valve bypasses the vaporizer to deliver only oxygen to the circuit. It should be used to fill the circuit as part of the preanesthetic "pressure check" before all anesthetic procedures. 90. a Increased ventilation increases the anesthetic concentration within the circuit. 91. e With in-circuit vaporizers, increasing fresh gas flow "washes" anesthetic out through the pop-off valve, decreasing the concentration within the circuit. 92. e Both one-way valves must be patent to ensure unidirectional gas flow.
© 1998 Mosby- Year Book, Inc. Photocopying is prohibited by law.
Anesthesiology
93. c The oxygen flush valve is located between the ~ource of compressed oxygen and the fresh gas inlet of the circuit. 94. c Without a patent inspiratory valve, exhaled gas would move into the inspiratory side of the rebreathing hose, so the patient would inspire gas containing carbon dioxide. 95. b If the one-way valves are patent, there is unidirectional gas flow from the Ypiece connector. 96. d Soda lime granules become harder when they are exhausted. 97. d Hypercapnia results in sympathetic nervous system stimulation. 98. b Occlusion of the tubing from the pop-off valve to the scavenger would cause pressure in the breathing circuit to increase. 99. d To check the circuit for leaks, it should be pressurized to 20 to 30 cm H20, and the manometer should be observed for 1 minute for pressure change. A pressure decrease indicates a leak. 100. b If the pop-off valve were closed, pressure would increase. It is possible to close the popoff valve in a low-flow system where the oxygen flow equals the patient's oxygen consumption (a closed circuit). 10 1. e Theoretically, the volume of the soda lime container should be at least twice the patient's tidal volume. 102. b The reservoir bag is the compliant (collapsible) part of the circuit and usually should have a volume of about 5 times the patient's tidal volume (approximately the functional residual capacity). 103. a These circuits require high fresh gas flows and so are not economical for larger patients. 104. b B?th are examples ofnonrebreathing circuits, so hIgh fresh gas flow is necessary to prevent rebreathing of expired carbon dioxide. 105. c Fresh gas flow rates of approximately 200 ml/kg/min are necessary to prevent rebreathing of carbon dioxide. 106. d With nonrebreathing circuits, there is no drop in anesthetic concentration from the vaporizer to the patient. 107. e For semiclosed circuits, oxygen flow is usually 30 ml/kg/min. However, for closed or low-flow circuits, the fresh oxygen flow can be as low as 4 to 6 ml/kg/min for dogs. 108. d This is approximately 4 to 6 ml/kg/min in dogs.
29
109. a For closed-circuit anesthesia the system must incorporate a soda lime carbon dioxide absorber. 1l0. c The other answers are all advantages of closed-circuitllow-flow systems. The closed system permits inhalation of warm gases, maintenance of a constant anesthetic level, economical use of volatile agents, and maintenance of high humidity of inspired gases. 111. d An endotracheal tube extending well beyond the animal's nose increases dead space. 112. e With this maneuver, stomach inflation/ deflation can be mistaken for chest excursion. 113. a Early in anesthesia, the patient removes most of the anesthetic from the inspired gas and exhales gas with a low anesthetic concentration. This gas goes back around the circuit and dilutes the concentration of gas coming from the vaporizer. 114. d Tissue oxygen saturation (Sao2, measured with pulse oximetry) will not change perceptibly unless there is a marked change in tissue perfusion or hemoglobin saturation with oxygen. Neither will change enough within the range of clinical anesthetic levels to change Sa0 2, especially if the animal is breathing a high inspired concentration of oxygen. 115. a A corneal reflex can persist even at deep levels of anesthesia. Loss of the corneal reflex is a sign of very deep anesthesia. Repeated testing of the corneal reflex can damage the cornea. 116. c The esophageal stethoscope should be positioned in the esophagus at the level where heart and lung sounds are best heard. These sounds allow evaluation of heart and respiratory rates, neither of which alone is a useful guide to anesthetic depth. 117. b The bradycardia is most likely the result of excessive vagal tone and could be treated with intravenous atropine. It should also respond to cessation of the noxious stimulus (i.e., release of traction on the ovary). 118. c "'2-adrenergic agonists cause marked bradycardia. In dogs and cats, they should be used after premedication with an anticholinergic drug, such as atropine. 119. b A high blood carbon dioxide level causes cardiovascular stimulation, mediated via the sympathetic nervous system.
30
SECTION 1
120. d Assessment of pulse quality does not enable you to estimate systolic or diastolic pressure. 121. d Arterial pressure =Cardiac output x Peripheral resistance. Cardiac output = Heart rate X Stroke volume. 122. a The response time of pulse oximeters is currently not fast enough to enable them to be used with a pneumatic cuff to detect blood pressure. 123. e Currently available indirect methods all have inaccuracies associated with cuff size, limb size, the amount of hair covering the site, and the animal's position. Some of these effects can be standardized for more repeatable measurements. In general, indirect methods most reliably determine systolic arterial pressure. 124. cAn overly wide cuff yields spuriously low readings. An overly narrow cuff and a loose cuff yield spuriously high readings. 125. e As the pressure wave travels distally, impedance causes the systolic pressure to increase, although mean arterial pressure tends to decrease. 126. a Hydrostatic force (gravity) causes a false elevation in pressure, proportional to the height difference (1.36 mm Hg/cm difference). 127. b Mean arterial pressure is the mean perfusion pressure (not blood flow) . It is the area under the pressure waveform. About one third of the cardiac cycle is systole, two thirds is diastole. Therefore, to calculate mean arterial pressure, systolic and diastolic pressure must be weighted. 128. e Hypotension is usually defined as a systolic pressure below 80 to 90 mrn Hg, which would generally result in a mean arterial pressure below 60 to 65 mrn Hg. 129. d Doxapram increases the metabolic rate of the brain and may stimulate ventilation. It does not directly improve cardiovascular function unless its administration has caused or follows hypoxemia. 130. e Cardiac output = Heart rate X Stroke volume. 131. d Phenylephrine and methoxamine are regarded as pure alpha -adrenergic stimulators. Isoproterenol has powerful positive inotropic and chronotropic myocardial effects, causing increased cardiac output. Peripheral vascular dilation and reduced resistance are other prominent effects of this beta-adrenergic stimulant. Ephedrine causes central nervous system-mediated release of norepinephrine; therefore, its effects are similar to those caused
www.vet4arab.co.cc
by norepinephrine administration, including increased stroke volume, presumably from increased contractility. 132. a At a dosage of 1 to 3 f.l.g/kg/minute, dopamine increases renal output by increasing glomerular filtration via dilation of the afferent arterioles. 133. a Phenylephrine may be given in reasonable doses without fear of ventricular irritability. However, epinephrine is likely to cause premature ventricular contractions. 134. c We assume that a hypotensive patient has poor tissue perfusion (not always a true assumption) . The objective of therapy is to improve tissue perfusion with oxygenated blood. 135. c Anatomic dead space comprises the area of the respiratory tract not involved in gas exchange, including the oronasal cavity and conducting airways. In healthy unanesthetized animals, it is equal to about one third of the tidal volume but increases during anesthesia, particularly as a result of atelectasis. 136. e The minute volume depends on both tidal volume and respiratory rate (Vt x Respiratory rate). 137. b An average range is 10 to 15 ml/kg/breath. 138. c Minute ventilation and arterial carbon dioxide levels are equivalent measures of the effectiveness of ventilation. 139. e Increased arterial carbon dioxide levels increase the depth and usually the rate of ventilation; increase heart rate, cardiac output, and blood pressure; and cause injection of the mucous membranes. 140. d Compliance is a measure of static pressurevolume relationships (Llcm) , whereas resistance is a measure of dynamic pressure flows (Llcm H,O/sec). 141. b Carbon dioxide causes cardiac epinephrine to be released from the sympathetic nerve endings, resulting in increased cardiac output. 142. d In general, anesthetic drugs cause dosedependent respiratory depression. 143. b Paco, is used to assess ventilation, whereas Pao, is used to assess oxygenation. 144. e CO, + H,O ... H,C0 3 ... H+ + HC03 • Large changes in blood bicarbonate levels produce only small changes in blood pH. Changes in hydrogen ion concentration have a direct effect on blood pH.
© 1998 Mosby-Year Book, Inc. Photocopying is prohibited by law.
Anesthesiology
31
145. c Oxygen delivery is related to cardiac output, of this, the anesthetic effect produced by NO, hemoglobin concentration, and the percent has a relatively rapid onset of action. saturation with oxygen. 155. b A canister containing activated charcoal will 146. b Shifting of the oxyhemoglobin curve to the left absorb halogenated anesthetic agents. increases the affinity of oxygen for hemoglobin, However, canister use must be monitored to reducing tissue oxygen availability. As blood pH prevent overuse (exhaustion of the activated decreases, the oxygen dissociation curve shifts to carbon), resulting in reduced scavenging and the right. Carbon dioxide and lactic acid pollution. Recommended governmental levels liberated during metabolism cause this of exposure to the halogenated hydrocarbons is phenomenon (the Bohr effect). increasing body no greater than 2 ppm for volatile agents and 25 temperature above normal, an increased organic ppm for nitrous oxide. Nitrous oxide is not phosphate level (2,3-diphosphoglycerate and absorbed by activated charcoal and must be adenosine triphosphate), and an acidic pH all vented to the outside. cause a shift to the right. 156. c Propofol is a sedative-hypnotic and is 147. e When the Pao, is above 100 mm Hg, all the unrelated to other anesthetic agents. Because hemoglobin (which carries over 97% of the propofol has minimal analgesic properties, oxygen) is saturated. analgesics should be administered concurrently 148. c Reducing the Paco, by 5 mm Hg would only for painful procedures. raise the Pao, by about 5 mm Hg. Answers b, d, 157. a Slow administration ofpropofol requires a and e would raise cardiac output, which would lesser amount of drug necessary to induce improve ventilation/perfusion in the lung. general anesthesia and reduces the degree of 149. c This dog is likely cyanotic and, if not hypotension and apnea that commonly occur anesthetized, may be unconscious because of on induction. Opiate premedication tends to hypoxemia. increase the likelihood of apnea on induction 150. e Cerebral blood flow and pressure are with propofol, but is not of consequence as long as ventilation is possible. enhanced by increasing Paco, or decreased by decreasing Paco,. Brain volume increases as 158. d The duration of analgesia produced by more blood flows into the closed cranial cavity, morphine administered epidurally is much which also increases cerebrospinal fluid longer than from other routes of pressure. Carbon dioxide is an anesthetic at administration. higher concentrations (e.g., 30%), although it 159. d Analgesia produced by morphine becomes causes marked cardiovascular responses. apparent approximately 1 hour after epidural 151. a These are the anatomic landmarks for administration. The delay in onset of effect is identifying the epidural space in dogs. Answer e related to its relatively lipophobic nature. It is states the landmarks to identify the epidural slowly absorbed into the spinal cord. space in horses. 160. d The blood/gas partition coefficient indicates 152. a The dose oflocal anesthetic should be that desflurane is the least soluble in blood of reduced by approximately half (1 ml reduced to those listed. This solubility reflects the time 0.5 ml per 4.5 kg) to prevent a higher level of necessary for the drug to exert its effect. The anesthetic block than is desired. By decreasing less soluble the drug, the more rapid the onset the dose, the incidence of "total spinal of effect. The order of onset of effect for this anesthesia" is reduced, decreasing the possibility group of drugs would be as follows: desflurane of respiratory and cardiovascular collapse. > sevoflurane > isoflurane > halothane> methoxyflurane. 153. b There is no possibility of "total spinal anesthesia" when an opioid is administered 161. e The lower the MAC, the more potent the into the subarachnoid space. There are drug. MAC reflects the potency of inhalant currently no documented problems from anesthetic agents. administration of the entire dose into the 162. c Methoxyflurane may be administered using a subarachnoid space in dogs. nonprecision vaporizer, such as the Ohio #8 154. d NO, is a relatively insoluble agent, with a vaporizer. blood:gas partition coefficient of 0.47. Because
32
SECTION 1
163. d The copper kettle vaporizer must be manually adjusted to compensate for changes in temperature and fresh gas flow to deliver the desired amount of anesthetic vapor. These units are still available for use in veterinary hospitals. The Ohio #8 is a nonprecision vaporizer and has no means to compensate for temperature and changes in fresh gas flow. The Vapor (North American Drager) is a precision vaporizer that is flow compensated, but manual adjustments must be made to correct for temperature changes. The newer precision vaporizers are both temperature and flow compensated. Examples of these are the "Tec" and "Matic" types of vaporizers, which automatically compensate for changes in temperature and fresh gas flows within flow ranges normally used in veterinary anesthesia. 164. e The appropriate volume per breath can be determined by simple observation of rise of the patient's chest wall when squeezing the rebreathing bag to inflate the lungs. If a pressure manometer is located in the airway circuit, a peak airway pressure approximating 15 to 20 cm of water pressure indicates adequate lung expansion. 165. c Wright's respirometer is used to measure tidal and minute volumes in small animals. It is commonly inserted into the breathing circuit of the anesthetic machine for these measurements. 166. d The pulse oximeter measures hemoglobin's saturation with oxygen. With 90% saturation (P90), the partial pressure of oxygen is approximately 60 rnrn Hg in dogs. 167. b Butorphanol is an opioid agonist; yohimbine is an ",, -antagonist. Yohimbine does not antagonize the effects ofbutorphanol. 168. c Butorphanol has antagonistic activity at the fL-receptor and has agonistic activity at K- and 0-- receptors. When butorphanol is given to an animal that has received morphine (a fLagonist), morphine's effects at this receptor are antagonized. However, the agonistic action of butorphanol at the K-receptor produces analgesia; therefore butorphanol can be administered to partially antagonize morphine's effects. 169. d Swine are the least susceptible of all species to the effects of xylazine. It is usually used in combination with other anesthetic agents to produce the desired effect in pigs. For sedation, pigs require 20 to 30 times the low dosage used in ruminants (0.1 mglkg 1M).
www.vet4arab.co.cc
170. e Cattle are most susceptible to the effects of xylazine. Sheep and goats are also very sensitive to xylazine. 171. e This is calculated using the equation [(% oxygen inspired) x (Barometric pressure Water vapor pressure)] - Respiratory quotient. 20.9% x (760 - 47) - 40/0.8 =99 mm Hg. 172. c This is calculated using the equation [(% oxygen inspired) x (Barometric pressure Water vapor pressure)] - Respiratory quotient. 100% x (760 - 47) - 40/0.8 = 663 mm Hg. 173. c Hemoglobin content is approximately one third the packed cell volume. 174. b A packed cell volume of21% would indicate a hemoglobin content of approximately 7 gl dl, which is the lowest acceptable hemoglobin content necessary for adequate oxygen content. 175. e Intermittent positive-pressure ventilation may decrease cardiac output. All the other answers are potential treatments. Usually treatment progresses as follows if the above measures have not corrected the situation: decrease the percent of inhalation anesthetic administered, administer a fluid bolus, administer ephedrine, and, last, administer a more potent inotrope, such as dobutamine. 176. d Succinylcholine interacts with the nicotinic receptor, just as acetylcholine does, and stimulates muscle depolarization. Because succinylcholine is not rapidly metabolized, the duration of receptor occupation is longer than acetylcholine and muscle stimulation (fasciculation) is followed by flaccidity. 177. d SUCcinylcholine is hydrolyzed by cholinesterases. It must be metabolized to end its effects. 178. a A full E cylinder contains approximately 700 L, with a service pressure of approximately 2000 to 2200 psi. The pressure is proportional to the gas volume (L) in the cylinder. 179. c The pin index safety system is used to prevent inadvertent interchange of nitrous oxide and oxygen on the hanger yoke of the anesthetic machine. It consists of two pins and a nipple, with spacing specific for each anesthetic gas. 180. d Tranquilizers administered as preanesthetics help to decrease the amount of anesthetic induction agent necessary. 181. a It is only administered intravenously to induce anesthesia.
© 1998 Mosby-Year Book, Inc. Photocopying is prohibited by law.
Anesthesiology
182. e Hypercapnia may increase intracranial pressure by causing vasodilation of cerebral blood vessels, resulting in increased blood flow and leading to possible brain herniation if a mass is present. Inappropriately managed anesthesia could result in this. Also see answer 185. 183. d Because the dog is very depressed, it is probably best not to administer any of the premedicants listed. Oxyrnorphone is a potent oplOld and depresses respiration; atracurium is a neuromuscular relaxant and should not be given as a premedicant (it is administered to anesthetized patients); ketarnine increases cerebral blood flow and could further increase intracranial pressure and may induce seizure activity; acepromazine can induce seizures. 184. d Isoflurane in oxygen is the most appropriate choice in this case. Also see answer 185. 185. a Intermittent positive-pressure ventilation is used to prevent accumulation of carbon dioxide which could increase intracranial pressure. ' When controlled ventilation and isoflurane are used in cases such as this, an increase in cerebral blood flow, resulting in increased intracranial pressure, is minimized. If halothane were administered instead, even with intermittent positive-pressure ventilation, cerebral blood flow is less well controlled and there is potential for increased intracranial pressure. 186. e Capnometry allows measurement of end tidal carbon dioxide. This estimates the arterial carbon dioxide partial pressure. End tidal carbon dioxide pressure is approximately 5 mm of Hg less than the arterial value. End tidal carbon dioxide pressure should be maintained in the range of 25 to 30 mm Hg. 187. c Local anesthetics produce their effects through interaction with the sodium channel of cells. They are thought to prevent action potential generation by obstructing the sodium c~annel from inside the membrane or by dlsruptmg electrical activity by lodging within the lipid membrane of the axon. 188. b Lidocaine, mepivacaine, and bupivacaine are all amide types of local anesthetics. Procaine and chloroprocaine are ester types of local anesthetics.
33
189. e Bupivacaine has an intermediate onset of action (20 to 30 minutes) and lasts approximately 4 to 6 hours. 190. a Pr?caine produces no effect following topical appitcatlOn. 191. a Mean arterial pressure is calculated using the equation [(Systolic blood pressure - Diastolic blood pressure)/3] + Diastolic blood pressure. [(100 - 551 3) + 55] = 70 mm Hg. 192. c A mean arterial pressure less than 60 mm of Hg of pressure indicates the potential for inadequate perfusion of the vital organs, such as the heart, liver, and kidneys. 193. d Using the equation cited in answer 191, [(70 - 30) 13] + 30 = 43 mm Hg. 194. b The blood volume of dogs is approximately 7.5:ro to 9% of body weight (in kilograms). A safe estimate for general use is 8%. To estimate blood volume, multiply the body weight (in kilograms) by 80 ml/kg. 40 x 80 =3200 mi. 195. e A packed cell volume of 25% indicates a hemog!obin content of approximately 8 gl dl. At thiS pomt, more hemoglobin (in the form ofred blood cells in whole blood) is needed. 196. d The dog initially has 1440 mI of red blood cells (Blood volume x Packed cell volume = Red blood cells [mill. When the packed cell volume falls to 25%, there is 800 mI of red blood cells. Therefore the dog has lost 640 mI of red blood cells (1440 - 800 = 640). 197. c The total plasma protein concentration is low and would result in hypovolemia because of inadequate protein to maintain intravascular osmotic pressure and retain fluids in the vascular compartment. 198. c HypertOnic (7%) saline can be rapidly infused N and should be followed by slower infusion of isotonic fluids (e.g., 0.9% saline). Hypertonic salme IS generally infused over approximately 10 minutes. 199. a Midazolam is a water-soluble benzodiazepine in the same class as diazepam. 200. d M~detomidine is an ",,-agonist approved for vetennary use in 19?6. It is the most potent "', agent currently available.
34
SECTI ON 1
www.vet4arab.co.cc
SECTION
2
NOTES ,
,
Clinical Pathology C.B. Grindem, B.T. Mitzner, R.E. Raskin
Recommended Reading Cowell RL, 'lYler RD: Cytology and hematology of the horse, St Louis, 1992, Mosby. Cowell RL, 'lYler RD: Diagnostic cytology of the dog and cat, ed 2, St Louis, 1997, Mosby. Duncan JR et al: Veterinary laboratory medicine, ed 3, Ames, Iowa, 1994, Iowa State University Press. Eades SC, Bounous Dr: Laboratory profiles of equine diseases, ed 2, St Louis, 1997, Mosby. Jain NC: Essentials of veterinary hematology, Baltimore, 1993, Williams & Wilkins. Meyer DJ et al: Veterinary laboratory medicine: interpretation and diagnosis, ed 2, Philadelphia, 1997, WB Saunders. SodikoffCH: Laboratory profiles of small animal diseases, ed 2, St Louis, 1996, Mosby. Willard MD et al: Small animal clinical diagnosis by laboratory methods, ed 2, Philadelphia, 1994, WB Saunders.
Questions C.B. Grindem 1. The anemia of chronic inflammatory disease is caused by:
c. deficiency of erythropoietin d. deficiency of transferrin e. ineffective use of stored iron
a. chronic blood loss b. deficiency of iron
Questions 2 and 3 Equine patient's sample Red blood cells (RBC) ( x 1()6/!1L) Hemoglobin (Hb) (g/rll) Packed cell volume (PCV) (%) Mean corpuscular volume (MCV) (tI) Mean corpuscular hemoglobin concentration (MCHC) (gl rIl) Total protein (gl rIl)
2.70 5.5 13
B.B
Reference range 6.0-10.43 10.1-16.1 27-43 37-49 35.3-39.3 6.0-B.5
Heinz bodies seen on blood smear
© 1998 Mosby-Year Book, Inc. Photocopying is prohibited by law.
Correct answers are on pages 55-62.
35
www.vet4arab.co.cc
SECTION 2
36
2. How is the anemia in this horse most accurately described?
a. b. c. d. e.
normocytic, hyperchromic normocytic, normochromic macrocytic, hyperchromic macrocytic, hypochromic microcytic, hypochromic
3. What is the most likely cause of the anemia?
a. b. c. d. e.
equine infectious anemia lead poisoning red maple poisoning bracken fern poisoning blister beetle poisoning
4. Which of the following causes extravascular hemolytic anemia? .
a. b. c. d. e.
babesiosis anaplasmosis leptospirosis copper toxicity hypophosphatemia
5. Neutrophils remain in the peripheral circulation for approximately:
a. b. c. d. e.
2 days 14 days 120 days 10 hours 30 minutes
8. If the automated white blood cell (WBC) count in a cat is 30,OOO/II'.L and 200 nucleated RBCsllOO WBCs were observed on the blood smear, the corrected WBC count would be:
a. b. c. d. e.
1000/fJ.L 1500/fJ.L 7500/fJ.L
10,OOO/fJ.L 15,0001 fJ.L
9. An 8-year-old dog has a fever, weight loss, and oculonasal discharge. Vaccinations are current (given 6 months ago). The WBC count is 20,OOO/fJ.L, with 50% lymphocytes that are morphologically normal. The platelet count is decreased, and plasma globulin levels are increased. What is the most likely cause of these findings?
a. b. c. d. e.
ehrlichiosis trypanosomiasis physiologic lymphocytosis acute lymphoblastic leukemia postvaccinationallymphocytosis
10. What is the best test to differentiate iron deficiency of chronic blood loss from relative iron deficiency of chronic inflammatory disease?
6. Neutrophil functional defects have been reported with all of the following except:
a. b. c. d. e.
c. in vivo hemolysis d. in vitro hemolysis e. extreme overanticoagulation with EDTA
estrogen toxicity after vaccination diabetes mellitus selenium deficiency bovine neutrophil CDllb/CDI8 surface marker deficiency
7. Causes of erroneous refractive index-derived total plasma protein values include all the following except:
a. b. c. d. e.
serum iron red cell indexes transferrin saturation bone marrow iron stores total iron-binding capacity
11. A dog has a PCVof 78% and normal blood urea nitrogen (BUN), total plasma protein, and erythropoietin levels. What is the most likely cause of these findings?
a. b. c. d. e.
hypoxia dehydration renal disease splenic contraction polycythemia rubra vera
d. transfusion oftype-DEA 1.1 blood into a dog with type-DEA 7 (tr) blood e. transfusion of the dam's RBCs into a foal
12. A false-negative Coombs' test can occur with any of the following except:
a. b. c. d.
glucocorticoid therapy improper antigenl antibody ratio insufficient quantity of antibody on RBCs testing an unseparated, refrigerated serum sample e. not performing the test at various temperatures
15. What is the best test to confirm failure of passive transfer of maternal immunoglobulins in a foal?
a. b. c. d. e.
13. What is the preferred test to confirm a diagnosis of systemic lupus erythematosus?
a. b. c. d. e.
37
Clinical Pathology
Coombs' test rheumatoid factor test antinuclear antibody test lupus erythematosus cell test direct immunofluorescence test
serum IgG levels serum IgM levels serum gamma glutamyltransferase (GGT) levels serum total protein serum albumin levels
16. Neonatal isoerythrolysis in foals is best diagnosed by:
a. cross matching the foal's RBCs and the mare's serum b. cross matching the mare's RBCs and the foal's serum c. blood typing the mare for Aa and Qa RBC antigens d. crossmatching the stallion's RBCs and the foal's serum e. crossmatching the mare's colostrum against the stallion's RBCs
14. Which patient is most likely to develop an immediate transfusion reaction?
a. transfusion of type-A blood into a cat with type-B blood b. transfusion of type-B blood into a cat with type-A blood c. transfusion oftype-DEA 1.2 blood into a dog with type-DEA 1.1 blood
For Questions 17 through 21, match the most likely cause with the following five canine hemograms. Test
WBCs (per ILL) PCV(%) MCV(fI) MCHC (g/dl) Platelets (x 103 ILL) Reticulocytes (%) Total plasma protein (gl dl)
a
b
c
d
e
2000 10 70 36 50 0 7.0
25,000 28 64 34 600 0 8.5
20,000 15 74 42 150 15 8
12,000 58 70 36 300 0 8.5
20,000 20 74 33 450 5 5.0
17. Acute blood loss anemia persisting for more than 3 days
19. Anemia of chronic inflammatory disease 20. Relative polycythemia
18. Hemolytic anemia 21. Aplasticanemia
a. lipemia b. Heinz bodies
© 1998 Mosby-Year Book, Inc. Photocopying is prohibited by law.
Reference range 5000-18,000 33-56 63-73 34-38 200-500 <1.5 6.0-7.5
Correct answers are on pages 55-62.
www.vet4arab.co.cc
SECTION 2
38
29. Amyloidosis
For Questions 22 through 26, match the most likely cause with the following five canine hemograms.
36. In horses, which of the following is most helpful in distinguishing changes in peritoneal fluid caused by pathologic peritonitis versus those related to recent abdominal surgery?
30. Myoglobinuria Test
WECs (per fl.L) Segmented neutrophils (per fl.L) Band neutrophils (per fl.L) Lymphocytes (per fl.L) Monocytes (per fl.L) Eosinophils (per I'll Platelets ( X 10' I'L) Total plasma protein (gt dl)
b
c
20,000 17,200 0 800 2000 0 250 6.5
14,000 9800 0 2800 700 700 500 6.8
2000 600 0 1000 300 100 50 7.0
22,000 17,160 2200 660 1980 0 450 8.5
23. Physiologic response to chronic stress (glucocorticoid release)
25. Inflammation, with a degenerative left shift
Reference range
4500 1125 1350 2025 0 0 38 8.7
5000-18,000 3000-6000 0-300 1000-5000 150-1350 100-750 200-500 6.0-7.5
32. Diabetes mellitus
27. Which of the following causes a positive reaction for blood with a urine dipstick?
34. In a dog with ethylene glycol toxicity, what is the least likely finding on urinalysis? a. b. c. d. e.
1.030 1.060 2.030 2.060
a
Assay
Color Turbidity Specific gravity pH Glucose Ketones Bilirubin Protein Blood WECsthigh-power field RBCsthigh-power field Other observations
b
c
d
e
Red-yellow Clear
Yellow Slightly turbid
Yellow Turbid
Yellow Turbid
Brown Turbid
1.037 6.5
1.033 7.0
1.017 7.0
1.022 6.5
Negative Negative Negative
Negative Negative
Negative Negative Negative
1.020 5.5 4+ 3+
1+ 3+ 10 Too numerous to count
Triple phosphate crystals
1+ 1+ 1+ 30-40 5-10
Negative
Negative Trace Negative
3-5 1-2
0-3 1-2
4+
Granular casts
Negative Negative Negative 2+ 4+ 0-2 0-2
ether extraction fluid/serum cholesterol ratio fluid/ serum triglyceride ratio Sudan IV staining of the cytologic smear cytomorphology and differential cell count
38. Which species normally has neutrophils in its abdominal fluid? a. b. c. d. e.
transudate septic exudate nonseptic exudate modified transudate hemorrhagic effusion
a. only hematuria b. only intravascular hemolytic anemia For Questions 29 through 33, match the most likely cause with the following five urinalyses.
37. What is the best procedure to confirm a cytologic diagnosis of chylous effusion? a. b. c. d. e.
bilirubin crystals pH5.5 5 WBCs/high-power field specific gravity 1.009 negative ketones
35. What type of effusion is most likely to be observed in a cat with feline infectious peritonitis (PIP)? a. b. c. d. e.
a. 1.015 b. c. d. e.
total plasma protein RBCcount neutrophil morphology percentage of neutrophils total nucleated cell count
33. Iatrogenic hematuria
28. If a urine sample is diluted 1:1 (because the reading is higher than the refractometer scale) and the diluted sample has a specific gravity of 1.030, what is the actual specific gravity of the sample?
26. Inflammation and stress
a. b. c. d. e.
31. Pyelonephritis
c. only intravascular and extravascular hemolytic anemia d. only hematuria and hemoglobinuria e. hematuria, myoglobinuria, and hemoglobinuria
22. Physiologic response to acute stress (epinephrine release)
24. Bone marrow suppression
e
d
a
cats dogs cattle sheep horses
For Questions 39 through 43, match the most likely cause with the findings from the follOwing five peritoneal fluid samples. a
Assay
Color Turbidity WECs (per I'L) Differential count Mononuclear cells Polymorphonuclear cells Blast cells Other cells
Protein (gt ell)
d
c
b
e
Red Cloudy
Yellow Cloudy
Milky white Cloudy
Green Cloudy
Colorless Clear
5600
89,000
8000
1000
1000
30% 70% 0
5% 95% 0
5% 5% 90%
35% 65%
98% 2%
o
o
Extracellular bacteria, ciliate protozoa, plant material
Platelet clumps
4.5
5.0
2.5
1.0
1.0
39. Hypoalbuminemia
42. Contamination of the sample with blood
40. Lymphosarcoma
43. Enterocentesis
41. Peritonitis
© 1998 Mosby-Year Book, Inc. Photocopying is prohibited by law.
39
Clinical Pathology
Correct answers are on pages 55-62.
www.vet4arab.co.cc
SECTION 2
40
44. Which two enzyme assays are best for diagnosing muscle disease? a. alanine aminotransferase (ALT) and lactate dehydrogenase (LDH) b. LDH and aspartate aminotransferase (AST) c. creatine phosphokinase (CK) and AST d. CK and LDH e. CKandALT
45. Which test is most specific for diagnosing exocrine pancreatic insufficiency? a. b. c. d. e.
fecal fat corn oil absorption D-xylose absorption trypsinlike immunoreactivity bentiromide (BT-PABA) absorption
a. b. c. d. e.
uroabdomen chylothorax hypothyroidism hypo aldosteronism whipworm infection
Question 48 Canine patient's samp'"
Reference range
2.8 25 14.1 7.2
2.3-3.1 8-28 9.1-11.7 2.9-5.3
Albumin (gl dl) BUN (mg/dl) Ca (mg/dl) P (mg/dl)
46. Thyroid hormone levels are likely to be decreased in animals in any of the following except: a. b. c. d. e.
47. A decreased serum Na/K ratio is likely to be observed with any of the following disorders except:
young animals lymphocytic thyroiditis phenobarbital administration glucocorticoid administration animals with concurrent illness
48. What is the most likely cause of these findings? a. renal failure b. hypoadrenocorticism c. pseudohyperparathryoidism d . primary hyperparathyroidism e. toxicity from a rodenticide containing cholecalciferol For Questions 49 through 52, match the most likely cause with the findings from the following four blood acid-base profiles.
a
pH Pco, (mmHg) HCO, (mEq / L) Total CO, (mIDollL)
7.3 27 14 15
49. Metabolic alkalosis with respiratory compensation 50. Respiratory alkalosis
b 7.5 18 19 19.5
c
d
7.5 53 37 38.6
7.0 53 13 14.6
Reference range 7.31-7.42 29-42 17-24 14-26
51. Metabolic acidosis and respiratory acidosis 52. Metabolic acidosis with compensatory respiratory alkalosis
Question 53
Patient's samp'"
Reference range
7 0.8 0.2 160 54 269 190
8-28 0.5-1.7 0.0-0.3 1-114 10-109 0-8 19· 120 2.3-3.1 0-5
BUN (mg/dl) Creatinine (mgl dl) Total bilirubin (mgl dl) ALP (lUlL) ALT (lUlL) Bile acid (llIDollL) Ammonia ()lg/dl) Albumin ()lg/dl) Bromsulphalein (BSP) (% retention)
1.1
6
53. What is the most likely cause of these findings? a. shock b. hepatitis c. cholestasis d. hemolytic anemia e. portosysternic shunt 54. Using an ion-selective electrode method for assay, which of the following causes a falsely elevated chloride value? lipemia icterus hemolysis bromide poisoning metabolic acidosis
acidosis hypoproteinemia acute pancreatitis blister beetle toxicity ethylene glycol toxicity
Na
a. 39.5 b. 40.4
c. 50.5 d.56.0 e. 61.5
a. b. c. d. e.
renal failure lactic acidosis hyperadrenocorticism diabetic ketoacidosis ethylene glycol intoxication
For Questions 58 through 62, select the correct answer from the five choices below. a. b. c. d. e.
ammonia alkaline phosphatase (ALP) alanine aminotransferase (ALT) gamma-glutamyltransferase (GGT) aspartate aminotransferase (AST)
59. A cytosolic and mitochondrial leakage enzyme used to evaluate liver damage in domestic animals 60. A membrane-associated enzyme used to detect cholestatic liver disease in small animals 61. A cytosolic leakage enzyme used to evaluate liver damage in small animals
Questions 56 and 57
K
56. What is the anion gap in this dog?
58. A test used to evaluate liver function
55. Hypocalcemia has been associated with all of the following conditions except: a. b. c. d. e.
93m£q/L 11 mEq/ L 411 mOsm / kg nmg/dl 170 mg/dl 10.1 mg/dl
57. What is the most likely cause of these findings?
A dog has the following blood chemistry profile:
© 1998 Mosby-Year Book, Inc. Photocopying is prohibited by law.
CI Total CO, Serum osmolality BUN Glucose Phosphorus
Blood samples from an 8-month-old mixed-breed dog with poor weight gain yield the following laboratory results.
a. b. c. d. e.
41
Clinical Pathology
62. Urinary enzyme used to evaluate renal tubular disease
149 mEq/L 5.5mEq/L
Correct answers are on pages 55-62.
www.vet4arab.co.cc
SECTION 2
42
65. Used to recover larvae ofStrongyloides,
Question 63 Canine patient's sample BUN (m g/dl) Creatinine (mg/dt) Urine specific gravity Total bilimbin (mg/dl)
Totail)lasma protein (g/ d l) PCV(%)
Reference range
39
8-24
I..
1.033 0.5 4.7 30
5.4-7.5 33-56
63. Wilat is the most likely cause of tllese findings? u. dehydrat ion b. chronic renal fail ure c. intravascular hemolysis d. extravascular hemolysis e. gastrointestinal hemorrhage Question 64 Can ine patient's sample Alkaline phosphatase {lUlL) Alkaline p hosphatase-levamisole resistance (%) Alanine aminotransferase (lUl L) Glucose (mg/dl) ACTH stim ula tion Pre-ACfH conisol (..,.g/dl) Post-ACTI-I cortisol (~g/ dl)
Referenre range
67. Used to idelllify trophozoites o/Balantidium, Giardia, trichomonads, and amoebae 68. Used to estimnte 'lUmbers 0/ parasite eggs or cysts per gram o/feces 69. Used to distinguish Eimeriafrom Isospora
70. What is tile most effectiveflolation solution for recovery of pa rasile eggs from feces?
a. b. c. d. e.
salt (sodium chloride) s ugar (sucrose) d istilled water zinc sulfate sodium nitrate
1-1 14
394
10-109
the microfilariae ofDirofilaria/rom those of Dipetalonema?
220
76- 119
a. s ize
<.
b. head s hape c. body shape d. buttonhook tail e. relative numbers
0.5 0.8
73. Recomme,u:lo.tiolls for submission offecal samples for culture incl/lde all ofthe following except Else of
a. b. c. d. e.
swabs fresh feces 5 to 10 9 offeces sterile container air-tight containers
74. Concerning microbial sensitivity to arrtimicrobiaf drugs, whiell statement is most correct?
a. Sensitivity test results correlate well with the clinical response to antimicrobial treatment. b. Drugs that do not inhibit bacterial growth in vitro will not likely be useful in vivo. c. The antimicrobial wilh the largest zone of inhibition is the most suitable drug. d. A Kirby-Bauer zone of inhibition of6 mm (none beyond perimeter of the disk) indicates complete inhibition of bacterial growth. e. Tube dilution is the most commonly used method in veterinary medicine to determine minim um inhibitory concentratio n of antimicrobial agents.
6-17
diabetes mellitus ialTogenic Cushing's disease adrenal-dependent Cushing's disease pituitary-dependent Cushing's disease
Baermann technique McMaster technique cellophane tape preparation sporulation technique staining of a wet mount direct smear with iodine solution
72. What is the preferred test to diagnose heartworm infection i1l dogs receiving macrolide heartworm preventive?
a. b. c. d. e.
filter test Knott's test direct blood smear antigen test antibody test
Ci 1998 Mosby-Year Book, Inc. PllOtocopying is prohibited by law.
75. If aerobic microbiologic swabs canllot be immediately (within several hours) irweulated oruo culture medium, the recommended procedure is to:
a. freeze the swab specimen b. refrigerate the swab specimen c. store the swab s pecimen anaerobicaIJy at room temperature d. inoculate the swab specimen into transport medium and store at room temperature e. inoculate the swab specimen into transport medium and store at refrigerator temperature 76. Concerning skill scrapings fo r diagnosis 0/ ectoparasitism, which stateme"t is least accurate?
a. Select an area near the margin of a lesion. b. Select an unmedicated, nonexcoriated area. c.. Pinch lhe affected skin ben.veen the lhumb and fo refinger and then scrape with an oily scalpel blade until a s mall amount of blood appears. d. Transfer the scraping from the scalpel blade into a drop of saline on the microscope slide. e. Apply a coverslip to the scraping on the microscope slide.
43
77. Which procedure is recommendedfordefi"itive
diag"osis ofdermatomycoses?
a. b. c. d. e.
71. Which criterion is most reliable/or differen tiating
For Questions 65 through 69, select the correct answer from the five choices below. a. b. c. d. e.
66. Used to confirm ?xyuris equi infecrion
1080 94
64. W1wt is tile most likely ca/lse of thesefilldings? a. hepatitis
b. c. d. e.
Aeluroslrongylus abstrusus. Metaslrongylus, and Dictyocaulus from feces
Clinica l Pathology
India ink preparations Wood's light examination culture in Mueller-Hinton medium culture in Sabouraud's dextrose agar potassium hydroxide slide preparations
For Questions 78 through 82, select the cor rect answer from the five choices below. a. Demodex s pp b. Psoroptes ovis c. Sarcoptes scabiei d. Otodectes cy"otis e. Knemidokoptes mutans 78. Causes otitis extema ill camivores 79. Causes ~scaly leg" and depillming scabies in birds
80. Tunnels deeply ifllo the epidermis and causes intense irritation 81. Remains 011 the skill surface or under scabs and
may cause ear mallge 82. Cigar-shaped mite that lives ill the hair follicles, sebaceous glands, and epidermis
For Questions 83 through 87. select the correct answer from the five choices below. a. EDTA b. citrate c. heparin d. oxalate e. fluoride
83. Anticoagularrt ofchoice for blood gas analysis 84. Anticoagulant ofchoice for a routine complete blood count (CBC) 85. Anticoagulam of choice for routine coagulation tests (prothrombin time {PTJ, actillated partial thromboplastin time {AP1TlJ
Correct answers are on pages 55·62.
44
www.vet4arab.co.cc
SECTION 2
86. Anticoagulant that inhibits glycolysis for accurate glucose testing
93. Vitamin K antagonism is most readily diagnosed WiTh which test?
a. prothrombin time b. activated partial thromboplastin time c. PrvKA test (pI;oteins induced by vitamin K absence or antagonism) d. antithrombin III assay e. factor VIII assay
B7. Anticoagulant thatean be used/or serum electrolyte assay For Questions 88 through 92, select the COlTect
answer from the five choices below.
Clinical Pathology
102. What is the most importalll periodic maimenance procedure required by impedancetype hematology analyzers?
a. b. c. d. e.
oiling the vacuum pump changing the vacuum tubing deproteinizing the counting aperture cleaning the diluter probe adjusting the vacuum pressure
a. new methylene blue stain 94. Which group ofclottingfactors is vitamin K dependent?
h. Wright's stain
c. Papanicolaou stain d. Nat! and Herrick's solution e. periodic acid-Schiff stain
a. b. c. d. e.
88. Used for identifying fungal structures on cytologic smears
89. Used/or staining reticulocytes
a. b. c. d. e.
I, V, IX, XII II, V. VIII, X IV; VI, Xl, XII II, VII, VITI , X, antithrombin III II, VII , IX, X, protein C
95. Wllat is the best test to diagnose von Willebrand's disease?
90. Used for routine staining ofcytologic smears
a. b. c. d. e.
91. Used for WBC examination in avian blood samples
92. Used/or routine stainingofblood smears
prothrombin time activated partial thromboplastin time factor VIII assay dilute clot retraction von Willebrand's factor antigen (vWF) assay
For Questions 96 through 100, select the correcl answer from the foUowing five coagulation profiJes, a
,
b
Prothrombin time Activated partial thromboplastin time Fibrin degradation products Platelet numbers
Normal Normal Normal
1
96. Factor Vll deficiency
,
d
Normal Normal Normal Normal
t
Normal Normal
Normal Normal
99. Disseminated intravascular coagulation
97. Factor Vlll deficiency
100. Tmmlllle-mediated thrombocytopenia
98. Factor X deficiency
B.T. Mitzner 101. Specimellsfor hematologic analysis should be collecred in vacuum tubes with a stopper of what color?
c. red d . green e. gray
a. lavender b. blue
103. When using an impedance-type hematology analyzer, the background count is performed on:
a normal blood sample a dilmed control sample deionized water an aliquot of isotonic diluent an aliquot of cleaning solution
104. Hematology quality-control teslS are used to assess the:
a. b. c. d. e.
performance of the ope rator accuracy of the counting method accuracy of the sample dilutions integrity of the reagents performance of the entire system
105. Most errors tllm occur with automated hematology analyzers can be traced back to:
a. b. c. d. e.
improper specimen collection inadequate premixing of the specimen improper dilution of the specimen lack of familiarity with the analyzer improper transposition of results
106. Avian blood smears may colltain all tlte following cell types except:
a. b. c. d. e.
erythrocytes thrombocytes heterophils neutrophils eosinophils
a. prorubricytes b. metamyelocytes
© 1998 Mosby-Year Book, Inc. Phorocopyillg is prohibired by law.
c. rubricytes d. reticulocytes e. erythrocytes 108. What arnfactual change in the complete blood COUTit is most likely to occur if the blood sample is collected from an excited or agitated patient?
a. b. c. d. e.
leukocytosis decreased hematocrit platelet aggregation left shift leukopenia
109. In which species are nucleated erytllrocytes a Ilormalfinding?
a. cats b. pigs c. rabbits d. chickens e. horses 110. Whic/, cell type is kast likely to befound on a peripheral blood smear from a dog with autoimmune hemolytic anemia?
a. reticulocyte b. spherocyte c. target cell d. erythrocyte e. metamyelocyte 111. Til a complete blood count, a left shift refers to:
a. b. c. d. e.
movement of the microscope slide to the left decreased numbers of platelets an abundance of immature WBC forms increased numbers of nucleated RBCs a trend toward macrocytosis
112. An abundanceofeosinophils on a peripheral blood smear is most commonly associated with:
107. The erythrocyte (red cell) series comprises all of the following cell types except:
45
a. b. c. d. e.
neoplastic disease infection trauma allergic conditions stress
Correct answers are on pages 55-62.
www.vet4arab.co.cc
SECTION 2
46 113. How long should microJiemmocrir rubes be cenrrifuged?
a. b. c. d. c.
d. bleeding time e. erythrocyte sedimentation rate
1 minute
Sminutes to minutes 3 minutes Calibrate centrifuge to determine time.
J 14. lNhich stain is not appropriate for rOil cine
staining otblODd smears?
a. h. c. d. e.
Wright's Wright's-Gicmsa Diff-Quik trichrome Giemsa
to the top
one fourth of the way to the top two thirds of the way to the top three quarters of the way to the top to me level s pecified by the collection tube manufacturer
116. TI,e most commoll bleeding disorders seen in companion animals are reInted to: a. b. c. d. e.
decreased clotting factor activity lhrombocytopenia platelet dysfun ction hemophilia A and B hypocalcemia
117. Bleeding time is the best test to detect:
a. b. c. d. e.
a nonseparated blqod sample collected for serum chemistry analysis?
a. b. c. d. e.
increased potassium level increascd aspartate aminotransferase activity increased phosphorus level decreased glucose level decreased cholesterol level
120. Wllich technique is most likely to prevent hemolysis during blood collection?
115. As a minimum standard. blood collection tubes colltaining EDTA should befiUed:
a. b. c. d. c.
119. Wllich abnormality is least likely to befound in
platelet dysfunction anemia platelet deficiency clotting factor deficiency reticulocytosis
a. Use a large-gauge (19-gauge or larger) needle. b. Use a small-gauge (21-gauge or smalle r) needle. c. Soak the skin well with alcohol before venipuncture. d. Shake the specimen vigorously after collection. c. Use a large (I0-ml or larger) collection tube. 121 . Wllich clinicopatliologic abnormality is least
lilcely to befound ill a patient with advanced renal disease?
a. b. c. d. e.
increased BUN level increased serum creatinin e level decreased serum calcium level decreased hematocrit increased serum phosphorus level
122. Wllich of tile following does rwt reflect liver function?
a. b. c. d. c.
serum albumin level serum alanine aminotransferase activity serum alkaline phosphatase activity serum creatinine level serum aspartate aminotransferase activity
123. Serum amylase activity generally is not
increased in animals with: 118. Which of the following is rwt a test afhemostasis?
a. activated clotting time b. prothrombin time c. partial thromboplastin time
e
a. b. c. d. c.
pancreatitis renal disease dehydration pancreatic abscess colitis
1998 Mosby-Year Book, Illc. Photocopying is prohibited by law.
47
Cllnicld Pathology
124. Decreased serum cholinesterase activity often
129. Which method is most reliable for confirmation
accompanies:
ofactive Ehrlichia canis infeerion?
a. b. c. d. e.
a. b. c. d.
renaJ disease trauma organophosphate insecticide toxicity diabetes mellitus pancreatitis
blood smear examination buffy coat examination indirect fluorescent antibody test on serum wet preparation of peripheral blood stained with new methylene blue e. serum titer
125. Wh ich of the following is not used to calculate
anion gap?
a. b. c. d. c.
130. Which method is not IIsedfordetection of heartworm microfilariae?
serum sodium level serum caJcium level serum CO 2 level serum potassium level serum chloride level
a. b. c. d. e.
126. The constituents in most serum chemistry control samples are stabl€ for apprOXimately 7 days after reconstitution. Wlliell constiwem is stable for tile shortest time after reconstitution?
a. b. c. d. e.
glucose aJbumin creatinine carbon dioxide urea n itrogen
127. Which immersion oil is m ost appropriate/or
normal light microscopy?
a. b. c. d. e.
typeA type B mineral oil linseed oil SAE 30 a utomotive oil
131. 11le test of choice for rourine silroeillance ofdogs treated monthly with heartworm preventives is:
a. Knott's technique b. filtration/concentration c. enzyme-linked immunosorbent assay of high sensitivity d. enzyme-linked immunosorbent or immunochromatographic assay of high specificity e. filrration/concentration plus enzyme-linked immunosorbent assay 132. w:llich met/lod is least appropriate for urine collection?
128. In an infected animal, life cycle stages of which
parasite are least likely to be found on a fecal flotation preparation? a. Giardia mti b. Toxocara canis c. 1hchuris vulpis d. Allcylostoma caninum e. Isospora
direct blood s mear buffy coat examination modified Knott's technique filtration/concentration method enzyme-linked immunosorbent assay
a. midstream free catch b. manuaJ expression of the bladder c. aspiration of urine from the cage floor or Iitterbox d. bladder catheterization e. cystocentesis 133. In which type of patient should cystocentesis not be performed?
a. b. c. d.
3-month-old puppy obese beagle female dog in heat adult male cat with disease of the bladder wall e. old dog with a fever
Correct answers are on pages 55-62.
www.vet4arab.co.cc
SECTION 2
48
134. A urine specimen with afruity odor is most likely
to cantail!: a. b. c. d.
metabolized fruit juice bacteria hemolyzed blood
c. I minute at 3000 rpm d. 400 RCF (relative centrifugal force) for 5 minutes e. Time and speed are of no significance. 139. Cystine crystals are most likely to befound in the urine 0/
acetone e. myoglobin 135. WllUl is tile nomlUl pH of canine and feline urine?
a. 7 or greater b. 7 or less c. 8to 10 d. 10 or greater e. highly variable p H, from 4 to 9
136. Wilen present in urine, wlliell substance can mask a positiueglucose reaction 011 urine dipsticks? a. kelOnes b. blood c. penicillin d. ascorbic acid e. creatinine
a. b. c. d. c.
female collies fema le Doberman pinschers female cocker spaniels male dachshunds male labrador retrievers
140. CAlcium oxalate crystals in urine sediment are mosloftell associated with:
a. b. c. d. e.
ethylene glycol (antifreeze) loxiciry gout bacterial cystitis end-stage renal disease organophosphate insecticide poisoning
141. All commerciallyavailable/efine leukemia virus (FeLV) lest kits designed for in-clinic use detect:
137. The urine nitrite test is based 011 tile ability of hacteria co convert nitrate to nitrite and is a crude indicator of urinary tmct infection itl people. Why is it a/limited use in dogs and cats?
a. Dogs and cats are rarely affected by bacterial infections of the urinary tract. b. The acidic p H of dog and cat urine prevents this reaction. c. The bacteria tend to react with Olher proteins in the urine of dogs and cats. d . The diets of most dogs and cats do not contain sufficient nitrate 10 result in a positive reaction. e. Enzymes in the urine of dogs and cats break down nitrate.
a. b. c. d.
FelVantibodies FelV-infected red blood cells FelVantigens Feline oncornavirus-associated cell membrane antigen (FOCMA) antibodies e. FelV-infected lymphoid cells
143. You test a eat's serum for feline leukemia virus usillg a microlVelf-type enzyme-linked immullosorbelH assay kit. Color develops in the negative con trol and pariellt sample wells, as well as in the positive well. What is the most likely cause of these results?
a. failu re to properly time the test b. inadequate washing of wells after addition of the enzyme conjugate c. prolonged storage of the kit at room temperature d. nonspecific cross-reactive antibodies in the eat's serum e. failure to add the enzyme conjugate 144. A 7-year-old outdoor male cat has a historyalld clinical sigm strongly suggestive of heartworm infection. However,filter testsfor heartworm microfilariae arid enzyme-finked immunosorberlt assay are both negative. MlDt is the most likeiy explanarion?
a. The cal probably does not have heartworm infection. b. The cat may have small numbers of adult heartworl11s. c. The test procedures were perform ed incorrectly. d. The tests are designed for use only in dogs. e. The cat is probably infected with feline leukemia virus. 145. You test a 2-year-old clinically normal catfrom a
142. A positiveenzyme-finked immullosorberIC assay for feline leukemia virus (FeLV) may indicate allY of tile following except:
a. b. c. d. e.
The cat may be transiently infected. The cat may be chronically infected. The cat will definitely die from FelV infection. The cat may become a latent carrier. The cat may be contagious for other cats.
138. For best results, urine specimens for microscopic
analysis should be centrifuged for:
49
Cli"ical Pathology
single-cat household for feline leukemia I!ims infection with an enzyme-linked immunosorbent assay kit designed for ill-office use. The test is positive. What is the mosl appropriate advice for the catS owner?
a. Isolate the cat and repeat the test in I or 2 months. b. Euthanize the cat before it develops full blown infection. c. The result was probably inaccurate. d. Isolate the cat but do not bother to retest because the second test will probably be positjve. e. Do not breed this cat.
146. Concerning cats whosesafiva tests are positive for feline leukemia virus, which statemellt is most accurate?
a. They will soon die. b. c. d. e.
They are only tranSiently infected. They have a latent infection. They can transmit the virus to other cats. They are probably not contagious.
147. When imerpretillg the results ofan antimicrobial susceptibility test, a drug that is appropriate for treatment is indicated bya disk:
a. with a large zone of inhibition b. with a small zone of inhibition c. wi th a zone diameter designared as inhibitory, according to standard charts for ant imicrobial susceptibility d. with a zone of inhibition indicating poor ability to diffi.lse through the agar e. that inhibits growth of the greatest numbe r of organisms 148. Preciseamimicrobial susceptibility testing requires tlwt the agar plme containing the antimicrobial disks be inoculated:
a. and incubated immediately after the differential media plates have been inoculated b. with a standard suspension ofa single organism selected from the preincubated blood agar plate c. with a mixed c ulture of all the organisms present on the preincubated blood agar plate d. after p rei ncubation of triple sugar-iron agar slants and broth subcultures e. after preincubation ofSabouraud's dextrose slants and Hektoen broth subcultures 149. For IOrlg-term storage, antimicrobial sensitivity
disks should be held at:
a. room tempe rature b.2"to25"C c. below 2" C d. above 5" C e. any convenient temperature
a. 5 minutes at 10,000 rpm b. 5 minutes at 6000 rpm
C 1998 Mosby-Year Book, Inc. Photocopying i5 prohibited by law.
Correct answers are on pages 55-62.
www.vet4arab.co.cc
SECTI O N 2
50
c. exposure to formalin vapors d. smear that is too thick e. stain or diluent that is too acidic
150. III cytologic specimens stained with
Romanowsky-type sWins (e.g., Diff-Quik,
DipStat, Wright's), Wl l iell aftlle/allowing is most likely to result ill excessive pin k staining?
161. Associated with ethylene glycol poisoning 162. MIen presellf in urine, Bence-Jones protein is
detected by:
a. inadequate washing of stained slide b. delayed fixation
a. b. c. d. e.
R.E. Ras kin 151. A dog is polydipsic. and you suspect renal disease. You perform a urinalysis alldfind the specijicgravity is 1.010. You conclude tllat this
animal's urinc specijicgraviry: a. b. c. d.
is within normal limits for dogs indicates isosth en uria has been affected by consumption of water indicates the patie nt 's kidneys retained their abiliry to dilute urine e. is insufficie nt to determine the concentrating ability of the kidneys without additional tests /52. /n a horse with suspected excreianaf myopathy.
tile red-brown u rinc is positive for occult blood and protein using a dipstick test. Wh ich additio nal piece of inform arion supports the diagllosisofmyoglobinuria and 1Iot Ilemogiobinuria?
a. A d ear supe rnatant occurs when ammonium sulfa te is added to the urine. b. There is an increase in serum sorbitol dehydrogenase activity. c. There is an increase in serum creatine kinase activity. d. Horses do not exhi bit hemoglobinuria. e. The plasma appears pink. 153. Tile urine of horses is normally thick a nd cloudy. This is ca used by the presellce of
a. b. c. d. e.
e pithelial cells and mucus epithelial cells and crystals mucus and crystals leukocytes and normal bacte rial fl ora leukocytes and hyaline casts
154. Proteinuria may befound;1/ animals with:
a. a fever b. diabetes inSipidus c. a portosyste mic shunt
Clin ical PalllOlogy
d. hyperadrenocorticism e. hypothyroid ism ISS. Bilirubill itl tlleuritleofdogs:
a. is abnormal, even whe n present in trace amounts b. is only detected aft er th e dog becomes icteric c. consists primarily of th e unconjugated form of bilirubin d. is increased with mild anemia e. is fou nd with severe cholestatic d isease
spectrophotometry suJfosalicylic acid test urine dipstick assay for protein microscopic examination of urine sediment freezing the urine to detect cryoprecipitares
163. Whicll test is useful in detecting retial tubular dysfunction?
a. b. c. d. e.
BUN assay
urine protein /creatinine ratio creatinine clearance fractional albumin clearance fractio nal sodium excretion
156. WIlen urine is collected from a normal atlimal by cystocelltesis, any of tile following is likely to be found except:
a. b. c. d. e.
sperm le ukocytes (3/high -power fie ld) bacteria hyaline casts (2/ Iow-power field) e pithelial cells (2/ 10w-power fi eld)
For Questions 157 through 161, select the correct answe r from the five choices below.
following except:
a. b. c. d. e.
heartworm disease urinary hemorrhage nephrotoxins causing renal tu bular disease uri nary obstruction in cats intravenous infusio n of dextrose-containing fluids
165. Wldch anioll is specifically measured by the urine reagent strip fo r ketones?
a. o-Iactate b. o-glucuronate
a. triple phosphate b. calcium carbonate c. calcium oxalate d. ammon ium biurate e. bilirubin
167. A dog has severe thrombocytopenia (20,{J(X) p lateletsliJ-L}. Bone marrow evaluation shows megakaryocytes with normal morphology present i" normal to increased numbers. The most appropriate course ofaction is to:
a. submit blood fo r an antimegakaryocyte antibody test b. dete rm ine the activated clotting time c. carefully scan the blood smear for evide nce of e hrlichiosis d. submit blood fo r prothrombin time, activated partial throm boplasin time, and fib rin degradation products assay e. give corticosteroids to control immunem ediated destruction of platelets 168. WI/icl/flo tatioll solution is best to usefor qualitative exam ination offeces?
/ 64. Glucosuria has been associated with all of tile
c. acetone d . acetoacetate e. l3- hydroxybutyrate
a. distilled water (specific gravity 1.000) b. sod ium chloride solution (specific gravity 1.050) c. acetic acid solution (specific gravity 1.100) d. sugar solution (specific gravity 1.300) e. copper sulfate solution (specific gravity 1.500) 169. How soon after infection with hearrworm
m icrofilariae are serolOgic test results positive?
a. b. c. d. e.
2 weeks I month 3 mo nths 6 months 1 year
166. A practical a nd sensitiue way to evaluate a
157. Foulld normally in herbiuores (e.g., horses, rabbits)
patie1ll's platelet function in the clinic without senditlK blood samples to tlte laboratory is by:
158. Associated with portosystemicsh unts
a. b. c. d. e.
159. Forms golden·yellow needle-shaped crystals
uritle 160. Found
jtl
j tl
clot relraction bleeding time von Willebrand's factor assay platelet count activated clotting time
alkaline urine ofdogs and cats
10 1998 Mosby-Year Book, Inc. Photocopying is pro/libited by law.
51
Correct answers are on pages 55-62.
www.vet4arab.co.cc
SECTI O N 2
52
170. Omcemirlg the microjiiariaeofDirofilaria immitis alld Dipetalonema reconditum, which statement is most accumte?
a. Microfilariae of Dirofilaria immitis move progressively. whereas those of Dipelalonema recondilllm move but remain in place. b. Microfilariae of Dirofilaria immitis are longer and wider than those of Dipetalonema recOflditum.
c. The tail of Dirofilaria immitis microfilariae is curved or hooked, whereas th at of Dipetalonema reconditum microfilariae is straight. d. Microfilariae of Dipetalonema reconditum are more common in canine blood than those of Dirofilaria imm itis. c. Microfilariae of Dipetalonema reconditum have a tapered head, whereas those of Dirofilaria immitis have a blunt head. 171. Conceming the direct Coombs' test, which statement is least accurate?
a. It is performed at 4°, 20", and 37" C. b. It requires use of species-specific reagen lS. c. It is affected by administration of corticosteroids. d. It is perfonned on serum from the patient. e. It is positive when hemagglutination is observed. 172. Cotlceming the antinuclear antibody test, which statement is most accurate?
a. It may be less sensitive than the lupus erythematosus ceUtest. b. It is not affected by large doses of exogenous corticosteroids. c. It does not require use of species-specific reagents. d. It is not positive in healthy dogs or cats. e. It is not specific for systemic lupus erythematosus. 173. Formalinfumes in the vicinityoffreshly made blood films:
a. have no adverse effects on staining b. alter the staining features of erythrocytes c. enhance the staining features of leukocyte nuclei
d. have no adverse effects on cellular detail,
e. give cell oUllines a crisper image
178. !fa dog's PCVis 15% and the reticulocytecounr is 12%, the corrected reticulocyte count is:
174. Particle smears made from bone marrow aspirate samples provide all the following information except:
a. degree of marrow fi brosis b. proportion of hematopoietic cells to adipose tissue c. megakaryocyte numbers d. hemosiderin content e. myeloid/erythroid ratio 175. Equipmem needed to perform a complete blood COlt /It i"cludes all ofllle/ollowing except a:
a. b. c. d. e.
microscope cell counter high-speed centrifuge refractometer 37"-C waler bath
176. Conceming hemograms seer! ill IIQrious conditiollS, which statement is least accurate?
a. Eosinophilia may be associated with allergic or hypersensitivity reactions. b. Eosinophilia may be associated with parasitic infections. c. Basophilia is associated with IgG-mediated disorders. d. Monocytosis is associated with chronic inflammatory d iseases. e. Persistent lymphocytosis may be associated with bovine leukemia virus infection in cattle. 177. Concemillg loxic neutTophils, which statement is least accurate?
a. Their presence suggests inflammation. b. Their presence is usually related to localized or systemic bacterial infections. c. Changes in the neutrophil cytoplasm arise from disturbed maturation of neutrophil precursors. d. Dohle bodies indicate severe toxicity. e. The prognosis is more favorable when neutrophil numbers and the severity of toxicity dimin ish over time.
C 1998 Mosby-Year Book, Inc. Pllotocopying is prohibited by law.
Clinica l PatilOlogy
CI Ca P
a. 10% b.8%
Total CO 2
c.6%
Glucose
BUN
53 87 mEq/L 8.7 mg/dl 12.1 mg/dl 14 mEq/L 86 mg/dl 73 mg/ dl
d.4% e.2%
182. W1wt is (he anion gap in this animal?
179. CBC in a cat reveals the/aI/owing: PCV 1096; MCV 48 fl; MCHC 34 gldI; aggregate reticulocytes 0.1% or IO,OOO/iJL 50 nucleated RBCsIlOO WBCs; normal erythrocyte morphology; and 90{)() WHCs/Ili-. Concerning this hemogram, which statemer/l is most aCCllrate?
a. The n umber of aggregate reticulocytes indicates regenerative anemia. b. The number of nucleated RBCs indicates regenerative anemia. c. The MCV indicates the bone marrow is responding to anemia by RBC regeneration. d. The PCV suggests moderate anemia for a cat. e. The corrected WBC count is 6000/ J.lL
a. 20 mEq/ L b. 32 mEq/L c. 45 mEq/L d. 57 mEq/L e. 66 mEq/L Question 183 A dog has IITefollolVing blood chemistry values. Ca Total plasma protein
Albumin Globulin
7.9 mg/dl 4.0 g/dl 1.8 g/dl 2.2 g/d]
183. W1lat is the corrected calcium value? 180. Increased serum alanilleaminotTansferase activity is common with all of the/allowing
l!XCept: a. abdominal trauma b. immune-mediated thrombocytopenia c. aClite pancreatitis d. severe skeletal muscle injury e. hemolysis of the blood sample 181. Acidosis is most likely to produce which change?
a. shift of potassium from extracellular to intracellular b. shift of potassium from intracellular to extracellular c. shift of sodium from extracellular to intracellular d. decrease in serum ionized calcium level e. increase in total serum calcium level Question 182 A patient has the /ollowing blood chemistry profile:
N. K
153 mEq/L 5,4 mEq/L
a. b. c. d.
7.6 mg/ dl 8.3 mg/dl 8.6 mg/ dl 9.3 mg/dl e. 9.6 mg/dl 184. Hypercalcemia has been associated with all of the following conditions excep t:
a. b. c. d. e.
lymphoid malignancies anal sac apocrine-gland carcinoma hypoadrenocorticism fungal infections milk fever
185. You suspect that a neonatal/oal has a ruptured bladder; and you collect fluid by abdominocentesis. ASsay for which biochemical constituent of lhe fluid is most likely to aid diagnosis ofbladder rupture?
a. b. c. d. e.
sodium phosphorus potassium urea nitrogen creatinine
Correct answers are on pages 55·62.
www.vet4arab.co.cc
SECTION 2
54 186. A direct smear preparation is generally not
advisable for cytologic examination of: a. pleural fluid h. pericardial fluid c. peritoneal fluid
d. cerebrospinal fluid e. synovial fluid
187. Qmcerning trallSudates, which statemem is least accurate?
n. Their protein concenuation is <2.5 gtdl. b. Cell counts are <500/ fJ.L in normal dogs. c. In normal dogs most of the cells are neutrophils. d . They arc associated with hypoalbuminemia <1.0 g/d!. e. They are associated with portal hypertension secondary to hepatic insufficiency. 188. uJrlceming an acute or recem hemorrhagic effusion, which scatement is least accurate?
a. It has a clear supernatant and red sediment. b. Intact eryth rocytes may be present. c. It contains macrophages with phagocytized erythrocytes. d. It contains hemosiderin -laden macrophages. e. It contains platelets. 189. Concemillg tile viscosity ofjoiTll fluid in horses, wllich statement is least accurate?
a. Hyaluronic acid is most responsible for joint fl u id viscosity. b. Viscosity is decreased by certain bacterial enzymes. c. Viscosity is decreased by dilurion of synovial fl uid with blood. d. Synovial fl uid appears yellow because of decreased viscosi£}'. e. Viscosi£}, may be estimated by the string test or mucin clot test 190. [n a dog hypersegmemed neurropllils i" an effusion indicate:
c. a long-standing pathologic condition d. normal canine neutrophils e. a degenerative change /91. Mononuclear pleocytosiS is mosllikely to be observed in an animal willi:
a. b. c. d. e.
canine distemper feline infectious peritonitis streptococcal meningitis cryptococcal meningitis toxoplasmosis
192. Concerning protein in cerebrospinal fluid, which statemelH is least accurate?
a. Protein levels can be estimated using tuin ary reagent strips. b. An increased protein level is seen with inflammatory conditions. c. The Pandy test is used to measure globulin levels. d. Protein levels are routinely measured as total solids with a refractometer. e. Protein levels may be increased without an increase in cell counts in degenerative condi tions. 193. WI,iel, cell type is likely to be prese1lt in highest numbers in a normal transtracheal wash?
a. b. c. d. e.
goblet cells ciliated columnar epithelial cells alveolar macrophages neutroph ils lymphocytes
Clinical Pathology
/95. A dog's transtracheal wash maycofllain farge numbers ofeosinophiis in any of tile following conditions excep t:
a. b. c. d. e.
b. c. d. e.
epithelial cells and mucus mast cells and epithelial cells neutrophils and mucus neutrophils and eosinophils eosinophils and mast cells
a. sepsis b. immune-mediated disease
C 1998 Mosby-Year Book, Inc. Photocopyillg is prollibilCd by law.
antimicrobial agents, a bsellceofa clear ring around tllealltimicrobial disks indicates:
a. b. c. d. e.
is least
accurate?
a. Cystocelllesis is preferred to catheterization to obtain urine. b. Transtracheal aspiration may be used to obtain lower airway specimens. c. A su rgical scrub of the skin is required before collection of samples from a lesion of supecficial pyoderma. d. Exudate from deep cutaneous lesions may be aspirated with a sterile needle and syringe. c. Several blood samples should be obtained for aerobic and anaerobic cultures 10 rule out bacterial septicemia. 197. 171e O/IQ of wllich parasite have a flattened operculum at one end?
contaminalll bacteria a slow-growing organism antimicrobial susceptibili£}' intermediate antimicrobial susceptibi lity antimicrobial res istance
199. Concerning examination ofskin scrapillgsfor mites, which stalemel/t is least accurate?
a. AHIO scalpel blade may be used for skin scrapings. b. Mineral oil may be placed on the lesion to aid recovery of parasites. c. Whcn looking for Demodex mites, the skin should be firmly pinched to help express the mites from hair follicles. d. Scrapings for Sarcoptes must be made deep into the dermis because th e parasites reside within hair follicles. e. TIle slide is examined under low power (lO x J. 200. Collcernillgdiagllosis ofdermatomycosis, which statement is least accllrate?
a. When present, ultraviolet fluorescence is best seen at the tip of intact hairs. b. Hairs should be plucked from the periphery of the lesion. c. A few drops of 10% potassium hydroxide may be used to clear debris from the specimen. d. Arth rospores on the hair shaft are best seen wi th the 40 X lens. e. The appearance of conidia, location of arth rospores, and a bility to fluoresce are lIsed to distinguish the different dermatophytes.
a. Eimeria b. Paragonimus kellicotti c. Trichuris vulpis d. Strongyloides stercoralis e. Giardia
194. A transtracheal wash from a dog wich a chronic cough associated with a collapsed trachea is likely to be characterized by: il.
198. III tests determinillg the semitiuityofbacleria to
heamvorm·d isease a1lecgic reaction lung fluke infection Toxocara migration Mke nnel cough~
196. Concemillgcollection of samples for microbiologic CIIlture, which statemem
55
Answers 1. e Bone marrow iron stores are normal to increased, but macrophages sequester the iron, so iI is not available fo r erythropoiesis. 2. a MCV=PCV/ RBC x 1O=48%,MCHC=Hb/PCV X 100 = 42 gfdJ.
3. c Red maple causes hemolytic anemia in horses, often associated with Heinz bodies andlor methemoglobin.
4. b The other diseases listed are associated with intravascular hemolysis. 5. d Neutrophils remain in circulation about 10 hours. 6. a Estrogen toxicity is associated with early neutrophilia, followed later by decreased production (aplastic anemia) and neutropenia, anemia, and thrombocytopenia.
56
SECTION 2
7. b Heinz bodies do not interfere with plasma protein values, but they can erroneously elevate hemoglobin values. 8. d Corrected WBC count = ObservedWBC x 100
30,000 x 100
30,000
Nucleated RBe + 100
200 + 100
300
=10,000
9. a Ehrilichiosis can be associated with benign lymphocytosis and monoclonal or polyclonal gammopathy. 10. d Ferritin is the preferred lest, hut it is species specific and often not available. Bone marrow iron stores are increased in chronic inflammatory disease. 11. e The PCV is higher than anticipated for relative polycythemia (answers b and d) . Erythropoietin levels should be elevated with secondary polycythemia (answers a and c) , 12. d A blood sample anticoagulated with EDTA is required. C3 coating ofRBCs can occur if clotted blood is left in the refrigerator. 13. c The antinuclear antibody test is more specific and sensi tive than (he other tests listed. 14. a Cats with rype-B blood have naturally occurring anti-A antibodies. 15. a IgG is the prevalent immunoglobulin in mare colostrum. 16. a In neonatal isoerythrolysis, the mare's serum contains antibodies against the foal's RBCs. 17. e Acute blood loss causes regenerative anemia (increased reticulocytes. often with macrocytic hypochromic indices) and decreased total plasma protein levels. 18. c He molytiCanemia is regenerative, often with macrocytic indices and decreased platelets. The total plasma protein level and MCHC are artificially elevated in he molyzed plasma. 19. b Anemia of ch ronic inflammatory disease is associated with le ukocytosis (ne utrophilia). increased total plasma protein level, mild anemia (often normocytic normochromic. with a tendency toward microcytic hypochromic indices), and often increased platelet numbers. 20. d Relative polycythemia has an increased PCV (not marked) and total p lasma protein level when associated with dehydration. 2 1. a Aplastic anemia is associated with pancytopenia (decreased WBC. RBC, and platelet numbers).
www.vet4arab.co.cc
22. b Epine phrine causes mature, mild leukocytosis (neulrophilia in dogs and lymphocytosis in cats and horses) and an increased PCV 23. a Glucocorticoids cause mature neutrophilia. lymphopenia, monocytosis, and eosinopenia. 24. c Decreased bone marrow production is associated with neutropenia without a left shift. 25. e The hallmarks of inflammation with a degenerat ive left shirt are decreased numbers of WBCs. with immature cells (bands and metamyelocytes, etc.) oU (llumbe ring mature neutrophils. 26. d Inflammation and stress are typified by an inflamm atory leukogram (neutrophilia, left shift. toxic changes) and lymphopenia . 27. e A dipstick test for blood detects RBCs, hemoglobin, and myoglobin. 28. c The last two digits of the specific gravity of the diluted sample (30) are multiplied by the dilutio n factor (2) to yield the specific gravity for the und iluted sample. 29. c Amyloidosis causes marked proteinuria, with granular casts and retained a bili ty to concentrate the urine (glomerulotubular imbalance). 30. e Myoglobin imparts a brown color to the urine. 4+ blood without RBCs indicates myoglobinuria or he maglobinuria. An ammonium sulfate preCipitation test is needed to differentiate these two (precipitates the hemoglobin). Also, the blood serum should be colorless rather than pink with myoglobinuria. and the history should be compatible with a muscle disorder. 31. b Pyelonephritis is inflammatory; increased WEC numbers with relatively few RBCs indicate inflammation. 32. d 4+ glucose with ketones suggests diabetes mellitus. Glucose in the urine increases the specific gravity. 33. a This combination ofresuhs indicates hematuria. 34. a Calcium oxalate crystals support a diagnos is of ethyle ne glycol toxiciry. 35. c FlP is associated with a moderately cellular effusion , often of mixed cell type and with a high protein level. On smears the background has a characteristic eosinophilic s tippled a ppearance. 36. c Postsurgical abdominal fluid is normally associated with high cell counts (often above 100.000/ .... L. with more than 90% neutrophils). Degenerative neutrophils. intracellular bacteria, and fecal material indicate a pathologic process.
o 1998 Mosby- Year Book, Inc. Photocopying is prohibited by law.
Clinical Pathology
37. c An increased triglyceride level. with a fluidlse rum triglyceride ratio a bove I, is the most reliable test indication of chylous effusion. 38. e Horses normally have neutrophils in their abdominal flu id. 39. e Hypoalbuminemia causes a clear transudate. 40. c Lymphosarcoma produces a modified transudate characte rized by blast cells. 41. b Peritonitis causes a suppurative exudate. 42. a Red fluid with platelet clumps but no erythrophagia typifies contamination with blood. 43. d Green fluid with plant material and bacteria but no inflammatory cells typifies bowel puncture. 44. c CK is the most sensitive and specific indicator of muscle damage. Activities of AST and LDH both rise more slowly and are nonspecific (m uscle and liver). Activity ofLDH is high in RBCs. so minor hemolysis may give erroneous elevations, especially in dogs but less likely in horses. 45. d Trypsinlike immunoreactivity reflects circulating trypsinogen and tryps in levels. his not affected by oral administration of pancreatic extracts; there is no need to halt use o f such products before testing. 46. a Young animals have high levels of thyroid hormone. 47. c Potassium excretion is decreased with hypoadrenocorticism and renal failure / urethral obstruction. Potassium is redistributed from cells to serum in acidosis. crush injuries. or cytosis. Sodium loss can occur with hypoadrenocorticism , diarrhea, vomiting, polyuria, or bums. 48. e Cholecalciferol causes vitamin D tOxicity, with hype rcalcemia. Primary hyperparathyroidism and pseudohyperparathyroidism are associated with hypophosphate mia. 49. c Metabolic alkalosis with respiratory compensation shows an alkaline pH with increased HCOl and Pc0 2. 50. b Respiratory alkalosis shows an alkaline pH with decreased Pc0 2 • 5 1. d Metabolic acidosis and respiratory acidosis show an acidic pH, decreased HeOl • and increased Pc0 2• 52. a Metabolic acidosis with respiratory compensation shows an acidic pH \vith decreased HC0 3 and Pco 2 •
57
53. e A portosystemic shunt causes increased bile acid and ammonia levels and low albumin and BUN levels. Other common findings include ammonium biurate crystals in the urine, target celis, microcytosis. hypoglycemia, and hypocholeste rolem ia. 54. d Bromide (used to treat seizures) poisoning causes a falsely elevated chloride level. 55. a Acidosis is associated with increased calcium levels. especially ionized calcium. 56. c Anion gap = (Na + K) - (Cl + total CO 2), 57. e Ethylene glycol causes a greater anion and osmolar gap than the other disorders listed. An estimate of osmolality is l.86(Na + K) + BUN/2.8 + Glucosell8 =323, yielding a gap of 411 - 323 = 88. 58. a Ammonia levels are a measure of liver function (rema ining viable cells). Activities of some enzymes are a measure of liver dysfunction (damaged cells) . 59. eAST is used to evaluate liver disease in large and small animals. 60. b ALP activity is used to evaluate cholestatic disease in small animals but is not a valuable test for cholestatic disease in large animals. 6 1. c ALT is used to evaluate liver disease in small animals but not in large animals. 62. d Elevated urinary GGT activity indicates tubula r damage. Serum GGT activity is also valuable in evaluating liver disease. especially in large animals. 63. e GI hemorrhage can increase the BUN level without increasing the creatinine level. A low total plasma protein level supports a diagnOSis of blood loss. 64. c Low pre- and POSl-ACTH cortisol levels characterize iatrogenic Cushing's disease. 65. a Larvae recove red from fres h feces of large animals are almost always those ofiungworms. The Baermann technique is not recommended fo r recovery of Filaroides from the fe ces of dogs. 66. c The cellophane tape method is used to detect eggs of pinworms. Horses are the only domestic species infected by pinworms. 67. e Trophozoites lack rigid walls and may collapse and be difficult to identify on fecal flotation . Wet mounts allow recognition of movement patterns to differentiate groups of protozoa. Addition of iodine facil itates identification of protozoa. 68. b The McMaster technique is the most accurate and commonly used method to quantitate eggs in feces.
58
SECTION 2
69. d Fecal culture causes sporulation of coccidial oocysts and is used to distinguish Eimeria (four sporocysts) from Isospora (two sporocysts). 70. e Sodium nitrate is the most efficient solution for fecal flotations. 71. a Size is the best criterion to differentiate the microfilariae of Dirofilaria from those of Dipetalonema (microfiiariae of Dirofilaria are longer and wider). 72. d Macrolides (milbemycin and ivermcctin) interfere with production of m icrofilariae by adult heartwonns. The antigen test is highly sensitive and specific and does not produce as many false· positive and false-negative reactions as the antibody test when screening fo r infections. 73. a Insufficient sample is the m ost common error associated with submission of fecal samples. Fastidious microaerophilic or anaerobic organisms cannot usually be recovered from swabs. 74. b Antimicrobial testing distinguishes drugs that maybe useful from those that a re not. 75. e Samples for aerobic culture should be refrigerated and stored in transport medium. Samples for anaerobic culture should be stored in transport medium at room tempe rature. Tissues should be f.roze n. 76. d Scraped mate rial should be added to mineral oil. 77. d Culture is me ultimate diagnostic test for ring-...onn. Microsporum ctmis is the only common dennatophyte of dogs and cats that fluoresces on exposure to a Wood's (ultraviolet) light. 78. d Otodectes causes otitis externa in dogs and cats. 79. e Knemidokoptes causes scaly leg a nd depluming scabies in birds. 80. c Sarcoptes mites deposit their eggs in the deep e pidermis and cause intense pruritus. 81. b Psoroptes mites do not burrow. 82. a Demodex mites have a classic cigar shape and are found in hair follicles, sebaceous glands, and the e pide rmis. 83. c Heparin is the anticoagula nt of choice for blood gas analysis. 84. a EDTA is the anticoagulant of choice for routine CBCs. 85. b Citrate is the anticoagula nt of choice for routine coagulation tests.
www.vet4arab.co.cc
86. e Fluoride inhibits glycolysis and should be used for glucose testing if serum sample cannot I?e immediately separated from the red cells or if a routine plasma sample cannot be immediately analyzed for glucose. 87. c A heparinized sanlple can be used for electrolyte analysiS, but serum is the preferred sample. 88. e Periodic acid-5chiff stain is used to stain fungal structures. 89. a New methylene blue is used to stain reticulocytes. 90. b Romanowsky-type stains (e.g., Diff-Quik, DipStat , Wright's) are used for routine staining of cytologic specime ns. Papanicolaou staining is more time consuming and technically difficult. 9 1. d Natt and Herrick's solution or phloxine B stain is used for avia n WBC counts. 92. b Romanowsky-type stains (e.g., Diff-Quik, DipStat) are used for routine staining of blood smears. 93. c The PIVICA test (proteins induced by vitamin K antagonism) is a modified prothrombin test that is very sensitive in diagnosing vitamin K toxicity. 94. e Factors II, VII, IX, a nd X a nd protein Care vitamin K dependent. 95. e Assay for vWF antigen is the best test to diagnose von Willebrand's d isease. 96. c Factor VII deficiency causes increased extrinsic (prothrombin time) test times. 97. e Factor VITI deficiency causes increased intrinsic (activated partial thromboplastin time) test times. 98. d Factor X defici ency causes increased extrinsic (prothrombin time) , intrinsic (activated partial thromboplastin time), and common pathway test times. 99. a Disseminated intravascular coagulation is characterized by increased prothrombin and activated partial thromboplastin times, increased fibrin degradation products, and decreased platelet numbers and fibrinogen levels. 100. b Immune-mediated thrombocytopenia causes decreased platelet numbers but does not affect prothrombin time, activated pa rtial thromboplastin time, or fibrin degradation products. 101. a Lavender-top tubes contain EDTA anticoagulant.
10 1998 Mosby-Year Boole, Inc. Photocopying is prohibited by law.
Cli nical Pathology
102. c Protein accumulation in the aperture can result in e rro neous hematocrit readings and an increased freq uency of obstructions. 103. d Con tamination of the diluent with bacte ria or other particles results in countin g e rrors. As the count inc reases, errors increase expone ntially. 104. e The enti re system includes the reagent, instrument, and operator. 105. b Although a ll of the other choices can also cause errors, inadequate mixing is the most common error. An automatic specimen rotator improves mixing. 106. d The avian heterophil is analogous to the mammalian neutrophil. 107. b The metamyelocyte is a granulocyte precursor. The other cells a re etyth rocyte precu rsors. 108. a Physiologic leukocytosis occurs whe n marginated granulocytes e nter the general circulatio n as a result of excitemen t or stress. 109. d All avian erythrocytes are nucleated. 110. e The metamyelocyte is a granulocyte precursor. The other cells are a Uof the erythrocyte series. Ill. c Immature forms include band and stab cells. 112. d Eosinophilia occurs commonly in response to antigen-antibody reactions, as well as with inflammation of certain organs, such as the lungs, which tend to be allergy targets. 113. e Calibration should be performed every I to 3 months to account fo r changes in the centrifuge that occur as a result of wear a nd tear. 114. d Trichrome stain is typically used for visualization of parasites in feces. 1l5. e Because blood collection tube manufacturers are now producing fewer tube s izes than previously, it is more importan t than ever to p ay attention to the recommended fill volume printed on the package. For example, several commonly used lavender-top (EDTA) tubes o n the market have an internal volume of 5 m1 but are used to collect only 2 to 2.5 ml of blood a nd contain enough EDTA to anticoagulate only 2 to 2.5 mJ of blood. Filling such a 5-mJ tube completely could result in a clotted specimen; therefore 2.5 ml (o r half the tube volume) is the m aximum fi ll recomme nded by the manufacturer. 116. b Th rombocytope nia refers to a reduct jon in platele t numbers.
59
11 7. a Bleeding time is also prolonged with platelet deficiency, but a direct count of platelets is a better method to assess thrombocytopenia. 11 8. e Erythrocyte sedimentation rate is not a test of hemostasis. 119. e Serum values of potassium, aspartate aminotransferase, and phosphorus increase as a result of red blood cell leakage a nd hemolysis. Glucose levels decrease up to 5% pe r hour as a result of anaerobic glycolysis by red blood cells. 120. b Large-bore needles cause "spiraling~ of blood cells as they enter the needle. Alcohol and rough handling can result in cell lysis. A 10-ml tube would likely create too much negative pressure with subseque nt hemolysis, when collecting blood from a small a nimal. 121. c Some renal diseases are related to increased serum calcium levels. 122. d The serum creatinine level is a bener assessment of renal function. 123. e Because amylase is eliminated through the kidneys, any disorder that reduces renal blood flow or impairs kidney function can result in elevated serum amylase activity. 124. c Organophosphate insecticides can be cholinesterase inhibitors. 125. b Anion gap = (Na + K) - (el + Total CO 2), expressed in mEq/L 126. d Ammonia (NH]) is also unstable. Such enzymes as alanine aminotransferase and aspartate aminotransferase usually show some loss of activity after a few days unless the reconstituted control is frozen. 127. b Type B is a high-viscosity oil and is the best choice for light microscopy. 128. a Giardia organisms a re more likely to be found in direct saline smears unless zinc sulfate is used as the flotation medium. 129. a In terms of reliability, finding the characteristic Ehrlichia morulae on a blood smear examination confimls an active infection. A negative blood smear, however, is inconclusive. Elevated Ehrfic/tia antibody titers can persist for many months after the animal has apparently recovered. A1mough suggestive of infection, antibody titers are not as sensitive for detecting active infection. 130. e Enzyme-lin ked immunosorbent assays and the newer immunochromatographic assays detect adul t heartwonn antigen, not micro filaria e. These tests are not affec ted by the presence nor a bsence of microfilariae.
60
SECTION 2
131. d In a low-incidence population, a test with a high positive pred ictive value is the best choice. Tests of high specificity have a high positive predictive value. 132. c Urine specime ns collected from the floo r or Iiller box arc likely to be contaminated. 133. d Needle puncture of the wall of a diseased. bladde r may predispose 10 bladder rupture. 134. d The urine of diabetic patients with ketoacidosis may have a ufruity" odor. 135. b The diet of carnivores usually produces acidic urine (pH less than 7.0), 136. d Dogs and cats can synthesize ascorbic acid; therefore an ascorb ic acid- resistant urine dipstick is best for veterinary use. 137. d Nitrate occurs naturally in plants. Dogs and cats are primarily carn ivores. 138. d ReF refers to relative centrifugal fo rce. An RCF of 400 can be obtained using a centrifuge with a 6-inch- radius arm rotated at 1500 rpm. 139. d Cystinuria occurs almost exclusively in male dogs. Dachshunds, basset hounds, Chihuahuas. Yorkshire terrie rs, and Irish terriers have been affected. 140. a Calcium oxalate crystals may also be found in small numbers in normaJ urine. 141. c All these kits test fo r the p27 FeLV antigen. 142. c Some infected cats will die fro m FeLV infection; however, many more wiU recover and become immune. 143. b Unbound enzyme conjugate was probably left be hind in all three wells. The unbound conjugate reacted with the added s ubstrate to produce color. 144. b Although several kits on the market have demonstrated enhanced sensitivity and have been approved for testing cats for heartworm infection, detection of s mall numbers of adult heartworms and the low levels of associated antigen are still problematic in cats. 145. a Many of these animals are transiently infected and seroconvert in time. 146. d FeLV is transmitted primarily through the saliva of infected cats. 147. c The extent to which an antimicrobial diffuses through agar is a property of the individual drug. Drugs tllat diffuse more slowly yield smaller zones of inhibition; however, they may still be effective choices for treatment.
www.vet4arab.co.cc
148. b Direct sensitivity tests (those performed with mixed cultures) may only yield equivocal results. 149. c Unopened disk cartridges are best stored in the freezer. Once cartridges are opened, however, it may be more convenient to store them with the dispenser in tlle refrigerator. 150. e All other choices listed would tend to cause excessive blue staining. 15 1. e Additional tests suggested include BUN or serum creatinine assay, repeat urine specific gravity testing, and a water deprivation test, if clinically advised. 152. c Myoglobinuria is best diagnosed when there is clinical or laboratory evide nce of muscle damage, as indicated by an increase in serum creatine kinase activity. 153 . c Mucus and crystals tend to make equine urine thick and cloudy, especially if the sample cools for some time after collection. 154. a Transitory, mild proteinuria may be related to extrarenal factors that can affect glomerular permeability. 155. e Bilirubinuria is a common finding with biliary obstruction. 156. c One of the advantages of cystocentesis is that the sample is not co ntaminated by microorganisms fro m the distal urinary tract. 157. b Calcium carbonate crystals are common in the urine of herbivores. 158. d Ammonium b iurate crystals may be found in animals with a portosystemic shunt. 159. e Bilirubin crystals have this characteristic appearance. 160. a Triple phosphate crystals are the most common crystals in canine and feline urine. 161 . c Calcium oxalate crystals are seen in the urine of animals that have in gested antifreeze. 162. b The sulfosalicylic acid p recipitation test detects many types of protein, including globulins and Bence-Jones protein. The reagent dipstick fo r urine protein only detects albumin. 163. e Frac(ional electrolyte excretion detects increased or decreased resorption by the renal tubuJes. 164. a Glucosuria may be seen in hyperglycemic patients with urinary hemorrhage and in cats with urinary obstruction.
C 1998 Mosby-Year Book, Inc. Photocopying is prollibited by law.
Clinical Pathology
165. d Some reagent strips can detect acetone as well as acetoacetate, but the reactions are most specific fo r acetoacetate.l3-hydroxybutyrate is not detected in tllese reactions. 16G. b ~si ng the buccal mucosa and a cutting deVIce, bleeding time is highly sensitive in detecting platelet function abnormalities and severe thrombocytopenia. Clot retraction also may be used but is more crude and less quantitative than bleeding time. 167. d Disseminated intravascular coagulation must first be ruled out before immune-mediated thrombocytopenia is considered. Ehrlichiosis is best diagnosed by serologic testing. 168. d The preferred specific gravity is between 1. 100 and 1.350. 169. d Results may be positive as early as 5 or 6 mon ths aft e r infection but usually not until 6 or 7 months after infection. 170. b Microfilariae of Dirofilaria immitis are approximately as wide as the diameter of a cani.n.e erythrocyte. They exhibit stationary motlhty and have a straight tail and a tapered head. 171. d The direct Coombs' test is perfomled on red blood cells collected in EDTA anticoagulant. 172. ~ The ~ntinuclear antibody test may be positive m a .v~ n ety of chronic inflammatory diseases, in addition to systemic lupus erythematosus. 173. b In smears stained with Romanowsky s tains formal in fumes cause red blood cells to stain' green ish bl ue. Cellular detail appears indistinct. 174. a Alth ough connective tissue cells may occasionally exfoliate, the best method to detect fibro tic change in bone marrow is core biopsy. 175. e A 56°~C ".'ater bath is used for fibrinogen determlllatlOn by the heat precipitation method. A high-speed centrifuge is required for measuring the packed cell volume. 176. c Basophilia is associated with IgE-mediated disorders. 177. d DohJe bodies indicate mild tOxicity. 178. d Corrected % reticulocytes = % reticuJocytes X Patient's PCV/Normal PCv. In this dog, corrected reticulocytes = 12% x 15%/45% = 4%. 179. e CorrectedWBC/J.l.L=(WBCcoum X 100)/ 100 + Nucleated RBCs. In this cat, corrected WBC count = (9000 x 100)lloo + 50 =6Ooo/J.l.L
61
180. b Alanine aminotransferase is not liver specific; serum ALT activity may be increased with severe skeletal muscle disorders. 181. b Acidosis increases ionized calcium levels but does not change total calcium levels. 182. d Anion gap = (Na + K) - (CI + bicarbonate). Tota] CO 2 can be substituted for bicarbonate In this case, anion gap = (I 53 + 5) - (87 + 14) = '158 - 101 =57. 183. e Corrected calcium (mg/eIl) = Calcium Albwn..in + 3.5. In this case, corrected calcium = 7.9 - 1.8 + 3.5 =9.6 mg/d1. Alternatively, Corrected calcium (mg/dl) == Calcium - (0.4 X tota] protein) + 3.3, which in tllis case = 7.9 - 1.6 + 3.3 = 9.6 mg/dl. 184. e Certain fu ngal infections, such as blas tomycosis, have been associated wi th hype rcalcemia. 185. e Urea is easily diffusible across the peritoneal membrane and has a low rate of synthesis in fo.als. Creatinine is a relatively larger molecule With poor diffusion across the peritoneum. 186. d Cell cou nts in uncentrifuged cerebrospinal flui d are .usually too low to count accurately; preparations must be centrifuged before examination. 187. c In dogs. mononuclear cells predominate in transudales. J88. d Hemosiderin-laden macrophages are seen in long-standing or chronic hemorrhagic effusions. 189. d Equi ne joint fluid is normally yellow. 190. c Hypersegmentalion of neulrophils is an indication of ch ronicity. 19 L. a Pleocytosis refers to an increased leukocyte coun t in cerebrospinal fluid . Mononuclear cells, such as lymphocytes, are most commonly seen in viral in fectio ns. 192. d A refractometer is not sensitive enough to detect the small amount of protein in cerebrospinal fluid. 193. b Alveolar macrophages are more commonly observed in bronchoalveolar lavages. 194. c These are the most common findings in washes from dogs with a collapsed trachea. 195. e Eosinophils are not characteristic of washes from dogs with tracheobronchitis. 196. c ~is in!ectio n is not necessary for s uperficial skin leSions because it interferes with bacterial growth in culture.
SECTION 2
62 197. b The ova of the lung fluke, Paragonimus
kellicotti, have a characteristic flattened opcrcuJum at one end. 198. e Antimicrobial growth (lack of a clear zone of inhibition) around the antimicrobial disk indicates bacterial resistance.
www.vet4arab.co.cc
199. d Sarcoptes miles arc found in the superficial hyperkeratotic portions of the epidermis. 200. a Fluorescence appears best on the deeper hair shaft portions, especially those below the skin surface. For this reason , hairs should be plucked and then examined for fluorescence.
NOTES
SECTION
3 Diagnostic Imaging and Recordings J.M. Bowen, S.T. Finn-Bodner, JA Hudson, W.R. Klemm, M.L. Moon, P.w. Pratt, D.E. Thrall, L.P. Tilley, E.R. Wisner
RADIOGRAPHY Rewmmeluled Reading Burk RL. Acke rman N: Small animal radiology and ulzrasoTlograp/lY. cd 2, Philadelphia. 1996, WB Saun ders. FarrowCS el al: Rndiologyoflilecat, SI. Louis, 1994, Mosby. Morgan JP. Silverman S: Tecliniques of veterinary radiography, ed 5, Davis, Calif, 1993, Velerinary Radio logy Associales. Thrall DE: Textbook of veterinary diagnostic radiology, ed 2, Philadelphia, 1994, WB Saunders. Practice answer sheets are on pages 259-260.
Questions M.L. Moon 1. The silhouette effect occurs on a radiograph wilen: a. nvo structures are superimposed and are thus doubled in density b. two structures of the same radiographic density are in contact and their margins cannot be distinguished radiographically c, two structures of the same radiographic density are not in close contact and are separated by a s ubstance of a differing radiopacity and their borders can be distinguished radiographically
@1998Mosby-YearBook,Inc.Photocopyingisprohibitedbylaw.
d. blood vessels are seen "end on," forming fo cal densities e. the two crura of the diaphragm are superimposed on lateral thoracic views 2. The silhouette effecr is frequently seen in animals witll: a. b. c. d. e.
pleural effusion chronic bronchitis hearnvorm d isease normal thoracic radiographs age-related changes of the lungs
Cm7l!ct answers are on pages 92- 100.
63
SECTION 3
64 3. Concerning teclllliqlle for radiography of the thorax, which statement is least accurate?
a. Routine thoracic films should be made at full inspiration, if possible. h. High-kVp techniques produce films \-vith low contrast and a long gray scale. c. A grid is usually not necessary, even in horses and large dogs. d. At least three separate radiographic views are needed to completely evaluate the thorax of an aduil horse. e. Low-mAs techniques produce films with less motion artifaci. 4. Radiographic signs a/pleural effusion include all of tile following except;
a. h. c. d. e.
a widened, radiopaque pleural space widened, opaque interlobar fissure lines partial collapse of the lung lobes radiolucent air surrounding the lung lobes blunting of lhe costophrenic angles on ventrodorsal views
5. Pneumomediastinum may be caused by any tile fo/lowingexcept:
a. b. c. d. e.
tracheal rupture esophageal rupture pneumothorax injury to lung parenchyma injury to cervical soft tissues
www.vet4arab.co.cc
7. Concerning radiograpllicelJaluation of tracheal collapse, which statemellt is least accurate?
a. The intrathoracic trachea collapses most on expiration. b. The eXlrathoracic trachea collapses most on inspiration . c. Tracheal collapse can only be diagnosed on flu oroscopy. d. In normal animals th e tracheal diameter does not change significantly between inspiration and expiration. e. Tracheal collapse is most often seen in toy dog breeds. 8. COllcernillgdijferentiatioll Of pneumonia (fobar
consolidation) and col/apse (atelectasis) of lung lobes Oil radiographs, which stateme1lt is most accurate?
a. Lung lobe atelectasis typically causes a mediastinal shift, best seen on a ventrodorsal or dorsoventral view. b. Pneumonia is characterized by an alveolar pattern; atelectasis is not. c. Lobar consolidation and atelectasis cannot be reliably differentiated on radiographs. d. Atelectasis always causes a secondary pleural effusion; pneumonia does not. e. Atelectasis may cause no radiographic changes, but pneumonia is always radiographically evident. 9. Pneumothorax may cause any of the following
6. A lesion ill the right middle lung lobe is best visualized on a left lateral radiograph because:
a. the lesion would be overlying the heart b. the lesion would be farther away from the cassette and hence magnified c. the lesion in the nondependent (uppermost), better-inflated lung lobe is better visualized d. there is less superimposition of ribs in this position e. air in the dependent lung helps to provide contrast
radiographic signs except:
a. a widened, radiolucent pleural space b. partial collapse of lung lobes c. absence of vascular and interstitial markings outside the lung lobes d. apparent elevation ofthe heart off the sternum on lateral views e. visible, opaque fissure lines
Diagnostic Imaging and Recordings
10. An air bronchogram is a:
a. thickened, prominent bronchial wall b. bronchus surrounded by dilated arteries and veins c. calcified bronchial wall d. bronchus containing air e. radiolucent bronchial lumen visible because it is surrounded by fluid-filled lung tissue
c. caudodorsal slant to the caudal vena cava d. caudal lung fields extending caudally to the first lumbar vertebra e. heart appearing relatively enlarged 15. A 12-year-old cocker spaniel;s seen because of
ofluflgpattern?
coughing and a murmur. Thoracic radiographs reIJeal left atrial and left ventricular enlargement, pulmonary venous enlargement, and a mixed alveolar and linear ;rlterstitial infiltrate in the dorsal and caudallungfields. What is the most likely calise of these findings?
a. b. c. d. e.
a. b. c. d. e.
11 . An air bronchogram is consistent with which type
alveolar vascular linear interstitial nodular interstitial bronchial
12. Wl1icl, disorder is not characterized by nodular lesions 011 thoracic radiographs? 3.
b. c. d. e.
primary pulmonary neoplasia metastatic pulmonary neoplasia fungal granuloma pulmonary abscess chronic bronchitis
13. Concerni1lg eIJallUlriOIi of peripheral pulmonary
arteries and /Jeins 011 thoracic radiographs, which statement is least accurate?
a. Enlarged arteries may indicate heartworm disease. b. Enlarged veins may indicate left-sided congestive heart failure. c. Enlarged arteries and veins indicate pulmon ary overperfusion associated with a left-to- right shunt. d. Small arteries and veins may indicate hypovolemia and shock. e. Peripheral arteries are typically larger than veins.
aspiration pneumonia cardiogenic pulmonary edema chronic bronchitis distemper pneumonia metastatic pulmonary neoplasia
16. On a dorsoventral radiographic IJiew, the heart may bediIJided into segments representing the face ofa clock. A prominent bulge at the 1- to 2-o'cfock position most likely represents the elliarged: a. b. c. d. e.
ascending aorta left atriwn pulmonary trwa left auricle right atrium
17. All of the following are consistent with left heart elllargemellt except:
a. an elongated, straight caudal border (lateral view) b. elevation of the entire trachea, including the main-stem bronchi (lateral view) c. decreased distance between the left ventricle and left thoracic wall (ventrodorsal view) d. "reversed-D" appearance on ventrodorsal view e. bulge of left auricle at the 2- to 3-o'c1ock position on ventrodorsal view
J4. All of ti,e followillg are signs ofexpiration on
thoracic radiographs except:
a. decreased distance between the heart and diaphragnl b. small, opaque lung fields
11:1 1998 Mosby-Year Book. Inc. Photocopying is prohibited by law.
65
Correct ariSlwrs are on pages 92-Joo.
www.vet4arab.co.cc
SECTION 3
66
18. 1710racic radiograpllS ofa puppy witli a murmur show marked left atrial and left ventricular enlargement. enlarged pulmonary arteries and veins, and a bulge;n rhe pulmonary trunk and descending aorta. Wllaf is the most likely calise of
Ihesejindings?
a. pulmonic stenosis b. patent ductus arteriosus c. aortic stenosis d. tetralogyofFallot e. ventricular seplal defect
19. Causes of generalized cardiomegaly inelude all the following except: a. b. c. d. e.
pericardial effusion pe ricardial-peritoneal diaphragmatic hernia mitral and tricuspid insufficiency dilative cardiomyopathy hypovolemia
20. 17lOracic mdiograpllS ofa 4-year-old dog with citron;c coughing reuenf a "reversed-D~ appearance to the hean 011 dorsoventral views. a bulge at the 1to 2-o'clock position all the cardiac sWlOuerteoll the dorsovelltrallJiew, and enlarged, tortuous pulmonary arteries Oil both lateral and dorsoventral views. WlIat is the mosllikely cause o/these findings?
a. b. c. d. e.
pulmonic stenosis heartworm disease ventricular septal defect tricuspid insufficie ncy mitral insufficiency
21. On abdomillal radiographs of dogs. tile normal kidney length is approximately: a. I to 1.5 times the length of the sixth lumbar vertebra b. 2.5 to 3.5 times the le ngth of the second lumbar vertebra c. 4 to 5 times the length of the fourth lumbar vertebra d. half the distance of the combined lengths of the third and fourth lumbar vertebrae e. twice the width of the first lumbar vertebra
22. Loss o/inrranbdominal C01lrrast on radiographs may be ca/lsed by any of the following except: a. b. c. d. e.
peritoneal effusion lack of imraabdominal fat peritonitiS peritoneal neoplasia free abdominal air
23. What is the earliest stage ofgestation at which mineralized fetal skeletons may be evident in abdominal radiographs ofa pregnant female dog?
a. 7 to 9 days b. 10to 19 days c. 20 to 30 days d. 40 to 45 days
e. 46 to 50 days 24. Conceming the appearance ofaccumulated fluid and gas 011 right lateral radiographs of the canine abdomen. wllich statement is most accurate? a. Fluid accumuJates in the pylorus, whereas gas accumulates in the fundus. b. Fluid accumulates in both the pylorus and fundus. c. Gas accumulates in both the pylorus and fundus. d. Fluid accumulates in the fundus. whereas gas accumuJates in the pylorus. e. A horizontal l1uid-gas interface is evident at the pylorus. 25. You wam to perform an upper gastroimestinal
contrast series 011 a dog with suspected perforation of the jejunum. W1/at is the contrast agent of choice for this procedure? a. barium sulfate because of its superior coating properties b. barium sulfate because of its lack of hype rtonicity c. a water-soluble organiC iodine agent because it will not irritate the pe ritoneum d. a water-soluble organic iodine agent because of its high radiopacity e. room air because it will not irritate the peritoneum
@ 1998Mosby-YearBook,lnc.Photocopyingisprohibitedbylaw.
Diagnostic Imaging and Recordl1Jgs
26. After bone trauma or infection, periosteal new bone formation first becomes radiographically evidellt after:
a. 1 to 2 days b. 7 to 10 days c. 2 to 3 weeks d. 4 to 6 weeks e. Sto 10 weeks
67
30. You note a large crallial abdominal mass 011 abdominal radiographs ofa cm. The stomach is displaced dorsalfyand caudallJI by tllis mass. Wllat is the most likely site of this mass? a. liver b. ovary c. kidney d. prosl'ate e. bladder
27. W11at is all important colltraindication to imravcllous urography? a. severe dehydration b. pyelonephritis c. hyposthenuria d. absence of one kidney e. renal calculi
31. W11at is the most common site ofosteocllOndrosis ill dogs?
u. distal medial humeral condyle b. c. d. e.
28. Radiographic sig1iS ofacute gastric fOrsion include all tile following except: a. gas and fluid distension ofthe stomach b. d isplacement of the pylorus dorsally and to the left c. compartmentalization of the stomach (fold ing of the s tomach on itself) d. displacement of the spleen e. accumulation offluid in the pylorus on right lateral views 29. Concemillg evaluation of the bladder using cystography. which statement is least accurate? a. Positive-contrast cystography is best for detection of bladder rupture. b. Double-contrast cystography is best for evaluating the bladder wall and intraluminal filling defects. c. Double-contrast cystography uses a large volume of positive-contrast medium to distend the bladder. with a small amount of negative-contrast medium (air. carbon dioxide. nitrous oxide). d. When negative· contrast agents are used, carbon dioxide and nitrous oxide are safer than room air in preventing air embolism. e. Survey abdominal radiographs should always be made before cystography of any kind.
lateral femoral condyle medial trochlear ridge of the talus caudal humeral head lateral femoral condyle
32. Collcemi1lgosreocllOndrosis in dogs. which statemelll is least accurate? a. Lesions may heal spontaneously. b. Lesions are often bilateral. c. Lesions are caused by a defect in enchondral ossification. d. Cartilage flaps or free-floating cartilage fragments may be present. e. Lesions are primarily seen in young. smallbreed dogs. 33. A 9-month-old German shepherd has shifting forelimb lameness. with pain on palpation over bot" hllmeral diaphyses. Radiographs reveal mllltlfocal, ,latchy increases in opacity in the medllilarycavity oftlie right and left humeri. What is the most likely cause of these findings?
a. b. c. d. e.
panosteitis osteochondritis dissecans hypertrophic osteodystrophy ununited anconeal process fragmented coronoid process
OJrrect answers are on pages 92·100.
www.vet4arab.co.cc
SECTION 3
68 34. A 9-mantll-old miniature poodfeexhibits
progressive rigllt rear limb lnmeness and pain. There;s no IIistoryoftrauma. On radiographs of the pelvis, tile rigllt femoral head is flattened and irregular. with areas of lysis. and the coxofemoral joillt space is widened. Mw! is tile mosllikely callse afthese findings?
a. h. c. d. e.
hip dysplasia osteochond rosis Legg-Calve-Perlhes disease panostcitis degenerative joint disease
35. Concerning osteosarcoma in dogs, which statemellt is least accurate?
a. It can occur in young dogs, as young as I or 2
years of age. b. The radiographic appearance is variable; lysis, proliferation, or both may be seen . c. It is the most common primary bone tumor in dogs. d. It usually occurs in mid-diaphysis aflong bones. e. Large-breed dogs are most commonly affected. 36. W1Ull is tile proper dosage ofbarium sulfate contrast medium for a routine upper gasrrointesrirlal series?
a. b. c. d. e.
I to 2 mllkg body weight 6 to 12 mllkg body weight 10 to 20 mllkg body weight 20 to 40 mllkg bodywcight 50 mil kg body weight
37. COllcerning Salter cfass- V fractures. which statement is most accurate?
a. They extend across the physis. b. They extcnd from the physis into the metaphysis. c. They extend from the physis into the epiphysis. d. They extend across the physis. from the metaphysis into the epiphysis. e. They are usually not seen on radiographs. because they are compression fractures.
38. W1lat is the most consistent radiographic sign of diskospondylitis?
a. b. c. d. e.
lysis and irregularity of the vertebral end plates ventral spondylosis calcification of the intervertebral disk lytic changes of the dorsal spinous processes dorsal spondylosis
39. Concerning degenerative joint disease of the equine tarms. which statement is least accurate?
a. It is most commonly seen in the d istal intertarsal and tarsometatarsal joints. b. Periarticular osteophyte formation is common in affectcd joints. c. Joint -space narrOwing or fusio n of affected joints may occur. d. There is not always a good correlation between clinical and radiographic signs. e. Extensive lysis of the tarsal bones is a consistent Sign. 40. A radiographic view is rlamed by tlte direction in
whidl the central ray oftile primary beam penetrates the body part. from tlte point ofentraru:e to tile poim ofexit. W1lidl of the following most accurately describes passage ofthe x -ray beam in a dorsal 60degn?e lateral-palmaromedial oblique view?
a. Starting laterally, the beam travels 60 degrees dorsally, to exit o n the palmaromedial aspect. b. Starting dorsally, the beam travels 60 degrees laterally. to exit on the palmaromedial aspect. c. Starting on the palmar aspect, the beam travels 30 degrees laterally, to exit dorsolaterally. d. Starting on the palmar aspect, the beam travels 90 degrees laterally, to exit dorsolaterally. e. Starling on the lateral aspect, the beam travels 45 degrees dorsally, to exit on the palmaromedial aspect.
41. A slab fracture of the dorsomedial aspect of the
third carpal bone is best evaluated llsing which two radiographic views?
a. dorsolateral-palmaromedial oblique and dorsoproximal-dorsodistal oblique b. dorsopalmar and lateral-medial c. dorsomedial-palmarolateral oblique and flexed lateral d. dorsopalmar and flexed lateral e. fl exed medial and ventromedialventroproximal oblique 42. Concerning radiographic evaluation ofnavicular disease inllorses, which statement is least accurate?
a. Large numbers of lollipop-shaped, dilated b. c. d. e.
channels in the distal border of the navicular bone are consistent with navicular disease. Cystlike lesions in the medullary cavity are consistent with navicular disease. Erosion of the cortical bone on the flexor surface is consistent with navicular disease. Horses \vith clinical signs of navicular disease may have normal radiographs. Navicular disease is best diagnosed on a single lateral-medial view of the foot.
69
45. Concerning radiographic technique. which statement is most accurate?
a. Low kVp results in high contrast and a short gray scale. b. High kVp results in high contrast and a short gray scale. c. When taking a radjograph of extremities (bone). a low-contrast, long-gray scale technique is preferred. d. When taking a radiograph of the thorax, a high-contrast, short-gray scale technique is preferred. e. Radiographic contrast depends only on kVp. 46. When processing x-ray film, the developer:
a. removes unexposed silver salts from the film emulSion, making unexposed areas clear b. converts exposed silver halide to metallic silver, making exposed areas black c. proceeds at a slower rate at higher temperatures d. never becomes exhausted and therefore does not need replenishment e. serves p rimarily to harden the film, preventing sweUing of the emulsion 47. After processing a radiograph. you note mUltiple,
43. Concerning use of grids in veterinary radiology, which statemem is least accurate?
a. Grids are placed between the x-ray film and the screens. b. Grids absorb any radiation traveling on a course not parallel to the primal)' x- ray beam. c. Grids are used to radiograph body parts thicker than approximately 10 cm. d. When grids are used, exposure factors must be increased. e. Grids Illay be stational)' or moving during the cxposure.
branching, black lines extending across the film. What ;s the most likely cause of this radiographic artifact?
a. b. c. d. e.
light leaking into the darkroom static electricity poor screen-film contact overexposure overly warm developer solution
48. What is the maximum annual legally pemJissible radiation dose for occupational exposure?
a. 5 rem 44. Radiographic density, the degree ofblackness of the finished film, can be decreased by:
• C 1998 Mosby-Year Book. Inc. Photocopying is prollibited by law.
Diagnostic Imagillg and Recordings
a. b. c. d. e.
increasing the milliamperage (mA) increasing the time exposure increasing tile kilovoltage (kVp) decreasing the developer solution temperature increasing the time in the developer solution
b. c. d. e.
10 rem 15 rem 20 rem 25 rem
Correct answers are on pages 92-100.
www.vet4arab.co.cc
SECTION 3
70 49. All the following are important principles of radiation safety except wllieh one?
a. Use optimal filtration of the x-ray beam. b. Use mechanical restraints when possible to minimize manual restraint of the patient. c. Collimate to the smallest area consistent with the clinical requirements. d. Use nonscreen film to achieve optimal detail. e. Always wear lead aprons and gloves when in the radiology roo m during the exposure.
50. All of the following are important principles of radiatiOIl safety durillg equille radiography except wllic11 olle?
a. Use cassette holders. b. Lead gloves protec t hands only from scatter radia tion, not from the primary beam. c. AJways ma nually hold the portable x-ray machine to achieve high-qua1ity radiographs. d. Minimize personnel in the radiology room during the exposure. e. If necessary, sedate the anima1 to reduce the need for repeat exposures.
D.E. Thrall 51. For which of tile following radiographic
examinations would a grid provide the greatest improvement in detail?
a. b. c. d. e.
lateral view, body, parakeet craniocaudal view, antebrachium, great dane lateral view, abdomen, German shepherd lateral view, morax. 6-week-old Siamese la teral view, metacarpal bones, aduJt thoroughbred
52. Which technique would produce an abdominal radiograph with the greatest contrast? 1 mAs, 108 kVp 2 mAs, 90 kVp 4 mAs, 72 kVp 8 mAs, 58 kVp e. 16 mAs, 46 kVp
a. b. c. d.
b. 20 mAs, 40 kVp c. 10 mAs, 80 kVp d. 40 mAs, 80 kVp e. 40 mAs, 60 kVp
a. b. c. d. e.
increased detail increased contrast increased cost increased speed dec reased cost
55. You manually develop a 14 x 17-inch sheet ofxray film followi"g exposure to x-rays. When the film is dry. you notice a green stripe at the edge of olleofthe 14·inch ends. What is the most likely cause of this discoloration? a. b. c. d.
53. A radiograph made using 20 mAs and 80 kVp is too dark. Which technique is reasonable to usefor the second attempt?
a. 20 mAs, 160 kVp
54. Rare-earth intellsifyillg screellS were illtroduced in the 1970s. Their biggest advantage over calcium-tungstate intellSifying screens is:
The fixer solution level is too low. The green region was not exposed to x·rays. The developer solution level is too low. Both the developer and fixer solution levels are too low. e. The wash water level is too low.
56. After processing a radiograph , you notice an imellSely black artifact witll the appearance of arborized black lines. The most likely cause of this artifact is:
a. b. c. d.
underdevelopment overdevelopment light rog developer splashed on the film in the darkroom e. static electricity
C 1998 Mosby-Year Book. Inc. Photocopying is prollibited by law.
Diagnostic Imaging alld Recordings
57. Detail is rhe degree of sharpness ofan object on a radiograph. Which ofthefoflowillgdoes not affect detail?
a. b. c. d. e.
roca1 Spot size object-film distance film-screen contact foca l spot-object distance milliamperage
58. You are presented witlt a 6·momlt-old Irish seuer with bilareralforelimb lameness persisringfor 1 month. You note swelling of the carpal and tarsal joints. On taking a radiograph ofone carpus, you note a region ofdecreased opacity in the metaphysis of tile radius and ulna and regions of soft tissue calCification circumferenriallyaround tlte radial and ulnar metaphyses. The most likely cause of these findings is:
a. b. c. d. e.
61. You take radiographs ofllle stifle alan 8-year-old dog with a 5 -week history ofsrifle swelling and pai1l. The radiographs show joint effusion and mulliple subchondral and periarticular erosive regiollS. Based 011 the radiographic signs. what is the least likely diagnosis?
a. b. c. d. e.
septic arthritis rheumatoid arthritis degenerative joint disease synovial sarcoma lupus erythematosus
62. Active periosteal reaction 011 the pafmarolateral and palmaromedial aspeC1S oflhe proximal phalanx in a horse is most caused by abnormal fJlIllingofthe:
a. suspensory ligament b. c. d. e.
scurvy panosteitis osteosarcoma hypertrophic osteodystrophy blas tomycosis
superficial digital flexor tendon deep digital flexor tendon impar ligament middle sesamoidean ligament
63. YOII are presenced with a 6-year-old collie with a
59. A 3-month -old dogfallsfrom a height of 15 feet and lands on all four feet. I1lere are 110 fractures , but the injury crushes one of the distal ulnar physes. Wlilch ofthefollowing is least likely to be observed 011 radiographs made 4 m01l111s later?
a. valgus deformity of the manus b. subluxation of the humeroulnar articulation c. fragmented medial coronoid process d. ununited a nconeal process e. subluxation of the ulnar-carpal articulation 60. Which radiographiC sign is wast likely to be associated wirh afragmellted medial ulnar coronoid process?
a. new bo ne fonnation on the proximal margin of the anconeal process b. incongruity (subluxation) of the humeroradial articulation c. a cleavage line between the coronoid process and the ulna d . degenerative jOint disease of the e lbow e. lysis of the proxima1 aspect of the ulna
71
2-momh history ofa serosanguineous discharge from the left naris. Wlliell radiographic view would most accllrareiyassess the nasal cavity?
a. b. c. d.
ventrodorsal view of the entire nasal region dorsoventral view of the entire nasal region rostrocaudal view of the ironta1 sinuses dorsoventra1 view of the nasal region using intraoral film e. velllrodorsal view ofthe nasal region using intraoral film
64. The hallmark radiographic sign of diskospondyliris is:
n. spondylosis b. c. d. e.
osteophyte formation venebral body end-plate lysis narrowed intervenebra1 disk space subluxation of the verte bral column
Correct answers are on pages 92· 100.
www.vet4arab.co.cc
SECTION 3
72 65. Wh ich althefallowing is not a common radiographic sign o[vertebraf mafalignmentrnalarticulation (wobbler) syndrome jn dogs?
a. b. c. d. c.
stenosis of the vertebral canal spondylosis intervertebral disk mineralization osteophyte formation on articular processes vertebral body end-plate lysis
69. You are presented witll a 7-yea r-ofdgofden retriever exhibiting weakness, regurgitation, arul coughing. You rake a radiograph ofthe thorax and note generalized megaesophagus, ventral alveolar lung disease, and a cranial mediastinal11UlSS ventral to the trachea. The mediastinal mass is m ost likely:
a. an esophageal tumor b. a thymoma
c. a lymphosarcoma d. a rib tumor
66. In dogs, at wllich site is herniation o[an
e. a hematoma
imeruertebraf disk least likely to occur?
a. C I-2
70. Which parenchymal pattern is commonly observed on radiographs of dogs with bacterial pllewllonia?
b. C3-4
c. T13-Ll d.13-4 e. L7-S1
a. bronchial
67. Which radiographic sign is least likely to result from hemiatioll a/an interuertebral disk? a. vertebral body end-plate lys is b. narrowing of lhe intervertebral disk space c. narrowing of the s pace be tween a rticular
71. /n dogs the right cran ial lobar pulmonary artery and vein are best seen on which radiographic projection?
processes d. mineralized material in the vertebral canal c. sclerosis ofvcrtebral body end-plates 68. YOIl are presented with a 2-year-old German shepherd soon after it is hit by a car. T1lere is pelvic limb paralysis, and you suspect a lumbar spine fracture. You 1I0te riO abnormalities on a lateml radiograph made with the dog ill left lateral recllmbencyand a vertically d irected x-ray beam. The most appropriate course ofaction is to:
a. perfonn a hemilaminectomy from Ll through L4 b. hospilalize the dog for observation c. use a horizontally directed x-ray beam and make a ventrodorsal projection of the lumbar spine with the dog in lateral rC(:umbency d. gently roll the dog into right late ral recumbency and make another lateral radiograph using a vertically directed x-ray beam e. use a horiw ntally directed x-ray beam and make a nother lateral projection of the lumbar spine after gently rolling the dog into dorsal recumbency
Q 1998 Mosby-Year Book,
b. interstitial c. alveolar d. vascular e. mediastinal
a. b. c. d. e.
left laleral right lateral dorsoventral ventrodorsal standing latera!
72. Which condition results in a rigllt cranial lobar vein that is larger than the right craninllobar artery?
a. b. c. d. e.
pulmonic stenosis pulmonic insufficiency mitral insufficiency heartworm disease aortic stenosis
73. W1lich ofthe following is not a common radiograpllic sign in dogs with left heart failure caused by mitral regurgitation?
a. b. c. d. e.
enlarged left atrium enlarged pulmonary veins pulmonary edema pleural effusion elevated tracheal bifurcation
Inc. Photocopying is prollibited by law.
Diagnostic Im aging and Recordillgs
74. You are presellled witli a 6-momh-ofd male Centum shepherd for vaccination. During routine physical examinarion)'Ou notea systolic lieart murmur. On radiographs of the thorax you note elongation ofthe cardiac silhouette, enlargemem of the main pulmonary artery segment, enlargement ofthe left atrium, and enlargement ofparenchymal pulmonary arteries and veins. The most likely calise ofthese findings is:
a. b. c. d. e.
patent ductus arteriosus subaortic ste nosis pulmonic stenosis ventricular septal defect congenital mitral insufficiency
Questions 75 and 76 You are preserlted with a 12-year-old miniature poodle witli a 6-month historyofa "honking" cougli. You SIlSpect collapse of the intrathoracic trachea. 75. To increase )'our clianGeS ofdetecting this a bnormality with conventional radiographs, YOIl should make the exposure:
a. b. c. d. e.
at the e nd of expiration at thc e nd of inspiration \vith the dog under gene ral anesthesia \vith the neck flexed at both the end of expiration and the end of inspiration
76. 011 thoracic radiographs of this dog,YOIl also !lote gas accllmulation within the midthoracic esophagus. The most appropriate course ofaction is to:
a. perform esophagography using barium paste b. perform esophagography using liquid barium c. perform no other diagnostic procedures, because this is an acceptable finding in this patient d. repeat the thoracic radiographs and see if the gas is stillihere e. perform esophageal endoscopy
73
77. Which oflhefollowing is the least reliable radiographic sign of pleural effusion?
a. retraction of the lung away from the thoracic wall b. inability to visualize the cardiac silhollctte c. scalloped soft tissue opacity dorsal to the sternum on the lateral view d. blunted costophrenicsuJcus e. elevation ohhe heart from the sternum on the lateral view 78. Concernillga dog Wilh a farge voillmeofflllid i/l the pleural space, which statement is least accurate?
a. The fluid should be obvious on vcntrodorsal and dorsoventral radiographs ofthe thorax. b. The fluid should be obvious on eith er a right or left lateral radiograph of the thorax. c. The heart should be bette r seen on a dorsoventral rathe r than a ventrodorsal radiograph of the thorax. d. The thoracic surface of the dia phragm will be impossible to assess completely in radiographs. e. The lung parenchyma should be increased in opacity because of partial collapse. 79. You take a radiograph of tlie thorax ofa lO-year-old
dog with a6-week historyofdYSP1lea and weight loss. 71le cardiac silliouette is greatly elliarged and has a spheric shape. What is the least likely calise of this radiographic appearance of the heart?
a. b. c. d. e.
pcricardial -peritoneal diaphragmatic hernia pericarditis heart-base tumor heartwo rm d isease pericardia! tumor
80. Masses within tlte IImg parenchyma are frequently not apparefll Oil radiographs made with the affected lung in a dependent positioll. S ilch obscuring ofa mass is related to:
a. b. c. d. e.
increased air volume in the dependent lung decreased air volume in the dependent lung increased flu id volume in the dependent lung decreased flui d volume in the dependent lung compreSSion of the mass in the dependent lung
Correct answers are on pages 92-100.
81. You take a radiograph of tile tflOrax ofa 3-year-ofd cat with a 2-montIJ "isforya/a productive (mucous) cough. You observe three cavitary lung nodules, about 1.5 em in diameter; in the caudal lung lobes. TIle most likely cause a/these findings is:
a. Paragonimus infection b. Aleurostrongylus infection c. primary lung tumor d. metastatic lung tumor e. pulmonary thromboembolism 82. COllcemiflg pulmollary metastasis. which statemenl is least accurate?
a. It often res ults in interstitial nodules. b. It may produce an unstructured interstitial lung pattern. c. It usually does not result in only one visible mass. d. It often produces coughing early in its development. e. It is often misdiagnosed when summation shadows created by vessels overlying ribs are seen. 83.
www.vet4arab.co.cc
SECTION 3
74
Concerning splenic torsion, which statemem is least accurate?
a. It does not occur wilhout gastric torsion. b. II frequentl y causes splenomegaly. c. II frequently results in an abnonnal splenic shape. d. II may resuil in peritoneal effusion. e. II may result in displacement of the duodenum.
85. Concerning emphysematous cystitis, which statemelll is least accu rate?
a. It is often associated with bacterial cystitis. b. II cannot be detected on survey abdominal radiographs. c. It can be secondary to diabetes mellitus. d. It can resull in pneumoperitoneum. e. It is often associated with hematuria. 86. You are presetlted with a lO-year-oldfemale German shepherd that straim on attempts to defecate. Another veterinarian had removed an unidelltified mass from the anal region 2 momhs previously. On pe/vic radiographs you note a large mass in the caudodorsal abdomen , ventral to LS-7 and compressing tile cololl. It is most appropriate to tell theowIler that the:
a, dog has cancer b. mass probably represents an enlarged lymph node c. mass probably represents a colonic tumor d. mass is normal e. mass should be treated with radiation therapy 87. In cats, which of the following is least likely to impair visualization of serosal margin detail on abdominal radiographs?
a. b. c. d, e.
steatitis right hean failu re emaciation peritoneal d ialysis ruptured urethra
88. TIle major colltrail.dicatioll to negative-contrast
84. Which ofthefollowing is the least common radiographic sigll associared with chronic renal failure?
a. b. c. d. e.
mineralization of the gastric rugae patchy alveolar lung disease mineralization of the foot pads small, mineralized kidneys hydronephrosis
cystography using room air as thecol/trast medium ill a cat witll hematuria is:
a. b, c. d. e.
the possibility of inducing bladder infection the possibility of inducing bladder rupture high cost the possibility of inducing air embolism the possibility of inducing vesicoureteral reflux
Diagnostlc Imaging and Recordings
89. COflcerningexcretory urography in a dog with normal renal junction, using water-soluble, ionic, iodinated contrast medium, which statement is least accurate?
a. This type of contrast medium is excreted by glomeruJar filtration. b. This type of contrast medium is secreted by the renal tubuJes. c. This type of contrast medium is concentrated by tubular resorption of water. d. Alternate organs of excretion of this type of contrast medium are the liver and gasnointestinal mucosa. e, This type of contrast medium is delivered to Ule kidney by the renal artery. 90. A 6-year-old male German shepherd, hit bya car 1 hour prelliously, has hematuria. Abdominal radiographs show a loss of serosal margin detail and no visible urinary bladder. No fluid is obtained on abdominocentesis. Tile most appropriate course ofaction is to:
a. tell the mvner the bladder is ruprured and explore the abdomen surgically b, tell the owner the bladder has not been ruptured because the dog is urinating and no flu id was obtained on abdominocentesis c. perform excretory urography to assess the bladder d, perform positive-contrast cystography to assess the bladder e. perform negative-contrast cystography to assess the bladder 91. Mlat is the most appropriarecontrast medium for positive-contrastcystographx if! dogs?
a. b, c, d,
barium sulfate paste barium sulfate solution air nonionk, iodinated, water-soluble contrast medium e. ionic, iodinated, water-soluble contrast medium 92. TIle best radiographic contrast procedure to assess tile urinary bladder mucosa is: a. excretory urography b, positive-contrast cystography
@ I998Mosby-YearBook,Inc, Photocopying is prohibited by law.
75
c. negative-contrast cystography d. double-contrast cystography e. retrograde urography 93. The contrast medium of clloke for a negatiueCOntrast cystogram is:
a. room air b. carbon dioxide
c. helium d. argon e. 100% oxygen
94. Itlnegative-contrast cystography, urine should be withdrawn before injection of the COntrast medium to prevent:
a, b. c. d. e,
bladder ruptu.re vesicoureteral reflux vesicourelhral refl ux air embolism bubbles on the radiograph
95. Extravasation ofCOntrast medium from tile urethra imo the parellchyma of the prostate gland during retrograde positive-contrast urethrography indicates:
a. b. c, d. e.
neoplasia nothing of diagnostic significance p rostatitis hyperplasia a normal prostate gland
96, On radiographs ofa 9-year-old male German shepherd with chronic hematuria, you observe two distinct spheric, large (8 cm diameter), soft tissue masses if! the caudoventral abdomen. The most reasonable conclusion is that:
a, the cranial mass is the bladder and the caudal mass is an enlarged prostate gland b. the caudal mass is the bladder and the cranial mass is an enlarged prostate gland c. exploratory surgery shouJd be performed d, retrograde positive-contrast urethrography should be performed e. this is a normal finding and there is no need for further studies or treatment
Correct answers are on pages 92-100,
www.vet4arab.co.cc
SECTION 3
76 97. What is the major radiographic difference between the canine and feline stomach? a. b. c. d. e.
more prominent rugal fold s ill dogs more prominent rugal folds in cats more medially positioned pylorus in dogs more medially positioned pylorus in cats more laterally positioned fundus in cats
98. You make left lateral and uentrodorsal abdominal radiographs ofa 3-year-old German shepherd wilh a J -day history ofabdomjnal pain and discemion. YOllnote extreme dilation ofrhe stomach witllfluid and gas, bill you cannot specifically idemify che fundic and pyloric regions of the stomach The most appropriace action at tllis time is 10:
a. rehydrate and observe the patient for 24 hours b. give 500 m.I of barium sulfate PO and repeat your attempt to identify the fundu s and pylorus c. make right lateral and dorsoventral radiographs of the abdomen in an attempt to identify the fundus and pylorus d. perform an exploratory laparotomy e. perform a complete upper gastrointestinal examinatio n
99. During all IIpper gastrointestintll radiographic examinarioll,you note two crater-shaped outpollcl!ings of barium from the Qntimesenteric
surface ollhe descendingduodenlltn ofa 4-yearold German sllepherd willi a history ofchronic melena. These outpouchings most likely
represent: a. a normal finding b. neoplasia
c. phycomycosis d. peptic ulcers e. areas of infarction 101). Water-soluble, iodinated, ionic contrast medium
can be used for lipper gastrointestinal radiograpl/icexaminatiolls. W1/Ut is the most important contraituiication to use of these agents in a dog with sewre diarrhea?
a. b. c. d. e.
high cost length of time involved poor mucosal detail exacerbation of fluid-electrolyte imbalance irritability to the gut mucosa
ULTRASONOGRAPHY Recommended Reading
Burk RL. Ackerman N: Small (.IlIimal radiologyalld u/rrasonogrnplly, ed 2, Philadelphia, 1996, WB Saunders. Cartee RE et a1: Practical veterinary ultrasound, Baltimore, 1995, Williams &Wukins. Green RW: Small animal ultrasound, Philadelphia, 1996, Lippincott-Haven. Nyland TG, Mattoon IS: Veterinary diagnostic ultrasound, Philadelphia, 1995, WB Saunders. RcefVB: Equine diagnostic ultrasound, Philadelphia, 1997, WB Saunders. Reimer 1M: Atlas o/equine ultrasonography, St. Louis, 1998, Mosby Practice answer sheets are on pages 259-260.
Diagnostic Imaging and Recordings
Questions JA Hudson and S.T. Finn-Bodner 101. Concerning choice ofa 7.S-MHz or a S.D-MHz
probe when performing diagnostic ultrasound, wllicll statement is most accurate?
a. Either probe can be used, but the probe should not be changed during the examination. b. High-frequency probes provide better penetration but poorer axial resolution than low-frequency probes. c. Low-frequency probes provide more penetration than high-frequency probes and may be required to examine deeper structures adequately. d. AJways use as Iowa frequency as possible to provide better axial resolution. e. Use lower-frequency probes when possible because probes with frequenCies greater than 10 MHz increase the risk of biohazards. 102. Concerning acoustic impedance as related
co
ultrasonography, which statement is most accurate?
a. Ultrasound is reflected when there is an acoustic impedance difference between two adjacent s tructures. b. The acoustic impedance of a tissue can be altered by changing depth (time) gain compensation. c. Structures with a high acoustic impedance reflect ultrasound and are anechoic. d. Acoustic impedance describes the intensity of ultrasound. e. Acoustic impedance describes the frequency d wavelength of ultrasound produced by an ultrasound transducer. 103. Concerning tile relative differences in echogenicity ofabdominal organs in dogs and cats, which statement is most accurate?
a. The renal cortex is more hyperechoic than the spleen. b. Relative echogenicity is affected by such factors as probe frequency and interposing of other organs between the probe and the organ being imaged.
e 1998 Mosby-Year Book, Inc. Photocopying is prollibited by law.
77
c. The renal medulla is more echogenic than the renal cortex and Liver. d. The spleen is generally more hypo echoic than other abdominal organs, such as the liver and renal cortex. e. Echogenicity of structures in the near field can be compared with that of structures in the far field as long as depth gain compensation is used to produce a uniform image. 104. The kidney isa complex organ comprised of capsule, cortex, medullary pyramids, pelvis, and pelvic diverticula. WI/icll ofthefollowing best describes tile relative ecllOgenicity of tile cortex, medullary pyramids, and pelvis/pelvic diverticula, from the most lzyperec/lOic co the least Izypereclloic? Assllme tlull the kidney is normal and the animal is not undergoing physiologic diuresis.
a. cortex > medullary pyramids >pelvis and diverticula b. pelvis and diverticula> medullary pyramids > cortex c. pelvis and diverticula> cortex > medullary pyramids d. medullary pyramids> pelvis and diverticula > cortex e. The kidney has a uniform hypoechoic appearance, with differences in echogenicity seen only with pathologic changes. 105. Concerning the ultrasonographicappearance of
tile normal canine kidney, which statemerlf is most accurate?
a. The renal cortex is hyperechoic as compared with the spleen. b. The renal medulla is hyperechoic as compared with the renal cortex. c. The renal cortex of obese cats has reduced echogenicity. d. Echogenicity of the renal medulla is similar to that of the liver. e. Echogenicity of the renal cortex is similar to that of the liver.
QJrrect answers are on pages 92· 100.
78
www.vet4arab.co.cc
SECTION 3
106. Bright dOl$ at thecorticomedullary junction in gray-scale images ofa canine kidney usually represent:
a. normal mineral deposits of no clinical significance h. arcuate arteries c. afferent arterioles d. abnormal mineraJ deposits that warrant further investigation
e. hippomanes 107. You obtain an ultrasound image ofa renal calculus. The area deep to the calculm is completely black. IN'llat term is used to describe tltis black artifact?
a. acoustic enhancement b. off- incidence angle artifact c. slice thickness artifact d. refraction artifact e. acoustic shadowing lOB. Concerning the ulrrasonographic appearance of the normal canine spleen, which statement is most accurate?
a. Echogenic flow in large splenic veins is an abnomlal finding requiring further investigation. b. The splenic capsule cannot be visualized because it has the same echogenicity as the splenic parenchyma. c. The spleen has the same echogenicity as the liver. d. When imaged oblique to the ultrasound beam, splenic veins are characteristically anechoic to hypoechoic, with poorly delineated walls. e. Splenic arteries and veins aTe not usually visible. 109. Which statement best describes the ulrrasonographic appearance of the wall of the small intestine in a normal dagar cat?
c. The muscularis is hyperechoic and is the thickest layer. d. The muscularis and serosa can only be distinguished when pathologic changes are present. e. The wall is composed of five equally thick layers. 110. Which statement best describes the ultrasanographic appearance of the luminal cOrllents of the small intestine?
a. When fluid is present, the mucosal layers of the near and far walls cannot be distinguished; the black mucosal layers ~b l end" with the black fluid-filled lumen to fonn a single black layer. b. Mucus in the lumen is hyperechoic and does not allow imaging of the far wall. c. Gas in the lumen causes acoustic enhancement, a phenomenon in which deeper structures are bright because of reverberations between gas and the probe. d. Intraluminal gas cannot be differentiated from intraluminal mucus. e. Fluid and mucus allow imaging of the far wall of the intestine. 111. Considering ultrasound imaging of the urinary bladder, which statement is most accurate?
a. A catheter should be placed and the bladder maximally distended before imaging. b. Acoustic shadowing is frequently seen deep to a normal bladder. c. The slightly distended bladder wall exhibits five layers of alternating echogenicity: serosa, muscularis, submucosa, mucosa, and mucosal surface. d. Ultrasonography, rather than cystography, should be used to evaluate the cranial border (apex) of the bladder because this area is clearly visualized with ultrasonography but not with cystography. e. Acoustic enhancement is frequently seen deep to the urinary bladder.
a. The serosa, submucosa, and mucosal surface are hyperechoic. b. Only the serosa is hypoechoic; all the other layers are hyperechoic.
© 1998 Mosby-Year Book, Inc. Photocopying is prohibited by law.
Diagnostic Imaging and Recordings
112. Which statement best describes the appearance of the canine prostate gland in ultrasound images?
a. The prostate gland is a peanut-shaped organ best identified by imaging craniomedial to the prostatic artery. b. Each prostatic lobe appears as a hypoechoic oval, with a hyperechoic line located centraUywithin the lobe. c. The prostate gland is less echogenic than the renal medulla. d. Intraparenchymal prostatic arteries appear to have hyperechoic walls, particularly when imaged at right angles to the ultrasound beam. e. The prostate has a uniform, somewhat coarse echogenicity. 113. Concerning the ultrasonographic appearance of the normal canine uterus, which statement is most accurate?
a. During estrus, the uterus in longitudinal section is a homogeneous hyperechoic tube \'lith an anechoic lumen. b. The anestrual uterus is difficult to distinguish from normal small intestine because both have a five-layered wall and a fluid-filled lumen. c. The walls of the normal small intestine and postpartum uterus are identical, with two layers evident. d. Pregnancy can be diagnosed in dogs by ultrasonographic visualization of the chorionic cavity at 17 to 20 days of gestation, after the luteinizing hormone peak:. e. The uterus can be differentiated from the aorta because the aorta bifurcates cranially, whereas the uterus bifurcates caudally. 114. Concerning the pancreas ofa normal dog, which statement is most accurate?
79
c. The ph renicoabdominal vein serves as a vascular landmark. d. The pancreas can only be seen when imaging from the left side of the abdomen; the ascending colon prevents visualization on the right. e. The pan creas is normally hyperechoic, wilh a hypoechoic central core. 115. COllcerning the lllrrasollograpliic appearance of the gallbladderalJd biledllcts inllealthydogs and cats, which statement is most accurate?
a. The gallbladder has a thick hyperechoic wall. b. The common bile duct is less than 2 mm wide and is difficult to visualize on its course to the duodenum. c. The feline gallbladder is difficult to visualize because it is usually empty. d. Cats have large cystic and common bile ducts that are easily visualized, unlike those of dogs. e. The lumen of the gallbladder is hyperechoic. 116. Concerning ultrasonograplJicartifacts associated with the normal liueror gallbladder, which statement is most accurate?
a. Distal acoustic enhancement occurs when sound is repeatedly reflected between the lung/diaphragm interface and the hepatic parenchyma. b. Mirror-image anifact causes vertical black streaks extending from the curved surfaces of the gallbladder into the hepatic parenchyma, deep to the gallbladder. c. Distal acoustic shadowing causes an artifactual increase in echogenidty of the liver, deep to the gallbladder. d. Distal acoustic enhancement occurs deep to the gallbladder because of minimal attenuation of ultrasound by fluid in the gallbladder. e. Artifacts are only seen when the liver or gallbladder is abnormal.
a. The pancreas is easily imaged because it is an anechoic organ surrounded by hyperechoic mesentery. b. In acute pancreatitis the pancreas is hypoechoic and enlarged.
Correct answers are on pages 92-100.
www.vet4arab.co.cc
SECTION 3
80 Questions 117 and 118
You perform an ecllOcardiographic examination on a 2-year-old, S-kg male domestic shortlzair cat with a grade-ll systolic murmur over the mitra/llalve area and a diastolic gallop rhythm. Findings afM-mode ecilOcardiography include ti,e following:
Left arrial diameter 15 mm (normal, 8.5 to 12.5 mm) uft l!entriCllfar imemal diastolic diameter 10 mm, (normal, 11 to 16 mm)
u[t ventricular waU diastolic diameter 6.5 mm (normal. 2.5 to 5 mm) IflferlJelltriwlar septum diastolic diameter 7 mm (normal, 2.5 to 5 mm) FractiOllal shortening 70% (normal, 29% to 55%) Two-dimensional examinatioll shows asymmetric tllickeningofthe interventricular septum at che base, narrowing of the left ventricu{aroutftow tract in systole (subaortic), and cranial molioll aftlle mitral valve crallial leaflet toward the septum during systole. The mitral valve leaflets are thichmed. Color Doppler eualuatio" sllOws turbulence ifl tile left vemricular outflow velltral to tlleaorcic valve and a mitral regurgiltlllrjer illto the left atrium. 117. Wllar is tile most likel), cause of these
eclwcardiograpllic fi ndi ngs?
a. mitral valve endocardiosis and regurgitation and secondary left ventricular hypertrophy b. congenital hean disease, with subaortic stenosis and left ventricular hypertrophy c. thyrotoxic cardiomyopathy d. hypertrophic cardiomyopathy (obstructive) e. mitral valve vegetative endocarditis, chordae tendineae rupfUre, and st'Condary left ventricular hypertrophy 118. What is tile most appropriate treatmemfor this condition?
a. administration of furosemide and digoxin b. adminisrration of methimazole, with thyroidectomy or use of radioactive iodine c. antimicrobial treatment based on urine or blood culfUre and sensitivity tests, and administration of an angiotensin-converting enzyme inhibitor to decrease preload and afterload
d. septal myectomy e. administration of a ~ -b lo cke r or calciumchannel blocker 11 9. You obtain a trallsverse ultrasound image of the flexor surface ofa horse's forelimb ar rile level of tllejunctioll of the proximal and middle thirds of rile metacarpus. Wlljc/I of the following accurately lists tile structures present in the image, ill order from superficial (closest to the probe) to deep (farthest from the probe)?
a. superficial digital flexo r tendon, deep digital flexor tendon, interosseous (suspensory) ligament b. superficial digital fl exor tendon, deep digital flexor tendon, inferior check ligament, interosseous (suspensory) ligament c. superficial digital flexor tendon, deep digital flexor tendon, interosseous (suspensory) ligament, inferior check ligament d. superficial digital flexor tendon , inferior check ligament, deep digital fl exor tendon, interosseous (suspensory) ligament e. superficial d igital flexor tendon. deep digital flexor tendon, inferior check ligament 120. Concerning the llltrasonograpilic appearance of a normal canine eye imaged Willi a 7.5- or 10MHz trallsdllcerand a standoffpad (orojfset), which statement is most accurate?
a. The lens appears as an elliptiC hyperechoic structure between the anechoic anterior chamber and anechoic vitreous chamber. b. The vitreous chamber is hyperechoic, whereas the aqueous anterior and posterior chambers are anechoic. c. Severe retinal detachment is manifested as Vshaped echogenic lines with points of aHachment at the optic disk or ora serrata. d. The optic nelVe appears as a uniformly hyperechoic tubular structure; the optic disk appears as an anechoic structure at the caudal aspect of the globe. e. The ciliary body cannot be visualized with ultrasound.
Diagnostic Imaging and Recordillgs
81
E.R. Wisner 121. Ultrasound waves propagating through tissues are reflected to a greater degree as the difference in the acoustic impedanceofadjacem tisslles increases. Theacollstic impedance ofa tissue increases with:
a. increaSing tissue density and increasing ultrasound propagation velocity b. decreasing tissue density and increasing ultrasound frequency c. increasing tissue density and decreasing ultrasound propagation velocity d. decreasing tissue density and increasing ultrasound propagation velocity e. increasing tissue density and decreasing ultraso und freq uency 122. As transducer frequency increases (e.g., from 5 to
10 MHz):
a. b. c. d. e.
spatial resolution of the image is degraded image contrast increases spatial resolution ohhe image improves ultrasound propagation velocity decreases ultrasound propagation velocity increases
123. Ultrasound images are prodl~ced b)' transmitting ultrasound waves of a gillen frequency into tissue in siron bllrsts or pulses. Each pulse con tains a fixed number of ultrasound waves or cycles. Tile axial resolution ofall image improves (sma ller objects can be detected} wlrell:
a. the transducer frequency is increased and the number of cycles per pulse is decreased b. the sound wave amplitude is increased and the number of cycles per pulse is increased c. the transducer frequency is decreased and the sound wave amplitude is increased d. the transducer frequency is increased and the sound wave amplitude is decreased e. the sound wave amplitude is increased and the number of cycles per pulse is decreased
124. Wllich of tile following best describes the basis for
prodl1Ction ofa tIoo-dimensional ultrasound image?
a. Changes in the ultrasound wave velocity are measured as the echoes propagate through different tissues. b. Changes in ultrasound wave frequency are measured as they propagate through d ifferent tissues. c. The amplitude of transmitted ultr.lsound waves is measured as the waves propagate through different tissues. d. Changes in ultrasound wave attenuation are measured and are determined by viscosity, stiffness, and scattering properties of different tissues. e. The time requ ired for reflected ultrasound waves to return to the transducer is determined, and a constant velocity of wave propagation within the tissues is assumed. For Questions 125 through 129, select the correct answer from the five choices below. a. b. c. d. e.
gain pulsed Doppler power B-mode time-gain compensation
125. Metllod ofde rem lining the velociryofmoving StruCtllres, sllch as flowing blood, using the magflimde oftile frequency shift ofreflected SOllnd
waves 126. Ultrasound control that uniformly regulates tile
amplitude of ultrasound output 127. Uniform electronic amplification ofall echoes
returning to the trallsducer 128. Comparatively greater increase in electronic amplification of sound waves rerumingfrom deeper (more distallt) structures 129. UltraSOllnd imagegellerared by return illgechoes as dots wllose brightness is proportional to the amplitude of the returning echo and whose position corresponds to the depth at wllich rile echo origillated
@I998 Mosby-YearBook,Inc.Phorocopyingisprohibitedbylaw.
Correcr answers are on pages 92-100.
www.vet4arab.co.cc
SECTION 3
82
130. Wlliell transducer is most appropriate for examining a small, superficial structure, such as a thyroid gland in a dog?
a. 3.0-MHz mechanical seclor-scanning transducer b. to-MHz linear-array transducer
c. 7.S-MHz mechanical sector-scanning transducer
d. S.O-MHz phased-array transducer e. 3.5-MHz linear-array transducer 131. Mlich strJIcture is mosllikely to produce a slladowingarrifacl in arl ultrasound image?
a. a urinary bladder calculus b. a renal/sple nic interface that is not perpendicular to the ultrasound beam c. a complex splenic mass d. a bile-distended gallbladder c. a fluid -distended small intestine 132. Which strrlctllre is most likely to produce distal enliaflcemem jn all ultrasound image?
a. b. c. d. e.
re nal cortical mineralization a gas-filled s tomach lead buckshot a renal pelvic calculus a hepatic cyst
133. Wlu'cll of tile following best describes the cause of mirror-image artifacts in an ultrasound image?
a. return of multiple, nonperpendicularuitrasolUld beam reflections to the transducer, resulting in duplication of structures in the image b. reflection of excessive ultrasound waves back to the transducer by homogeneous £issues in lhe imaging field c. mismapping of reflected ultrasound waves because of excessive fat in the imaging field d. m ismapping of reflected ultrasound waves from previous pulse repetitions when a high pulse repetition frequency is used e. multiple reflections betw"een nvo closely spaced tissue interfaces, producing a series of parallel lines within the image
@
134. Wlliell of the following accurately lists parenchymal organs in normal dogs, from most echogenic (briglltest) to least eciJOgenic (darkest)?
a. b. c. d. e.
renal cortex,. spleen ,. renal medulla,. liver liver ,. renal medulla ,. renal cortex" spleen spleen ,. liver,. renal cortex,. renal medulla spleen ,. renal cortex,. renal medulla,. liver renal medulla ,. liver " spleen ,. renal cortex
138. Which afthefollowing is most likely co be observed during ultrasound examination o/tlle abdomen ofa ctlt with lymphoma affecting the introobdominallympllnodes?
a. uniformly hyperechoic nodes, with signi.ficant distal enhancement b. unifonnly hyperechoic nodes, with linle or no distal enhancement c. uniformly hypoechoic nodes. with little or no distal enhancement d. complex lymph node internal architecture that is hyperechoic centrally and hypo echoic peripherally e. moderately echogenic nodes, wi th a coarse. heterogeneous echotexture
135. Wilich disorder is most likely to produce a generalized increase inliverechogenicity (brighmess)?
a. b. c. d. e.
Diagnostic Imaging and Recordings
acute hepatitis hepatic lipidosis biliary obstruction hepatic venous congestion multiple hepatic cysts
139. 1n which disorder would echocardiograpllic examination be least we/ul for definitive diagnosis?
a. b. c. d. e.
lefl -sided congestive heart failure pericardia! effusion dilative cardiomyopathy bacterial valvular endocarditis aortic stenosis
140. Wilat is the 1I0rmalieft IJentricular percelll fractional shortening ill flnsedated dogs?
a.
15%
b. 35%
c.
55%
d.70%
c. 85%
ENDOSCOPY
136. Which disordero/ten produces a marked illcrease ill renal cortictll echogenicjty (brightness)?
a. b. c. d. e.
83
Recommended Reading
renal lymphoma acute leptospirosis polycystic renal disease chronic obstructive hydronephrosis acute ethylene glycol tOxicity
BrcarJeyMJ et al: Colorat/aso/smaJianimal endoscopy, St Louis. 1991. Mosby. Tams TR: Small animal endoscopy. ed 2. 5t Louis. 1998, Mosby.
Traub-Dargatz It. Brown eM: Equine endoscopy. ed 2, 5t Louis, 1997, Mosby. Practice answer sheeg. .ilre on pages 259-260.
137. Wilich vascular structures within the normal liver have a circular or linear anechoic core representing the vessel lumen and a hypereclJOic margin representing the vessel wall and surrollnding connectilJe tissue?
a. hepatic vei ns b. portal veins c. hepatic arteries d. bile ducts e. lymphatics
1998 Mosby-Year Book. Inc. PllOtocopying is prohibited by law.
•
Questions P.w. Pratt
141. In a fiberoptic endoscope:
a. a coherent fiber bundle transmits light from the light source to the distal tip b. a noncoherent fiber bundle transmits images from the distal tip to the eyepiece c. about 80% of the light from the light source has been dissipated when it reaches the distal tip d. each glass fiber is clad with a substance of low refractive index e. water is circulated around the fiber bundles to reduce heat
142. Most pediatric gastrointestinal fiberoptic endoscopes designed for use in human infants:
a. have four-way tip deflection. with one angle deflecting at least 180 degrees b. are equipped with an operating or accessory channel 7.8 to 10 mm in diameter for passage of diagnostic accessories c. have an umbilical cord containing one coherent and tw"o noncoherent fiber bundles d. have an insertion tube 40 to 50 cm long e. are not equipped with a vacuum device
Correct answers are on pages 92-100.
84
www.vet4arab.co.cc
SECTION 3
143. For Japaroscopy ill which intraabliominai
electrocoagulation willlifcely be used, thegas of choice for insufflation is:
a. air
d. Narcotics can be given to decrease pyloriC and cranial duodenal tone. e. Food should be withheld for 72 hours before endoscopy.
151. eoncemingcolonoscopic examination of small animals using a flexible jiberoptic endoscope, which statement is most accurate?
a. The distal tip of the endoscope is inserted blindly for 10 cm, after which the colon is insufflated with air. b. It is not possible to pass the distal tip of the endoscope cranial to the ileocolic valve. c. The e ndoscope should be slowly withdrawn to fa cilitate careful visual examination. d. Biopsy of lesions in the hepatic flexu re of the colon can be done with alligator forceps or uterine biopsy forceps. e. General anesthesia is contraindicated because it significantly alters the appearance of the colonic mucosa.
h. carbo n dioxide 148. During gastric endoscopy. the pyloric antrum and pylorus are in view but tile endoscope tip cannot be advanced, or it repeatedly flips over the angularis toward the cardia. What is a,e usual cause of this phenomenon?
c. nitrous oxide d. oxygen e. hydrogen
144. Which condition in smaff animals is least likely to he diagnosed by IIpper gastrointestinal endoscopy? a. esophagitis h. pyloric hypertrophy
c. hypertrophic gastritis d. abnormal motility e. gastric carcinoma
gastric overinsufflation gastric mural disease atrophic gastritis linear forei gn body traversing the pylorus anticholinergic overdosage
149. Wllat is tile major reasoll for unproductive or unsuccessful colonoscopic examination?
145. A coll trail1diootion/or IIpper gastrointestilWl
endoscopy is: a. h. c. d.
a. b. c. d. e.
hematcmesis esophageal stricture bowel perforation chronic vomiling
e. hypersecrctory disorder 146. Which phenomenon is observed whetl tlte
endoscope is passed into tile small intestine oJ small-breed dogs?
a. bradycardia h. gastric dilatation-volvulus
c. pyloric nutter d. rectal prolapse e. gallbladder evacuation
a. b. c. d. e.
152. A contrai"dicatiorl for Japaroscopy is:
a. b. c. d. e.
bowel perforation iatrogenic intussusception inadequate bowel cleansing underinsufflation of the bowel at resia ani
150. Concerning preparation of small animals for co!Otloscopy. which statemem is most accurate?
a. Magnesium citrate should not be given to cats fo r colonic cleansing. b. Polyelhylene glycol should not be given to dogs for colonic cleansing. c. Several warm-water enemas are superior to oral electrolyte lavage solutions. d. Metoclopramide is given to stimulate vomiting and delay gastric emptying. e. Enema solution should be repeatedly infused at 75 ml/kg.
147. Concerning upper gastroilltestillal endoscopy in small animals. wllicll statement is most accurate?
a. Physical restraint is usually sufficient. b. Anticholinergic premedication can be used to reduce gastric motility and secretion. c. Patients typically are placed in right latera] recumbe ncy so thai gastric juices gravitate toward the pyloric antrum.
@ 1998Mosby-YearBook.Inc.Photocopyingisprollibitedbylaw.
Diagllostic Imag/IIg alld Recordillgs
diaphragmatic herrtia hepatic cirrhosis bladder rupture renal amyloidosis suspected or confirmed neoplasia
153. Omceming rhinoscopic examination ofsmall a"imals, which statement is most accurate?
•
a. Flexible pediatric fiberoptic rhinolaryngoscopes, designed fo r use in human infants, are ideal because they allow large· volume irrigation. b. Light sedation usually is sufficient for restraint. c. Radiographs of the nasal cavity and frontal sinuses should be made before rhi noscopy. d. Any bleeding of the nasal mucosa during the procedure indicates inflammatory disease or a coagulation disorder. e. The preferred irrigam is 15% dextrose.
154. Which condition i"creases tile risk of complications after tracheobronchoscopy?
a. Oslerus osled infection b. vegetable-matter foreign body c. stabilized congestive heart failure d. chronic cough e. pulmonary hypertension
85
155. eoflcemi"g endoscopic examination of the right bronchial tree ofdogs, which statemem is mOst accurate?
a. The bronchus to the right cranial lung lobe exits the principal bronchus medially. b. The bronchus to the right middle lung lobe exits the principal bronchus venrrally. c. The bronchus to the right caudal lung lobe exits the principal bronchus ventromedially, just beyond the middle lung lobe bronchus. d. Numerous small tertiary bronchi exit the length of the right principal bronchus. e. Beyond the origin of the caudal lung lobe bronchus. the right principal bronchus becomes the accessory lung lobe bronchus. 156. Co nceming transurelhral cystoscopic examination of small animals, wllich statement is most accurate?
a. A rigid 2.7 mm (diameter) arthroscope can be used in female cats and female dogs. b. Urine samples for microbiologic culture can be collecled during cystoscopy. c. Gas insufflation of the bladder is contraindicated if bladder mucosal bleeding is suspected or confirmed. d. Biopsy of localized bladder lesions is best done through a rigid arthroscope. c. The most common complication is urethral perforation. 157. COllcerning prepubic percutaneous cystoscopy using a rigid arthroscope in small animals, which statemellt is most accurate?
a. Xylazine sedation is sufficient for restraint and analgesia. b. An incision must be made with a scalpel th rough the skin and abdominal wall. c. Penetration of the opposite (dorsal) bladder wall warrants immediate laparotomy to suture the bladder wall. d. An indwelling urethral catheter is left in place for 48 to 72 hours after the procedure. e. A small bladder with a very thick wall is ideal for the procedure.
Correct answers are Of! pages 92- 100.
www.vet4arab.co.cc
SECTION 3
86
158. As viewed with tile endoscope in the nasal passage ora llealthy horse, during swallowing: a. the soft palate is displaced rostrodorsally b. the pharyngeal openings of the auditory tube close and move la terally c. the caudal pharynx can be seen as it dilates d . the epiglollis can be seen as it occludes the glottiS e. the corniculate process of each arytenoid
cartilage can be seen as it is adducted 159. In elldoscOfJicexamiliarioll of tile guttural pouclJes:
a. the endoscope is int roduced into the dorsal nasal meatus
h. general anesthesia is necessary c. the slitlike open ings a re located rostrovemral
to the pharyngeal recess d. the stylohyoid bone can be identified by its mediolateral course, dividing the gunura! pouch illla dorsal a nd ventral compartments e. the inte rna l carotid artery can be identified by its rostroventrallocation in the dorsal compartment 160. COllcemillge1ldoscop;cexam;Ilatioll of the esophagusatld stomach ofadult horses, which statemem is most accurate?
a. General anesthesia is usuaUy required because of the extreme sensitivity of the esophageal mucosa. b. Insufflation is co nt raindicated because of the danger of esophageal or gastric ru pture. c. The e ntrance to the cardia from the esophagus can be recognized as a tricuspid valve-like struc ture. d . The sacc us cecus portion of the stomach can be seen by directing the e ndoscope tip dorsally and to the horse's right side. e. Sufficient warm water is instilled to fully distend the stomach.
161. Colleeming endoscopic examination a/the duodenllm Ofhorses. which statemem is most accurate?
a. An endoscope that is 100 cm long is s ufficient fo r adult horses. b. The normal duode nal mucosa appears soft and velvety, \vith a gray- red or yellow- red color. c. The duodenal diverticuJum can be seen 24 to 30 inches distal to the pylorus. d. Pe ristalsis ceases because of vagovagaJ reflexes. e. The extreme na ture of the procedure necessita tes general anesthesia. 162. COllcemillgcfldoscopic examinatio n 0/ a mare's genital tmct. which statemeTil is most accurate?
Diagnostic Imagin g alld Recordings
164. eorlceming arthroscopic examination 0/ the equine carpal joint, which statement is most accurate?
165. In detemlillillg the gender o/a bird by laparascopy:
a. the laparoscope is inserted into the cloaca b. the abdominal air sac must be avoided c. the bird should be kept anesthetized for at least 30 minutes after laparoscopy to prevent air sac eventration d. the gonad can be found caudal to the caudal lobe of the kidney e. the bird should be placed in right lateraJ recumbency
a. The stopcock of the egress cannula must be kept open at aJl times until the procedure is com pleted. b. The trocar is inserted before the joint is dis tended with flu.id or gas. c. The trocar and arthroscopic shea th should be inserted at a 45-degree a ngle to the skin. d. The arthroscopic sheath and enclosed arthroscope should be "walked" along the a rticular cartilage to the s ite of suspected lesions. e. The joint is incised before the joint capsule is distended with fluid or gas.
a. The vagina and cervix should be thoroughly fl ushed with gl utaraldehyde solution before endoscope insertion. b. The diestrual cervix is flattened, partially o pe n, and shuated near the floor of the cranial vagina. c. The estrual cervix is closed and situated near the center of the c ranial vagina. d . The endometrium rapidJy becomes hyperemic when the ute rus is insufflated with air. e. The endometrial fo lds arc highly prominent, hyperemic, and edematous during diestrus.
87
ELECTROCARDIOGRAPHY Recommended Readillg Keene BW, Hamlin RL: Small animal carclioloKJ\ Philadelphia. 1997, \VB Saunders. Miller MS. TIUey LP: Mamml afsmall animal cardiology. ed 2. Philadelphia, 1995. WBSaunders. Physid::-Sheard PW:ln Colahan P1" et aI: eqUine metiicineand surgery. cd 5, SI. Louis. 1998, Mosby. Smith FWK. Tilley LP: Rapid imerpretatioll o/llean sounds, murmurs, alld arrhythmias, Baltimore, 1992.Wi.lliams & Wilki ns.
Tilley LP: Essentials ofcanine alld feline electrocardiography: illterpretation alld treatment. cd 3, Baltimore. 1992, Williams & Wilkins.
Practice answer sheets are on pages 259-260. 163. COllcemillgdistelltion 0/ajoi1lt capsule/or arthroscopic examination ofequine join/s, whicll statemellt is most accurate?
a. Distention with carbon dioxide or nitrous oxide is more practicaJ but also more damaging than diste ntion \vith fluid . b. Distention with sterile saline or lactated Ringer's solution is effective, rela tively harmless, and very practicaJ. c. DistentiOll with ai r offe rs the advantage of easier particle mobiLi7..ation and removal than distention with fl uid. d. Distention \vith flui d is complicated by the need for sophisticated pumps, ,vith sensitive gauges to continually monitor pressure. e. Distention with fl uid provides a sharper image a nd more accurate assessm e nt of cartilage than diste ntion \vith gas.
C 1998 Mosby-Year Book. Inc. Photocopying is prohibited by la w.
Questions loP. Tilley 166. A 10-mm upward deflection a/the ECG stylus in response to 1 m V 0/ electrical current is known as: a. b. c. d. e.
depolarization pola rity standardizalion conductivity aUlOmaticity
167. Tile action o/digitalis in treating atrial arrhythmias is best described as:
a. direct action on the atrial tissue to abolish a n irrita ble focus b. slowing of the ventricular rate by prolonging the refractory period of the atrioventricuJar node c. slowing the atriaJ rate so that the s inus node can take over as pacemaker d . increasing the s trength of ventricular contrac tion e. slowing the ventricular rate by prolonging the refr actory period of Purkinje fib e rs
Correct answers are on pages 92- 100.
www.vet4arab.co.cc
SECTI ON 3
88 168. Left-axis deviation occurs wilen:
a. b. c. d. e.
lead I is positive and aVF is positive lead I is positive and aV F is negative aVF is positive and lead J is negative aV F is negative and lead I is negative lead I is negative, aVF is negative, and lead II is negative
169. Which rhythm is characterized by slowing and speeding of the heart race related to respiration?
a. sinus arrhythmia
h. normal sinus rhythm c. sinus bradycardia d . sinoatrial block e. sinus tachycardia
atrial depolarization ventricular repoiarization ventricular depolarization atrioventricular conduction atrial repoiarization
171 . The P wave on the ECG represents: a. h. c. d. e.
174. When the sympatlletic nervous system is
stimulated:
a. the heart rate increases b. the speed of atrioventricular cond uction decreases and the heart rate decreases c. the force of contraction decreases d. the heart rate decreases e. the speed of atrioventricular conduction increases and the heart rate increases 175. A continuous rhythm strip showing each normal complex followed by a premature ventricular complex is most appropriately interpreted as:
170. The QRS complex 011 the ECG represents:
a. b. c. d. e.
c. propranolol, verapamil, and lidocaine d. propranolol, d igitalis, and d iltiazem e. digitalis, enalapril, and lidocaine
firing of the sinus node ventricular repolarization atrial repolarization atrial depolarization ventricular depolarization
172. The QT interval on the EeG represents:
a. b. c. d.
ventricular depolarization atrial depolarization ventricular repolarization conduction through the atrioventricular node e. ventricular depolarization and repolarizarion
173. Drugs most commonly used in treatment of
atrial arrhythmias include:
a. digitalis, procainamide, and lidocaine b. disopyramide, procainamide, and propranolol
a. b. c. d.
ventricular trigeminy irregular rhythm ventricular bigeminy infrequent premature ventricular contractions e. ventricular escapes 176. An arrhythmia that can progress to ventricular fibrillation and for which the clinician must continually monitor is:
a. b. c. d. e.
first-degree atriovenrricular block sinus tachycardia atrial premature complexes sinoatrial arrest ventricular tachycardia
177. Some atrial impulses are blocked in the atrioventricular junction and do nOl reach the ventricles, but the PR illlerval of those that do conduct are identical to each other. This statemerll best describes: a. h. c. d. e.
type-II atrioventricular block third-degree atrioventricular block first -degree atrioventricular block type-I atrioventricular block left bundle branch block
© 1998 Mosby-Year Book, Inc. Photocopying is prohibited by law.
Diagnostic Imaging and Recordings
178. ~arasympathetic stimulation of the heart results m:
a. increased heart rate b. increased force of contraction c. decreased heart rate and slowed atrioventricular conduction d. increased speed of atrioventricular cond uction e. increased atrioventricular conduction 179. The normal PR imerval jn dogs js: a. 0.06 to 0.13 second b.0.lOtoO.16second c. 0.12 to 0.20 second d. 0.1 6 to 0.24 second e. 0.04 to 0.10 second
R-on -T premature ventricular contraction fusion ce mplex sequential premature ventricular comraction ven tricular escape complex atrial premature complex
181. A single ventricular ectopic focus may fire once
or may fire in a series of rapid successive jmpulses to produce: a. b. c. d. e.
183. As the PR inrervallengthens, the RR interval shortells before "dropped complexes." 711is statemetlt best describes:
a. type-II atrioventricular block, with variable block b. third-degree atrioventricular block c. first-degree atrioventricular block d. type-I atrioventricular block (Wenckebach phenomenon) e. left bundle branch block 184. The three augmented wtipolnr limb leads are:
180. At times tile sinus impulse penetrates and contributes to ventricu!aractivation, along with a premature focus in the ventricles. The complex produced by this collision is referred to as:
a. b. c. d. e.
89
ventricular escape rhythm ventricular fusion complexes ventricular fibrillation ventricular tachycardia vemricular bigeminy
a. b. c. d. e.
185. In dogs and cats electrical allemalls is associated with:
a. pericard ial effusion o r supraventricular tachycardia b. ventricular tachycardia c. sinus arrest or atrioventricular block d. left ventricular enlargement or pericardial effusion e. ventricular fibrillation 186. Sick SifllLS syndrome is a term associated with:
a. b. c. d. c.
182. eoncerningfirst-degreeatrioventricular block in dogs, which statement is most accurate?
a. There is a 2:1 ratio ofPto QRS. b. Conduction through the junction is prolonged more than 0.13 second. c. The most common cause is heart failure. d. The rhythm is irregular. e. The heart rate is rapid.
aV!~ aVR, and aVF I, n, and III I, aV R, and III I, aVL> a nd aVF I, II, and aVL
pacemaker battery failure ventricular fibrillation sinus bradycardia and sinoatrial block heartworm disease respiratory disease and cor p ulmonale
187. Wolff-Parkinson- \.1Ihite syndrome consists ot
a. sinus bradycardia and ventricular premature complexes b. ventricular preexcitation with paroxysmal supraventricular tachycardia c. complete atrioventricular block and ventricular preexcita tion d. a trial tachycardia and heartworm disease e. ventricular escape complexes and sinoatrial arrest
Correct answers are on pages 92-100.
www.vet4arab.co.cc
SECTION 3
90 188. The important features of right ventricular etllflrgement are:
a. wide and positive QRS complexes in leads I,
II, III , and aVF b. S waves in leads 1, II, JII, and aV F and an axis of + 150 degrees c. S waves in leads I, II , III , and aV F and an axis of +30 degrees
d. deep Q waves in leads I, II , III , and aVF and an axis of +30 degrees
e. wide and negative QRS complexes in leads aV R• aVu and aV F
189. AtrioventriC/llar dissociation ca ll represellt a combination of
a. increased ventricular automaticity with an eClOpic ven tricular fo cus in control of the
ventricles, while the sinoatrial node continues to activate the atria b. control of atrial and ventricular activity by the sinoatrial node. with conduction from the sinoatrial node to the atria, atrioventricular node, and ventricles
c. variation in heart rate and rhythm secondary to increased vagal tone from respiration d. ventricular fib rillation during cardiac arrest after repeated a u em pts to convert the rhythm e. sinus bradycardia secondary to hypokalemia and decreased a utomaticity 190. Atrial standstill
is most likely to be associated
witll: a. hypoadre nocorticism, Iddney failure or diabetes mellitus b. hypoadrenocorticism, cor pulmonaJe or diabetes melli tus c. hearhvorm disease, hypoadrenocorticism or liver disease d. liver disease, congestive heart failure or diabetes mellitus e. gastric torsion, hypoadrenocorticism or cor pulmonale
Diagtlostic Im aging a /Ill Recordings
193. Tile cellular origin of tile EEG is predomillafl tiy
tile result of a. algebraic summation of postsynaptic membrane potentials b. summation of excitatory effects in cortical synapses c. changes in gl ial me mbrane potentials d. summation of action potentiaJs e. unknown factors not yet elucidated by research evidence
91
195. The brain structures most directly illvolved in maintaining tile EEGofalert wakefulness are the neocortex and tile: a. amygdaJa b. limbic system c. hypothalamus d. basaJ ganglia e. brain stem 196. The major use of electromyography is in: a. suppressing excessive electrical activity in muscle b. neurologic diagnosis c. acupuncture d. diagnosing electrical diseases of muscle e. promoting reinnervation of skeletal muscles
194. Tile EEG is most likely to provide mOrlitoringof all of tile following except:
a. anesthesia b. sleep
c. dream sleep d. coma
e. pain
ELECTROMYOGRAPHY Recommended Readi"g
ELECTROENCEPHALOGRAPHY Recommellded Readillg Klemm WR: Animal electroencephalography, New York, 1969. Academic Press. Klemm WR: API)/ied electronics for veterinary medicine and animal physiology, Springfield, Ill,
Braund KG: Clinical syndromes in veterinary neurology. cd 2, SI. Louis, 1994. Mosby. Chrisman Cl: Problems in small animal neurolOID\ ed 2, Ballimore, 1991, Williams &Wukins. Mayhew IG: I1lrgeanimal neurology. Ballimore, 1989, Williams & Wilkins. Oliver IE, Lorenz MD: Handbook o/veteri"ary "eurology. ed 2, Philadelphia, 1993, WB Saunders. Practice answer sheets are on pages 259-260.
1976, Charles C Thomas. Klemm Wit [n [ndrieri RJ: Epilepsy. Prob[Vet Med 1:535-556, 1989.
Questions
Practice answer sheets are on pages 259·260.
Questions W.R. Klemm 191. 71leeiecrroencepllalogram (EEG) can most accurately be described as: a. b. c. d. e.
the best way to measure intelligence the only way to monitor consciousness a plot of microvolts versus time a refl ection ofthe amount of brain activity the best way to detect epilepsy
192. WI,at causes tile major artifact associated witll EEG recording in animals? a. b. c. d. e.
I.M. Bowen 197. Use of needle electrodes in conducting all electromyographic examination:
epilepsy lighlning radio interference computer te rminals muscle spasms and movement
Cl 1998 Mosby--Year Book, Inc. Pilotocopying is prollibiled by law.
a. prevents insertion activity b. increases serum creatine phosphokinase (CPK) activi£y c. inhibits respiration d. stimulates interference patterns e. reduces blood potassium levels
198. After nerve injury,fibrillatioll potentials occur in denervated canine skeletal muscles witllin: a. I to 2 hours b. 12 to 24 hours c. 2 t03 days d. 5 to 7 days e. 10to 12 days
Correct answers are Of! pages 92-100.
www.vet4arab.co.cc
SECTION 3
92
199. If a peripheral motor nerve is partially damaged, which oft/le/ollowing is most likely to be
observed Oil eleclromyograplTicexaminarion 10 days after injury?
200. Peripllerai nerve conduction velocity. which can be measured electromyograpllically. is most likely to be slowed in all animal with:
a. myopathy
a. fibrillation potentials only b. motor unit potentials only c. combination of fibrillation potentials and motor unit pote ntials
b. c. d. e.
neuronopalhy axonopathy myelinopathy uppe r motor neuron disease
d. myolOnic discharges
e. absence of nerve conduction
Answers I. b This is the definition of the silhouette effect. 2. a Pleural effusion usually s ilhouettes the heart and diaphragm. 3. c Grids are used ifthe body part to be radiographed is greater than 10 cm thick. 4. d Radiolucent air surrounding the lung lobes is a sign of pneumothorax. 5. c Pneumomediastinum docs not occur secondary to pneumothorax. 6. c VVhen the soft tissue lesion is in the nondependent lung, it is surrounded by more air, giving it better cont rast and visibilily. 7. e Tracheal colla pse can be diagnosed on plain radiographs. Fluoroscopy is helpful but not essential in most cases. B. a Atelectasis (collapse) typically causes a mediastinal shift to compensate for loss of lung volume. There is no loss or lung volume in pneulllonia. 9. e Visible, opaque fissure lines are a sign of pleural effusion. 10. e Only the bronchial lumen is seen with an air bronchogram: the bronchial walls are silhouetted against fluid in the lungs. II. a Air bronchograms are only seen with an alveolar pattern. 12. e Chronic bronchitis typically causes a bronchial pattern, not a nodular pattern. 13. e Peripheral arteries and veins are tht same size. 14. d On an expiratory radiograph, the lung fields typically do not extend caudal to the l\velfth thoracic vertebra. 15. b The radiographic changes described are typical of left-sided congestive heart failure (pulmo nary edemal. The clinical signs and
@
16. 17.
lB.
19. 20.
21. 22. 23. 24.
25.
26.
physical exam ination findings support this diagnosis. e Only the pulmonary trunk is located at this site. d A "reversed- DO. appearance is typical ofright heart enlargement. b These cardiac changes are seen with patent ductus arteriosus. Aortic s te nosis is characte rized by a bulge in the ascending aorta. Ventricular septal defect does not cause a bulge in the descending aorta. Pulmonic stenosis and tetralogy or Fallol cause right heart enlargement. e Hypovolemia usually res ults in a small heart. b These changes are typical of heartworm disease. None of the other diseases listed would have enlarged, tortuous arteries. b The normal canine Jddney is 2.5 to 3.5 times the length of the second lumbar vertebra. e Free abdominal gas increases contrast in the abdomen. d Felal skeletons can be seen on radiographs by 40 to 45 days of gestation. a Fluid accumulates in the pylorus, which is dependent on right lateral views. Gas accumulates in the nondependent fundus. c Iodine contrast medium does not cause severe inflammation/peritonitis if it leaks into the peritoneal space. Barium sulfate causes severe peritonitis if it leaks into the peritoneal space and is contraindicated if bowel or gastric rupture is suspected. b Periosteal new bone formation is radiographically evident 7 to 10 days after injury.
1998 Mosby-Year Book. Inc. Photocopying is pro/libired by law.
Diagnostic Imaging and Recordings
27. a Intravenous contrast agents may cause severe side effects if given to markedly dehydrated patients. The other problems listed are not contraindications for intravenous urography. 28. e The pylorus is not dependent on right lateral views of patients with gastric IOrs ion and therefore does not accumulate fluid. 29. c On a double-contrast cyslOgram, the bladder is distended with negative-contrast medium. with a small amount of positive-contrast medium added. 30. a Hepatomegaly displaces the stomach caudodorsally. The other organs listed are caudal to the stomach. 31. d Osteochondrosis most commonly affects the caudal aspect of the humeral head. 32. e Os teochondritis dissecans is primarily seen in large-breed dogs. 33. a These signs are typical of panosteitis. The patient characteristics and physical examination findings support the diagnOSis. 34. c The radiographic findings and patient characleri! tics support a diagnosis of LeggCalve-Perthes disease. 35. d Osteosarcoma usually occurs in the metaphyseal area of long bones. 36. b Barium solution is given PO at 6to 12 m1 /kg. 37. e Salter class-V fractures are frequently not evident on radiographs. The other choices describe Salter fractures of class I through rv. 36. a Lysis of the vertebral end plates is consistent with diskospondylitis. 39. e Lysis of the tarsal bones is not a s ign of degenerative joint disease. 40. b None of tlle other descriptions is accurate. 41. a A dorsolateral-palmaromedial oblique projection delineates the dorsomedial aspect of the third carpal bone, whereas a dorsoproximaldorsodistal oblique projection provides an endon view of the tarsal bones. 42. e Navicular disease is best diagnosed using at least three different views of the navicular bone. 43. a Grids are placed between the patient and the x-ray film. 44. d DecreaSing the developer solution temperature makes the film lighter (underdeveloped). All other choices increase the radiographic density. 45. a High kVp results in low contrast, \vith a long gray scale. A high-contrast, short-gray scale technique is preferred for extremity films ; a low-
46.
47. 48.
49. 50.
5 1.
93
contrast, long-gray scale technique is preferred for thoracic films. Many factors affect contrast. b Film development is faster at higher temperatures. Developer solution must be replenished periodically. Developer solution does not harden the film. b Static electricity causes such branching artifacts. a The National Council on Radiation Protection and Measurements recommends a maximum permissible dose of 5 rem per year. d Nonscreen film requires longer exposure times and should not be used for routine procedures. c Portable x-ray machines should never be hand held during use, because this increases radiation exposure. c Grids are indicated for parts thicker than 10 Cill.
52. e High-mAs, low-kVp techniques produce the highest contrast. All listed techniques are equivalent in terms of film blackness. 53. c Halving the mAs is a reasonable adjustment to correct an exposure that is too dark. 54. d Rare-earth screens have increased speed as their main advan tage. 55. d Unprocessed x-ray film is naturally green to gray. 56. e Static often causes this type of artifact. 57. e MilJiamperage does not affect detail. 58. d No other option produces these radiographic signs. 59. c "is not related to insufficient growth of the ulna. 60. e It is not found in patients with a fragmented coronoid process. 61. e Lupus erythematosus is nonerosive in dogs. 62. e This ligament attaches at the affected region. 63. d This provides an unobstructed view of the nasal cavity. 64. c The other options are much less specific. 65. e It usually occurs only with discospondylitis. 66. a There is no disk at CI-C2. 67. a This usually occurs only with diskospondyliris. 68. c Patients with spinal fractures shouJd not be manipulated. An orthogonal view is necessary ror complete evaluation. 69. b Myasthen ia gravis, megaesophagus, and aspiration pneumonia are common in patients with thymoma.
94
SECTION 3
70. c Bacterial pneumonia produces an alveolar infiltrate. 7 1. a In the other options the vessels are not clearly seen. 72. c Enlargement of this vein is consistent with pulmonary venous hypertension. 73. d Pleural effusion is more common in right heart failure . 74. a These radiographic signs are classic for patent ductus arleriosliS. 75. e Comparing the tracheal appearance at inspiration and expiration is very helpful. 76. c Dyspneic, coughing patients often have gas in the esophagus. 77. e This is more often a sign of pneumothorax. 78. c Fluid pools around the heart. prod ucing a silhouette sign. 79 . d There is no obvious rcason why heamvorrn disease would produce a heart with the shape described . 80. b The depcndclH lung is partially collapsed and exhibits increased opacity but may silhouette a mass. 81. a Of the choices listed, Paragonimus is the most common cause of cavitary lung lesions in cats. 82. d Metastasis does not usually produce clinical s igns until late in the disease. 83. a Splenic torsion can occur as an independent event. 84. e Hydrone phrosis is not a reliable sign of chronic renal failure. 85. b Emphysematous cystitis is easily seen in survey radiographs. 86. b The location of the mass is most consistent \vith external iliac lymphadenopathy. 87. e Most of the urethra is extraperitoneal. 88. d The insolubility of air makes it hazardous in this situation. 89. b The type of contrast medium specified is not secreted by renal tubules. 90. d This is the most accurate method of assessing bladder integrity. 91. e This offers the optimal combination of safety and low cost. 92. d The other choices lis ted do not allow the mucosa to be visualized in comparable detail. 93. b The high solubility of carbon dioxide decreases the chance of fatal gas embolism.
www.vet4arab.co.cc
94. e Air bubbles compromise the quality of the study, impairing assessment of the bladder wall. 95. b Extravasation occurs in normal and abnormal prostate glands. 96. d This procedure allows identification of one mass as the bladder and more accurate assessment of the other mass. 97. d This is important to know so as not to misdiagnose stomach displacement in cats. 98. c Attempting to ide ntify the fundus and/or pylorus by using gastric gas as a contrast medium is the most appropriate next step in an attempt to rule out gastric volvulus. 99. a This is the typical appearance of pseudoulcers. 100. d The hypertonic nature of the contrast medium pu lls the fluid onto the bowel from the intravascular space. 101. c High.freque ncy probes provide better axial resolution hut poorer penetration than lowfrequency probes. Low-frequency probes provide poorer axial resolution and better penetration than high-frequency probes and may be needed to image deeper structures. For example, in imaging the liver of a large dog. the sonographer might use a 7.S-MHz probe to examine the s upe rficial portion of the liver but might swilch to a S.O-M Hz probe for examination of deeper portions of the liver. 102. a Acoustic impedance of a tissue can be calculated from the density of the tissue and the propagation speed of sound in the tissue. Sound is reflected from an interface behveen two tissues with different acoustic impedances. The intens ity (viewed as brightness) of sound reflected depends on the angle of the ultrasound beam and the difference in acoustic impedance. 103. b The relative echogenicities of these tissues are as follows: splee n > renal cortex and liver> renal medulla. In comparing echogenicities of two tissues, the sonographer must e nsure that the tissues are at the sam e depth (to avoid artifaci'caused by depth gain compensation), that the probe frequency is the same for each organ, and that structures are not present that might alter the echogenicity of one of the tissues (e.g., if fluid is superficial to the imaged tissue, echogenicity of the tissue is increased).
@ 1998Mosby-YearBook, lnc.Pho(ocopyingisprollibiredbylaw.
Diagnostic Imaging and Recordings
104. c The capsule is a bright linear echo when the ultrasound beam is perpendicular to the kidney. The cortex and medulla are both hypoechoic, but the cortex is always more echogenic than the medulla. Echogenicity of the renal cortex should be approximately the same as that of the liver and less than that of the spleen. The renal pelvis is hyperechoic unless distended. Diverticula are hyperechoic. 105. e The renal cortex is more echogenic than the renal medulla. Echogenicity of the renal cortex should be apprOximately the same as that of the liver and less than that of the spleen. The renal cortex of obese cats has increased echogenicity because offal in the renal tubules. 106. b Small bright dots a t the corticomedullary junction represent a rcuate arteries. 107. e Acoustic shadowing occurs when a ll or most oCthe ultrasound beam is attenuated (reflected, absorbed, or scattered) by a structure so that no ultrasound is reflected by deeper structures. 108. d The spleen has smooth borders and a welldefined capsule that appears as a fine echogenic line when imaged at right angles to the probe. TIle spleen has a finer and more hyperechoic echo pattern than the liver. Intrapare nchymal splenic venous branches are hypo echoic to anechoic when viewed at a n oblique angle to the ultrasound beam but the walls may appear hyperechoic when imaged at right angles to the probe. Echogenic flow is a normal finding in larger vessels and does not indicate pathologic changes. Splenic arteries are not visible within the parenchyma, although arterial rami may uncommonly be found at the hilus. Splenic veins are visible. 109. a The gastrointestinal tract has five layers, \'lith altemating hyperechoic and hypoechoic echoes. From th e lumen to the external serosa, the layers are as follows: mucosal surface (hyperechoic), mucosa (hypoechoic), submucosa Olyperechoic), muscularis propria (hypoechoic), and serosa (hyperechoic and includes the subserosa). 110. e Mucus and gas are both hyperechoic, but mucus allows visualization of the far wall of the intestine, whereas gas exhibits distal or deep acoustic shadowing. Reverberations between the transducer and gas appear as artifactual lines \'Iithin the shadow; acoustic shado\'ling associated with gas has been called '·dirty." Intraluminal fluid is anechoic and (like mucus)
95
allows visualization of the far wall. Mucus and gas have echogenicity s imilar to that of the mucosal layer (all are hyperechoic). so that the interface between the mucosal s urface and either gas or mucus cannot be dis tinguished. Because fluid is anechoic, the mucosal surfaces are clearly seen as a separate layer. Ill. e From the lumen to the external serosal layers, the four wall layers are as follows; a hypoechoic mucosal layer; an irregular, discontinuous echogenic lamina propria submucosa parallel to the mucosa (lamina epithelialis); slightly heterogeneous, hypoechoic smooth muscle; and a hyperechoic serosa/perivesicular fat interface. All four layers may be visible when the bladder is collapsed, but imaging is suboptimal because the wall is artificially thickened. When the bladder is viewed with a high-frequency transducer (7.5 MHz or higher), v.rith slight to moderate distention, three layers are seen: a hype rechoic inner layer representing the submucosa/ mucosallumen interface; a slightly heterogeneous or hypoechoic middle layer representing the smooth muscle layers; and a hyperechoic outer laye r representing the serosa/ perivesicular fat inte rface. Stretching the bladder wall by maximal distention can mask even severe bladder wall thickening. Acoustic enhancement enol shadowing) occurs deep to the urinary bladder because there is minimal anenuation of ultrasound as it passes through a cystic structure. The cranial border (apex) of the bladder is poorly imaged by many ultrasound probes because of ultrasound refraction in this area. This area can be seen in a cystogram. 112. e The canine prostate gland is a bilobed structure surround ing the proximal urethra at the neck of the bladder. The normal prostate gla nd has a uniform, somewhat coarse echogenicity similar in brightness to that of the spleen. The urethra is a linear structure located between the lobes and has been described variously as hyperechoic or hypoechoic. The description of a hypoechoic oval \'lith a hyperechoic line located centrally \'Iithin the lobe would better apply to the testicle. The testicle is a smooth, homogeneous, elliptic organ, \'lith the rete (mediastinum) testis appearing as a central hyperechoic line in longitudinal images and a hypcrechoic dot in transverse images.
SECTION 3
96
11 3. d The uterus is hypoechoic, not hyperechoic. During estrus the lumen may not be visible or a small amount orfluid may be presenl. The uterus may be differentiated from the aorta and caudal vena cava because these vessels bifurcate caudally, whereas the uterus bifurcates cranially. (Also, the vessels are uSlImly anechoic with a proper gain selling.) In contrast to the gastrointestinal tract, the uterine wall in anestrus does not exhibit layers. Also, hypcrechoic gas or ingesta in the ilHcstinal lumen and peristalsis are generally apparent. Immediately postpartum, the endometrium and myometrium can be distinguished. Three layers may be visible in the myometrium: an inne r hypoechoic circular muscle layer; a central hyperechoic fibrovascular layer; and an oLlter h ypoechoic longitudinal muscle layer. The waU of the small intestine has five layers. Pregnancy can be diagnosed in dogs by ultrasonographic visualization of the chorionic cavity at 17to 20 days after the luteinizing hormone peak. The chorionic cavity appears as a 1- to 2- mm spheric vesicle. 114. b The normal pancreas is difficult to image because it is thin, has an echogenicity similar to tha t of adjacent m esentery, and is proximal (cranial) lO gas in the bowel. II has a homogeneous echogenic texture that is isoechoic to slightly hyperecho ic to live r and hypoechoic to spleen. The right extremity is dorsomedial to the duodenum, the body is associated \vith the cranial duodenal flexure, and the left extremity is found in lile region bordered by the stomach, spleen, and transverse colon. The phrenicoabdominal vein serves as a vascular landmark for the adrenal glands. The pancreaticod uodenal vein may be seen centrally in the pancreas. In acute pancreatitis the pancreas becomes thicker and more hypoechoic. Pancreatic pseudocysls may be seen as anechoic areas lilal can become q uite large. Hyperechoic areas may be seen with chronic pancreatitis. 115. b The normal gallbladder is teardrop shaped, with a smooth, uniform wall less than 2 to 3 mm wide. The wall is isoechoic to norma1liver and is therefore difficu lt to see and measure. The gallbladder lumen is rare ly empty and usually contains anechoic bile. The common bile duct is less than 2 mm wide and is difficult to follow to the duodenum.
@
www.vet4arab.co.cc
11 6. d Artifacts are frequently seen when imaging the liver and ga1lbladder. Mirror-image artifact occurs when sound is repeatedly reflected between the lung/diaphragm interface and the hepatic parenchyma. Black streaks originating fTom the curved surfaces of the gallbladder and extending into the hepatic parenchyma deep to the gallbladder are caused by refraction of sound from a curved object. Distal acoustic enhancement ("through transmission~) often causes an artifactual increase in echogenicity of the live r deep to the gallbladder. 117. d Echocardiographic findings in hypertrophic cardiomyopathy include left ventricular hypertrophy, either symmetric or asymmetric, behveen the left ventricular wall and septum. Left ventricular chamber size is reduced and the wall is noncompliant, preventing diastolic filling and causing elevated pressures at the end of diastole. The high e nd-diastolic pressure, along with distortion of the mitral valve apparatus, results in mitral regurgitation and left atrial dilation. These changes are aggravated by ventricular hypercontractility, which increases cardiac workload and tachycardia, decreasing the diastolic time available to fill the ventricle. In this patient, narrowing of the left ventricular outflow tract created by disproportional septal hypertrophy is creating functional obstruction to outflow. resulting in high-velocity turbulent flow; this further distorts the mitral valve and increases afterload o n the ventricle. 118. e p-blockers slow the h eart rate, which would decrease cardiac workload and allow for inc reased diastolic filling of lile ventricle. This increases cardiac output and decreases cardiac muscle hypoxia. Calcium-channel blockers slow the heart rate and increase ventricular re laxation, which assists diastolic filling and decreases the workload of the ventricles.
1998 Mosby-Year Book, Inc. Pllotocopying is prollibited by law.
Diagnostic Imagitlg and Recordings
119. b The su perficial digital extensor tendon originates on the media1 epicondyle of the humerus and inserts d istally on the phaJanges. (An accessory ligament originates on the distal radius.) Deep to the superficial digital extensor tendon is the deep digital flexor tendon, which originates on the d istal hwnerus, proximal olecranon, and mid-radius/ulna. The inferior check ligament continues as the palmar carpal ligament, travels deep to the deep digital flexor tendon, and joins the deep digital flexor tendon in the mid-metacarpal region. The interosseous (suspensory) ligament originates on the palmar surface of the third metacarpal bone and lies deep to the flexor tendons and inferior check ligament. It bifurcates at the level of the junction of the middle and distal thirds of the metacarpus. 120. c Severe retinal detachment appears as Vshaped echogenic lines with points of attachment at the optic disk or ora serrata. The lens is anechoic, whereas the anterior and posterior lens capsules are hyperechoic linear structUres. The anterior, posterior (when seen), and vitreous chambers are anechoic. The optic nerve is hypoechoic, whereas the optic disk is hyperec hoic. The iris a nd ciliary body are seen as short echogenic lines on each side of the lens. 121. a Acoustic impedance for a given material (tissue) is defined as the sound wave velocity multiplied by the density of the material. As the difference in acoustic impedance between £\VO adjacent tissues inc reases, the proportion of sound waves reflected at the interface of the tissues increases. Increased reflection of sound waves results in a brighter image at the point of reflection. 122. c Both the axial resolution and lateral resolution of an image are improved by increasing the transducer frequency. 123. a Axial resolution improves with increasing transducer frequency and with fewer cycles per pulse. 124. e Production of an ultrasound image reHes on the transducers receiving sound waves reflected from tissue interfaces at various distances. The time for a given pulse to be emitted, reflected at an interface, and return to the transducer dete rmines the distance of the interface from the transducer. The intensity of reflected waves returning to the transducer determines the brightness of the ultrasound
97
image at the point of reflection . A constant sound wave velocity through soft tissues is assumed even though slight variations in propagation speed occur. 125. b Pulsed Doppler is a method for determining the velocity of moving structures, such as flowing blood. The technique relics on changes in the frequency of ultrasound waves reflected from objects moving toward or away from the transducer. In the example of flowing blood, ultrasound waves reflected off blood cells moving toward the transducer have a higher frequency than the original emitted sound wave frequency, whereas sound waves reflec ting off cells moving away from the transducer have a lower frequency. 126. c The power control on an ultrasou nd machine determines the amplitude or intensity of the sound waves produced. As power is inc reased, the brighlness of the image is increased because more sound waves are reflected back to the transducer. 127. a The gain control o n an ultrasound machine is used to electronically amplify the signal produced by reflected ultrasound waves returning 10 the transducer. Amplifying the signal increases the brightness of Ule ultrasound image. Most machines have a gain control that uniformly modifies the sound wave amplification. 128. e Time-gain compensation controls are used to e lectronically amplify reflected waves from deeper structures. Because sound waves emitted from a transducer are auenuated and reflected by superficial structures, deeper structures receive comparatively fewer waves. The time-gain compensation control compensates for this nonuniform Signal generated for image formation. 129. dB-mode, or brightness mode, is the standard for nvo-dimensional image generation. The brightness of a particular point in an image depends on the intensity of reflected echoes returning to the transducer from the point of reflection. 130. b A IO-MHz linear-array transducer is the best choice. High-frequency transducers provide better spatial resolution (better image detail) than low-frequency transducers. linear-array transducers, which produce a rectangular image, provide better image quality in the near field (superficially) than do sector-scanning transducers. which produce a wedge-shaped image.
98
SECTION 3
131. a A urinary bladder calculus would be most
likely to produce a shadowing a rtifact because of reflection or refraction of almost all ultrasound waves by the calculus. Because very few waves penetrate deep to the calculus, virtually no signal is generat ed in this region and an anechoic shadow is produced.
132. e Because cystic snuctures are fluid fill ed, sound waves penetrate these structures to a greater degree than for soft tissues. This results in morc sound waves reaching tissues deep to the cyst As a result, there is a comparatively greater reflection of sound waves in these deeper structures. producing a brighter image deep to the cyst. 133.
134. 135.
136.
137. 138.
139:
140.
a Mirror-image artifacts result from reflected sound waves traveling by multiple paths back to the transducer. This can delay arrival of some of the reflected waves at the transducer. Because the ultrasound image is prod uced by mapping structures based on time of travel of reflected sound waves, a delay results in apparent pOSitioning of the structure deeper than its actual location. c Spleen is most echogenic, and the renal medulla is least echogenic. b Animals with hepatic lipidosis have a generalized incrcase in hepatic parcnchymal echogenicity. e Animals with acute ethylene glycol intoxication have a marked increase in renal cortical echogenicity. The change in echogenicity is proba bly the resuJt of accumulation of calcium oxalate crystals in the re nal tubules. b Portal veins have tltis appearance on sonograms. c Lym ph nodes are generally uniformly hypoechoic and produce less distal enhancement than cystic structures. a Left ventricular failure resulting in pulmonary edema would be difficult or impossible to diagnose us ing cardiac ultrasound alone. Diagnosis of pulmonary edema would require thoracic radiographs. Echocardiographic findin gs of the othe r conditions are well described. b On sonograms of unsedated dogs, the left ventricular shortening fraction averages approximately 35%.
www.vet4arab.co.cc
141. d Unless the fibers are clad, light tends to leak as it traverses the fibers toward the distal tip of the endoscope. 142. a These features enhance maneuverability of the distal tip. 143. b Air should not be used because of the risk of air e mbolism. Nitrous oxide, oxygen and hydrogen can support combustion. 144. d Endoscopy can detect morphologic but not fu nctional disease. 145. c Surrounding tissues could be contaminated by leakage of ingesta during insufflation. 14 6. a It may be caused by vagovagal refl exes stimulated by overdistention of the bowel or by traction on the mesentery. 147. b Glycopyrrolate is commonly used. 148. a The endoscope should be withdrawn to the cardia and the stomach partially deflated before further attempts are made. 149. c Fasting and administration of oral electrolyte solutions help to cleanse the colonic mucosa in preparation fo r colonoscopy. 150. a Some treated cats may develop hypermagnesemia. 15 1. c The directional control knobs can be manipulated so as to view the entire ci rcumference of the colon. 152. a Gas could escape from the a bdominal cavity into the thoracic cavity, caus ing pneumotllorax. 153. c Preceding rhinoscopic examination may alter the radiographic appearance of structures. 154. e Biopsy during bronchoscopy could result in serious hemorrhage. 155. b None of the other answer choices accurately describes the endoscopic view of the right bronchial tree. 156. a This instrument can be inserted through the urethra of dogs and cats. 157. d The bladder must be kept undistended to allow the puncture sites to heal. 158. a The view caudally is obscured during swallowing. .. 159. c These openings are evident in the rostroventral portion ofche guttural pouches. 160. d The endoscope tip should be directed dorsally and to the right to view this portion of the stomach. 161. b Differences in appearance may indicate pathologic changes.
C 1998 Mosby-Year Book, 111C. Photocopying is prohibited by law.
Diagnostic Imaging and Recordings
162. d Introduction of air into the uterus produces a hyperemic response. 163. b Both cause minor pathologic changes in chondrocytcs that resolve in a few days. 164. e This preserves the anatomic position of tendons and their sheaths for accurate incision. 165. e Birds should be placed in right lateral recumbency for gender determination. 166. c The other possible choices describe electrical activity within the myocardial cell. 167. b Digitalis acts directly on the myocardium , vagus nerve, and atrioventricular junction . 168. b Us ing at least two limb leads (I, aVF) and the different angles at which they record the heart's electrical activity, one can estimate the mean electrical axis in the frontal plane. 169. a Sinus arrhythmia is represented by alternating pe riods of slower and more rapid heart rates, usually related to respiration . The heart rate increases with inspiration and decreases with expiration. 170. c Atrial depolarization and repolarization represent the P wave and subsequent T wave, whereas ventricular repolarization represents the T wave following a QRS complex. 171. d The depolarization waves spread from the sinoatrial node through the right atrium, toward the left atrium and the atrioventricular node, resulting in a P wave. This indicates depolarization. 172. e The QT interval is the summation of ventricular depolarization and repolarization. 173. d Procainamide, lidocaine, and disopyramide are ventricular antiarrhythmic agents. 174. e Sympathetic stimulation increases the si nus rate and increases tlle force of myocardial contraction. 175. c Ventricular bigeminy occurs when the rhythm alternates between a normal sinus complex and a ventricular premature complex. 176. e The othe r possible choices are not primary ventricular arrhythmias and are also arrhythmias that usually do not lead to ventricular fibrillation. 177. a One or more P waves not followed byQRS.Y complexes can be classified into two types of second-degree atrioventricular block: Mobitz type I and Mobitz type II. With Mobitz type I, the RR interval becomes progressively shorter as the PR interval becomes longer, until a P wave is blocked.
99
178. c The cardiac parasympathetic nerves form the vagus nerves, whose fibers terminate primarily in the s inoatrial node, atria, and atrioventricular node, slowing the heart rate and slowing conduction through the atrioventricular node. 179. a The normal PR interval range is 0.06 to 0. 13 seconds. 180. b A fus ion complex is simultaneous activation of the ventricles by impulses coming from the s inoatrial node and ventricular ectopic foc i. 181. d Ventricular tachycardia can be thought of as a continuous series of ventricular premature complexes resulting from stimulation of an ectopic ventricular focus. 182. b Delay in conduction of a supraventricular impulse through the atrioventricular junction is called first-degree atrioventricular block. 183. d Mobitz type-I atrioventricular block is present when the RR interval becomes progressively shorter as the PR interval becomes longer, until a P wave is blocked. 184. a Leads I, II, and III represent the s tandard bipolar leads. 185. a Electrical alternans is diagnosed when the Po QRS, or T complexes alter in configuration on every olher complex, every third complex, every fourth complex, etc., with each complex originating from the same focus. 186. c Sick sinus syndrome describes a number of electrocardiographic abnonnalities of the sinoatrial node, including severe sinus bradycardia a nd severe s inoatriaJ block andl or sinus arrest. 187. b Wolff-Parkinson-White syndrome consists of ventricular preexcitation, with episodes of paroxysmal supraventricular tachycardia associated with an accessory conduction pathway. 188. b In tlle normal canine heart, the axis lies betwee n +40 degrees and + 100 degrees, with right-axis deviation occurring when the axis is greater than +100 degrees. 189. a Atrioventricular dissociation implies that the atria and ventricles are discharged by two independent foci of impulse fonnation. 190. a Hyperkalemia is a very common cause of atrial standstill in animals with hypoadrenocorticism, kidney failure, or diabetes mellitus.
100
SECTION 3
191 . c All other choices are incomplete or ambiguous. Some of the other choices. such as
band e, are amy half-truths, because there are other ways to evaluate consciousness and epilepsy.
192. e All other answers refer to other sources of artifact, but these problems are not always present In fact, the problem of muscle spasms a nd movement is so great that many veterinary e lectroencephalographers insist that recordings be made with the patie nt under anesthesia. 193. a Although the other answers refer to other possible sources of EEG signals, these
influences are probably relatively minor. 194. e The EEG becomes activated (low voltage, fast activity) when an animal is in pain. However, it becomes Similarly activated when the animal is stimulated imensely by stimuli that a re not painful. Also, the EEG of dream sleep appears similar. 195. e Conscious awareness is e nabled by the excitatory action of the brain s tem reticular form ation upon the neocortex. Lesions or stimulation of the brain ste m produces predicta ble changes in the EEG.
www.vet4arab.co.cc
196. b Electromyography is an important adjunct to neurologic examination. By ilSelf, e lectromyography does not provide a definitive diagnosis of a nerve or muscle disease. 197. b Needle electromyographic e lectrodes cause focal muscle fiber damage, which results in release of CPK and elevation of serum CPK activities proportional to the number of needle insertions. 196. d Fibrillation potentials in canine skeletal muscles usually appear on day 5 after denervation. A direct re lations h.ip exists between size of the animal species and the time of appearance of fibrillation potentials. 199. c Fibrillation potentials appear in the denervated motor units. Motor unit potentials occur in the innervated motor units. Thus both types of potentials would be recorded from a partially innervated muscle. 200. d Demyelination of a periphe ral nerve results in conversion of conduction of impulses from a saltatory (nodal) process to continuous conduction, which is significantly slower than saltatory conduction.
SECTION
4 Hematology and Cytology R.L. Cowell,
W.I.
Dodds, B.L. Hines
Recommended Reading Cowell RI.., Tyler RD: Cytology and hematology of tile horse, St Louis, 1992. Mosby. Cowell RI.., Tyler RD, Meinkoth JH: Diagnostic cytologyoftlle dog and cat, ed 2. St Louis. 1998,
Mosby. Duncan JR et al: Veterinary labomtory medicine, ed 3. Ames, Iowa. 1994, Iowa State Uni\'ersity
Pre". fain NC: Essentials of veterinary Ilemat%gy, Baltimore, 1993. \Vllliams & Wllldns.
Meyer 0 1 el al: Vererinary laboratory medici/Ie: interpretation and d iagllosis, Philadelphia. i 992, we Saunders. Willard MO et al: Small animal clinical diagnosis by laboratory methods. ed 2, Philadelphia. 1994, WE Saunders.
NOTES
Practice answer sheetli are on pages 261-262.
Questions R.L. Cowell and B.L. Hines 1. Which anticoagulant does not act by binding calcium ?
a. I, II, V; Xl b. II, V, VII, IX
a . heparin
b. c. d. e.
sodium EDTA sodium citrate potassium EDTA potassium oxalate
c. II, IX. X. XII d. II. VII. IX, X e. III, VII, IX, X
2. Which type of red blood cell morphology is compatible with phenothiazine roxicity in horses? a. b. c. d. e. «)
1998 MoslJy-Year Book, In c. Photocopying is prohibited bylaw
target cells Heinz bodies stomatocylosis basophilic s tippling nucleated red blood ceUs
3. Which clottingfactors are affected in animals wilh dicumarol toxicosis?
4. lNhich factor in Ole clotting cascade has the shortest half-life? a. I b. III c. V d. VII
e. IX Correct aTlSUJers are on pages 124-130.
101
5. A 5-year-old St. Bernard is presenced to your
small-animal practice because of lethargy and anorexia 0[2 weeks' duration. Physical examination reveals generalized lymphadenopathy. Microscopic examination ofa fine-needle aspirate shows a monotonous population (>90%) ofround cells with a high nucleus/cytoplasm ratio and prominent nucleoli. These cells are larger than tile rare neutrophils you find. Based on this description. what is the most likely diagnosis?
a. b. c. d. e.
www.vet4arab.co.cc
SECTION 4
102
inflammation osteosarcoma lymphosarcoma benign hyperplasia immune stimulation
6. W1Jal is the predominant cell type seen in most chylous effusions?
a. mast cell b. eosinophil c. neutrophil d . lymphocyte e. macrophage 7. Wlljcll oftile following is typical offluid removed from tile peritoneal cavityofa cat with effusive feline infectious peritonitis?
a. clear. high-protein fluid containing primarily lymphocytes b. straw-colored, low-protein fluid containing primarily neutrophils c. straw-colored, high-protein flu id containing primarilyeosinophils d. straw-colored, high-protein fluid containing primarily neutrophils e. s traw-colored. high-protein fluid containing primarily lymphocytes
9. A KurfoJfbody is an eosinophilic-staining inclusion seen normally witllin the lymphocytes of
14. All exaggerated response to mild anemia, in the form of polychromasia, nucleated erythrocytes,
and basophilic stippling. is most often seen in
a. rats b. rabbits c. reptiles d. hamsters e. guinea pigs
animals wilh : a. iron toxicity b. iron deficien cy c. lead poisoning d. folate deficiency e. copper deficiency
10. Wllich of tile following is not classified as a round-cell tumor?
15. Larce '/limbers of spherocyres are characteristic of
a. histiocytoma b. mastocytoma c. osteosarcoma d. lymphosarcoma e. trans missible venereal tumor
a. anem ia of lead poisoning b. Heinz-body anemia c. iron-deficiency anemia d. anemia of chronic disease e. immune-mediated hemolytic anemia
11. Which of the following best describes the most
16. Tn dogs the normal activated clottjng rime is:
likely hematologic findings in a dog given corticosteroids 12 to 24 hours previously?
a. lymphoCYlOsis. neutropenia. monocytosis. eosinopenia b. lymphopenia, neutropenia, monocytopenia, eosinopenia c. lymphopenia, mature neutrophilia, monocytosis. eosinophilia d. lymphopenia. mature neutrophilia. monocytosis, eosinopenia e. lymphocytosis. mature neutrophilia, monocytosis, eoSinopenia 12. In horses, which of the following is the best indicator ofa bone marrow response to anemia?
a. b. c. d.
polychromasia increased reticulocyte count improved mucous membrane color increased packed cell volume on serial determinations , e. increased nucleated red blood cell count
8. Megakaryocytes are ti,e precursor cells to:
a. platelets only b. lymphocytes only c. red blood cells only d. segmented white blood cells only e. all circulating blood cells except lymphocytes
Hematology and Cytology
13. Von Wiflebrand's disease is most common in:
a. boxers b. Irish setters c. Labrador retrievers d. Doberman pinschers e. German short-haired pointers
Cl 1998 Mosby-Year Book, Inc. Photocopying is prohibited by law.
•.
,
less than I minute b. less than 2 minutes c. less than 3 minutes d. less than 4 minutes e. less than 5 minutes
18. A 6-momll-o/d puppy has had cOllghingand all OCu/ollasai discharge for the past week. TIle puppy was vaccinated at 10 and 12 weeks a/age. A blood smear stained Wilh Diff-Quik reveals homogeneous eosinophilic-staining inciusiom ill red blood cells and nemrophils. The most likely cause ofthis puppy's illness is:
a. b. c. d. e.
a. Activated c10tfing time is normaJ in both instances. b. Platelet count is generally elevated in DIC and normal in warfarin toxicosis. c. Platelet count is generaJly elevated in Die and decreased in warfarin toxicosis. d. Concentrations of fibrin degradation products are typically increased in DIC and normal in warfarin toxicosis. e. Concentrations of fibrin degradation products are typically normaJ in DIe and increased in warfarin toxicosis.
parvovirus infection ehrlichiosis borreliosis distempet pneumonia. \vith normaJ morphologic changes seen in the blood cells of young animals
19. On an impression smear made from an ulcerated area on tile forelimb ofa cat. you find multiple. small (2 to 4 ~m). basophilic-scainingorganisms whose shape varies from round to oval to fusiform (cigar shaped). wirh a thin, clear halo. The most likely calise of the lesion in this cat is:
a. b. c. d. e.
17. Concerning differentiation of warfarin toxicosis from disseminated illtravascularcoagulation (DIC), wh ich statemetlt is most accurate?
103
leishmaniasis toxoplasmosis sporotrichosis histoplasmosis cytauxzoonosis
20. In a cat witll a packed cel/volume (PCV) of5%. an adequate regenerative response is suggested by an aggregate reticulocyte cOlmt ofat least:
a. J% b. 2% c. 5% d.10% e. 20% 21. Whicll organism commonly affects the central nervous system and might be found on cytologic evaluation ofcerebrospinal fluid?
a. Sporothrix schenckii b. CoccidiQides immitis c. Histoplasma capslliatum d . Blastomyces dermatitidis
e. Cryptococcus neoformans
Corm;t arlSwerSQn- 0" pages 124- 130.
www.vet4arab.co.cc
SECTION -4
104 22. A cow in respiratory distress has pale mucous
membranes. Evaluation ofa blood sample reveals small, round, basophilic-staining inclusions on the margins a/many red blood cells. Based on this informatioll, what is the most likely diagnosis?
26. Red blood cell Willi no central pa/lorand much smaller thall normal 27. Differentiated lymphocyte that produces immunoglobulin
a. babesiosis h. leptospirosis c. molybdenum toxicosis
28. Eosinophilic inc/usion normally found in lymphoid cells ofguinea pigs
d. anaplasmosis e. bacillary hemoglobinuria 23. Avian species are hosts to several different blood parasites, which can be idelllified 011 examination afa blood smear. W1Iich of tile following is not a blood parasite a/birds? a. Prototlleca b. Plasmodium c. TIypanosoma d. Haemoproteus e. Leukocytozoon 24. Whicilleukocyce predominates in the peripheral blood ofhamsters?
a. basophil h. eosinophil
30. ImmwlOglobulin -fiffed vacuole ill tile cytoplasm of plasma ceffs
For Questions 31 through 35, select the correct answer from the five choices below.
a. metaplasia b. dysplaSia c. crenation d. carcinoma e. sarcoma 31. Morphologic change of red blood cel~picaffy caused by dehydration and characterized by multiple indentatiollS in the cell membrane 32. Malignant neoplasm ofepithelial-cell origin
25. Which term describes the sex chromatin lobe seen as a "drumstick" on the nucleus ofa neutrophil from afemaleanimal?
a. b. c. d. e.
8arrbody Heinz body Russell body Curschmann body Pappenheimer body
For Questions 26 through 30, select the correct answer from the five choices below. a. b. c. d. e.
Kurloffbody Russell body plasma cell spherocyte schistocyte
37. The major role of VOIl Willebrand's factor in the coagulation cascade is ill:
a. b. c. d. e.
lysis of clots formation of platelets activation of factor X activation of factor XII platelet adherence to damaged tissue
38. The normal Ufe span ofa canine red blood cell is: 29. Fragment ofa red blood cell
c. monocyte d. ne utrophil e. lymphocyte
Hematology and Cytology
33. Tissue challge caused by chronic irritation and characterized byf'eplacement ofolle mature cell type willi allother mature cell type
a. b. c. d. e.
30 days 60 days 90 days 120 days JSOdays
intrinsic pathway only extrinsic pathway only common pathway only intrinsic and common pathways extrinsic and common path,vays
40. TIle activated partial thromboplastin time (APIT) is used to evaluate which aspect of the
coagulation cascade?
a. b. c. d. e.
il1(rinsic pathway only extrinsic pathway only common pathway only intrinsic and common pa£hways extrinsic and common pathways
34. Maligllant neoplasm ofmese1lchymal-cell origin 35. Nonneoplastic tissue change associated with variOlLs factors, such as inflammation, and characterized by irregular, atypical, and proliferative changes in cell populations 36. Steatitis defines:
a. inflammation or the mammary glands b. excessive lipid globules in feces, associated with enteritis c. inflammation of adipose tissue d. excessive conjugated bilirubin in the serum, associated with hepatitis e. inflammation of lymph vessels C 1998 Mosby-Year Book. Inc. Phorocopying is prohibired by law.
42. Leukocytosis is all increase in the total white blood cell count above the normal range. An increased number of which cell type most commonly characterizes leukocytosis in dogs?
a. basophil b. eosinophil c. monocyte d. neutrophil e. lymphocyte 43. YOllfind rare, smail, roulld, pale blue- to gray_ staining. mulberry-like morulae in the cytoplasm of neutrophils ofa 3-year-old English poimer with polyarthritis. The most likely cause of this finding is:
39. One-slage prothrombin rime (OSPT) is used to evaluate which aspect ofthecoaglllntioll cascade?
a. b. c. d. e.
105
41. C0 11cern ing corticosteroid-i1lduced neutrophilia, which statemelll is kastaccurate?
a. Release of mature neutrophils from bone marrow is increased. b. The peak response is seen t2 to 15 hours after corticosteroid administration. c. The magnitude of neutrophilia diminishes with long-term corticosteroid therapy. d . Leukocyte values relUm to pretreatment levels within 24 hours after a single dose of corticosteroids. e. Neutrophils remain in circulation longer because of decreased margination of cells.
a. b. c. d. e.
distemper babesiosis ehrlichiosis leptospirosis histoplasmosis
44. Wllicll of the following is least likely to cause macrocytic anemia? a. b. c. d. e.
erythremic myelosis folic acid deficiency vitamin 8 12 defiCiency iron defiCiency cobalt deficiency in ruminants
45. Which type ofanemia is most likely to be macrocytic hypochromic?
a. b. c. d. e.
regenerative anemia anemia associated \vith iron deficiency anemia associated with copper deficiency anemia associated with folic acid deficiency anemia associated with bone marrow suppression
46. In a cat with leukocytosis induced by endogenous epinephrine (physiologic leukocytosis), the leukocytosis is attributable to:
a. b. c. d. e.
neulJ'ophilia only lymphocytosis only neUlJ'ophilia and monocytosis neutrophilia and lymphocytosis monocytosis and lymphocytosis
Correct answers are 0" pages 124·130.
www.vet4arab.co.cc
SECTION 4
106
47. Which leukocyte predominates in the peripheral blood ofnormal dogs?
a. basophil b. monocyte c. neutrophil d. eosinophil e. lymphocyte
53. Which of the following is an inherited disease causing a defect in leukocyresegme'ltatioll?
48. Wllieh leukocyte predominates in the peripheral blood ofa normal weanling pig?
a. basophil b. monocyte c. neutrophil d. eosinophil e. lymphocyte
a. b. c. d. e.
Evans' syndrome Pelger-Huet anomaly mucopolysaccharidosis CMdiak-Higashi syndrome neutTophii granulation anomaly of Birman cats
54. Which oflhe following causes microcytic hypochromic anemia?
49. Which leukocyte predominates in tile peripheral blood ofa normal adult cow?
a. basophil b. monocyte c. neutrophil d. eosinophil e. lymphocyte
a. b. c. d. e.
renal fail ure iron deficiency estrogen toxicity folic acid deficiency inflammatory disease
55. Avian tllrombocytes differ from mammalian tllrombocytes in that avian tllrombocytes:
50. Heinz bodies are most likely to be detected on a smear a/blood from healthy:
a. pigs b. cats c. dogs d. cattle e. horses
a. b. c. d. e.
are phagocytic are nucleated cells have no role in hemostasis can transport oxygen playa major role in immune responses
56. Concerning protamine sulfate, which statement is most accurate f
51. Wll ich type o[abnormaJ red blood cell indicates microangiopathic hemolysis?
a. target cell b. lep tocyte
c. schistocyte d. Slomatocyte e. Mott cell
a. It is an irreversible anticoagulant. b. It is a potentiator of sodium citrate in anticoagulation. c. It is a cause of Heinz-body anemia in cats. d. I{ is used in an early screening test for disseminated intravascular coagulation. e. II is a common cause of autoimmune hemolytic anemia in dogs. 57. Which cell type can divide by mirosis?
52. Which of the following is most difficult to d ifferentiate from an Anaplasma organism on a Romanowsky-slllined blood smear?
a. Heinz body b. Dohle body
c. Schmauch body d. Howell-Jolly body e. erythrocyte refractile body
a. b. c. d. e.
myelocyte reticulocyte metamyelocyte band neutrophil segmented neutrophil
C 1998 Mosby-Year Book, Inc. Photocopying is prohibited by law.
Hema tology and Cyto logy
58. Which type of hemorrhage is not typically seen in animals with platelet abnormalities or thrombocytopenia?
a. b. c. d. e.
epistaxis hematuria ecchymosis petechiation hemarthrosis
107
d. Kupffer cell e. T lymphocyte For Questions 63 through 67, select the correct answer from the five choices below.
59. Which of the followillg is IlOt a hereditary
a. b. c. d. e.
basophilic stippling Heinz body rouleaux agglutination Howell-Jolly body
coagulopathy?
a. b. c. d. e.
Pelger-Huet anomaly factor X deficiency von WiUebrand's disease hemophilia A (factor VIII deficiency) hemophilia B (factor IX deficiency)
60. In which species has cytauxzoonosis not been reported or described?
a. b. c. d. e.
dogs bobcats cheetahs antelopes domestic cats
63. Random clumping oferythrocytes
64. Organized grouping of red blood cells, resembling stacked COiTlS 65. Mulear remnaflts within mature (nonnucleated) red blood cells 66. Clumps of denatured hemoglobin within red
blood cells 67. Ribosomal clumping in red blood cells tliat stains basophilic witl! Wrights stain and may appear as distinct granules or fine dots
61. Tn a blood smear from a markedly anemic cal,
youfind large nllmbers ofllucleated red blood cells in the absence of polychromasia alld reticulocytosis. The most li/cely cause of this finding is:
a. b. c. d. e.
lead poisoning erythremic myelosis Pelger-Huet anomaly lymphoblastic leukemia Chediak-Higashi syndrome
a. b. c. d. e.
siderocyte Mott cell Kupffer cell ferrocyte Downey cell
69. Which of tile following is sometimes referred to as plumbism?
62. A differentiated B lymphocyte develops into a:
a. rubricyte b. siderocyte c. plasma cell
68. A mature red blood cell containing visible iron particles (Pappenheimer bodies) is known as a:
a. b. c. d. e.
iron deficiency copper toxicity lead poisoning lymphosarcoma multiple myeloma
Correcranswers are on pages 124-130.
108 70. Tile hematologic response associated with hypothyroidism in dogs is:
a. polycythemia b. erythroleukemia c. normocytic normochrom ic anemia wim leptocytosis d. microcytic hypochromic anemia with spherocytosis e. macrocytic hypochromic anemia with reticulocytosis 71. Which pair of species can become infected with Ehrlichia?
a. dogs and pigs b. horses and pigs c. dogs and horses d. cattle and sheep e. horses and canle
astrocytes plasma cells endothelial cells precursor cells of granulocytes myelin.producing cells of the central nervous system
73. Which ofthefollowillg is not associated with Heinz -body formmio,,? a. b. c. d. e.
d. increased erythropoietin production e. splenic contraction in stressed animals 75. What is the most common cause of polycythemia ;nanimals?
a. b. c. d. e.
onions red maple bracken fern new methylene blue phenothiazine dewormers
74. Polycythemia uera is associated with:
a. pulmonary hypoxia b. reduced atmospheric oxygen c. a malignant stem·cell disorder
polycythemia vera familial erythrocytosis disorders resulting in hypoxemia abnormal erythropoietin production dehydration and splenic contraction
77. eonceming the effect ofcorticosteroids on
..
a. 11ley have no effect on granulopoiesis or lymphopoiesis. b. 11ley inhibit granulopoiesis and lymphopoiesis. c. They stimulate granulopoiesis and lymphopoiesis. d. They inhibit granulopoiesis but stimulate lymphopoiesis. , e. They stimulate granulopoiesis but inhibit lymphopoiesis. 7B. The proteins responsible for blood coagulation are
produced primarily in the:
a. b. c. d. e.
liver spleen kidney pancreas small intestine
79. In a vaginal swab from a 2·year-old intact miniature schnauzer,youfind 96% superficial celis, with small pyknotic nuclei. Thisfinding indicates that this bitch is most likely:
a. b. c. d. e.
a. b. c. d. e.
estrogens thyroxine androgens corticosteroids growth hormone
grallulopoiesisand lymphopoiesis. which sMtemeru is most accurate?
Hematology and Cytology
in estrus in diestrus in anestrus in proestrus infected with a fungus
c. leukocytosis caused by neutrophilia and lymphocytosis d. leukocytos is caused by neutrophilia and monocytosis e. leukopenia primarily caused by neutropenia
aplastic anem ia myeloid leukemia sevcre le ukopenia erythrOid le ukemia iron-deficiency anemia
a. b. c. d. e.
immune-mediated thrombocytopenia feline infectious anemia equine infectious anemia immune-mediated hemolytic anemia microangiopathic hemolytic anemia
85. WI/icll abllormality is associated Will, Willebrand's disease?
81. In a transtraclleal wash from a couglling dog. you find columnar cells, macrophages, a mixed population of bacilli alld cocci, and numerous Simonsiella organisms. The most likely cause of these findings is:
a. b. c. d.
a hypersensitivity reaction oropharyngeal contamination a mixed fungal and bacterial infection a mixed bacterial infection associated with immunosuppression e. a fungal infection associated with overuse of antibiotics
a. b. c. d. e.
VOII
thrombocytosis thrombocytopenia thrombocytopathy deficiency of factor IX deficiency of factor VI I
For Questions 86 through 90, select the correct answer from the five choices below. a. b. c. d. e.
major crossmatch minor crossmatch Russell's viper venom time (Rvvr) intravascular hemolysis extravascular hemolysis
86. Associated Willi babesiosis in cattle
82. Which cell type may occasionally be observed in syrlOllial flllid and is diagnostic of systemic lupus erythematosus?
a. b. c. d. c.
lupus erythematosus (LE) cells Mott cells giant cell leplOcytcs siderocytes
87. Test using recipient's plasma and donor's cells 88. Associated with anaplasmosis in cattle 89. Used to differentiate factor VII deficiencies from common pathway coagillopathies
83. loW/icll hematologic finding is most likely to be observed in cottle with severe, acute inflammation?
90. Test using dOllor's plasma and recipient's cells
a. leukocytosis caused by neutrophilia only b. leukocytosis caused by lymphocytosis only
@1998Mosby-YearBook,Inc.Photocopyingisprohibitedbylaw.
109
84. A Coombs' test is used to help diagnose:
80. Which hematologic abnormality is associated willI IOllg-term bracken-fern jngestion in cattle?
76. Which of the following does not stimulate erythropoiesis?
a. b. c. d. e.
72. From which cell rype does multiple myeloma arise?
a. b. c. d. e.
www.vet4arab.co.cc
SECTION 4
Correcr answers are on pages 124-130.
110
www.vet4arab.co.cc
SECTION 4
91. Theanemia seen associated with:
a. b. c. d. c.
jn
chronic Tenalfailure is
lack of e rythropoietin production hemolysis caused by electrolyte imbalances splenic removal of abnormal red blood cells loss of red blood cells in the form o f hematuria stress· induced estrogen release and resultant bone marrow suppression
92. In dogs, w/lere is erythropoietin produced?
c. meat d. eggs e. legumes 97. Mwt protein rransports iron in tile plasma?
a. b. c. d. e.
transferrin lipofuscin hemosiderin chromaffin transcobalamin
a. liver
H ematology and Cytol ogy
102. A giallC cell isformed by /iJSion of
a. heterophils
98. A Serto{j-cell OImor is most likely to occur:
93. Which organisms are referred to as marginal bodies?
a. Babesia b. Anaplasma c. Eperythrozoon d. Hem obarlollclla e. Ehrlichia
a. b. c. d. e.
in the mouth of a cat around the eye of a horse on the distal limb of a 6-month-old dog in an ovary of an 8- to 12-year-old female dog in a retained (abdominal) testicle of a 7-yearold dog
103. III w/tich type of leukocyte is ~tox;c change" most commonly seen?
a. basophils
99. Wlljc/I disorder is least likely to produce peripheral neutropenia?
a. hyperadrenocorticism
c. monocytes d. neutrophils e. lymphocytes 104. In which dog breed do eosinophilsfrequently have more vacuoles t/Jan granules?
a. b. c. d. e.
akita boxer Samoyed greyhound A1askan malamute
h. parvoviral infection
94. Which of the [oI/owing does not increase nwnbers ofcirculating platelets in animals with
immune-mediated thrombocytopenia? a. vincristine
c. rupture of the cecum d. overwhelming bacterial infection e. Salmonella- induced endotoxemia in a horse
c. tetracyclines d. corticosteroids e. cyclophosphamide 95. Which blood type is most common in cats in the United Stales?
a. F b. Tr
a. leptocytosis b. anisocytosis c. poikilocytosis d. reticulocytosis e. stomatocytosis
96. Which of the followin g is a poor source of iron?
a. fo lded in half c. variable shapes d. organized like a stack of coins e. s malle r than normal, with no cen tral pallor 106. Concemillg histiocytomas. whicll statement is most accurate?
a. They are relatively more common in beagles. h. They contain distinct spindle-shaped ceUs.
/01. A heterophil is most likely to befollnd in a:
c. B d.AB e. A
105. Mlich of fhe following best describes anisocytosis oferythrocytes? h. variable sizes
100. A general term Ilsed to describe abnormal and bizarre shapes in erythrocytes is:
h. splenectomy
a. pig h. cat c. dog d. bird e. Uama
107. In wliich ofthefollowing is a Diilile body most likely to befoulld?
eosinophil in a greyhound cytoplasm of a toxic neutrophil cytoplasm of a thrombocyte in a bird nucle us of a liver cell in a puppy with infectious canine hepatitis e. cytoplasm of a lymphocyte stimulated to produce antibodies a. b. c. d.
b. neutrophils c. hepatocytes d. lymphocytes e. macrophages
h. eosinophils
b. kidney c. spleen d. thyroid e. bone marrow
c. They tend to occur in youn g dogs.
108. In a transtracheal wash from a horse,youfind ciliated and nOllciliated columnar and cuboidal cells. alveolar macropl/ages, an occasional neutrophil. and approximately 20% of cells as eosinop/ti/s. You observe no superficial squamolJS epithelial cells or Simonsiella organisms. Which of the following is tile most appropriate cytologic interpretation?
a. This is a normal ceUpopulation. b. There is evidence of malignant neoplaSia. c. There is evidence of a hypersensitivity reaction . d. There is evidence of bacterial or fu ngal inflammation. e. There is evidence of oropharyngeal contamination. 109. Concemillg the procedure ofpercutaneous trallStracheallbronchial wash, which statement is most accurate?
a. It cannot be done without general anesthesia. h. It is preferred in frac tious animals. c. It requires use of a sterilized endotracheal tube. d. It reduces the risk of oropharyngeal contamination. e. It involves passage of a needle and catheter through the thyroid cartilage.
d. They can metastasize to the lungs. e. Azurophilic granuJation can obscure the nucleus ofhlstiocytoma cells.
a. dirt b. milk
e 1998 Mosby-Year Book. Inc. PhotOCOpying is prollibiled by law.
III
Correct answers are on pages 124-130.
www.vet4arab.co.cc
SECTION 4
112
110. Which a/the following best characterizes a septic
exudate? a. low protein conlent (<3.0 g/dl), predominantly lymphocytes present, no bacteria seen or cultured h. low protein content (<J.O g/d]), predominandy
nondegenerate ncuU'ophils present, no bacteria seen or cultured c. high protein content (>3.0 g/dl), predominantly degenerate neutrophils present, intracellular and lor extracellular bacteria present d. low protein content (<3.0 g/dl), predominantly
nondegenerate neutrophils present, intracellular and/or extracellular bacteria present e. high protein content (>3.0 gtdl), predominantly
foamy macrophages and nondegenerate neutrophils present, intracellular andlor extracellula r bacteria present
c. monocytic leukemia d. p lasma-cell myeloma e. reticuloendotheliosis
117. Which term is used to identify the blue-black iron-containing particulate matter seen in bone marrow macrophages?
114. Which of tile following best describes an
osteoblast? a. arises from the granulocytic cell line b. a giant cell containing six to ten randomly arranged nucle i c. a cell that produces calcilOnin within the parathyroid gland d. the precursor cell to oth er skeletal system cells, such as chondrocytes and fibrocytes e. an ovoid, p lump cell resembling a large p lasma cell. with a round, eccentric nucleus and dark b lu e cytoplasm 115. Concemingcytologic evaluation of bone
Hema tology and Cytology
...
a. b. c. d. e.
bilirubin melanin hematoidin hemoglobin hemosiderin
c. myeloblast, promyelocyte, myelocyte, metamyelocyte, band, segmented neutrophil d. myelocyte, myeloblast. metamyelocyte, promyelocyte, band, segmented neutrophil e. myelocyte. metamyelocyte. promyelocyte. myeloblast, band, segmented neutrophil 122. In which species are the granules ofeosinophi/S
almost exclusively rod shaped? a. dogs b. cats c. cattle d . sbeep e. h orses
118. W1lich disorder is most likely to cause overall
lIyperceffularityofhematopoietic tissue in bone marrow?
a. myelofibrosis b. erythroleukemia c. aplastic anemia d. chronic ehrlichiosis e. chronic renal failure
123. Concerning immature canine red blood cells.
wllich statement is least accurate? a. Polychromatic cells are generally larger than mature red blood cells. b. All reticulocytes are polychromatophilic with Romanowsky stains. c. Polychromatophilic cells contain less hemoglobin than mature red blood cells. d. Mature erythrocytes stain poorly with supravital stains, such as new methylene blue. e. Reticulocyte counts are perfonned on smears stained with a supravital stain, such as new methylene b lue.
specimens. which statement is wast accurate? 111. Which cell type comprises the liningo/tlle pleural, peritoneal. and uisceral surfaces?
a. Kupffer cell b. epithelial cell c. endothelial cell d. mesothelial cell e. reticuloendothelial cell 112. Conceming normal cerebrospinal fluid in dogs.
a. Samples are relatively difficult to obtain. b. Coccidioidomycosis is a common cause of osteomyeBtis. c. Thmors of bone are difficult to differentiate cytologically. d . The most common finding in osteomyelitis is a n eutrophilic exudate. e. The most frequently identified bone tumor is osteosarcoma. closely followed by fibrosarcoma. ..
whicll statemem is most accurate? a. It should contain fewer than 8 nucleated cells/~L
113. Which neoplasm is most likely toS/IOW
radiograph ic evidence ofbone lysis? a. lymphosarcoma b. erythremic m yelosis
@ 1998
overall hypercellularity of hematopoietic tissue in bone marrow? irradia tion myelofibros is suppression by feline leukemia virus (FeLV) bone marrow necrosis anemia of chronic inflammatory disease
120. What is the correct sequence ofmaturation ;'1 the rOO
blood cell line,from least mature to most nUllure? 116. Concemingcytologic evaluation of synovial fluid
ofdogs, which statemefll is least accurate?
b. Nucleated cells should consist primarily of nondegene rate neutrophils. c. Cell counts should be performed 2 to 24 hours after collection of Ule sample. d. Normal cerebrospinal flu id is clear and pink, with a total protein concentration of 50 to 100 mg/ dL e. Normal cerebrospinal fluid is clear and yellow, with a total protein concentration of 50 to 100 m g/dL
119. Wllich oftllefollowing is most likely to cause
a. b. c. d. e.
113
a. The presence of lupus erythematosus (LE) cells in synovial flu id is diagnostic of lup u s erythematosus. b. Normal synovial fluid is of low cellularity «3000 ce ll s/ ~l). c. Osteoclasts are not unusual in synovial fluid because these cells are present in articular cartilage. d. Organisms are often difficult to identify in inflammatory arthropathies of bacterial origin . e. An increased nucleated cell count with more than 90% mononuclear cells su ggests degenerative joint disease.
Mosby-Year Book, Inc. Photocopying is prohibited by law.
a. reticulocyte, rubriblast, metarubricyte, prorubricyte. rubricyte. erythrocyte b. rubriblast, metarubricyte, prorubricyte, rubricyte, reticulocyte, erythrocyte c. rubriblast, prorubricyte, rub ricyte, metarubricyte, reticulocyte, erythrocyte d. rubriblast. rubricyte, metarubricyte, prorubricyte, reticulocyte, erythrocyte e. prorubricyte. metarubricyte, rubricyte, rubriblast, reticulocyte, erythrocyte
J24. Col1ceming cytologic elltlluation of lymph nodes,
121. Mlat is tile correct sequence of maturation in
the myeloid cell line, from least mature to most mature?
which statement is least accurate? a. Ruptured cells are a common artifact becau se of th e fragility of lymphoid tissue. b. There is no true cytologic difference between n ormal and hyperplastic lymph nodes. c. Small lymphocytes typically comprise at least 75% of a normal lymph node cell population. d. Lymphadenitis can be reliably diagnosed when neulrophils make up greater than 10% o f the nucleated cell population_ e. Aspiration of foamy epithelial cells in the area of the mandibular lymph node u sually indicates neoplasia.
a. promyelocyte. myeloblast, metamyelocyte. myelocyte, band, segmen ted neutrophil b. myeloblast, promyelocyte, metamyelocyte, m yelocyte, band, segmented neutrophil
OJrrect answers are on
pages 124-/30.
114
www.vet4arab.co.cc
SECT10N 4
Por Questions 125 through 129, select the correct answers from the five choices below.
a. h. c. d. e.
Charcot-Leyden crystals Curschmann spirals tingible bodies hematogones lymphoglandular bodies
125. Free nuclei shed from maturing red blood cells (metarubricytes) 126. Round, basophilic-staining structures, similar in size 10 platelets, produced by cytoplasmic fragmentation in lymphocytes 127. Elongated double pyramidal structures/armed by coalescence ofeasif/opllil granules
128. Prominent dark blue (basophilic) nuclear debris within macrophages
129. Mucous casts formed in small bronchioles 130. The swollen, eosinophilic, lacelike nuclei from ruptured cells are known as:
a. ragocytes b. schistocytes
c. pyriformation d. castellation e. coronification
is not a cytologic criterion o/malignancy for epithelial-cell tumors?
133. Which of the following
a. b. c. d. e.
angular nucleoli uniform nuclear size large, prominent nucleoli high nucleus/cytoplasm ratio large numbers of abnormal mitotic figures
d. caprocytes
ear canal, wliich statement is least accurate?
a. Cerumen, an oily, yellow secretion, stains poorly or not at all. b. A sweet-smelling, pale yellow exudate ismost likely attributed to Pseudomonas infection. c. Malassezia is a broad-based, budding yeast that is the most common cause of mycotiC otitis externa, d. The ear mite that produces a dry, black, granular discharge associated with parasitic otitis externa in companion animals is Otobius megnirli.
e. Some of the most corvmon causes of otitis externa (Maiassezia and bacteria) are often normal inhabitants of the external ear canal. 135. Concerning cytologic examination o/the eye,
which statement is least accurate?
131. Neutrophi/S containing small, dark intracytoplasmic granules, observed on unstained wet preparations 0/ synovial fluid, are known as:
a. ragocytes b. c. d. e.
schistocytes Fleischmann's cells synoviform cells procartiblasts
132. The linear distriburion of cells seen on pull smears o/fluids o/high viscosity is termed:
136. Concerning cytologic examination of nasal masses
and exudates, which statement is least accurate?
a. Most nasal tumors are of mesenchymal origin. b. Rhinosporidium infection in dogs causes a polypoid growth. c. Transmissible venereal tumor can occur in the nasal areas, particularly in males. d. Ciliated pseudostratified columnar epithelial cells originate from the nasal turbinates. e. Cryptococcus neofomums infections produce a mucoid exudate and are most common in cats. 137. Periallal-gland cells can be/ound if! all o/the
134. Concerning cytologic examination ofthe external
c. Fleischmann's celis e. basket cells
Hematology and Cytology
a. Keratoconjunctivitis sicca produces a lymphocytic exudate. b. The anterior uvea is a site for metastasis of systemic carcinomas. C. Neutrophilic infiltration of aqueous humor is characteristic of most cases of anterior uveitis. d. In cats, corneal scrapings containing primarily eosinophils indicate eosinophilic keratitis. e. Most intraocular tumors, either primary or secondary, do not exfoliate into aqueous humor; the exception is lymphosarcoma.
a. rouleaux b. windrowing
following areas except:
a. the tail b. c. d. e.
c. They tend to exfoliate only a few individual cells. d. They show very few characteristics of malignancy. e. They exfoliate small ce[ls with a scant amount of cytoplasm. 141. Each hemoglobin molewle is composed of
a. b. c. d. e.
1\o.ro globin chains and one heme group 1\o.ro globin chains and two heme groups 1\o.ro globin chains and four heme groups Four globin chains and one heme group Four globin chains and two heme groups
142. Which biochemical pathway uses glucose to
the ears the thighs the prepuce around the anus
generateATP in mature canineerylhrocytes?
138. COllcerning liposarcomas, which statement is
most accurate?
a. They are benign tumors of adipose tissue. b. They are almost always secondarily infected. C. Only very small lipid globules are observed in cells aspi rated from liposarcomas. d. They arise most commonly in subcutaneous tissue of the shoulders, thighs, and trunk. e. After alcohol fixation, cytologic samples can be stained with special stains, such as Sudan rv, to verify the presence of lipid.
a. b. c. d. e.
Embden-Meyerhofpathway aminolevulinic acid pathway Luebering-Rapoport pathway hexose monophosphate pathway methemoglobin reductase pathway
143. In dogs, what is the last erythrocyte stage in
which the nucleus is still evident?
a. b. c. d. e.
rubricyte erythrocyte prorubricyte reticulocyte metarubricyte
144. Which of the following best describes lipemic 139. Concerning cytologic examination of squnmous-
plasma from
a flormal, flon/asted dog?
cell carcinomas, which statement is least accurate?
a. clear and yellow
a. They are often secondarily infected. b. The [Umor masses tend to yield groups of cells. C. Perinuclear vacuolation is a common cytologic finding. d. They are fmUld only in poorly pigmented areas of the skin exposed to ultraviolet radiation. e. They are tumors of epithelial origin.
b. opaque and white c. clear and colorless d. opaque and yellow e. clear and pink to red
140. Concerning tumors o/epithelial origin, which
statemeTil is most accurate?
a. They include tumors of endocrine glands. b. When malignant, they are called sarcomas.
© 1998 Mosby-Year Book, Inc. Photocopying is prohibited by law.
115
145. Which species lias the smallest erythrocyte?
a. cats b. pigs c. dogs d. goats e. horses
Correct answers art! on pages 124-130.
1I6
www.vet4arab.co.cc
SECTION 4
146. Which species has tile largesl erythrocyte? b. pigs c. dogs d. horses
a.
e. sheep 147. What term describes tile random clumping (grapelike aggregation) oferythrocytes tliat occurs in some cases ofantibody-mediated
b. coalescence c. poikilocytosis d. agglutinatio n e. polychromasia
148. Concerning reticulocytes in the peripheral circulation a/horses, which statemeTll is most accurate?
a. Reticulocytes are absent in both health and regenerative anemia. b. RCliculocyrcs are a bscnI in health but present in regenerative anemia. c. Rcticulocytcs are present in health but morc numerous in regenerative anemia. d. Reticulocytes are a prominent feature in healthy foals, less numerous in adult horses, and more numerous in regenerative anemia. e. RCliculocytes are found in two forms: punctale reticulocytes are present in health, and aggregate reticulocytes are present in regenerative anemia. 149. Which species has two types of reticulocytes (aggregate and punctate)? b. cats c. dogs d. cattle e. goats
a. b. c. d. e.
I
b.2 c. 3 d. 7 e. 14
a. b. c. d. e.
acquired valvular insufficiency decreased Oleygen flow to the heart a congenital defect not detected earlier occult heartworm disease decreased blood viscOSity, causing turbule nce
,
152. Conceming the differences between Ilemalytic anemia alld hemorri!Ogicanemia, which statement is most accurate?
a. The reticuJocyte response is slower in hemolytic anemia. b. The plasma prOlein concentration is decreased in hemolytic anemia. c. The plasma protein concentration is normal to increased in hemolytic anemia. d. The plasma protein concentration is normal to increased in hemorrhagic anemia. e. Hyperbilirubinemia is often present with anemia caused by external hemorrhage. 153. Erythrocytesfrom which species are most likely to contaill erythrocyte refractile bodies in health?
a. b. c. d. e.
pigs cats dogs horses llamas
o 1998 Mosby- Year B(}()/c, Inc. Pll otocopying is prollibited by law.
117
159. Macropllages are derived from wliiclileukocyte?
a. basophil b. mo nocyte c. eosinophil d. n e utrophil e. lymphocyte
osmotic lysis physical injury oxidative injury complement-mediated destruction increased macrophage activity
160. Wllich of the following is not afunction of
previously, you detect a heart murmur. The dog also has tachycardia, taC#lypnea, and pale mucous membranes, with a packed cell voilime af 10%. W1lDt is the most likely cause afthe heart murmur?
a. rouJeaux
154. Which aftllefollowing is ,wta mechanism by
which intravascular 1/e1lJolysis may occur?
151. In a 3-year-old dog that was normal 2 months
anemia?
a. pigs
150. After a single acute anemic crisis (hemorrhage or
hemolysis) in a dog, how many days are generally required for peripheral blood reticu/ocytes to become most numerous?
a. cats
Hematology and Cytology
155. Wlilch of tile following best describes the mechanism ofanemia associated witll myeloplltlt isic disease?
a. lack of erythropoietin b. iron diversion to storage pools c. infection with such agents as panleukopenia virus d. dietary deficiencies, such as vitamin B l2 and folic acid e. physical replacement of bone marrow by abnormal proliferation of cells
eosinopllils? a. b. c. d.
activation of plasminogen synthesis of complement components phagocytic and bactericidal capabilities complement ·dependent parasiticidal properties e. inhibition of chemical mediators released from mast cells in anaphylactic reactions
161. Which cells are the only source ofendogenous
heparin? a. b. c. d. e.
156. All tile following coaglilation factors are produced in tile liver except:
a. h. c. d.
faClor X factor IX factor VII fibrinogen e. von Willebrand factor
162. In reference to the leukogram, which of the
followillg bestdefilles a left shift? a. b. c. d. e.
157. About how much time does a mature nelltropliil
spelld circlilating witllin the peripheral blood?
a. 5 hours b. c. d. e.
basophils only mas t cells only eosinophils only basophils and mast cells eosinophils and mast cells
10 ho urs 15 hours 24 hours 72 hours
toxic change in n e utrophils inc reased segmentation of all leukocytes increased numbers of mature neutrophils increased numbers of immature platelets increased numbers of immature neutrophils
163. W1licll oflllefollow/ngdoes not cause peripheral
blood lymphopenia?
158. The circlliating pool of segmented nelitTophils is
approximately one tllird tile size of the marginated pool ill:
a. pigs b. cats c. dogs d. cattle e. horses
Correct answers are on
a. b. c. d. e.
viral infections lohne's disease epinephrine res ponse ruptured thoracic duct immunosuppressive therapy
pages 124-130.
www.vet4arab.co.cc
SECTION 4
118
164. A neutrophil must contain how many distinct
nllclear lobes to be described as hypersegmented?
a. one or more b. two or more
c. three or more d. four or more e. five or more 165. What;s the average life span ofa circulating platelet?
a. b. c. d.
1 day
3 days 5 days 10 days e. 14 days 166. Omcerning thrombin. which statement is least
accurate? a. b. c. d. e.
It accelerates fibrinolysis.
It accelerates coagulation. It results in fibrin formaLion. It is a potent platelet antagonist. It is inunediately derived from factor II.
169. Which ofthefollowing is associated with an exudate?
a. b. c. d. e.
hypoproteinemia right heart failure lymphatic obstruction mycobacterial pneumonia congestive heart failure with ascites
170. Wllich of tile following is tile most defining characteristic ofan exudate?
a. b. c. d. e.
ceUcount protein concentration eryth rophagocytosis cholesterol/ triglyceride ratio >1 cholesterol/triglyceride ratio <1
171. W1,icll of the following is nota typicalfinding in a long-standing or resolving hemorrhagic effusion?
a. b. c. d. e.
thrombocytosis erythrophagocytosis hypersegmented neutrophils pink or xanthochromic supernatant macrophages containing black pigment (hemosiderin)
167. Whicli ofthe following is an in lJivo test of
platelet Junction? a. b. c. d. e.
platelet time bleeding time thromboplastin time activated clotting time Russell's viper venom time
168. Which althe/allowing best describes a
neutrophil that is swollen and exhibiting karyolysis, loss of nuclear membrane. and homogeneous pink-staining chromatin?
a. b. c. d. e.
band neutrophil toxic neutrophil reactive neutrophil neoplastic neutrophil degenerate neutrophil
172. In horses, neonatal isoerythrolysis is most commonly seen:
a. after parturition in primiparous dams b. in first-born offspring after ingestion of colostrum c. during the last half of gestation if the dam is sensitized to the foal's erythrocytes d. in the dam after breeding if the dam was previously exposed to the stud's erythrocytes e. in neonatal foals after ingestion of colostrum if the dam has been sensitized to fetal erythrocytes 173. Levels of which plasma protein of large animals are used to detect internal inflammatory lesions?
a. b. c. d. e.
kallikrein duombin serotonin fi brinogen plasminogen
C 1998 Mosby-Year Book, Inc. Photocopying is prohibited by law.
Hematology and Cytology
174. Which of tile following cannot be counted !vith a hemocytometer?
a. b. c. d. e.
platelets le ukocytes eryth rocytes reticulocytes erythrocytes and leukocytes
175. Which tenn describes the average volume ofa population of red blood cells, expressed in femtoliters?
a. b. c. d. e.
hematocril packed cell volume mean corpuscular volume mean corpuscular hemoglobin mean corpuscular hemoglobin concentration
179. Hematologic examination ofa bouine blood sample reveals a packed cell volume of 10%,10 nucleated RBCsIlOO WHCs, marked basophilic stippling, and marked polychromasia. What is the most likely cause of these findin gs?
a. b. c. d. e.
Plasma fibrinogen levels increase \vith age. It is produced in the spleen. It precipitates at 50° C. Plasma fibrinogen levels decrease with systemic inflam mation . e. Plasma fibrinogen levels cannOl be accurately determined on heparinized samples. 177. An iniJerited, semilethal condition resulting in cyclic neutropenia (cyclic hematopoiesis) occurs in:
a. b. c. d. e.
bull terriers Gordon setters Newfoundlands silver-gray collies Rhodesian ridgebacks
which statement is most accurate?
a. b. c. d. e.
Birds do not have thrombocytes. Birds lack the extrinsic pathway. Birds lack a complete intrinsic pathway. Birds do not have a common pathway. In addition to the intrinsic, extrinsic. and common pathways, birds have another dotting pathway.
181. Conceming hematopoiesis in auian bone
marrow, which statemellf is most accurate?
a. b. c. d.
Rubriblasts are not evident. Myeloblasts are not evident. Megakaryocytes are not evident. Lymphopoiesis occurs primarily in marrow, not in peripheral lymphoid tissue. e. The huge reserve stores of iron occupy approxim ately 90% of the medullary cavity. 182. WIJ iclt of the following is a hematopoietic
neoplasm of lymphoid tissue in chichms?
178. Eosinopllils from which species have a characteristic "raspberry" appearance?
a. p igs b. cats c. cattle d. deer e. horses
lead poisoni ng micro angiopathy regenerative anemia iron-deficiency anemia nonregenerative anemia
180. Conceming the coagulation cascade in birds.
176. Concerning/ibrinogen, which statement is most accurate?
a. b. c. d.
119
a. b. c. d. e.
Marek's disease Jahne's disease l'yzzer's disease pullorum diSease Newcastle d isease
183. Which of the following is a hematologic disorder seen in jntactfemaleferrers wilh repeated anovulatory cycles?
a. b. c. d. e.
aplastic anemia erythroleukemia lymphOid leukemia granulocytic leukemia immune- mediated hemolytic anemia
Correct anslwrs are on pages 124-130.
120
www.vet4arab.co.cc
SECTION 4
184. During the/eta' period ill cats, which structure does 1I0( directlycontribllle /0 blood/ormation?
a. liver b. spleen c. kidney d . yol k sac e. bone marrow
189. In a centrifuged microllematocrit tube containing whole blood. the area containing platelets and leukocytes is called the:
a. band layer b. white band c. buffy coat d . white line e. myelochrome band
185. \.VlIicll a/the/allowing is not afunction althe spleen?
a. reservoir of platelets b. reservoir of leukocytes
c. reservoir of erythrocytes d. reservoir of lymphoid cells with immune function s e. destruction of aged or defective erythrocytes 186. Which term describes the complex series of physical and biochemical events that both promote and inhibit blood clocting?
190. In carnivores tile icterus index is a measure of
a. b. c. d. e.
the amount of bilirubin in serum unconjugated bilirubin in the liver discoloration of the mucosa the amount of bilirubin in urine functional hepatic parenchymal mass
191. A structure released in clusters from tile cytoplasm of megakaryocytes is the:
a. micro karyocyte b. cytoplasmocyte
a. hemoprysis
c. thrombocyte
b. hemos tasis c. hemos iderosis d. he matochezia
c. Russell body
d. astrocyte
e. hematopoiesis
192. A reduction of blood volume below normal levels is known as:
187. Which tenn describes an intravasculardeposil comprised a/fibrin and fomled elements a/blood?
a. embolus b. c. d. e.
•
thrombin thrombus
a. b. c. d. e.
polycythemia relative erythrocytosis compensatory leukocytosis hypovolemia shock
Hematology ",nd Cytology
194. The formatioll alld development of blood cells are termed:
a. b. c. d. e.
hematuresis hemopexis he malOchromatosis hema phe resis hematopoiesis
d. factor VII e. factorVlII 198. A prolonged one-stage prothrombin timealld a normal StyPlJe1l time (Russell's viper vellom time) in a bleeding dog indicate a deficiency of'
a. b. c. d. e.
195. Itl avian species. what hematopoietic neoplasm is induced by a DNA Ilerpesvirus?
a. b. c. d. e.
leukosis-sarcoma complex Pacheco's disease Marek's disease Fanconi synd rome malignant erythroleukemoid syndrome
factor V fa ctor X fa ctor II factorVlI fa ctorVIII
199. Which of tile following does not prolong the thrombin time?
a. b. c. d. e.
196. A prolonged activated partial thromboplastin time and a normal one-stage prothrombin time iTl a bleeding dog indicate a deficiency of:
a. b. c. d. e.
factor V factor X fa ctor II faclOr VII factor VIII
heparin therapy dysfibrinogenemia factor II deficiency severe hypofibrinogenemia high concentration of fibrin degradation products
200. In reference to the leukogram, which of the fof/owing best describes a right shift?
a. increased numbers of immature neutrophils b. increased numbers of hypersegmented neurrophils c. toxic changes in neuuophils d. increased numbers ofJymphocytes with basophilic cytoplasm e. increased cytoplasmic vacuolation of monocytes
197. A normal activated partial thromboplastin time and a prolonged one-stage prothrombin rime in a bleeding dog jndicate a deficiellCY of'
a. factor V b. factor X c. factor II
infarct plaque
193. In aviatl species, what hematopoietic neoplasm is induced by all RNA retrovirus?
188. What;s the larges! aftlle bone marrow lIematopoieticcefls?
a. rubriblast b. myeloblast
c. lymphoblast d. metarubricyte e. megakaryocyte
a. b. c. d. e.
leukosis-sarcoma comp lex erythremic myelosis Marek's disease Fanconi syndrome malignant erythroleukemoid syndrome
W.I. Dodds 201. Viral jTlfectioli is classically associated with:
a. lymphocytosis b. lymphopenia c. eosinophilia d. neutrophilia e. target ceUs
C 1998 Mosby--Year Book. Inc. Photocopying is prohibited by /o.w.
c. small platelets d. autoagglutination e. platelet clumping 203. Autoagglurinated red blood cells usually indicate:
202. The imnllme-mediatedfoml of thrombocytopenia is characterized by:
a. leukocytosis b. giant platelets
121
a. b. c. d. e.
hypoproteinemia hyperosmolarity collection artifact erythrocyte fragiJjty immune-mediated hemolytic disease
Correct answers are on pages 124-130.
www.vet4arab.co.cc
SECTION 4
122 204. Disseminated illlravascularcoagulation prtXiuces red blood cell fragmentation These fragments afred blood cefls are called:
a. b. c. d. e.
schislocytes microcytes target cells leptocytes Howell-Jolly bodies
210. 71le test for erythrocyte antiglobulin is called the:
205. A regenerative response to allemia is indico.ted by:
a. b. c. d. e.
microcytosis and hypochromia normochromia and nucleated red blood cells hyperchromia and stomalocytes normocytosis and nucleated red blood cells macrocytosis and polychromasia
206. Red blood cells tilat are polychromatophilic when stained with Wrights stain arecaffed what when srained with new methylene blue?
207. Coagulation profiles ofanimals with liver
disease typically show: a. low von WLllebrand factor activity
reduced fibrinolysis prolonged prothrombin time thrombocytosis hypercoagulability
208. What are typical clinical sigm 0/
thrombocytopenia?
a. b. c. d. e.
hematoma formation and joim pain epistaxis and exercise imolerance petechiae and ecchymoses melena hematuria
209. A leukocyte with a nucleus in tile shape ofa spoked wheel is called a:
a. plasma cell b. leukemoid cell
a. b. c. d. e.
Schmidt's test Schirmer test Cooley's anemia test Howell-Jolly test Coombs' test
211. Which blood cells llave the longest life span? a. neutrophils b. erythrocytes c. thrombocytes d. megakaryocytes e. monocytes 212. Toxicanemia caused by oxidant drugs or toxins is characterized by deposits ofdenatured hemoglobin called:
a. reticulocytes b. poikilocytes c. leptocytes d. target cells e. acanthocytes
b. c. d. e.
c. chromatid body d. polychromatophilic cell e. monocyte
a. b. c. d. e.
Howell-Jolly bodies Heinz bodies LE bodies refractile bodies Ehrlichia bodies
213. Platelet adhesion, an importam illitial ellellt in the control of bleedillg, is primarily mediated by:
a. b. c. d. e.
fibrin platelet factor 3 von Willebrand factor thromboxanes subendothelial elastin
214. The inherifQnce pattern of hemophilia is:
a. b. c. d. e.
sex-linked recessive autosomal recessive sex-linked dominant autosomal incompletely dominam sex-linked codominam
• C 1998 Mosby-Year Book. Inc. Photocopying is prohibited by law.
Hematology MId Cyt ology
215. 71le resflllS of which hemostatic test are usually normal i" allimals with von Willebrand's disease?
a. b. c. d. e.
bleeding time platelet retention (adhesion) prothrombin time botrocetin cofa ctor activity von Willebrand factor antigen assay
216. Hemophilia A is characterized by:
n. b. c. d. e.
low factorVlJl activity lowvonWillebrand factor activity low factor IX activity prolonged primary bleeding time normal fac torVlII activity
2 17. Hemophilia B is characterized by:
a. b. c. d. e.
lowfactorVIII activity low von Willebrand factor activity prolonged primary bleeding time normal factor lX activity low factor IX activity
218. Fibrin clot formation is the result of'
a. b. c. d. e.
thrombin interaction with fibrinogen plasmin interaction with fibrinogen thrombin interaction with plasminogen plasmin interaction with plasminogen prothrombin interaction with thrombin
219. Vitamin K-dependem clotting/actors are sylllhesized in the:
a. b. c. d. e.
spleen liver pancreas kidney thymus
a. b. c. d. e.
hematoma formation epistaxis petechiae and ecchymoses melena hematuria
22 J. The characteristic type of bleeding in animals with von Willebrand'sdisease involves the:
a. b. c. d. e.
skin and subcutaneous tissue muscles mucosal surfaces joints medullary cavity of long bones
222. Anticoagulant rodellticide poisollingcauses
bleedillg by:
a. inhibiting platelet function and fibrin deposition b. inhibiting platelet function and fibrinolys is c. preventing formation of fibrin in the liver d. blocking synthesis of vitamin K-dependent clotting factors e. inhibiting prothrombin activation of blood vessels 223. 11Je most common inheritance pattern ill Wil/ebrand's disease is:
a. b. c. d. e.
VOrl
sex-linked recessive a utosomal recessive a utosomal incompletely dominant sex-linked dominant sex-linked codominant
224. The bolle marrow precursor cell of the blood platelet is the:
a. b. c. d. e.
megakaryocyte macrokaryoblast mononuclear giant cell Reed-Sternberg cell pluripotent stem cell
225. Wllat is the hereditary anemia ofbasenji dogs called?
220. TIle classic sigll ofbleeding ill animals with
hemophilia is:
123
a. glucose-6-phosphate dehydrogenase deficiency b. phosphofructokinase deficiency c. porphyria d. pyruvate kinase deficiency e. methemoglobinemia
Correct allswers are 011 pages 124·130.
124
www.vet4arab.co.cc
SECTION 4
Answers I. a Heparin serves as an anticoagulant by accelerating the activity of antithrombin III. The other anticoagulants listed chelate or bind calcium. 2. b Heinz bodies are clumps of denatured hemoglobin caused by oxidative injury, as with phenothiazine toxicity and onion toxicosis. 3. d FaclOrs II, VII, IX, and X are vitamin K dependent. Vitamin K is inhibited by dicumarol. 4. d FactorVII in the extrinsic pathway has the shortest half-life. Therefore the extrinsic pathway clotting time is prolonged before the intrinsic pathway is affected. 5. c St. Bemards are predisposed to lymphosarcoma. Finding over SO% lymphoblasts in a cytologic preparation of an enlarged lymph node is considered diagnostic of lymphosarcoma. 6. d Neutrophils can predominate. but this is much less common. 7. d ~Wet" feline infectious peritonitis typically produces a straw-colored effusion containing primarily neutrophils. 8. a Other precursor cells within the bone marrow give rise to other circulating cells. 9. e Kurloffbodies are normal findings in the lymphocytes of guinea pigs. 10. c Osteosarcoma is a spindJe-ceU tumor of mesenchymal origin. All the others listed are round-cell tumors. II . d Corticosteroid administration produces lymphopenia, mature neutrophilia, monocytosis, and eosinopenia. 12. d Horses do not release immature red cells from the bone marrow. Therefore polychromasia and reticulocytes are nO{ seen, nor are nucleated red blood cells seen. Mucous membrane color is a poor method of determining the response to anemia. 13. d This disorder is most common in Dobermans. 14. c Lead poisoning p roduces these hematologic findings. 15. e Spherocytosis is characteristic of immunemediated hemolytic anemia. 16. b Activated clotting time in normal dogs is under 2 minutes. 17. d Concentrations of fibrin degradation products increase in disseminated intravascular coagulation and are normal in warfarin toxicosis.
Activated clotting time is prolonged in both instances. Platelet counts are decreased in disseminated intravascular coagulation and normal in warfarin toxicosis. 18. d Respiratory disease is a common presenting sign of distemper. Characteristic inclusion bodies are sometimes seen in erythrocytes, neutrophils, lymphocytes, andlor monocytes. 19. c Many Spororllrix organisms are found in infected cats, but the organisms are difficult to find in infected dogs and horses. Fusiform (cigar) shapes are characteristic of Sporothrix schenckii. 20. d The aggregate reticulocyte count should be at least 10%. 1b estimate the minimum regenerative response in cats, the rule of thumb is as follows: PCV Aggngate n?ticulocytes 20%
2%
10% 5%
5% 10%
2 1. e Of the organisms listed, o nly Cryptococcus is likely to be found in cerebrospinal fluid. 22. d Anaplasma marginale causes a hemolytic crisis in cattle. The organism is transmitted by the one-host tick Boophilus annulatus. 23. a Prorotheca is an alga that can cause disseminated disease in dogs but usually only causes cutaneous lesions in cats. 24. e Lymphocytes are the predominant leukocyte in hamsters. 25. a This sex chromatin lobe on the nucleus is called a Barr body. 26. d Spherocytes are much smaller than normal RBCs. 27. c Plasma cells produce immunoglobulins. 28. a Kurloffbodies are found in the lymphocytes of guinea pigs. 29. e Schistocytes are RBC fragments. 30. b Russell bodies are found in plasma cells. 31. c Crenation ofRBCs is seen with dehydration. 32. d Carcinomas are of epithelial-cell origin. 33. a Metaplasia involves a change from one type of mature cell to another. 34. e Sarcomas are of mesenchymal-cell origin. 35. b Dysplasia is characterized by proliferation of atypical cell populations. 36. c None of the other answers correctly defines steatitis.
•
C 1998 Mosby-Year Book, 1nc. Photocopying is prohibited by law.
Hematology and Cytology
37. e Platelets are formed in the bone marrow by megalcaryocytes. 38. d Canine RBCs persist for up to 4 months. 39. e The OSPT evaluates the extrinsic and common pathways. 40. d The APTr evaluates the intrinsic and common pathways. 41. b The peak response is seen 4 to 8 hours after corticosteroid administration. 42. d leukocytosis in dogs primarily involves increased numbers ofneutrophils. 43. c Pale blue inclus ions in neutrophils are seen with some Ehrlichia ewingii infections. Parvoviral infection does not cause inclusion bodies in circulating cells. Distemper inclusions are homoge neous, basophilic- or eosinophilicstaining structures in neutrophils and erythrocytes or mononuclear cells. 44. d Iron defiCiency causes microcytic hypochromic anemia. 45. a Macrocytic hypochromic anemia is characterized by large, pale-staining cells, suggesting polychromasia and reticulocytosis. 46. d in cats, epinephrine release increases numbers of circulating neutrophils and lymphocytes. 47. c Mature neutrophils predominate in the blood of normal dogs. 48. e Lymphocytes predominate in the blood of weanling pigs. 49. e Lymphocytes predominate in the blood of normal adult cattle. 50. b Heinz bodies are most common in feline blood. 51. c Schistocytes are RBC fragments produced when membranes are cleaved by fibrin strands in capillary beds. 52. d Howell -Jolly bodies are nuclear remnants within red blood cells. 53. b The Pelger-Huet anomaly involves defective WBC segmentation. 54. b Iron deficiency causes microcytic hypochromic anemia. 55. b Avian thrombocytes are nucleated cells. 56. d The protamine sulfate test is used to detect fibrin monomers and early fibrin degradation products. 57. a Myelocytes have approximately two mitoses. The other cell types do not undergo mitotic divisions .
125
58. e Hemarthrosis and bleeding into body cavities are not typical of thrombocytopenia or thrombopathy. 59. a Pelger-Huet anomaly is an inherited d isorder associated with abnormal neutrophil segmentation. 60. a Cytauxzoonosis has not been reported in dogs. 61. b Lead poisoning occurs in cats but has been reponed only rarely and causes no anemia or only mild anemia. In add ition, polychromasia would be expected. Erythremic myelosis is reported in cats, and blood smears typically show large numbers of nucleated RBCs. 62. c A plasma cell is a differentiated B lymphocyte. If it contains vacuoles filled with immunoglobulin (Russell bodies), it is known as a Mort cell. 63. d Agglutination describes random clumping of RBCs. 64. c in French the term means rolls. 65. e Howell -Jolly bodies are nuclear remnants within mature RBCs. 66. b Heinz bodies are clumps of denatured hemoglobin in RBCs. 67. a Basophilic stippling ofRBes is evident with Wright's stain. 68. a Siderocytes contain iron particles. 69. c Plumbism is from the Latin term for lead, plumbum. 70. c Hypothyroidism typically causes normocytic normochromic anemia with leptocytosis. Answer e describes anemia that is regenerative (caused by hemorrhage or hemolysis). Answer d describes anemia caused by iron deficiency (without spherocytosis). 7 1. c EhrUchia infection has also been reported in cats and people. 72. b Multiple myeloma is a tumor of plasma cells, which generally produce excessive amounts of a s ingle type of immunoglobulin. Another characteristic of this tumor in dogs is bone lysis. 73. c All the other toxicants tend to cause formation of Heinz bodies. 74. c Polycythemias are classified as absolute and relative. Absolute polycythemias are further divided into primary and secondary. Polycythemia vera is the only cause of primary absolute polycythemia. 75. e Dehydration and splenic contraction are forms of relative polycythemia and resolve once stress is relieved and dehydration is corrected.
126
SECTION 4
76. a All the orner hormones s timulate erythropoiesis. 77. e Corticosteroids inhibit lymphopoiesis. 78. a The liver produces coagulation factors. 79. a These find ings are characteristic of~heat." 80. a Bracken-fern poisoning causes aplastic anemia. SI. b Oropharyngeal contamination is suggested by the presence of superficial epithelial cells and SimOllsiella organisms. $imOllSiella is a bacillus found in colonies formed by organizing in a characteristic pattern. 82. a Motl cells are vacuolated plasma cells. Giant cells are seen with chronic granulomatous disease. Siderocytes arc fed blood cells comaining iron panicles. LeplOcytes are thin, folded fed blood cells. 83. e Cattle have a limited reserve of neutrophils in thc i,r bone marrow and often develop leukopenia dunng severe, acute inflammation. 84. d . N.eonalal isoerythrolysis is diagnosed by clulical behavior and serologic tests. Equine infectious anemia is confirmed using the Coggins test. Feline infectious anemia can be confirmed by examination of the blood smear, as can microangiopathies. 85. c Thrombocytopathy or abnormal platelet function is associated with deficiencies of von Willebrand factor. 86. d Babesiosis is characterized by intravascular hemolysis. 87. a In a major cross match, the recipient's plasma is tested with the donor's blood cells. 88. e Anaplasmosis is characterized by extravascular hemolysis. 89. c The RVVf is prolonged with deficiency of fa ctor X orV. 90. b In a minor crossmatch the donor's plasma is tested with the recipient's blood cells. 91. a Lack of erythropoietin because of renal parenchymal loss is the mechanism underlying the anemia in chronic renal failure. 92. b In people and animals other than dogs, the liver produces approximately 10% of erythropoietin. 93. b Allaplasma organisms tend to be located on the periphery of infected RBCs. 94. c Tetracyclines are effective against Ehrlichiainduced thrombocytope nia. 95. e Most cats in America have type-A blood. 96. b Milk has a low iron content.
www.vet4arab.co.cc
97. a Transferrin is a plasma transport protein. Hemosiderin is a storage form of iron. Transcobalamin is a transport protein fo r vitamin BIZ' 98. e Sertoli·cell tumors are most commonly associated with feminization in males. 99. a Increased release of corticosteroids associated with hyperadrenocorticism causes peripheral neutrophilia. 100. c Poikilocytosis describes variously shaped RBCs. 101. d Heterophils can also be seen in reptiles and some laboratory animals. 102. e Giant cells may also be formed by fusion of monocytes or epithelioid cells. 103. d Other leukocytes rarely show toxic changes. The most common toxic changes are cytoplasmic vacuolization, cytoplasmic basophilia, toxic granulation, and Dohle bodies. 104. d The eosinophils of greyhounds tend to be more highly vacuolated. 105. b Spherocytes are smaller than normal cells and contain no central pallor. Poilcilocytes are of variable shape. Rouleaux refer to stacking of red blood cells. Stomatocytes are folded red blood cells. 106. c Histiocytomas are more common in young dogs. 107. b Dohle bodies are most likely to be observed in the cytoplasm of to:oc neutrophils. 108. c If more than 10% of the cells are eosinophils and no criteria of malignancy or oropharyngeal contamination exist (superficial squamous cells) . then an allergic or parasitic infection is most likely. 109. d It reduces the risk of oropharyngeal COlllamination. The landmark used for this technique is the c ricothyroid ligament. Because general anesthesia is usually not needed, this would not be a good choice for a fractious or aggressive animal. 110. c Septic exudates have a high protein content and contain many bacteria and degenerate neutfophils. 111. d These body cavities a re lined with mesothelial cells. 112. a Normal cerebrospinal fluid is also clear a nd colorless and has a total protein concentration less than 50 mg/dl. 113. d Plasmacytomas commonJy cause osteolysis.
0 1998 Mosby-Year Book, Inc. Photocopying is prollibited bJ' law.
Hematology and Cytology
114. e Osteoblasts are ovoid, plump cells with a rounded eccentric nucleus and dark blue cytoplasm. They resemble large plasma cells. Osteoclasts are large multinucleated cells that arise from the macrophage/monocyte cell line. There is not a s ingle precursor cell to all other skeletal system cells. 115. e The most frequently identified bone tumor is osteosarcoma, closely followed by fibrosarcoma. Although osteosarcoma is the most common tumor, chondrosarcoma (not fibrosarcoma) is the second most common. 116. c Finding an osteoclast is not unusual because these cells are present in articular cartilage. Osteoclasts are present in subchondral bone. Their presence in synovial fluid suggests erosion of articu lar cartilage to subchondral bone. 117. e. Hematoidin is a non-iron-containing, golden pigment often referred to as tissue bilirubin. 118. b Erythroleukemia is the most likely cause. All the other disorders listed cause hypocellular samples. 119. e Anemia of chronic inflammatory disease is the most likely cause. Myelofibrosis is the displacement of normal marrow e lements by fibrou s tissue. FeLV-induced suppression can result in a normocellular or hypoceUular sample. 120. c None of the other answers correctly cites the sequence of erythroid cell maturation. 121. c None of the other answers correctly cites the sequence of myeloid cell maturation. 122. b Dogs have pleomorphic granules, but they are generally round. Round granules arc seen in horses, cattle, and sheep. Horses have prominent granules that fill the cytoplasm. 123. b All reticulocytes are polychromatophilic with Romanowsky stains. Romanowsky s tains include Wright's stain and its derivatives. 124. e Aspiration of foamy epithelial cells in the area of the mandibular lymph node usually indicates neoplasia. The submandibular salivary gland is often aspirated in this area, revealing a uniform population of foamy cells in a pink background when stained with Wright's stain. 125. d Hematogones are nuclei shed from maturing
RBCs. 126. e Lymphoglandular bodies are fragments of lymphocyte cytoplasm. 127. a Charcot·Leyden crystals are formed by coalescence of eosinophil granules.
127
128. c Tingible bodies are nuclear debris in macrophages. 129. b Curschmann spirals are mucous casts in small bronchioles. 130. e Basket cells are not cells but rather are the nuclei ofruptllfed WBCs. 131. a Ragocytes may be seen in synovial fluid. 132. b This linear distribution is characteristic of synovial fluid of high viscosity. 133. b Uniform nuclear size is not a criterion. ~ic~lly, .malignant neoplasia shows a large variatIOn III nuclear size. An exception is lymphosarcoma. Lymphosarcoma is a roundcen tumor, not an epithelial-cell tumor. 134. d The ear mite that produces a dry, black, granula r discharge is Otodectes cynotis. Otobius megnini is the spinose ear tick of food animals. 135. a Keratoconjunctivitis sicca produces a neutrophilic exudate, usually containing bacteria. 136. a Most nasal tumors are of epithelial origin. 137. b Pe rianal gland cells can be found in the skin ofthe areas listed. They can also be found in the dorsum, but this does not specifically include the cars. 138. d Liposarcomas are malignant, may co ntain large to s mall lipid globuJes, are rarely secondarily infected, and can be stained with special stains only if alcohol or another lipid solvent has not been lIsed for fixation. 139. d Tumors are o nly found in poorly pigmented areas of the skin exposed 10 ultraviolet radiation. Squamous-cell carcinoma may be found in any area of the skin in dogs and cats. 140. a Epithelialtllmors are called carcinomas when malignant, and they exfoliate groups of cells because of formation of cellular "bridges." The cells exfoliated arc large, witlliarge amounts of cytoplasm and many criteria of malignancy. 141. d Each hemoglobin molecule comprises a heme group and four globin chains. 142. a The Embdcn-Meyerhof pathway uses glucose. The hexose monophosphate pathway maintains glutathione in a reduced state to neutralize oxidants. The methemoglobin reductase pathway keeps hemoglobin in a reduced state to transport oxygen. The Lucbering-Rapoport pathway allows formation of2,3-diphosphoglycerme, which regulates oxygen transport. 143. e Nuclei are evident in metarubricytes.
128
144. b Clear and colorless plasma is seen normally
145. 146. 147.
148. 149.
150. 151.
152 .
153.
154.
155.
156. 157. 158. 159.
www.vet4arab.co.cc
SECTION 4
in most dogs and cats. Horses and cattle have slightly yellow plasma. Icteric samples are yellow, whereas hcmoly7..ed samples are pink. to red. d The average caprine erythrocyte is 4 fJ..m in diameter. e The average canine erythrocyte is 7 fJ..m> in diameter. d Rouleaux are groupings of erythrocytes like a stack of coins. Agglutination pe rsists in a saline dilution test, whereas rouleaux do not. a Unlike other species, horses do not show reticulocytes. b Aggregate reticulocytes are similar to those seen in other species and generally increase in number during response to more severe anemias. Punctate reticulocytes occur during health, and their numbers remain increased even after numbers of aggregate reticuJocytes return to normal foll owi ng a response. d Reticulocytosis may nOt be evident until 2 or 3 days after an anemic crisis. e Decreased blood viscosity, causing turbule nce, is the most likely cause. Acquired problems could cause this murmur; however, given this description, acqui red problems and congenital defects would be less likely. c Plasma protein levels are normal to increased in hemolytic anemia. They are decreased in hemorrhagic anemia. Reticulocyte response is more rapid with hemolysis because of the more readily available iron stores. Bilirubin levels cannot increase via hemorrhage if the blood is lost outside the body. b Heinz bodies, Schmauch bodies, and erythrocyte refractile bodies are all the same thing (clumps of denalUred hemoglobin). e Extravascular hemolysis occurs outside the vascular system. Increased macrophage activity results in extravascular hemolysis. e In myelophthisic disease, hematopoietic cells in the bone marrow are replaced by abnormal cells, such as fibrous or neoplastic tissue. e Von Wtllebrand factor is produced by endothelial cells. b Neutrophils move from tile peripheral blood into tissue. They do not return to the blood. b About two thirds of intravascular neulrophils in cats are marginated. b Macrophages are de rived from monocytes.
160. b Eosinophils have phagocytic properties
similar to those of neutrophils but are not protective against bacte rial infection. J6l. d He parin is produced by basophils and mast
cells. 162. e Numbers of im mature neutrophils are increased in a left shift. 163. c All the other disorders cause lymphopenia. 164. e Hypersegmented neutrophils have five or more nuclear lobes. 165. d Platelets circulate for apprOximately 10 days. 166. d Thrombin is a potent platelet agonist. It accelerates fibrinol ysis because it activates plasminogen . It accelerates coagulation because it activates fa ctors II , IX, and X. 167. b The other tests are in vitro. 168. e Toxic neutrophils exhibil CYlOplasmic vacuolization, toxic granulation, and Dahle bodies. Band neutrophils appear normal, except for a broad nucleus without segmentation. 169. d Mycobacterial pne umonia is characterized by an exudate. The other disorders listed would produce a transudate or modified transudate. 170. a Exudates and transudates may have similar protein concentration. The cholesterol! triglyceride ratio is not used to d ifferentiate exudates from transudates. 171. a Platelets are generally absent. 172. e Antibodies in the colostrum cause destruction of the foal's RBCs. 173. d Fibrinogen levels tend to increase with inflammatory disorders. 174. d Reticulocytes are immature erythrocytes that must be stained with supravital stains, such as new methylene blue, and cou nted using light microscopy. There are many methods for calculating reticulocyte pe rcentages. 175. e Mean corpuscuJar hemoglobin concentration is a ratio of the weight of he moglobin to the volume of a given population of red blood cells. Mean corpuscular hemoglobin is the weight of he moglobin in an average popuJation. Packed cell volume and hematocrit are different terms for the pe rcentage of red blood cells in whole
Glood. 176. e Fibrinogen is produced in the liver, and plasma levels are not affected by age but increase during inflammatory processes. Fibrinogen precipitates at 56° 10 58° C. 177. d Cyclic neutropenia occurs in some gray collies.
e 1998 Mosby- Year Book, Inc. Pllotocopying is prollibited by law.
Hematology and Cytology
178. e The eosinophils of horses tend (0 resemble
plump ras pbe rries. 179. c Basophilic stippling is common i.1l canlc with a strong regenerative response to anemia. Lead poisoning typically produces only slight anemia or no anemia. 180. e Birds have an incomplete intrinsic pathway. 181. e If iron occupied 90% of the medullary cavity, flight might be difficult. 182. a Marek's disease is caused by a DNA herpesvirus and results in lymphoid proliferation in peripheral nerves and central nervous system inflammation. lohne's disease is an infiltrative disease of the bovine intestine. Newcastle disease is avian pneumoencephalitis. "Jyzzer's disease is seen in many animals and is caused by Bacillus pilifannis. Pullomm disease is Salmonella pulfarum infection in birds. 183. a PerSistently high estrogen levels produce aplastic anemia. 184. c The kidney is not directly involved in erythropoiesis in fetal kittens. 185. b Although lymphocytes are stored here, that is primarily related to the immunologic fun ction of the spleen . It is not a leukocyte storage area pe r se. 186. b Hemostasis is a dynamic series of events that control clotting. 187. c Thrombin is a coagulation protein. An infarct is an area of tissue necrosis, often caused by a thrombus or embolus. An embolus is a freely circulating thrombus. A plaque is a flat area or patch. 18B. e Megakaryocytes are comparatively large. 189. e The microhematocrit tube can be snapped off at the buffy coat and a smear made forWnC examination. 190. a A high icte rus index may indicate hemolysis or dehydration. 19l. e Thrombocytes or platelets are derived from megakaryocytes. 192. d Hypovolemia can be caused by hemorrhage or dehydration. 193. a An RNA retrovirus causes avian leukosis. 194. e Hematopoiesis describes formation and maturation of blood cells. 195. c Marek's d i sea~e is caused by a DNA herpesvirus. 196. e The defect is in the intrinsic system. Possible problems include factor VIII deficiency (hemophilia A), factor IX deficiency
129
(hemophilia B), or factor Xl deficiency (hemophilia C). A deficiency of factor X11 does
not cause a bleeding problem. 197. d Factor VII is the only soluble coagulation factor in the extrinsic system. 198. d Russell 's viper venom activates factor X and is used to evaluate the common pathway. 199. e Thrombin time tests fibrinogen level and fun ction. Factor II (prothrombin) is not involved in thrombin time. 200. b Increased numbers of hypersegme ntcd neutrophils indicate a right shift. 201. b Viral infections typically cause lymphopenia. 202. c Platelet size is reduced with immunemediated thrombocytopenia. 203. e Autoagglutination is characteristic of immune- mediated hemolytic disease. 204. a Fragments of ROCs are termed sch istocytes. 205. e Increased polychromasia indicates regeneration. Numbers of nucleated RBCs often increase with regenerative anemia; however, their numbers also increase \vith many nonregenerative anemias and therefore do not indicate regene ration unless polychromasia or reticulocytos is is e vident. 206. a Reticlilocyies are young polychromatic RBCs. 207. e Animals with liver disease may have a prolonged prothrombin time. 208. c Petechiae and ecchymoses are common in thrombocytopenia. 209. a Plas ma cells have a nucleus resembling a spoked wheel. 210. e The Coombs' test is used 10 detect RBC antiglobulin. 211. b Of all blood cells, RBCs have the longest life span (up to 120 days). 212. b Heinz bodies are typically observed in the RBCs of animals with toKic anemia. 213. e Platelet adhesion is mediated by von Willebrand factor. 214. a Hemophilia is inherited as a sex-linked recessive trait. 215. c Prothrombin time is usually normal in animals with von Wtllebrand's disease. 216. a Animals with he mophilia A have low factor VIII activity. 217. e Animals with hemophilia B have low factor IX activity. 21B. a Interaction of thrombin with fibrinogen produces clot formation .
130
www.vet4arab.co.cc
SECTION 4
219. b These clotting factors are synthesized in the
liver. 220. a Animals with hemophilia easily develop hematomas. 221. c Animals with von Willebrand's disease show
bleeding at mucosal surfaces. 222. d After repeated doses, lack of these factors leads to bleeding.
223. c Von Willebrand's disease is inherited as an autosomal incompletely dominant trait. 224. a Thrombocytes are derived from megakaryocytes. 225. d The anemia is macrocytic hypochromic and may develop before 6 months of age.
SECTION
5 Immunology
•
l.J. Gershwin
NOTES Recommended Reading Cae Clough NE, Roth fA: Understanding immunology, 51 Louis, 1998, Mosby. Gershwin LJ et a1: Immunology and immunopatllOfogy ofdomestic animals, ed 2, St Louis, 1995, Mosby. Lewis RM, Picut CA: Veterinary clinical immunology. Bahimore. 1989. Williams &WIlkins. T12.ard I: Veterinary immunology: an introduction. ed 5, PhUadelphia, 1996, WB Saunders. Practice answer sheet is on page 263.
Questions 1. MilCh immunoglobulin (lg) is most effective on mucosal surfaces? a.lgG b. lgM c. IgD d. IgE e. IgA
a. IgG b. lgM c.lgD d.lgE e. IgA
2. Mlich immunoglobulin (Ig) is present almost exclusively in tile intravascular compartmeru?
•
C 1998 Mosby-Year Book, Inc. Photocopying is prollibited by law.
a. IgG b. lgM c. JgD d.lgE e. IgA
3. Wlllch immwlOglobulin (/g) binds to mast cells and sellSitizes for antigen-induced degranulation?
4. WlIich immunoglobulin (lg) has its major role as a B lymphocyte differentiation marker?
a.lgG b.lgM c. IgD d.lgE e. IgA
Correcranswers are on pages 139-141.
131
132
www.vet4arab.co.cc
SECTION 5
5. Which immullogiobulin (lgJ predominates in
d. postcapillary venules e. subcapsular vessels
serum dllringa secondaryatltibody response?
a. IgG b. lgM
11. Which process occurs ill the thymlts?
c. 'gD d. IgE e. IgA 6. In allergic dermatitis, which cell is the sourceo! histamine?
a. eosinophil b. mast cell c. lymphocyte
B cell maturation antibody production T cell maturation stem-cell he matopoiesis thrombopoiesis
7. What is the most likely calise ofa high titer of antinuclearamibodies ill a dog with k idney failure?
Goodpasture's disease amyloidosis leptospirosis systemic lupus erythematosus nephrotoxic reaction
8. B /ympllocytes are predominarufy found in which
region of lymph nodes?
a. b. c. d. e.
insoluble solu ble membrane bound intracellular purified
13. Alltigen can be presellted to TcelIs by all of the following except:
a. b. c. d. e.
8 lymphocytes T lymphocytes macrophages dendritic cells Langerhans' cells
a. cortex b. c. d. c.
medulla paracortex white pulp red pulp
14. Which cytokilles are produced by the eD4' Tcells called type-l helper T cells?
9. T lympllOcyres are predominan tly [olllld i n which
region of lymph lIodes?
a. cortex b. medulla c. paracortex d. white pulp e. red pulp
a. efferent lymphatics b. afferent lymphatics c. sheathed arteries
a. interleuldn 4, interleukin 2. tumor necrosis #fo factor-o: b. interleukin 4, interferon '1, interleukin 2 c. interleukin 2. tumor necrosis faclor-o: , interle ukin 5 d. interleukin 4, interleukin 5, interleukin 10 e. interleukin 12, interferon-'Y. interleukin 2 15. Which cytokines are produced by the CD4+ T cells called type-2 helper T cells?
10. Lymphocytes exit Iympllnodes through:
16. Which Tcell cytoldne is considered all autocrine growth factor?
a. interleukin 1 b. interleukin 2 c. interleukin 3 d. interleukin 4 e. interleukin 5 17. Mlich cytokine is most importam in the immune
respoflSe to facultative intracellular bacteria? 12. 111e Coggills test for equine infectious anemia and the single radial immunodiffusion method of immunoglobulin quantitation have in common the requirefllellt that the antigen must be:
d. neutrophil e. dendri tic cell
n. b. c. d. e.
a. b. c. d. e.
Immull o logy
a. b. c. d. e.
interlcukin 4, ime rleukin 12, interfcron-y interleukin 10, interleukin 12, imerleukin 2 interleukin 4, imcrleukin 5, interleukin 10 imerlcukin 12, interferon-y, interleukin 2 interleukin 2, tumor necrosis factor-a, interleuldn 5
C 1998 Mosby-Year Book, Inc. Photocopying is prohibited by law.
a. interlc ukin J b. interleukin 2 c. interleukin 4 d. illle rieukin 10 c. interle ukin 12
18. Macrophages are activated by prodllction of
a. b. c. d. e.
interleukin 2 inte rleukin 4 inte rleukin6 interferon-a intcrferon-y
gammopatlly?
agglutination precipitation immunofluorescence immunoelcctrophoresis complement fixation
22. A Holstein calfwitlt persistelllIJelltrophilia and lack of pus formation at sites of bacterial infection is most IikelyaJJ1icred with:
a. b. c. d. e.
severe combined immunodeficiency primary agammaglobulin emia Chediak-Higashi syndrome bovine leukocyte adhesion deficiency cycl ic ne utropenia
23. Blood from all Arabian foal with severe combilled immunodeficiellCY is most likely to show:
variable IgG b. few to no lymphocytes. nonnal lgM, and no IgG c. few to no lymphocytes. no IgM, and variable IgG d. normal B lymphocyte numbers, low T lymphocyte numbers, and normallgG and IgM e. low 8 lymphocytc numbcrs, normal T lymphocyte numbers, and low IgM 24. Bence-fOlies proteins in urine comprise:
a. immunoglobulin heavy chains
20. Polyclonal gammoparhy is usualfycaused by:
a. b. c. d. e.
c. in ute ro lysis by the mare's natural killer (NK) cells d. des truction in the immediate neonatal period by antibody and complement c. consumption of very low levels of maternal immunoglobulins by me neonatal foal
a. normal lymphocyte n umbers. no IgM, and
19. Which technique is used to detect monoclonal
a. b. c. d. e.
133
a malignant plasma-cell tumor a benign plasma-cell tumor hemangiosarcoma anal-sac carcinoma persistent antigeniC stimulation
21. [n neonatal isoerythrolysis, destructioll of the foal's erythrocyres is a result of
a. destruction in utero by antibody and comple ment b. in utero sensitization of the foal 10 the mare's erythrocytes
b. immunoglobulin light chains c. immunoglobulin J chains d. secrelOry components c. Fub immunoglobulin fragments 25. Hyperviscosity syndrome is most likely to occur witll a myeloma producing which immunoglobulin (IgJ?
a. IgG b.lgM
c. IgA d.lgO
e. IgE
Correct answers arc 011 pnges 139· 141.
134
www.vet4arab.co.cc
SECTION 5
26. Physiologic effects a/histaminc illcJudeall olllle
31. All ofthefollowi1,g are it, vitro correlates ofcell-
following except:
mediated immllnityexcepl:
a. smooth muscle contraction b. increased capillary permeability c. vasodilation
a. delayed hypersensitivity skin test b. mixed le ukocyte culture c. lymphocyte stimulation with phytohemagglutinin d. NK cell assay e. cytotoxic T cell assay
d. increased mucous secretion e. chemotaxis of ncutrophils 27. Smooth mllscleconCTaction lImc occllrs 12 hours
after exposure ofa seflSitized horse to allergen
aerosol is caused by:
32. Qmlacl demlatitis Oil the muzzleofa cat, caused
by expoS/lre to aplastic food dish, is mediated by:
a. histamine
a. b. c. d. e.
b. seroto nin c. bradykinin d. lCllkotriene 8 4 e. prostaglandin D2
28. Lesions produced in immullccomplex disease involve all of tile following components except:
a. neutrophils b. antibodies
c. antigen d. complement e. macrophages
IgE-triggered mast-cell degranula[ion IgG- and complemelll -rnediated cell damage IgM- and complemen t-mediated cell damage immu ne-complex depOSition T lymphocyte-media ted delayed hypersensitivity
33. In enzyme-linked immwlOsorbent assay (EUSA) to rest for felille leukemia virus (FeLV) antigen in a cats serum, the plastic wells o/the test kit are coated with:
a. monoclonal antibody to FeLV b. purified FeLV c. horseradish peroxidase-conjugated antibody
29. Allligen specificity in delayed hypersensitivity reactiol/S is con/erred by wllieh celf type?
a. B lymphocyte b. T lymphocyte
to r-eLV
d. serum from the cat e. monoclonal antibody to feline IgG 34. Wllicll sample is used in EUSA to test for parvollims antigen ill dogs?
c. macrophage d. NK cell c. ep ithelioid cell
30. Development alamedar uveitis ("blue eye") following vaccination wilh a modified-live canille adenovirus 1 vaccine indicates tltat:
a. adenovirus-specific antibody is presenl in the eye b. the virus has become virulent and damaged the cornea c. there were contaminants in the vaccine d. the dog is allergic to some component of the vaccine e. there is hemagglutination of the virus in the cornea
a. b. c. d. e.
serum plasma fe ces saliva whole blood
35. lmmunoflilorescellce srainingofa skin biopsy wirh antica1line IgG shows illlercellular staining of the epithelial cells. 17,is pattem is consistent with:
a. b. c. d. e.
discoid lupus erythematosus chronic allergic dermatitis pemphigus vulgaris scleroderma vasculitis
C 1998 Mosby-Year Book, Illc. Photocopyi"g is prohibited by law.
Immullology
36. In dogs the direct Coombs' rest is performed by combining rhe:
a. patient's washed erythrocytes with guinea pig complement b. patient's washed erythrocytes with rabbit anticanine IgG c. patient's serum with normal erythrocytes and complement d. patient's serum with normal erythrocytes and anticani ne IgG e. patienl's washed erythrocytes with normal serum 37. A calf develops an acute respiratory infection. A serum sample/or serologic testing is obtained during tile ilIlless and again after 21 days. Acute and convalescent titers for four viruses areas /ollows: bovine respiratory syncytial virus 1:16 and 1:256; infectious bovine rhinotracheitis virus 1:32 and 1 :64; parainfluenza 3 virus 1:32 and 1: 16; and bovine virus diarrhea virus 1:128 and 1:256. WllOt is tlte most fikelycause ofrhis ill/ection?
a. infectious bovine rhinotracheitis virus b. bovine respiratory syncytial virus c_ parainfluenza 3 virus d. I10vine virus diarrhea virus e. None of these viruses is likely to have caused the illness. 38. Whicll sample is used for the standard immlUlofluorescem antinuclear alllibody test?
a. b. c. d. e.
skin Iddney serum peripheral blood leukocytes joint fluid
peripheral blood lymph node bone marrow spleen kidney
40. 1n autoimmllne myasthenia gravis,
autoantibodies attack:
a. b. c. d. e.
e pinephrine receptors acetylcholine receptors complement receptors Fc receptors histamine receptors
41. In autoimmune thyroiditis o/dogs, tisslledamage
is caused by:
a. b. c. d. e.
antibodies to T3 antibodiestoT~
antibodies to thyroid-stimulating hormone lymphocytic infiltration ofthyroid follicles antibodies to thyroid-stimulating hormone receptors
42. Prevemion of ref aI/us depends on amibodies specific/or:
a. Clostridium tetani b. Clostridium per/riflgem
c. Clostridium botulinum d. botulinus toxin
e. tetanus toxin 43. An anapl'ylactoid response to a vaccine is:
a. an IgE-mediated acute allergic reaction b. a shockUke reaction that resembles anaphylaxis c. an acute hemorrhagic response d. an anergic reaction e. an IgE-mediated late-phase response 44. Which pair o/cross-reactingantigens has been
used in a vaccination strategy?
39. WI'icll sample is llsedfor the immunofluorescent test for anlimegakaryocyte antibodies?
a. b. c. d. e.
135
a. group-B streptococci and myocardium b. feline infectious peritonitis virus and enteric coronavirus c. human ABO erythrocyte antigens and intestinal fl ora d. human and bovine strains of respiratory syncytial virus e. canine distemper virus and human measles virus
Correcra"swersuroon pages 139· 141.
, SECTION 5
136
45. A baboon heart grafted imo a human infant is an
exampleo! a. an autograft b. an allograft c. a xenograft
transported across the intestinal epithelium:
e. an isograft 46. Vasculitis detected with immwlOfluoresamceof
deposits of immunoglobulin and complement in the vascular endothelium and tissue spaces can occur with all ofthe following except:
a. booster immunization for hepatitis b. infection with a virus that causes chronic viremia c. allergic desensitization d. acute rhinovirus infection e. inhalation of fungal spores by a horse with high IgG titers 47. A small-molecular-weigll! compound thar binds roa hOS1 protein orcelf and rllell becomes
immunogenic is called:
an antigen a hapten complement an idiotope a paratope
a. IgG b. lgM c. IgE d. IgG and IgM e. immune complexes 49. In which species is failure of passive
immunoglobulin tramfer most common?
a. sample A, with hemolysis in all 12 tubes b. sample B, with hemolysis only in tubes 6 through 12 c. sample C, with no hemolysis in any tube d. sample D, with hemolysis only in tubes 2 through 12 e. sample E, with hemolysis only in rubes II and 12
51. Allthefollowingare mediated by immune
complexes except:
a. hypersensitivity pneumonitis b. anterior uveitis caused by can ine adenovirus I c. flea-bite hypersensitivity d. glomerulonephritis caused by viral infection e. Arthus reaction at the site of vaccination
56. Grafting which type of tissue is most likely to result
in graft-versus·llOst disease?
a. skin b. kidney
52. Plasma-cell tumors are most common in the:
a. an immunoglobulin b. an albumin c. a cytokine d. an a-globulin e. a j3-globulin 54. All of the following SIlggest a diagflOsis of monoclonal gammopathy except:
a. b. c. d. e.
hyperproteinemia hypercalcemia hyperglycemia anemia immunodepression
60. In horses, wllich blood group antigens are most immunogenic to horses lacking these determinants?
a. Cand U b. Kand P c. DandQ d.AandC e. Aand Q 61. Congenital deficiency of which complemem compOTlellt would have the greatest negative effect on a puppy's ability to kill pathogenic bacteria (i.e., make the animal most susceptible to bacterial infection)?
d. CB e. C9
antigens are commonly typed using:
a. b. c. d. e.
62. For some respimrory diseases, intranasal
lymphocyte stimulation mixed lymphocyte reaction agglutination complement fixation rnicrocytotoxicity
immuni:Ultion is prefemble to intramuscular immllfli:Ultion because it stimulates production ot
58. Class-II major histocompatibility complex
antigens are commonly typed using:
a. b. c. d. e.
lymphocyte stimulation mixed lymphocyte reaction agglutination complement fixation microcytotoxicity
a. b. c. d.
cytotoxic T ceUs opsonizing antibodies mucosal IgE antibodies mucosallgA. antibodies e. complement-fixing antibodies 63. Diagnosis of some fungal infections can be facilitated by injecting an extract of the organism intradermally and observing the injectioll site for swelling after:
59. Monoclonal antibodies are useful in diagnostic
tests because Ihey are:
a. broadly reactive against many antigenic determinants b. inexpensive and easy to produce
C 1998 Mosby-Year Book, Inc. Photocopying is prohibited by law.
c. very specific for particular antigeniC epitopes d. present in patients with monoclonal gammopathies e. present in patients with polyclonal gammopathies
b. C2 c. C3
57. Class-I major histocompatibUitycomplex
•
137
a. C l q
c. bone marrow d. heart e. liver
a. b. c. d.
53. What is myeloma protein, the product of plasmacell tumors?
48. An acute, shocklifce episode following consumption o/a drug is most likely to bea druginduced hypersensitivity reaction caused by:
a. horses b. dogs c. people
a. only during the first 6 hours of life b. during the first 12 to 24 hours of life c. during the first week of life d. during the firSl2 weeks of life e. during the first 3 weeks of life
skin lymph nodes liver bone marrow e. thymus
Immunology
55. The complement-fixation test uses hemolytic complement and antibody-sensitized sheep erythrocytes as an indicator system. In a complement-fixation test to tktennine the titer of antibody against Sa1monella antigen, the test tubes contai" serum dilutions starting at 1:10 in tube 1, with twofold dilutions through tube 12. Wllfch serum sample has the highest antibody titer?
d. cats e. nonhuman primates 50. 111 foals and calves, colostral antibodies are
d. a syngraft
a. b. c. d. e.
www.vet4arab.co.cc
a. b. c. d. e.
30 minutes 4 hours 8 hours 24 hours 72 hours
Comet answers are on pages 139-141.
64. TI,e reaction described in Question 63 is all
example ofa IrypersensirilJity reaction mediated by;
a. IgE b. c. d. e.
d . Three loci code for class-I, -II , and -III antigens. e. There are many alleles for each loclls.
a. b. c. d. e.
IgG and complement NK cells T lymphocytes
versus a killed whole-organism vaccine include all of tile following except:
a. immune response is to only one protein b. no adjuvant is necessary c. antibodies are not made to non protective antigenic epitopes d. it decreases adverse reactions to vaccination e. it allows for d iscrimination between vaccinated and infected animals 66. All of the following are amigenic differences between t1Imorcefls and " ormai cells in the same tissue or individual except:
a. feline oncornavirus cellmcmbr3 ne antigen (FOCMA) expression on fcline cells b. loss of histocompatibility antigens c. a-fetoprotein expression on liver cells d . carcinoembryonic antigen expression on colon cells e. addition of new histocompatibility antige ns 67. Vaccinia virus is all excel/em vector fo r recombinant vaccines because it:
has a large genome does not integrate into the host genome is atten uated and sta ble does not multiply in nonhuman species is easy 10 administer
68. Concerning the major II istocompatibility complex, which SUllemem is least accurate?
a. Inheritance of class- I antigens is recessive. b. Inheritance of class-J antigens is codominant. c. Alleles from both parents form the haplotype.
69. What is tile function of tile class-/l major h istocompatibility complex a migen ?
a. b. c. d. e.
serves as a complement receptor presents antigen to T helper cells presents antigen to T cytotOxic cells serves as an Fc receptor presents antigen to B cells
70. Which disorder can have either an autoimmune or a congeniral etiology?
a. b. c. d. e.
myasthenia gravis pemphigus foliaceus Waldenstrom's macroglobuline mia bovine leukocyte adhesion deficiency Cushing's disease
1. e IgA is a secretory antibody. It is protected fro m
2.
3.
develop disease related to which body systems?
musculoskeletal and nervous systems cardiovascular and respiratory systems gas(fointestinal and respiratory syste ms urinary and reproductive syste ms lymphoid and hematopo ietic systems
72. Tile barrier function of the skin asan innate defense mechanism is best illustrated by:
a. b. c. d.
allergic dermatitis discoid lupus erythematosus contact dermatitis Fllsobacterium necroplJOrum infection ohhe equine frog e. hypersensitivity to Culicoides species 73. Indigenous flora are desirable in tile gasrroimestinal tract because they:
a. b. c. d. e.
prevent adherence of pathogenic bacteria decrease the transit time of ingesta promote secretion of digestive enzymes stimulate IgA production act as mitogens for Peyer's patch lymphocytes
e 1998 Mosby-Year Book, Inc. PllOfocopying is prohibited by law.
simian immunodeficiency virus sequencing the human genome major hi stocompatibility complex restriction bovine leukocyte adhesion deficiency foot-and -mouth disease
75. III treatillg patiellts with some types of tumors, rhe patient's T cells arecOlwerted to lymphokilleacci(I(Ited killercelfs by in vitro exposure to:
a. b. c. d. e.
inte rferon -a interferon--y interle ukin 3 interleukin 4 interle ukin 2
Answers
71. A dog deficient inIgA would be most likely to
a. b. c. d. e.
139
Immunology
74. A veterinariall sllared ill tile 1996 Nobel Pri2.efar Medicine, based Oil his research on:
immune complexes
65. Advantages ofa noninfectious subunit vaccine
a. b. c. d. e.
www.vet4arab.co.cc
SECTIO N 5
138
4.
5.
6.
7.
8. 9. 10.
II.
digestion by its secretory component, a peptide produced by epithelial cells on the mucosal surface. b IgM has a molecular weight of about 900,000 daltons and is too large to easily d iffuse through the vascular endothelium. d IgE is the only immunoglobulin that binds to high-affinity Fc receptors on mast celis. Some species have an IgG subclass thai binds to mast cells with lower affinity, but this is inconsistent. Therefore IgE is the best answer. c IgD is the only immunoglobulin for which the re is no identified function other than as a receptor in B cell development. IgM also serves as a B cell receptor during development, but this is not its major role. a IgG predominates in the seco ndary antibody response, whe reas IgM predominates in the primary response. b A mast cell degranulated by an IgE-antigen reaction on its surface releases histamine (type-I hype rsensitivity reaction). d Antinuclear antibodies are present in patients with syste mic lupus erythematos us. One of the complications associated with this diso rder is development of immune complexes and subsequellt type-III hypersens itivity reaction in the glomeruli, which ultimately results in glomerulonephritis and kidney failure. a Germinal centers that produce B lymphocytes are located in the cortex of lymph nodes. e T cells are mainly found in the para cortex of lymph nodes. a Lymphocytes exit the lymph node through the efferent lymphatics. c T cells mature and acquire theirT cell markers in the thymus.
12. b Both of these are immunodiffusion tests
13. 14. 15. 16.
17.
18.
19.
20.
21.
requiring soluble antigen and antibody so as to form insol uble complexes at the equivalence point. b T lymphocytes are incapa ble of presenting antigen. e Interleukin 12 and interferon gamma are both T helper type- l cytokines. e Interleukins 4, 10. and 5 are all T helper type-2 cytokines. b Interleukin 2 is produced byT cells and binds to interle ukin 2 receptors on T cells. This binding stimulates T cells to divide; thus interlcukin 2 is an aUlocrine growth factor. e lnte rleukin 12 induces T helper 1 cells to produce ...,-interferon, which is also known as macrophage-activating factor. Macrophages must be activalCd to kill facultative intracellular bacteria. e -v- inte rferon increases ruffled me mbrane activity. activates e nzyme systems, and enhances the killing ability of macro phages. d Im munoelectrophoresis distributes the serum protein fractions. Then, using agar gel diffusion with antiserum to specific immunoglobulin heavy chains, one can identify the de nse arc characteristic of the monoclonal antibody (paraprotein). e Polyclonal gammopathy can occur with pyometra, chronic abscess, dirofilariasis, and syste miC lupus erythematosus. These conditions all produce ch ronic antigenic stimulation. Plasma-cell tumors produce monoclonal gammopathy. d After a foal consumes colostrum that contains antibodies 10 the foal's e rythrocyte antigens, complement is fIxed and the foal's erythrocytes are lysed or removed by the reticuloendothelial system .
140
SECT ION 5
22 . d This calf has an inherited trait that makes it unab le to make the COI l/COlS adhesion molecule; thus the leukocytes cannot adhere to the vascular endothelium and canoO[ undergo diapedesis and move into the tissues. 23. c Foals with combined immunodeficiency lack lymphocytes and therefore cannot make antibody. Affected foal s have some IgG from the maternal antibodies in lhe colostrum but have no IgM because that must be made by the foaL 24 . b Bence-Jones proteins are light chains. 25. b IgM molecules are very large and impart very high viscosity to the blood. IfIgA is bivalent, it can also increase viscosity. The best answer, h owever, is IgM. 26. e Histamine does not cause chemotaxis of neutrophils. 27. c Histamine. serolOnin, and bradykinin act during the immediate response. Leukotriene B4 and prostaglandin O2 are part of the late-phase type- I hypersensitivity response. Only prostaglandin D z causes smooth muscle contraction. 28. e Macrophages are not a part of rype-III hypersensitivity reactions, although they may move into the area later to help clean up the tissue damage. 29. b Sensitized T lymphocytes are responsible for antigen specificity. 30. a This is an Arthus reaction that occurs when antigen and antibody form complexes in the anterior chamber of the eye. 3 1. a A skin test is an in vivo test, not an in vitro test. 32. e Contact dermatitis is a T ceU- mediated type-IV hypersensitivity reaction. 33. a Most antigen-catching EUSAs use a bound virus antigen-specific antibody to "catch" any antigen in the sample. The antigen is then detected by an enzyme-conjugated virus-specific a ntibody (double antibody sandwich). 34. c The virus is passed in the feces of infected dogs. 35. c Pemphigus vuJgaris is caused by antibody to the intercellular material. 36. b If the test is positive for autoantibody, the cells are agglutinated when combined with rabbit anticanine IgG. 37. b The titer for this virus has increased more than fourfold. Any increase of titer between acute and convalescent serum samples must be greater
38.
39. 40.
41.
42. 43.
44.
45. 46.
47.
48. 49.
50. 51.
• www.vet4arab.co.cc
than one dilution for it to be a significant increase. A decreasing titer indicates normal metabol ism of preexisting antibody. c Senun is diluted and used as primary antibody on a fixed cell line (usually Hep-2 cells) on slides that are later labeled with anti-IgG fluorochrome dye (species specific). c Megakaryocytes are found in bone marrow. b Antibodies to the acetylcholine receptors on the motor end plate prevent the signal from crossing the neuromuscular junction and in itiating a muscle contraction; thus the patient appears weak. d Al though autoantibodies to thyroid components are formed, pathologic changes are thought to be caused by the cellular infiltrate. e Tetanus toxin causes the damage, and therefore it is the target for the immune response. b An anaphylactoid response resembles anaphylaxis, but it is not initiated by binding of IgE antibody to mast cells. It is usually initiated by mast-cell degranulation via other mechanisms, such as the alternate complement pathway. e Distemper in dogs and measles in people are both caused bypammyxoviruses.lnduction of antibodies in pups by vaccination with measles virus can protect against distemper caused by cross-reacting epitopes. Fortunately, maternal antibodies w distemper virus do not interfere \vith immunization. c A xenograft is a graft performed between two different species. d The antigen is limited to the respirawry tract. High levels of precipitating aO(ibodies would not be present. b A small-molecular-weight compound that is not immunogenic by itself may become immunogenic by binding to cells or serum proteins that become carrie r molecules and induce an immune response to the hapten. c An IgE response to a drug produces anaphylactic shock. a Horses must obtain all their maternal antibodies from colostrum, whereas people and nonhuman primates obtain it by placental transfer. Dogs obtain most oftheir antibodies from colostrum, with some placental transfer. b After 24 hours of life, antibodies can no longer be transported across intestinal epithelial cells. c Flea-bite hypersensitivity is rypically a type-I reaction, with or without a rype-IV reaction.
© 1998 Mosby-Year Book, Inc. Photocopying is proll/bleed by law.
Immunology
52. d Bone marrow is most commonly affected. 53. a Plasma-cell tumors produce monoclonal immunoglobulin. 54. c Myelomas do not cause hyperglycemia. 55. c Hemolysis indicates that no antigen-specific antibody is present to fix the complement. Sanlple A has no Salmonella-specific antibody (hemolysis in aJll 2 tubes). Sample C has the most antibody because, even at the greatest dilution (tube 12), there is no residual complement (no hemolysis). The other samples fall between these two extremes. 56. c Bone marrow contains immunocompetent cells that respond to foreign major histocompatibility complex antigens on the host's cells. 57. e Antisera for class-I major histocompatibility complex antigens are combined with the cells, followed by complement and a blue dye. Exclusion of the dye (no microcytotoxicity) means that the cell does not have that particular determinant because the complement failed to lyse the cell. 58. b Class-II antigens are typed using a cellular reaction, that is, response of host lymphocytes w donor (or known type) cells. 59. c This answer choice is most accurate. Answer d is true but not correct because monoclonal antibodies detected in patients' sera are not useful in diagnostic tests. 60. e Groups A and Q are most often associated with neonatal isoerythrolysis. 61. c A defi ciency ofC3 inactivates both the classical and alternate pathways. 62. d IgA is the secretory antibody found on mucosal surfaces. 63. e In animals with a fungal infection , injection of the fungal antigen typically causes a type-IV
141
hypersensitivity reaction 48 to 72 hours after antigen injection . 64. e Sensitized T lymphocytes react with antigen and produce cytokines that attract macrophages to the area. 65 . b Some type of adjuvant is needed to increase the immunogenkity of subunit vaccines. 66. e Items listed in answers a through d are found with some tumors, but new histocompatibility antigens are not expressed because the genetic capability is present only fo r those normally expressed by that individual. 67. d Vaccinia virus does multiply in nonhuman species. 68. a Inheritance is codom inant. 69. b Class- II antigens present antigens to helper cells. 70. a Myasthenia gravis may be congenital and inherited. 71. c Affected animals would most likely develop gastrointestinal or respiratory infection because these have mucosal surfaces where secretory 19A is important in preventing infection. 72. d ~ Thrus h" develops when moist conditions compromise the barrier function of the kerati n covering the frog, allowing entry and growth of the causative bacteria. 73. a By occupying a niche in the enteric environment, these organisms prevent adhesion and subsequent infection by pathogens. 74. c The Nobel Prize for Medicine was presented to Drs. Zinkernagel and Dougherty for their work on major histocompatibility complex restriction. Dr. Doughe rty is a veterinarian. 75. e Lymphokine-activated killer cells are produced by stimulating the patient'sT cells in vitro with interleukin 2 and the n returning them to the patient.
NOTES
142
SECTION 5
www.vet4arab.co.cc
SECTION
NOTES
6 Nutrition L.D. Baker, EA Moser
Recommended Reading Case LP el a1: Canine and feline nutrition, 51 Louis, 1995, Mosby. Guilford WG: Strombeck's small animal gastroenterology, cd 3, Philadelphia. 1996, WE Saunders. KeUy NC, Wills JM: Manualofcompanion animal !llIfririo" and feeding, Ames, Iowa, 1997, Iowa
State University Press. Lewis In: Equine clinical nutrition, Baltimore. 1995, Williams & Wilkins. Lewis LD el a1: Small animal d inirol nutrition 111, Topeka, Kan, 1987, Mark Morris Associales. Naylor , N, Ralston 51.: Large animal clinical nutrition. 51 Louis, 1992, Mosby.
Nutrient requirements of domestic animals, Was hingto n , DC. Na tional Academ y Press. (TItles in series: Cats 1986; Dogs, 1985; Dairycattle, 1989; Bee/amle. 1996; Horses, 1989; Sheep, 1985: Swine, 1988; Goats, 1981.)
Practice answer sheet is on page 265.
Questions L.D. Baker 1. 111e approximate energy content of hay is beSt indicated by: a. b. c. d. e.
stem size color smell number of weeds present number ofleaves present
2. Nonprotein nitrogen is used least efficiemiy in a:
a. b. c. d. e.
C 1998 Mosby-Year Book,Inc. Photocopying is prohibited by law.
high-producing dairy cow low-producing dairy cow pregnant ewe gelding on pasture feedlot steer
3. Whicll feed has the greatest energy density?
a. shelled corn b. barley c. whole cottonseeds d. soybean meal e. cottonseed meal 4. Which ofthe followillg is considered a protein supplement?
a. shelled corn b. corn gluten meal
c. soybean hulls d. oats e. barley
Correct answers are on pages 151-153.
143
www.vet4arab.co.cc
SECTION 6
144
5. WI,jel, vitamin is stored in the liver ill significant
10. VVh icll feed has tile lIigllest percentage of rumen
amoullts?
escape (bypass) protein ?
a. fo late
a. b. c. d. e.
b. cobalamin c. thiamin d. pyridoxine
e. niacin 6. WI/jell afthe/ollowing is not used by rumen
distillers' dried grains blood meal whole cotto nseeds urea roasted soybeans
IS. In ruminants what is the primary source of glucose?
a. am ino acids b. acetate c. lactate d. glycerol e. propionate
11. Wllicllfeed has the lowest percentage of rumen
16. Which of the following is not a nonnal end product
microbes as an energy source?
escape (bypass) protein ?
a. ammonia
a. distillers' dried grains
a. lactate
b. cellulose
b. blood meal c. whole cottonseeds d. urea e. roasted soybeans
b. methane
c. peptides d. fat e. carbohydrate 7. When rumen unprotected fats are added 10 dairy cattle rarions. which minerals should be
a. cal cium and phosphorus b. calcium and magnesium c. calcium and potassium
d. magnesium and phosphorus
e. magnesium and potassium 8. A producer wall15lOfeed 0.1 kg ofcalcium to his cow. How milch Iimesrolle con raining36 %
12. A nonpregnantiactating dairy cow is producing 45 L of milk per day, with a calcium content of 1.3
a. 36g b.72g c. 80g d. I18g e. l 65g
d. 2.8 kg e. 3.6 kg 9. A prodllcer is / eeding 21 kg of corn silage to his cow daily. The dry-matter content is 30%. How muell dry mOiler is being/ed daily?
a. b. c. d. e.
a. b. c. d. c.
acetate glucose propionate amino acids lactose
14. Which o/the/ollowing is most resistant
ferm elllative digestion in the rumen?
70 kg 0.63 kg 7.0 kg 6.3 kg 0.7 kg
a. b. c. d. e.
@ 1998
cellulose hemicellulose pectin lignin starch
Mosby-Year Book, Inc. Photocopyi/lg is prollibited by law.
17. In a well-managed dairy operation, Holstein heifers are reared to adljew what ideal body weight at breeding (12 to 14 momlrs ofage)?
a. 270kg b. 320 kg c. 350 kg d. 385 kg e. 400 kg
A dairy farmer has ctllled on you to evaluate tire nutritional status of early-lactation cows. At the farm you determine Ihat this group ofcows has access to a total mixed ration at all times. However, the cows appear very thin and milk production is suboptimal. The forage/grain ratio, on a dry-mauer basis, is 50:50. 18. To minim ize bodyconditionioss and improve milk production, tire most appropriate recommendation is to:
co
a. ad libitum to a ll cows in early lactation, regardless of milk production levels b. ad libitum only to cows with high milk produc tion c. ad libitum only to thin cows d. in limite d amounts in early lactation until cows prove the ir milk production potential e. in limited amounts to all cows, regardless of milk production or stage of lactation 20. Excessi/)(! j"takeofllOllStrucwral carbohydrate is conducive to all of the following except:
Questions 18 and 19
13. The volwne of milk produced is determined mainly by tlie mammary glands uptake of:
c. 0.36 kg
c. increase the caloric density of the diet by e levating the protein concentration in the grain mix without changing the grain/ fo rage ratio d. increase the caloric density by inc reasing the amount of grain and adding fat to the ration to maintain forage levels at 40% or more of dry matter e. reduce the caloric density by increasing the amount of forage in the ration to 60% or m ore of dry ma tter to supply sufficient d ie rary fib e r 19. Tile revised ration should befed:
c. carbon dioxide d. glycerol e. acetate
calcium should be/ed? a. 0.036 kg b. 0.28 kg
145
ofctlrbohydrate femlentation in the rumen?
gIL This cow excretes 1.5 gofcalcium daily in her urine, and her endogenous fecal ctllcium loss is 6 glday. If dietary calcium is 40% aooilable, what is this cows daily dietary ctllcium requirement?
supplemellted?
Nlltritlon
a. increase the caloric density of the diet by reducing the amount of forage in the ration to 30% of dry m atte r so that m ore grai.n can be fed b. maintain the same caloric density by substituting better-quality forages in the ration, allO\ving the amount of grain to be reduced
a. b. c. d. e.
increased milk volume decreased milk fat content decreased milk protein content decreased rumen pH increased rumen turnover
2 1. Which cow is 11I0silikely to develop milk fever ?
a. 2-year-old Jersey fed alfalfa hay before calving b. 5-year-old Holstein fed timothy hay during the d ry period c. 2-yea r-old Holstein fed alfalfa hay before calving d. 5-year-old Holstein fed alfalfa hay during the dry period e. 3-year-old Jersey fed timothy hay during the dry period
Correct answers are on pages 151- 153.
146
www.vet4arab.co.cc
SECTION 6
22. Which pastllTecomposition grazed by cattle and sheep is mosllikeiy to predispose to grass tetany? a. high phosphorus and calcium. low pmassiulll and sod ium b. high potassium and c..'l1cium, low phosphorus and sodium c. high nitrogen and calcium, low phosphorus and sod ium d. high nitrogen and phosphorus, low potassium and calcium
e. high nitrogen and potassium, low sodium and
d, 133% of its requirement for protein c. 150% of its requirement for protein
24. lWlicllfeedstllfffor IlOrses lias tile highest amount ofdigestiblee"ergy per u"it I/Olullle?
a. b. c. d. e.
rolled oats timothy hay alfalfa hay shelled corn beet pulp
calcium 23. A race horse is beingfed 10 lb of 18% cnlde protein
alfalfa Ill/yand 121bofa 10% sweet feed. The horse requires 2.51bofprorein per day. In relation to recommellded levels. tlJis horse is being/ed: a. 80% of its requirement for protein b. 100% of its requirement for protein c. 120% of its requi rement for protein
25. Witll adequate "utrition, a "ormai400- to 550-kg mare call be expected to produce how much milk dailydllring thefirsI3 months oflactatio,,?
a. 5 to 8 kg/day b. 7to 10 kg/day c. 9to 12 kg/day d. II to 14 kg/day e.15toI8kg/day
LA. Moser 26. A diet contains nutrient A in tile amount of5 mglkg dry matter. flow many parts per million of nlltrierll A are illcluded in tile diet? a. 0.0005 b. 0.05 c. 0.5 d.5 e. 500
29. WJ,ic" amino acid is esselltial for cats but not for dogs?
a. valine b. threonine
c. lysine d. methionine
c. taurine 30. Which amino acid contains sulfur?
27. Tile most abundanl fauyacid ill com oil is:
a. b. c. d. e.
stearic acid palmitic acid oleic acid linoleic acid arachidonic acid
28. flow is 10,000 parIS per million expressed as a percentage (parts per lIundred)?
a. l % b. 10%
c.O.I% d.l OO% e. 10,000%
glycine valine leucine isoleucine e. methionine a. b. c. d.
31. Wlmt is tIle majorcarboltydrate-digesti"genzyme secrered in pallcrearicjuice?
a. b. c. d. e.
a-amylase chyme deXlrinase trypsin chymotrypsin
e 1998 Mosby- Year Book, Inc. Plwtocopying is proll/'bilcd by law.
Nutrition
32. Wllat is the primary copper-containing protein in mamnmIian plasma?
a. plasferrin b. "y-globulin c. albumin d. ceruloplasmin e. transferrin
a. arginine, lysine, and glycine b. valine. leucine, and isoleucine c. tyrosine, tryptophan, and arginine d. taurine, methionine, and lysine e. carnitine and lysine 34. What is the protein equivalent of urea?
a. vitamin A b. vilaminC c. thiamin d. folic acid e. vitamin D 39. Wllich vitamin promotes growth and mineralization of bones and increases calcium absorption from tile gut?
•.
vitamin A
b. vitamin D
a. 16% b. 100% c. 281% d. 0.015% e. 625%
c. thiamin d. choline
e. niacin Questions 40 and 41
35. lActose is a disaccharide made up of which cwo ,nonosaccharides?
a. b. c. d.
glucose and fructose glucose and glucose fructose and raffinose sucrose and maltose c. glucose and galactose
A dog owner comes to your clinic with the following infomUltion about a dogfood she obtained from a pet food company represemative: Moisture 75.0% Crude protein 9.0% Crudefat5.0% Crude fiber 1.0%
36. Whicll of the following are B vitamins?
tryptophan aJld thiamin
40. MJat is the approximate crude protein content on a dry-matter basis?
a. 6.75% b. 12.0%
b. thiamin and vitamin K
c. riboflavin and choline d. thiamin and pantothenic acid e. pantothenic acid and choline
c. 22.5% d. 36.0% e. 63.0%
37. Which vitamin is involved in formation ofactive prothrombin and plays an important role in blood clorting?
a. vitaminA b. vitamin B, (pyridoxine) c. niacin
d. vitamin K e. choline 38. Which vitamin isa constituent of rhodopsin and is involved in mucopolysaccharide synthesis and maintenance ofepithelial tissues?
33. TIle branched-chain amino acids include:
3.
147
41. What is the percent dry marter in the food?
a.90.0%
b. 25.0%
c. 75.0% d. 10.0%
e. 60.0%
Correct omwers are 011 pages 151 -153.
148
www.vet4arab.co.cc
SECTION 6
42. lonophores are low-molecular-weight molecules tlult hind ions of various minerals and modulate
rlleir movement across membranes. A major effect QII
mmen/ermenration ofionophores is:
a. b. c. d.
increased methane production decreased molar ratio of acetate [Q propionate increased molar ratio of acetate to propionate decreased production of propionate
e. decreased feed efficiency 43. When dairy cattle are fed unprotected long-chain fOlty acids, precautions must be taken to provide supplemental sources of a. b. c. d. e.
calcium and phosphorus zinc and copper manganese and magnesium iron and phosphorus calcium and magnesium
44. Digestible etiler extract fat cotltains approximately lIow much energy per unit weiglu,
as compared with carbohydrates? a. b. c. d.
about the same about 6.25 times as much about 2.25 times as much the energy from carbohydrate and fat cannot be compared on a per unit weight basis e. about half as much
45. Wllich ofthefoflowing most acc:urately indicates tlie prolein and fat cOlllerlts (80% moisture as fed) of tile milk ofcats?
a. b. c. d. e.
10 g protein and 6.5 g fat per 100 g milk 15 g protein and 7.5 g fat per 100 g milk 4 g protein and 6.0 g fat per 100 g milk 8 g protein and 5.0 g fat per 100 g milk 45 g protein and 45 g fat per 100 g milk
46. faIry acids are classified by the number of double
bonds they contain. Afalty acid with one double bond is classified as:
a. saturated b. polyunsaturated
47. "Vllat is the longest portiorl of tile small intestine in companiorl animals?
a. duodenum b. pylorus c. jejunum d. ile um e. ileocolic 48. For wll ich of the three classes of nutrients (carbohydrates, proteirlS,[at) are digestion and absorption most complex and most efficient?
a. protein b. fat c. carbohydrate d. protein and fat e. carbohydrate and fat 49. Mlich IlOrmone stimulates pancreatic enzyme secretion and causes contraction o/the gaflbladder, aflowing lipase and bile to enter the diloderillm?
a. b. c. d. e.
cholecystokinin insulin chymotrypsin glucagon trypsin
52. During lactation in queeru,[ood imake typically:
a. b. c. d. e.
stays the same increases to about 1.5 times normal increases to about 2 to 3 times normal increases to about 5 to 6 times normal decreases to a bout 50% of normal
53. Halogeton and greasewood are poisonous plants that grow in the arid and semiarid saline regions of the westem United States. I1IeY are toxic to sheep because of their high content of:
a. b. c. d. e.
55. Gross energy minus energy lost infeces, urine, and
a. b. c. d. e.
to capillary beds to the centrallacteals of the villi into the bowel lumen into the mucosal lining into the brush border region
a. b. c. d. e.
sunlight on skin, changed to the active form?
a. b. c. d. e.
digestible energy metabolizable energy net energy available energy gross energy
ergosterol 7-dehyd rocholesterol cholesterol cholecalciferol zearalenone
58. Which of the following most accurately describes the/undamenral role o/vitamin E?
antirachitic precursor of coenzyme E antioxidant antired uctant required for prothrombin formation
59. What is the primary nutritional cause of simple
goiter (enlargement of the thyroid gland)?
a. b. c. d. e.
ruminants carnivores nonruminant herbivores obligate omnivores variable omnivores
gaseous products ofdigestion is equal to: SO. Chylomicroru exit the intestinal mucosal cells into the lamina propria and are transported:
57. In animals what is the predomirumrform of provitamin D, wllich is, through the action of
a. b. c. d. e.
copper oxalate calcium selenium ketone bodies
54. Based on digestive anatomy and physiology. rabbits are classified as:
a. b. c. d. e.
149
Nutrition
iodjne deficiency iodine toxicity vitamin K deficiency vitamin K toxicity sodium toxicity
60. What is the generally recommended minimum rlutricnt standard for crude protein for canine adult maintenance (units = dry-matter basis at 3.5 kroVgJ? a. 10% b. 15%
c. 18% d. 23%
56. The standard metl/Od of diagnosing food
e. 25%
hypersensitivity in dogs involves: 51. Unlike most ocher mammals, cats cannot convert ~-carotene
a. b. c. d. e.
to:
vitamin C vitamin B\ vitamin A folacin arachidonic acid
c. unsaturated d. essential e. nonessential
C J998 Mosby-Year Book, Inc. Pllotocopying is prohibited by law.
a. b. c. d. e.
serologic examination fasting, purgation, and challenge elimination and sequential challenge enzyme-linked immunosorbent assay intradermal testing
61. MUll is the generally recommended minimllm nutrient standard for crude protein for canine growth arid reproduction (units = dry-matter basis at 3.5 kroVg)?
a. 15% b.23% c. 28% d.30% e. 43%
Correct answers are on pages 151·153.
62. In dogs, tlJeanemiQ associated with copper deficiency is classified as: a. b. c. d. e.
hypochromic microcytic hyperchromic microcytic normochromic normocytic hypochromic m acrocytic hype rchromic macrocytic
63. 111eftrst step in metabolicacrivation a/vitamin D is conversion of cllOfecalciJerol to 25hydroxycllOlecalciferol in the: a. kidney
b. c. d. c.
www.vet4arab.co.cc
SECTION 6
150
hypothalamus brush border of the small intestine liver skin
64. Oftllefollowingfeedstl1/fs, which isgenemlly rich in camitille?
a. meat b. algae
67. In wllich disease ofdogs can serum folate and vitamin B,z levels be used as screening tests?
a. b. c. d. e.
intestinal malabsorption syndrome hepatic steatitis syndrome hyperthyroidism diabetes m ellitus diabetes insipidus
68. MlQt are tile most common nutritional deficits in
fed home-cooked diets for dogs atld cats?
a. protein and fat b. vitamin A and copper c. vitamin A a nd essen tial fatty acids d. salt and protein e. fat and copper 69. The toxic methylxallllline derivative, tlleobromine, m ay cause vom iting, diarrhea, sudden collapse, and death in dogs and cats. Mlat common hOlLselloldfoodstuffcontains this marerial?
c. legumes
a. cocoa
d. yeasts e. fat
b. onions c. whole wheat Oour d. cornstarch e. vanilla
65. MlQt is the primary site ofcholesterol syn thesis in most animals? a. brush border o f the small intestine b. adipose tissue c. kidney d. panc reas e. liver
66. In normal dogs, ingestion a/which type of commercially form ulated dogfood results in the highest immediate postprandial p lasma glucose and insulin concemrations?
a. canned b. c. d. e.
dry (lObble) dry (kibble) soaked in wate r semimoist a mixture of canned and dry (equal calories from each)
@ 1998
70. How many carIS ofdogfood (SOO-kcal metabolizable energy per can) are needed da ily for maintenance ofan auerage 13.6-kg (30-/b) dog? a. 2 b.3 c. 4 d.5 e. 6
71. An animal will die most quickly from iackofwhar nutrient?
a. b. c. d. e.
water protein carbohyd rate fat e ne rgy
Mosby-Year Book, Inc. Pllotocopying is prollibited by klw.
151
Nutrition
72. Cobalt is a component of
74. What is the calciumJpllOsplJOrus (CalP) ratio in mammalian bone?
a. folic acid b. biotin c. camiti ne d. vitamin B il c. taurine
a. 1:2
b. 201 c. 3: 1 d. 0.5: 1 e. 5: 1
73. Iii which cwo dog breeds does primnry hepatic copper storage disease occur as a result ofa hereditary autosomal recessive disorder?
a. collie and spitz b. dac hsh und and golde n retriever c. Bedlington terrier and West Highland white terrier d. bulldog and pug e. borzo i and beagle
75. A dog has a thin hair coat, scaly dennatitis (especially around the eyes, mouth, and ears), pamkerarosis, and hair depigmematioli. l1,ese signs are most likely related to a dietary deficiency of
a. b. c. d. e.
zinc iron sele nium calcium cadmium
Answers I . a Larger s tem s izes refl ect forage maturity, high
fiber levels. and lower digestibility and available energy. 2. d Nonprotein n itrogen has little benefit for hindgut fe rmenters, such as horses. 3. c This feed contains the greatest amount of fa t. 4. b This feed contains more than 20% of crude protein. 5. b This is vita min 8 ,2 , 6. d Rumen microbes cannot use fat as an e nergy source. 7. b Calcium a nd m agnesium form insoluble soaps with rumen unprotected fats. 8. b 0.1/0.36 =0.277 kg. 9. d 2 1 kg X 0.30 = 6.3 kg. 10. b Approximately 80% of the protein bypasses the rumen. 11. d Urea is 100% degradable in the rumen. 12. e 45L x l.3g/L=58.Sg+L5g+6g= 66g/0.4 = l 6Sg. 13. b Lactose is the osmotic regulator of m ilk volume and is synthesized from glucose in the mammary gland. 14. d Lignin is not a carbohydrate and is not fennentab le. 15. e Glucose is produced from propionate via gluconeogenesis.
16. d Glycerol is produced from triglycerides a nd not from carbohydrates. 17. c This is the ideal body weight fo r heifers to be bred. 18. d Th is strategy sho uld red uce body condition loss and improve milk production. 19. a The ai m is to maximize caloric intake to s uppor! early-lactation milk production and to lim it mobilization of adipose tissue reserves. 20. c Increased fermentable carbohydrate in the rumen generally increases microbial yield and increases milk protein content. 2 1. d Excessive calcium fed during the dry period plays tlle largest role in development of milk fever in multiparous cows. 22. e These factors inhibit the availability of magnesium. 23. c I.B + 1.2 =3 1b of protein consumed ; 3/2.5 required = 1.2 = 120%. 24. d Corn is more dense tha n oats a nd weighs more per volume. 25. d Mares in early lactation can produce 25 to 30 lb of milk daily. 26. d 5 mgl kg = 5 ppm (pans per million). 27. d More tha n 50% o f the fatty acids in corn oil is linoleic acid . 28. a 10,000 ppm = 1%.
www.vet4arab.co.cc
SECTION 6
152
29. e Taurine is an essential amino acid for cats but not for dogs. 30. e Methionine, cysteine, and cystine arc sulfurcontaining amino acids. 31. a a-Amylase is the primary carbohydrate digestive enzyme secreted in pancreatic juice. 32. d Ceruloplasmin is the primary coppercontaining protein in mammalian plasma. 33. b Valine, leucine. and isoleucine are branchedchain amino acids. 34. c 45% nitrogen x 6.25 about 281 % protein equivalent. 35. e Lactose is comprised of glucose and galactose. 36. d Thiamin and pantothenic acid are B vitamins. 37. d Vitamin K is involved in formation of active prothrombin and is important in blood clotting. 3B. a Vitamin A is a constituent of rhodopsin, is involved in maintenance of epithelial tissues, and has a role in mucopolysaccharide synthesis. 39. b Vitamin D promotes growth and mineralization of hones and increases calcium deposition. 40. d 9.0 divided by 0.25 36% protein on a drymatter basis. Divide the as-fed protein by the percentage of dry matter to get the percentage of protein on a dry-matter basis. 4 1. b 100 minus water (75) dry matter of 25%. 42. b A decreased molar ratio of acetate to propionate decreases methane production and increases propionate production. 43. e Dietary fat may form a complex with calcium and magnesium, making them unavailable for absorption by the gastrointestinal system. 44. c Digestible ether extract fat contains about 9 kcal/g, whereas digestible carbohydrate contains about 4 kcal/g. 45. a On a dry-malter bas is, this equates to about 44% protein and 28% fat. Compare this with cow's milk, which contains about 26% protein and 36% fat and bitch's milk, which contains about 35% protein and 43% fat. 46. c Fatty acids wim no double bonds arc saturated. Those with one or two double bonds are unsaturated, whereas those with more than !\vo double bonds are polyunsaturated. 47. c The jejun um is the longest part of the small intestine and fi Us the mid-abdomen. 48. b In good-quality diets. fats are usually digested wilh more than 90% efficiency in dogs and cats.
=
=
=
49. a Cholecystokinin , aJongwith secretin, is released into the blood when add and partly digested food from the stomach e nte r the duodemun. stimulating secretion of pancreatic juice. 50. b Chyiomicro ns exit intestinal mucosal cells. enter ]acleals, and then move into larger lymphatic channels. 51. c Cats cannot convert j3-carotene to vitamin A. 52. e During lactation a Queen's food intake typically increases 2 to 3 times normal. 53. b These plants contai n a high level of oxalates. 54. c Rabbits are cecal fe rmenters. 55. b Gross energy - Fecal energy Digestible energy. Gross e ne rgy - Fecal/ urinary/gas energy Metabolizable energy. 56. c Food hypersensitivity is diagnosed by feeding an elim ination diet, followed by challenge with individual fo od ingredients. 57. b In animals, provitamin D is 7dehydrocholesterol. In plants, provitamin D is e rgosterol. 5B. e Vitamin E acts as an antioxidant in food and in the body. 59. a Goiter is primarily caused by insufficient iodine. 60. c The generally recommended level of crude protein for maintenance is 18%. 61. b The generally recommended level of crude protein for growth and reproduction is 23%. 62. c Copper deficiency causes normochromic normocytic anemia. 63 . a The principal active metabolite is formed in the kidney. 64. a Meats are a rich source of carnitine, whereas plan ts are a poor source. 65. e The liver is the primary site of cholesterol synthesis from acetate. 66. d Semimoist dog food co ntains large amounts of simple carbohydrates. 67. a Low serum levels of these nutrients indicate intestinal malabsorption. 68. b Most home-cooked diets have high levels of protein and fat but insufficient vitamin A and copper. 69. a Cocoa and related products (chocolate) contain theobromine. 70. a A 13.6-kg dog requires 900 to 1000 kcal of metabolizable energy per day and so would require two cans of this food.
=
=
e 1998 Mosby-Year Book, Inc. Photocopying is prollibited by law.
Nlltrition
71. a Fat-free muscle is composed of75% to 80% water. Water is required for essentially all body functions. 72. d Cobalt is a component of vitamin BI2 (cyanocobalamin).
153
73. c Bedlingtons and Westies may inherit copper storage d isease. 74. b There is little variation in the elementary composition of bone ash from the 2: 1 Ca/P ratjo. 75. a A deficiency of zinc would cause these clinical signs.
NOTES
154
SECTION 6
www.vet4arab.co.cc
SECTION
7
NOTES
Pharmacology T.P. Clark, L.E. Davis, S.H. Duran, J.E. Riviere
Recommended Reading Adams HR: Veterinary pharmacology and therapeutics, cd 7, Ames, Iowa, 1995, Iowa State University Press.
Allen DG: Handbook o!veterinary drugs, Philadelphia, 1993, Lippincott, Barragry TC: Veterinary drug therapy. Baltimore. 1994, Williams & Wilkins. Duran SH , Lin He: Mosby's IIf!rcrinarydrug re/erenre, 51 Louis, 1998, Mosby.
Ha rdman IG et aI: Goodman alld Gilman's rIle pJwrmarologica/ basis olrlierapeutics, cd 9, New \ork, 1996, McGraw-Hill. Me lmon KL et al: Melmon and Morrelli's clinical pharmacology: basic principles in therapeutics. cd 3, NcwYork, 1992. McGraw·HilI.
Physicians'desk reference, Oradell, NJ. 199B, Medical Economics. PI!}'sician's genRx, 5t Louis, 1998, Mos by. Plumb DC: Veterinary drug handbook, ed 2, Ames, Iowa, 1995, Iowa Stale University Press.
Prescott IF. Baggol ID: Antimicrobial /hempy in veterinary medicine, cd 2, Ames, Iowa, 1993, Iowa Slate University Press.
Ve/erinQrypllllnnaceuticalsand biologicals 199711998, ed 10, Lenexa, Kan, 1997, Ve lerinary Medicine Publishing. Practice answer sheets are on pages 267-268.
Questions T.P. Clark 1. A partial agonist is a drug tllat:
a. blocks the response produced by an agonist b. produces a maximal response by occupying all or a fraction of the receptors c. produces a half-maximal effect d. produces less than a maximal response even when it occup ies all of the receptors e. produces half-maximal inhibition
01998 Mosby-Year Boolc,lnc. Photocopying is prohibited by law.
2. Five drugs are known to bind to a,-adrenergic receptors. Using radiolabeled drug-binding studies, equilibrium dissociation constants (KrJ are determined for each of rile drugs. Which hypothetic drug has the highest affiniry for tile aI-adrenergic receptor?
a. b. c. d. e.
drug 1_ KD = 10 nanomolar drug 2 _ Ko = 55 nanomolar drug 3 _ ~ = 0.21 micromolar drug 4 _ Ko = 5.5 micromolar drug 5 _ Ko = 0.12 molar
Correct answers are on pages 178· 184.
155
156
www.vet4arab.co.cc
SECTION 7
is a weak acid (pK" = 4) and must penetrate the blood-brain-barrier to exert its effects. WI,e" used in a dog Wilh respiratory acidosis (blood pH 6.8), how should the drug dose be adjusted. compared with the dose givell to a normal dog (blood pH 7.4), to achieve tiJerapemic levels ill the brai,,?
6. Drug X and drug Y have identical whole-body clearance rates (4 ml/min· kg). However. tile plasma eliminatioll/ialf-lifefor drngX is I hour;
a. Increase the dose because more of the drug is ionized. b. Decrease the dose because more of the drug is ionized. c. No dose adjustments are necessary. d. Increase the dose because more of the drug is nonionized. e. Decrease the dose because more of the drug is nonionized .
b. Drug X is eliminated from the kidney and
3. A newllnaigesic compound
4. A cow has mastitis caused bya Staphylococcus
species. Based on bacterial culture and sensitivity testillg. the following antibiorics are indicated. Based on rile prillciples o/ioTlic trapping, which one will achieve the hig11est concentration ill milk (pH 6.8) when used systemically (blood pH 7.4j?
a. cloxacillin (acid, pK. 2.7) b. ampicillin (acid, pK. 3.4) c. cephaloridine (aCid. pK. 4.4) d. sulfadimethoxine (acid. pK. 6.1) e. erythromycin (base. pK. B.B)
whereas the plasma elimination half-life for drug Y is 48 hours. W1Jat accounts for these drugs havillg identical whole-body clearance?
a. Drug X is metabolized. and drug Y is not. drugY in the bile.
c. Drug X has a smaller volume of distribution (han drugY. d. Drug X is more lipophilic than drugY. e. Drug X is an acid, and drug Y is a base.
sylllhetic pathway functions at a very slow rate?
a. horses b. cattle
c. pigs d. calS e. dogs
rapid renal excretion. Peak blood levels after a single injection are 16 W1ml. l/tlleal1tibiotic has a Iialflife 0/ 1.5 1I0urs. what would the predicted blood levels be 7.5 hours after an injection?
a. 12fJ.g /ml b. 8 ILg/ ml
c. 3IJ.g/ ml d. 1.5ILg/ml
e. 0.5 jl.g/ml
;n a dog. Using a J:1000 epinephrine solution, how sllOuld tliis beadmjllistered toa 2S-kgdog?
a. DiluteO.1 ml of the l:IOOOsolution to I mI, and inject 0.25 ml. b. Dilute 0.1 mJ of the 1:1000 solution to 10 mJ. and adm.inister 0.25 mt c. Dilute 0.1 Illl of the 1:1000 solution to 100 mi. and administer 0.25 mJ. d. Administer 0.25 ml of the I: 1000 solution. c. Administer 2.5 ml of the 1:1000 solution. 11. Epinephrine is used with local anesthetics
a. a-adrenergic receptor agonist propenies b. 132-adrenergic receptor agonist properties c. 131 -adrenergic receptor agonists properties d. muscarinic receptor agonist properties c. nicotinic receptor agonist propenies 12. Wliich drug would potentiate the effects 0/
8. A drug tllat mimics parasympatheticstimulatioll o/thegastrointestinal tract would cause:
..
c. d. e.
increased motility and sphincter constriction decreased motility and sphincter constriction increased motility and sphincter relaxation decreased motility and sphincter relaxation no effect
9. What is the order of potency/or I3radrenergic receptors, from most potent to least potent?
a. norepinephrine> isoproterenol > epine phrine b. isoproterenol > norepinephrine> epinephrine c. All have equal potency. d. epinephrine> norepinephrine> isoproterenol e. isoproterenol> epinephrine> norepinephrine
a. b. c. d. e.
bromine; kidney bromine; liver fluorine; kidney fluorine; liver methanol; centraJ nervous system
16. W1Jat effects does halothane hatJe all the
cardiovascillar system?
u. depresses myocardiaJ contractility but maintains normal blood pressurc by s timulating the sympathetic nervous system b. maintains normal blood pressure by stimulating the baroreceptor rcflex c. protects the heart from the arrhythmogenic effects of epinephrine d. can induce spontaneous arrhythmias, especially at light surgical anesthesia (stage III, plane I) e. causes dose-dependent cardiomyopathy 17. W11a' is a primary metabolite a/halothane. and
IlOw is tile toxic effect o/this m etabolite l7lallifested?
b. neostigmine
c. atropine d. scopolamine
c. pilocarpine 13. Tile effect a/an intravenous injection of acetylcholine all blood pressure call be blocked by:
a. metoclopramide b. propranolol c. d-tubocurare d. atropine c. nitroprusside
infusion a/dopamine Oil the renal vasculature is caused byacrivotioll of
dopamine receptors a-adrenergic receptors 132-adrenergic receptors muscarinic receptors nicotinic receptors
a. b. c. d. e.
bromine: lethargy and ataxia bromine; renal damage chlorine; metabolic alkalosis fluo rine: hepatic damage fluorine; re naJ damage
18. All tlle/ollowing are associated with routi/Ie. properly administered halothane anesthesia except:
14. III dogs tile vasodilator effect 0/ a low-dosage
a. b. c. d. e.
C 1998 Mosby-Year Book, Inc. Photocopying is prollibited by law.
15. W1lnt is a primary metaboliteo/methoxyfluralle. and where does this metabolite exert its toxicity?
succillylc/lOlille?
a. methacholine
b. 5. A highly polar, parenterally administered alltibiotic lias a sllort plasma eliminatioll half-life because 0/
10. A" epinephrine dosage of0.01 mg/kg;s indicated
because a/its; 7. Drug X undergoes phase-[J metabolism by being conjugated with glucuronic acid. Drug X carmot be used ill which species because the glucurollide
157
Pharm4cology
a. transient decrease in the ability to concentrate urine because of mild renal hypoxia b. transient increase in blood urea nitrogen and creatinine because of decreased glomerular filtration rate during anesthesia c. transient liver damage because of mild hepatic hypoxia d. increased cerebral blood flow e. decreased intracranial pressure
Q)rrecranswers are on pages 178- 184.
158
SECTION 7
19. Concerning nitrous oxide, which statement is most accurate?
a. Most dogs can be successfulJy anesthetized using only a mixture of80% nitrous oxide and 20% oxygen. b. Nitrous oxide causes distention and increased pressure inside any closed, air-containing space within the body. c. Nitrous oxide slows uptake of other anesthetic gases from the alveoli to the blood by competing for anesthetic transport molecules. d. Because of its excitatory effects on the central nervous system, nitrous oxide use increases the dose of other anesthetic agents required to attain surgical anesthesia. e. Nitrous oxide decreases activity of the sympathetic nervous system and therefore has desirable antiarrhythmic effects. 20. Prop%l causes centrallleruous system
depression and:
a. is an anticonvulsant b. does not cause respiratory depression c. decreases intracranial and intraocular pressure d. is metabolized in the liver e. is a glutamate receptor agonist 21 . Barbiturates produce anesthesia through:
a. increased synaptic transmission b. increased seroton in release c. action at the -y-aminobutyric acid (GABA) receptor (GABA mimetic) d. depression ofthe thalamoneocortical system in conjunction with activation of the limbic system e. decreased oxygen uptake by neurons 22. All thefollowing are associated with thiobarbiturates except:
a. rapid distribution across the blood-brain barrier b. initial rapid distribution to highly perfused tissues, followed by redistribution to other tissues c. rapid recovery because of redistribution of the anesthetic into adipose tissue
www.vet4arab.co.cc
d. distribution across the placenta and into fetal tissue e. prolonged recovery in obese animals 23. What is a cardiovascular side effect of most barbit1lrates?
a. bradycardia in dogs, caused by stimulation of the vagus nerve b. tachycardia in dogs, secondary to peripheral vasodilation and decreased arterial pressure c. prevention of catecholamine-induced arrhythmias d. increased mean arterial pressure c. increased cardiac output 24. Mlich one of the following most accurately
describes ketamine anesthesia?
a. Ketamine causes rapid induction to stage III of anesthesia. b. Peripheral reflexes are lost at surgical planes of anesthesia. c. Ketamine has a wider therapeutic index (LD 50 / ED 50 ) than barbiturates. d. Ketamine is a potent visceral analgesic. e. Ketamine induces potent muscle relaxation. 25. Concerning ketamine, which statement is least accurate?
a. Ketamine is derived from phencyclidine. b. Ketamine is a GABA agonist and glutamate receptor antagonist. c. Ketamine stimulates £he cardiovascular system through a sympathomimecic effect. d. Ketamine increases cardiac output. e. Ketamine is excreted in the bile; therefore poor renal function does not preclude its use. 26. Concerning phenobarbital, which statement is most accurate?
a. It is metabolized by hepatic microsomal enzymes. b. It is not metabolized but rather is excreted unchanged in the urine. c. Long-term therapy with phenobarbital decreases its rate of metabolism. d. It is a potent inhibitor of hepatic microsomal enzymes. e. It is excreted in the bil e.
© 1998 Mosby- Year Book, fllc. Photocopying is prohibited by mw.
159
Pharmacology
27. Concerning primidone, which statement is most accurate?
a. It is metabolized to phenobarbital and phenylethylmalonamide. b. It is the anticonvulsant drug of choice in cats. c. It is a potent inhibitor of hepatic microsomal enzymes. d. It causes none of the side effects reported with phenobarbital. e. Serum primidone levels can be used to guide dosage regimens. 28. Conceming phenytoin, which statement is most accurate?
a. It is 100% bioavaiJable. b. In dogs its elimination half-life is longer than that of phenobarbital. c. It induces hepatic microsomal enzymes. d. It is metabolized to phenobarbital and phenylethylmalonamide. e. It is the anticonvulsant drug of choice in cats.
31. Which of the following most accurately explains why second-generation competitive H, receptorblocking drugs do not cause drowsiness?
a. They compete for H2 receptors as well as HJ receptors. b. They increase cardiac output and oxygen levels in the brain. c. They have serotonin agonist properties. d. They have caffeinelike effects. e. They are less lipophilic and do not cross the blood-brain barrier. 32. Which drug would be most appropriate to give before surgical removal of a mast-cell tumor in a dog?
a. b. c. d. e.
epinephrine prostaglandin F2n cimetidine neostigmine aspirin
33. Omeprazole blocks gastric acid secretion by: 29. Benzodiazepines exert their amicorlvulsam effect by:
a. blocking neuronal fast sodium channels b. enhancing the inhibitory effect of GABA c. inhibiting the excitatory neurotransmitter glutamate d. decreasing influx of calcium into neurons e. blocking neuronal potassium channels 30. Concerning use ofbromide as an anticOlllmlsant,
which statement is most accurate?
a. It hyperpolarizes neurons by replacing intracellular chloride. b. Its elimination half-life is 7 days. c. Without a loading dose, steady-state dynamics can be reached within I week. d. It has no side effects. e. It competitively blocks fast sodium channels.
a. competitively blocking gastrin receptors b. competitively blocking H2 histamine receptors c. noncompetitively blocking acetylcholine receptors d. inhibiting the gastric parietal cell H+K+ATPase pump e. inhibiting gastrin release from pyloric glands 34. Effects of antiinflammatory doses of glucocorticoids include all the following except:
a. binding membrane glucocorticoid receptors b. inducing cellular lipocortin synthesis c. interfering with cellular phospholipase activation d. decreasing cellular leukotriene synthesis e. decreasing cellular prostaglandin synthesis
Correct answers are on pages 178-184.
35. Concerning aspirirl, which statement is most accurate?
a. After absorption, it is metabolized to salicylic acid. h. It is excreted unchanged in the bile. c. It is a weak base with a pKa of 8. d. There is no difference in oral absorption of regular and buffered aspirin. e. It blocks leukotriene synthesis. 36. In which species does aspirin have the longest elimination half-life?
a. ruminants h. c. d. e.
horses dogs
cats There are no species differences in elimination half-life.
37. Which drug inhibits the lipoxygenase pathway as well as the cyclooxygenase pathway?
a. b. c. d. e.
acetaminophen aspirin flunixin
naproxen
d. It is a synthetic compound not present in normal animals. e. It requires calcium to exert anticoagulant activity. 40. A dog has difficulty breathing secondary to pulmonary thromboembolism. Which treatment would have the most rapid effect in preventing furrherdeposition offibrin in the clot?
a. b. c. d. e.
intravenous heparin oral synthetic coumarin derivative intravenous protamine sulfate fresh whole blood transfusion plasma transfusion
41. Cardiac glycosides exert their cellular effects on the heart by:
3. blocking fast sodium channels b. inactivating troponin and tropomyosin c. inhibiting membrane vohage-dependent calcium channels d . inhibiting membrane sodium-potassium ATPase e. inhibiting membrane calcium ATPase 42. At therapeutic levels. cardiac glycosides produce all the following effects except:
ketoprofen
38. A dog has a recent history of spontaneous epistaxis, ecchymosis, and petechial hemorrhage secondary to warfarin i1ltoxication. Which drug is most appropriate for therapy?
a. b. c. d. e.
www.vet4arab.co.cc
SECTION 7
160
heparin vitamin K] (phytonadione) vitam in K2 (menaquinone) vitamin K3 (menadione) protamine sulfate
39. Concerning heparin, which statement is most accurate?
a. When given intravenously, it can directly break down an existing clot. b. It requires antithrombin III to have functional anticoagulant activity. c. It prevents synthesis of some clotting factors by the liver.
a. increased cardiac output at all levels of ventricular end-diastolic volume b. reflex withdrawal of sympathetic vasomotor lone (vasodilation) c. increased myocardial oxygen consumption d. slowed heart rate e. slowed atrioventricular conduction 43. How does digitalis therapy increase urine output in patients with heart failure?
a. direct blockade of sodium resorption in the proximal tubule b. competition for antidiuretic hormone receptors in the collecting duct c. stimulation of central thirst centers. with secondary polydipsia d. secondary to increased renal perfusion because of improved hemodynamics e. renal medullary washout
© 1998 Mosby-Year Book, Inc. Photocopying is prohibited by law.
Pharmacology
44. Digitalis therapy is beneficial in treatingarrial fibrillnrion because it:
a. decreases the refractory period in atrial muscle b. slows atrioventricular conduction c. converts the rhythm to normal sinus rhythm d. increases atrial muscle contractility e. blocks sympathetic input to the heart
161
49. Angiotensin-converting enzyme (ACE) inhibitors may cause renal failure in dogs wilh congestive heart/ailure through:
a. b. c. d. e.
afferent glomerular arteriolar constriction afferent glomerular arteriolar dilation efferent glomerular arteriolar constriction efferent glomerular arteriolar dilation constriction of both afferent and efferent glomerular arterioles
45. In which situation is digitalis contraindicated?
a. b. c. d. e.
congestive heart failure first- or second-degree atrioventricular block atrial fibrillat ion and atrial flurter supraventricular tachycardia concurrent use with diuretics
46. Which drug dilates blood vessels by competitive antagonism of(XI-adrenergic receptors?
a. b. c. d. e.
nitroglycerin nitroprusside sodium prazosin diltiazem captopril
a. b. c. d. e.
hypovolemia vomiting hypotension diuretic therapy oxygen therapy
51. Conceming enalnpril. which statement is most accurate?
47. The intravenous infusion rate/or nitroprusside is 5/Jog/kg/min. Using the 35 jJ,g/ml solution and a 15 drop/ml drip set, how many drops per minute should beadministered to a 22-lb dog?
a. b. c. d. e.
50. Which of the following does not increase the risk of renal failure associated with angiolensinconverting enzyme (ACE) inhibitor therapy?
12 drops/min 21 drops/min 47 drops/min 84 drops/min 116drops/min
a. Enalapril is the active form of the drug. b. It is associated with a Significant first-pass effect. c. It has a more rapid onset of action than captopri!. d . It has a longer duration of action than captopril. e. It is not approved by the U.S. Food and Drug Administration (FDA) for use in dogs. 52. Concerning the action ofverapamil on the heart. which statement is least accurate?
48. Blood volume is reduced after therapy with angiotensin-converting enzyme (ACE) inhibitors because of
a. inc reased aldosterone secretion b. decreased aldosterone and antidiuretic hormone secretion c. increased aldosterone and antidiuretic hormone secretion d. decreased antidiuretic hormone secretion e. decreased renal potassium reabsorption
a. It is a negative inotrope. b. It is a negative chronotrope. c. It speeds conduction through the atrioventricular node. d. It decreases the rate of sinoatrial discharge. e. It decreases coronary vascular resistance. 53. What is the predominant mechanism ofaction of class-1 antiarrhythmics?
a. b. c. d. e.
f3-adrenergic receptor blockade calcium channel blockade potassium channel blockade sodium channel blockade sodium-potassium ATPase blockade
Correct answers are on pages 178-184.
54. What is the predominant mechanism afaction of class-3 antiarrhythmics? a. h. c. d. e.
l3-adrenergic receptor blockade calcium channel blockade potassium channel blockade sodium channel blockade sodium-potassium ATPase blockade
55. What is the major electrophysiologic effect of quinidine on atrial and lIelUriwlar muscle that accounts for its antiarrhythmic properties?
a. h. c. d. e.
decreased phase 0 depolarization decreased phase I depolarization decreased phase 2 depolarization decreased phase 3 repoiarization increased spontaneous phase 4 depolarization
56. In which condition is use ofmannirol contraindicated? a. b. c. d. e.
www.vet4arab.co.cc
SECTION 7
162
acute oliguric renal failure intracranial hemorrhage acute glaucoma cerebral edema increased intracerebral pressure
57. By what mechanism does ammonium chloride cause diuresis? a. increased tubular ammonium load and increased osmotic force in the tubular lumen b. sodium channel blockade in the proximal tubule c. decreased chloride resorption in the loop of Henle d. ionic interference with aldosterone receptor binding in the distal tubule and collecting duct e. increased glucose secretion in the proximal tubule
58. By what mechanism do carbonic anhydrase inhibitors cause diuresis? a. increased potassium secretion in the collecting duct b. competitive inhibition of aldosterone receptors
c. competitive inhibition of antidiuretic hormone receptors d. inhibition of chloride transport in the thick ascending loop of Henle e. decreased sodium resorption in the distal tubule
59. Aside from diuresis, carbonic anhydrase inhibitors result in: a. hyperchloremic metabolic alkalosis and alkaline urine b. hyperchloremic metabolic acidosis and alkaline urine c. hypochloremic metabolic alkalosis and acidic Q urine d. hypochloremic metabolic acidosis and alkaline urine e. hyperchloremic metabolic alkalosis and acidic urine
60. What is the mechanism ofaction of thiazide diuretics? a. competitive inhibition of antidiuretic hormone receptors b. decreased sodium and chloride resorption in the distal tubule c. competitive inhibition of aldosterone receplOrs d. inhibition of chloride transport in the thick ascending loop of Henle e. increased potassium secretion in the collecting duct
61. What is the mechanism ofaction of spirorlOlactone? a. competitive inhibition of antidiuretic hormone receplOrs b. decreased sodium and chloride resorption in the distal tubule c. competitive inhibition of aldosterone receptors d. inhibition of chloride transport in the thick ascending loop of Henle e. increased potassium secretion in the collecting duct
© 1998 Mosby-Year Book. Inc. Photocopying is prohibited by law.
Pharmacology
62. What is the mechanism ofaction offurosemide? a. competitive inhibition of antidiuretic hormone receptors b. decreased bicarbonate resorption in the tubule c. competitive inhibition of aldosterone receptors d. inhibition of chloride transport in the thick ascending loop of Henle e. decreased potassium secretion in the collecting duct
63. Effects ofmetoclopramide include all the following except: a. b. c. d. e.
increased lower esophageal sphincter tone increased force of gastric contraction dopamine receptor antagonism increased colonic motility peripheral antiemetic action
64. Which antacid increases the risk ofgastric ruplllre? a. b. c. d. e.
aluminum hydroxide magnesium hydroxide magnesium sulfate sodium bicarbonate sodium sulfate
65. Concerning apomorphine, which statement is least accurate? a. Its emetic effects are countered by opiate antagonists. b. It is a synthetic derivative of morphine. c. It exerts its effects at the chemoreceptor trigger zone. d. It is a dopamine agonist. e. It produces emesis within 20 minutes of administration.
163
67. Concerning the effects of narcotic antitussive agents, which statement is least accurate? a. They cause constipation. b. They depress peripheral cough receptors in the bronchi. c. They are indicated for nonproductive cough . d. They are contraindicated when copious respiratory secretions are present. e. They can cause respiratory depression.
68. Sulfollamides are strucfUra/ analogs of a. b. c. d. e.
tetrahydrofolate reductase folic acid reductase para-aminobenzoic acid (PABA) pteridine glutamic acid
69. The 2,4-diamillopyrimidille compounds, trimerhoprim and ormetoprim, exert their antimicrobial effecrs through: a. competitive inhibition of dihydrofolate reductase b. decreased dihydropteridine uptake c. competitive inhibi('ion or para-aminobenzoic acid d. irreversible binding to transpeptidase e. irreversible binding to the 305 ribosomal subunit
70. Sulfonamides may produce any of the following side effects except: a. b. c. d. e.
hypoprothrombinemia keratoconjunctivitis sicca aplastic anemia and thrombocytopenia yellow staining of dental enamel hypothyroidism
71. What is the mechanism ofactio1l ofpenicillills? 66. Jmestinal effects ofanticholinergics include all the following except: a. b. c. d.
decreased intestinal secretions decreased segmental intestinal movements decreased propulsive intestinal movements antagonism of the purgative effects of neostigmine e. emesis
a. irreversible binding to cell wall porins b. irreversible binding to the 505 ribosomal subunit c. prevention of peptidoglycan synthesis by binding to transpeptidase d. interference with folic add synthesis e. competitive inhibition of dihydrofolate reductase
OJrrect answers are on pages 178-184.
164
SECTIO N 7
72. Concerning penicillin G, which statement is most accurate?
a. Its spectrum of activity is limited to gramnegative bacteria. b. It is resistant to 13-lactamases and is effective against Staphylococcus species. c. II is rapidly destroyed by gastric acid. d. It is combined with procaine to prevent pain at the injection site. e. It is a semisynthetic penicillin. 73. eoncem ingcepltaiosporins, which srateme1lt is least accurate?
a. They are bactericidal. b. They penet'rale gram- negat ive bacteria better than penicillins. c. They are more s usceptible to j3-lactamases produced by Staphylococcus species than are
penicillins. d. T hey are produced from Cephalosporium acremonium.
e. They interfere with peptidoglycan synthesis. 74. WI/ar is theamimicrouiaf mechanism a/action of tetracyclines?
a. binding to the bacterial 50S ribosomal subunit and inhibition ofpeptidyhransferase activity b. reversible binding 10 Ihe baClerial 50S ribosomal subunit and inhibition of ribosomal translocation c. binding to the bacterial 305 subunit. resulting in interference with aminoacyl tRNA binding to the acceptor (Al si le d. irreversi ble binding to the bacterial30S and possibly 50S subunit. resulting in abnormal codon:anticodon recognition e. inhibition of DNA synthesis by disruption of DNA gyrase activity 75. Possible toxic effects of tetracycline include all tile following except:
a. tympanitis or otitis media b. staini ng of dental enamel c. photo(Qxicity and photosensitization
www.vet4arab.co.cc
d. acute collapse following intravenolls injection e. pancytopenia 76. What is llJeantimicrobial mechanism ofaction of chloramphenicol?
a. binding 10 the bacte rial 50S ribosomal subunit and inhibition of peptidyltransferase activity b. reversible binding to the bacterial 50S ribosomal subunit and inhibition of ribosomal translocation c. binding to the bacterial30S subu nit, resulting in interference with aminoacyl tRNA binding to the acceptor (A) site d. irreversible binding to the bacterial 305 and possibly 50S subunit, resulting in abnormal codon:anticodon recognition e. inhibhion of DNA synth esis by disruption of DNA gyrase activity 77. All the following are associated with chloramphenicol toxicity except:
a. reversible dose·dependent bone marrow suppression b. reduced gonadal function in males and females c. dose-dependent leukopenia in neonates d. irreversible dose- independent bone marrow suppression e. cholestasis and arthropathy
Pharmacology
79. Con cerning metabolism and disposition of erythromycin, which statement is least accurate?
a. Oral erythromycin base must be enteric coated to prevent acid-induced inactivation. b. It is well distributed to the brain and cerebrospinal fluid. c. It is primarily excreted in the bile as an inactive metabolite. d. It must be administered 3 to 4 times daily because of its shon half-life. e. It causes severe diarrhea in horses. 80. Which drug should not be placed in an automatic, powered syringe for injection because of the risk of cardiovascular roxicity from accidental self-injection?
a. tylosin b. e rythromycin c. lincomycin d. tilmicosin e. clindamycin
Clostridium overgrowth in people, horses. and rabbits? tylosin
b. erythromycin c. lincomycin d. tilmicosin
e. clindamycin 78. Mlat is the Q/ltimicrohialmechallism of action of
erythromycin?
a. binding to the bacterial 50S ribosomal subunit and inhibition of peptidyltransferase activity b. reversible binding to the bacterial 50S ribosomal subunil and inhibition of ribosomal translocation c. binding to the bacterial30S subunit. resulting in interference with aminoacyl tRNA binding to the acceptor (A) site d . irreversible binding to the bacterial30S and possibly 50s s ubunit. resulting in abnormal codon:anticodon recognition e. inhibition of ON A synthesis by disruption of DNA gyrase activity
CI 1998 Mosby-Year Book. Inc. Photocopying is pro/libited by law.
83. Mlich of the following is a toxic effect of /luoroqllinolones?
a. degenerative chondrocyte changes in young animals b. staining of dental enamel c. cardiotoxicity and arrhythmia d. fatal d iarrhea e. pancytopenia 84. Concem ingaminoglycosideantibiotics, which statemeTll is most accurate?
a. They are highly lipid soluble. b. They easily cross biologic membranes. c. They are poorly absorbed following oral ad ministration. d. They are highly metabolized in the liver. e. They easily cross the blood-brain barrier. 85. Mlat is the antimicrobial mechanism of action of aminogiycosides?
81. Which drug causesfaral diarrhea secondary 10
..
165
82. WIJar is the antimicrobial mechanism ofaction of /luoroquinololle antibiotics?
a. binding to the bacterial 50S ribosomal subunit and inhibition ofpeptidyltransferase activity b. reversible binding to the bacterial 50S ribosomal subunit and inhibition of ribosomal translocation c. binding to the bacterial 305 subunit. resulting in interference with aminoacyl tRNA binding 10 the acceptor (A) s ite d. irreversible binding to the bacterial 305 and pOSSibly 50S subunit. resulting in abnormal codon:anticodon recognition e. inhibition of DNA synthesis by disruption of DNA gyrase activity
a. binding to the bacterial 50S ribosomal SUbWlit and inhibition of peptidyltransferase activity b. reversible binding to me bacterial 50S ribosomal subunit and inhibition of ribosomal translocation c. binding to the bacterial 30S subunit. resulting in interference with aminoacyl tRNA binding to the acceptor (A) site d. irreversible binding to the bacterial 305 and possibly 50S subunit, resulting in abnormal codon:anticodon recognition e. inhibition of ON A synthesis by disruption of DNA gyrase activity 86. COflcemillgaminogiycoside-induced neuromuscular blockade, which statemem is least accurate?
a. It results from inhibition of prejunctional ace£y1choline release. b. Paralysis of the diaphragm has been associated with intrapleural injections. c. Of all aminoglycosides, neomycin is most likely to cause neuromuscular blockade. d. Calcium infusion can reverse neuromuscular blockade. e. Aminoglycosides competitively inhibit nicotinic receptors.
UJrrect answers are on pages 178-184.
www.vet4arab.co.cc
SECTION 7
166 87. W1llch aminoglycoside is most nephrotoxic?
92. Cyclic amines, such as amantadine and
rimantadine, exert their antiviral activity by:
a. gentamicin
a. b. c. d. e.
b. kanamycin
c. neomycin d. s treptomycin e. amikacin 88. Concerning vancomycin, which
semement is most
blockade of viral fusion and release incorporation into growing mHNA inhibition of RNA polymerase inhibition of viral e nzymes inhibition of DNA polymerase in herpes viruses
accurate? a. It is bacteriostatic. b. Its use should be limited to penicillinresistant Staphylococcus infections. c. Only oral formulations are used clinically. d. It is hepatotoxic. e. It is eliminated through biliary excretion.
89. Concerning rilampin, which statement is most
93. Amiviml 'Iucleoside analogs are usually in the form of prod rugs and must be activated by: a. incorporalion iOlo growing polypeptide chains b. phosphorylation c. glycosylation d. incorporation into viral DNA e. protease inhibitors
accurate?
a. It is highly lipid soluble. h. It is degraded in an acidic environment.
c. It does not penetrate neutrophils or macrophages. d. Its spectrum of activity is limited to gramnegative bacteria. e. It should not be used in horses. 90. Concern ing melTonidaz<)le, which statement is kast accurate?
a. It is effective against anaerobic bacteria. b. It has an extended antimicrobial spectnun against protozoa, trichomonads, and amoebae. c. It does not penetrate the central nervous system. d. It is potentially mutagenic and teratogenic. e. It is clinically useful in hepatic encephalopathy. 91. Which class ofantiviral compounds prevents
attachment ofviruses to "receptors" on tile host cell?
a. b. c. d.
nucleoside analogs interferons cyclic amines synthetic pep tides e. interleukins
94. eoncerning amphotericin B, which statement is
most accurate?
a. Alopecia is the most serious common adverse effect and is self-limiting. b. Nephrotoxicity is the most serious common adverse effect and is predictable and dose related. c. Nephrotoxicity is the most serious common adverse effect, is not reversible, and is usually fatal. d. Hepatocellular toxicity is the most serious common adverse effect and is more severe in dogs than in cats. e. Hepatocellular toxicity is the most serious common adverse effect and is usually fatal.
It cannot be administered orally. It must undergo hepatic activation. It is a severe vesicant. It is cardiotoxic. It is cell cycle phase specific.
99. \oWlic/1 drug has been associated with direct mast-cell degra"uitltion, independent of IgE?
a. Ketoconazole is also used to treat hyperadrenocorticism. h. Altered steroid biosynthesis is considered a serious adverse effect of miconazole. c. Ketoconazole causes increased testosterone production . d. Potassium iodide causes hypothyroidism. e. ltraconazole causes diuresis.
C 1998 Mosby-Year Book, Inc. Photocopying is prohibited by law.
a. b. c. d. e.
97. WlII·cll drug is particu/arlyeffective in treating cllro"icgranuiocytic leukemia?
a. mechlorethamine b. cyclophosphamide c. melphalan d. busulfan e. carmustine
cytosine arabinoside taxol vincristine bleomycin doxorubicin
100. Which drug has acute cardiac arrest and cardiomyopatllyas toxic effects?
a. etoposide b. doxOfubicin
c. cisplatin d. l- asparaginase e. tamoxifen
98. Which drug is characterized bya high degree of lipid solubilitya"d is used to treat brain and meningeal tumors?
a. chlorambucil b. busulfan
l.E. Davis 101. Large doses ofcorticosteroids may produce:
a. b. c. d. e.
nephritis hepalOpathy encephalomalacia neutropenia amaurosis
a. unaffected by variation in renal blood flow b. generally more rapid if the dmg is not extensively protein bound c. more rapid with dmgs that are actively secreted by tubular cells d. always increased by administration of diuretics e. a minor pathway for elimination of dmgs 103. A toxic manifestation ofdibucai"e toxicity is:
a. agranulocytosis b. coma c. seizures
d. urticaria e. hepatic lipidosis 104. Prochlorperazine is least effective in prevention ofuomitingassociated with:
102. Excretion of drugs by the kidneys is: 95. eoncerningantifungals and their effects on steroid biosymilesis. which statement is most accurate?
c. melphalan d. thiote pa e. carmustine
96. Concerningcyc/opJlOsphamide, which statement is most accurate?
a. b. c. d. e.
167
Pharmacology
a. b. c. d. e.
vestibular stimulation uremia gastroenteritis stimulation of the chemoreceptor trigger zone radiation sickness
105. Urinary pH might be intentionally altered as part ofa therapeutic regimen to accomplish all the following except to:
a. prevent formation of some calculi b. decrease irritant properties of urine c. increase the effectiveness of certain chemotherapeutic drugs d. modify the pH of extraceliuJar fluid e. alter the rate of urinary excretion of another drug
Correct answers are on pages 178- 184.
168
www.vet4arab.co.cc
SECTION 7
106. Morphine sulfate:
a. h. c. d. e.
stimulates the respiratory center dilates the bronchioles depresses the spinal cord stimulates intestinal smooth muscle dilates the pupil in dogs
107. You wish to dilute strong tincture of iodine (7%) with 70% ethanol to prepare 4 oz a/tincture of iodine USP (2%) for use on small animals. How much ethanol and 7% iodine must be used? a. h. c. d. e.
3 fl oz of ethanol a nd 1 fl oz of 7% iodine 50 ml of e thanol and 45 ml of 7% iodine 160 ml of ethanol and 80 m1 of7% iodine 86 ml of ethanol and 34 ml of 7% iodine 45 ml of ethanol and 55 mI of 7% iodine
108. Abnormal elimination behavior in a dog, caused by separation anxiety, may be effectively treated with:
a. estradiol b. amphetamine c. phenobarbital
c. an emetic response d. suppression of conditioned avoidance behavior e. hypertension when given in large doses 112. Long-term cllronic administration ofbarbiturates to an epileptic dog may reduce the respome to otller anticonvulsant drugs because of
a. tolerance caused by decreased receptor sensitivity b. increased rate of excretion c. accelerated rate of biotransformation d. increased volume of distribution e. impaired intestinal absorption 113. What is tile effect of propantlieline on tile gut?
a. b. c. d. e.
spasmogenic antimuscarinic and ganglioplegic stimula tes motility increases secretion by intestinal glands evacuates the colo n
114. An anticoagulant chat is suitable for systemic administration is:
d. primidone e. amitriptyline J09. VitaminKis:
a. b. c. d.
synthesized in the small intestine found in significant quantity in cereal grains absorbed from th e gut in the absence of bile essential for thrombin formation in blood vessels e. found in alfalfa as phytonadione 110. Which dog breed is most likely to exhibit an idiosyncratic reaction to ivermectin? a. great Dane b. golden retriever c. dalmatian d. cocker spaniel e. collie
a. b. c. d. e.
calcium EDTA dicumarol sodium fluoride thromboplastin sodium oxalate
115. Norepinephrine and phenylephrine decrease tile
heart rate by:
a. directly depressing the cardioaccelerator center in the medulla b. decreasing peri pheral blood flow c. activating a baroreceptor reflex d. decreasing bronchial muscle tone e. exerting a direct negative inotropic effect 116. Which eicosanoid induces aggregation of placelets in tllecirwlation?
111. Tranquilizers produce:
a. anesthesia when given in large doses b. decreased seizure activity
a. b. c. d. e.
prostacyclin prostaglandin ~ leukotriene C4 thromboxane A2 prostaglandin H2
© 1998 Mosby-Year Book, Inc. Photocopying is prohibited by law.
117. Burow's solution is employed in wet dresSings because of its:
a. b. c. d. e.
emollient effect depilatory action demulcent effect astringent action effect on pigmentation
increased gluconeogenesis depressed protein catabolism eosinopenia monocytopenia neutrophilia
cascara aloe castor oil liquid petrolatum croton oil
acepromazine methylphenidate phenytoin mephenesin valpro ic acid
a. liver b. skin c. kidney d. gut e. bo ne 125. Heartworm disease can he prevented in dogs living in enzootic areas by administering, at monthly intervals, the larvicidal drug:
120. Tile renal toxicity associated witl1 long-term sulfollamide therapy is:
a. related to water solubility at a particular urinary pH b. diminished by using a single sulfonamide c. independent of urinary volume d. related to neural function e. least common with sulfathiazole 121. Aspirin:
a. b. c. d. e.
a. b. c. d. e.
124. 1,25-DihydroxycI10Iecalciferol, the actilleform of viUlmin 0, is synthesized in the:
119. Which catl1artic may be given safely toa lactating sow nursing a litter of piglets?
a. b. c. d. e.
d. can cause renal necrosis e. is expensive to use 123. Overactivityand intolerance to restrairll (hyperkinesis) in a dog are appropriately created with:
118. Which of tile following is not produced by glucocorticoid administration?
a. b. c. d. e.
169
Pharmacology
has no antiinflammatory effects produces hypothermia with overdosage is eliminated more rapidly by dogs than by pigs is effective in reducing visceral pain inhibits cyclooxygenase
122. Although halothane is a widely used inhalant anesthetic, a disadvantage is that it:
a. can depress the medulla b. has powerful hypotensive effects c. is a respiratory depressant
a. dithiazanine b. ivermectin c. thiacetarsamide d . diethylcarbamazine e. levamisole 126. A drug tllat has affinity for and inceracts with a cell receptor 10 elicit a response is called:
a. b. c. d. e.
an agonist a competitive antagonist an incomplete antagonist a potentiating agent an enzyme inducer
127. A cationic detergent is one in which the hydrophobic portion is positively charged. An example is:
a. b. c. d. e.
sodium orthophenylphenate tincture of green soap benzalkonium chloride sodium hypochlorite sapinated cresol
OJrrect answers are on pages 178-184.
170
www.vet4arab.co.cc
SECTIO N 7
128. Atropine exerts its main therapeutic effect at
receptors in the:
d. incompatibility e. drug interaction
a. central nervous system h. neuromuscular junction c. autonomic ganglia d . parasympathetic effector organs
e. postganglionic parasympathetic nerves
129. To dispense or prescribe controlled substances i" tlte U"itedStates, a veterinarian must be
licensed to practice and be registered with tile Drug Enforcemem Administration (DEA), wi/jell is a division of the: a. U.S. Department ofJustice b. U.S. Food and Drug Administration
c.
U.s. Department of the Treasury
d . Customs Department e. U.S. Department of Agriculture /30. Eicosanoid,s are involved in all the following
pl1ysiologic /llnctiollS except: a. h. c. d. e.
regulation of heat production regression of the corpus lu teum blood dotting renal failure hepatic failure
131. Whitfield's ointment is used as a: a. b. c. d. e.
protective sunscreen keratolytic depilatory disinfectant
relaxation of the pyloric sphincter, and increased tone ofrhe esophageal sphincter is: a. b. c. d. e.
metodopramide a tropine morphine epinephri ne haloperidol
134. Furosemide indl~ces diuresis by: a. inhibiting carbonic anhydrase b. antagonizing aldosterone c. inhibiting resorption of chloride in the loop of Henle d. decreasing permeability of the renal collecting ducts e. inhibiting sod ium resorption in the proximal tubule J 35. Bone marrow suppression produced by
methotrexate can be ameliorated by administration of a. b. c. d. e.
stanozolol daunorubicin leucovorin prednisolone cobaltous chloride
137. Which dassof drugs lowers the seizure threshold and may induce extrapyramidal motor effects? a. b. c. d. e.
phenothiazines benzodiazepines monoamine oxidase inhibitors butyrophenones tricydics
138. Ifa drug has a b iologic half-life of 17 hours in cattle, approximately how long should the drug be withheld before sillughter, so that the meat does not contain drug residue? a. b. c. d. e.
17 hours 8.5 hours 34 hours 7 days 85 hours
139. Which cholinergic alkaloid is commonly used topicaJJy to produce miosis? a. arecoline b. muscarine c. pilocarpine d. methacholine e. neostigmine 140. In which species does salicylate exhibit the shorlest elimination half-life? a. cattle
136. A horse that has been treated with chloral hydrate
may subsequently show glucosuria because: a. chloral hydrate stimulates release of epinephrine from the adren al gland b. chlo ral hydrate decreases glucose u tilization by the tissues c. the metabolite urochloralic acid is a reducing substance d. chloral hydrate decreases the renal threshold for glucose e. the renal tubules are damaged by chloral hydrate
© 1998 Mosby-Year Book, Inc. PllOtocopying is prohibited by law.
c. dogs d. cats e. people
141. The best route for administration ofa drug that forms a very alkaline solution is: a. b. c. d. e.
intravenous subcutaneous intramuscular intraperitoneal intrathecal
142. Use of which antimicrobial drug ill foodproducing animals is specifically forbidden by U.S. Food and Drug Administration regulations? a. b. c. d. e.
oxytetracycline ampicillin chloramphenicol sulfamethazine tylosin
143. Rapid induction ofanesthesia by administration ofan inltalalll anesthetic is primarily caused by the low: a. potency of the gas b. partial pressure of the gas in the central nervous system c . solubility of the gas in blood d. rate of metabolism of the gas e. respiratory rate 144. Phenylpropanolamine is often effective in control of urinary incontinence because it: a . stimulates release of vasopressin by th e pituitary gland b. inhibits contraction oCthe detrusor muscle by blocking cholinergiC receptors c. increases tone of the internal urethral sphincter by stimulating a-receptors d. decreases renal blood flow e. blocks serotonin receptors in the hypothalamus 145. A drug used to treat atrial fibrillation is:
b. rabbits
132. The phenomenon caused by long-term administration of certain drugs and characterized by hepatomegaly, increased prolein content of the liver, and increased microsomal enzyme activity is known as: a. induction b. inhibition c. enhancement
133. A drug that stimulates gastric emptying,
171
Pharmacology
a. b. c. d. e.
aminophylline lidocaine nitroglycerin epinephrin e procainamide
146. Most drugs are absorbed into the circulation by: a. b. c. d. e.
facilitated d iffusion pinocytosis simple diffusion active transport osmosis
Correct answers are on pages 178-184.
172
www.vet4arab.co.cc
SECTION 7
147. Which condition is an absolute contraindication to use ofcorticosteroids?
a. b. c. d. e.
149. \Nhich preparation has the longest duration of action?
a. b. c. d. e.
ketos is ulcerative keratitis allergic dermatitis arthritis lupus erythematosus
148. Dose-related cardiac damage is an important toxic effect oJ lite chemotherapeutic agent:
a. b. c. d. e.
posterior pi tuitary injection USP vasopressin injection USP vasopressin tannate injection USP arginine vasopressin lysine vasopressin
150. What is II complication of insulin therapy in diabetic patients?
a. b. c. d. e.
mithramycin doxoru bicin 5-fluorouradl chlorambucil cytosine arabinoside
hypokalemia metabolic acid osis dehydration hyponatremia acute renal shutdown
S. H. Duran 151. All the following are examples of extra label use ofa veteriflary drug except: a. drug used exac!ly as indicated on the label h. d rug used for a species other than those indicated on the label c. route of administration different than that indicate d on the label d. disease treated not indicated on the label e. dosage interval d ifferent from that ind icated on the label 152. A mi/liequivalem (mEq) is calculated by dividing the milligram (mgJ molecular weight by the valence, whicli gives mg/mEq. Ifpomssium chloride (KC/) has a molecular weightof74.5, how many milliequivalents are in 1 gofKCn
a. 74.5 mEq b. 7.45 mEq c. 13.4 mEq d. 20 mEq e. 134 mEq
156. Gentamicin and amikacin are antibiotics with a na"ow therapeutic index, meaning the dosage must be carefully controlled to prevent toxicity. What is the safest way to monitor the response to these antibiotics?
a. Measure levels in serum to make sure they are within the safe range. b. Give very low dosages. c. Use these drugs for no more than I day. d . Use these drugs for 3 days, withh old the drugs for another 3 days, and then resume treatment for 3 more days. e . Keep the animal well nourished while the drugs are be ing given.
a.4ml b.2ml c. I m l rl.20m] e. 5 ml 154. You plan to dispense an external parasiticide to a eiient.ln wha.t type olcontainer should the insecticide be dispensed?
a. b. c. d. e.
the original container, with a safety cap a clear glass bottle, ap propriately labeled an amber glass bottle, ap p ropriately labeled a plastic I-gallon cOnlainer a plastic container with a wide mouth
155. What is the osmolarity ofan isotonic solution?
a. b. c. d. e.
750 mOsm/L 300 mOsm/L 1000 mOsm/ L 200 mOsm/L 150 mOsm/L
© 1998 Mosby-Year Book, Inc. Photocopying is prohibited by law.
c. thiamylal sodium d. guaifenesin e. delOmidine 161. If you wish to prepare 1 L ofa 5% guaifenesin solution in 5% dextrose, how much ofeach ingredient will the solution c01ltain?
a. b. c. d. e.
100 g of dextrose and 100 g ofguaifenesin 5 g of dextrose and 5 g of guaifenesin 50 g of dextrose and 50 g of guaifenes in IO g of dextrose and IO g of guaifenesin 0.5 g of dextrose and 0.5 g of guaifenesin
162. Cimetidineand ranitidineare: 157. A dog has a serum creatinine level of3 mgldl. You p lan to begin ampicillin therapy using a dosage of25 mglkg body weight. How often should you give the drug?
a. every 4 to 6 hours
153. A vinl ofsodium penicillin G powdercontairu 5 million units. Ifyou add 18 ml ofsterile water to the vin/, the concentration will be 250,000 UllitsJmL What volume ofpowder was originally in the vial?
173
Pharmacology
h. every 6 to 8 h ours
c. every 8 to IO hours d. every 1 to 2 hours e. every 12 to 18 hours
sterile, distilled distilled, bacteriostatic sterile, pyrogen free autoc1aved, distilled bacteriostatic, irrad iated
calcium chloride potassiu m chloride sodium chloride sodium bicarbonate potassium acetate
a. b. c. d. e.
infections with gram-positive bacteria intestinal malabsorption squamous-cell carcinoma chronic obstructive pulmonary d isease diabetes insipidus
165. Whole blood, plasma, and blood substitutes should:
160. An animal has an allergic reaction to xyiazine. What other drug should not be administered to this animal?
a. ketamine b. acepromazine
a. with calcium products b. very slowly intravenously c. by any convenient route d. with blood products e. very rapidly as an intravenous bolus 164. Aminophylline, prednisone, and terbutalineare used in combination to treat:
159. Lactated Ringer's injection USP is not compatible with:
a. b. c. d. e.
c. antibiotics used against gram-positive bacteria d. H2 antagonists used for gastrointestinal ulcers e. injectable laxatives 163. When injected, oxytetracycline should be given:
158. What key words should you look for when purchasingfluids for intravenous injection?
a. b. c. d. e.
a. an tacids used for reflux esophagitis
b. anthelmin tics used for hookworm infections
a. be kept at room temperature for 24 hours before use b. be given in the same intravenous line as other medications c. not be used to dilute any drugs d. be mixed with calcium products before use e. be given with dextrose solutions
Correct answers are on pages J 78-184.
174
www.vet4arab.co.cc
SECTION 7
166. Afreratimillistralioll ofheracillin porassiwn, the drug is meUlbofized and appears in tissue and blood as:
a. b. c. d. e.
ampicillin and amoxicillin hetacillin and carbenicillin ampicillin and hetacillin amoxicillin and penicillin penicillin and carbenicillin
167. Strong tinclIIreo/iodine USP is comprised
a. b. c. d.
c. It is permitted if they have been employed by the hospital for al least 3 years. d. It is permiued if lhey are blood relatives. e. It is nOI pe rmitted under any circumstances.
175. Therapeutic drug monitoring involves:
and cats bill is flat absorbed?
ot
2% iod ine in alcohol 7% iodine in water 7% iodine in alcohol e. 15.6% iodine in water
168. O/tlle/allowing, with which drug can epinephrine be giuell safely? a. dextrose 5% injection USP b. sodium bicarbonate c. warfarin sod ium d. ascorbic acid
e. hetaSlarch
169. Records Oil use of COli trolled substances in a
a. b. c. d. e.
cimetidine ranitidine neomycin sucralfate omeprazo[e
172. A ciiem calls and says she left tire container of
4iethylcarbamazine citrate oral liquid inlrercar for 3 weeks during the summer. Concerning the patency aft/!edrugafter such storage, which statement is most accurate?
a. It has probably lost potency. b. It has probably gained potency. c. It will regain any losl potency if placed in the refrigerator. d. It \villiose any excessive potency if placed in the refrigerator. e. It is probably unaffected and can be used \vithout concern.
veterinary practice should be maintained by: a. a veterinary technician with a license in that Slate
b. a clerk with experience in inventory control c. a hospital administrator with a key to the controlled substances cabinet d. an office manager e. a veterinarian registered with the Drug Enforcement Administration 170. COllcernillga ucterinariall'sdispemingof
medication (0 1I0spifai staff members for use in treating tlwtnselves, wllicll statement is most accurate?
a. h is permitted if they present a prescription written by a physician. b. It is penniued if the drugs are used on the premises.
J.E. 176. All the fol/owing physiologic processes result in
decreasing plasma concentrations ofdrug except:
phenylbutazone?
pUlmonary edema crystals in the urine gastric ulcers cardiac decompensation dryness of the mouth
Riviere 180. A IO-kgdog is gillen an antibiotic at a dosage of
5 mglkg. If the drug's half-life is 30 minutes, how long will it takefor the animal to have less thm!
a. absorption
I mgofdrug remaining in its body
b. distribution c. metabolism d. elimination e. tissue sequestration
a. b. c. d. e.
177. Which factor is most important in determining the extent and rate ofdrugdisrributian?
a. route of elimination b. route of administration c. drug's physiochemical properties d. size of the dose administered e. dosage fo rm used
173. Mlat is the main adverse effectof
a. b. c. d. e.
175
c. measurement of serum or plasma levels of a drug to maintain levels in the optimal range d. close visual observation of animals in the first 8 hours after a drug is administered e. periodically obtaining tissue or exudate specimens for culture during a course of drug therapy
a. periodic physical examination of animals that have been treated with drugs b. testing of animals for possible anaphylactic reaction before treatment with a drug
171. Whic" drug is used (a treat gastric ulcers in dogs
2% iod ine in water
@
Pharmacology
178. Which of the fallowing sites has the lowest ratio
ofblood flow to tissue mass
30 minutes 60 minutes 90 minutes 120 minutes 180 minutes
181. All animal is brought to yourciinic in renal failure. It is severely dehydrated, and its glomerular filtration rate is only one quarter of nonna/. 71!e animal reqllires antibiotic trr!O.tment. l1!e antibiotic)'Ou select is cleared solely by renal glomerular filtration; its ooiume ofdistribution is limited to extracellular fluid such that its oolwne ofdistribution is halfofnormal. If the drng's half life in a normal animal is 60 minutes, what would its half-life be in this animal?
a. 30 minutes b. 60 minutes c. 120 minutes d. 240 minutes e. 360 minutes
a. brain b. muscle c. skin d. bone e. kidney
174. A dehydrated animal is best treated by
intravenalls infusion of
a. b. c. d. e.
dext rose 15% inj ection USP dextrose 10% injection USP dextrose 20% injection USP dextrose 50% injection USP acetated Ringer's injection USP
1998 Mosby-Year Book, Inc. Photocopying is prollibiled by law.
179. The rate afelimination of drugs with a high
hepatic extraction ratio is limited by:
a. b. c. d. e.
rate of hepatic blood flow rate of bile flow liver size extent of serum protein binding glomerular filtration rate
182. If a drug's volume ofdistribution is doubled in a diseased animal, which a/tire following would flat occur?
a. b. c. d. e.
Half-life would double. Serum concentrations would be halved. Clearance would double. Elimination rate constant would be halved. Plasma protein binding would double.
Correct answers are on pages 178-184.
www.vet4arab.co.cc
SECTION 7
176
183. All the followingfactors would double the free (non-protein bound) effective plasma concentration ofa drug except:
a. b. c. d. e.
doubling the dose halving the rate of hepatic metabolism halving the volume of distribution halving the extent of protein binding doubling the extent of protein binding
184. Ifol/ edoes not want a drug to accumulate in the body, the:
a. dosage interval should be less than or equal to the half-life b. dosage interval should be much longer than the half-life . c. dose given should be as small as possible d. dosage should be divided into smaller units and administered more often e. dosage form should be of the sustainedrelease type 185. Whjcl, factor influences the magnitude of the steady-state concentration achieved after drug administration?
a. b. c. d. e.
dose route of administration dosage form (capsule, tablet, injection, e tc.) drug potency route of elimination
186. What is the most important factor in determining how long it will take a drug to achieve steadystate serum concentrations in a multiple-dose regimen?
a. b. c. d. e.
size of the dose number of doses route of elimination route of administration half-life
187. If you double tile dose of drug given to afood
animal, the normal withdrawal time should be:
c. increased by 50% d. increased by one half-life e. unchanged 188. If a disease process doubles the half-life ofa drug in a food animal, the withdrawal time should be:
a. b. c. d. e.
doubled halved increased by 50% increased by one half-life unchanged
189. From tile sfandpoinr of increased antibacterial efficacy, which dmg combination is least rational?
a. b. c. d. e.
ampicillin with kanamycin ampicillin \'lith gentamicin chloramphenicol with oleandomycin penicillin with streptomycin clindamycin with gentamicin
190. Which ampicillin dosage regimen would be optimalfor surgical prophylaxis?
a. one dose the night before surgery and one after skin closure b. a 5-day regimen beginning during surgery c. a single dose after skin closure d. one dose at anesthetic induction and one after skin closure e. Ampicillin is never indicated for surgical prophylaxis.
Pllarmacology
192. A lactating Holstein cow is presented to your clinic with afever, regenerative neutrophilia, and "harsh" lung sounds. Diagnostic tests suggest bacterial pneumonia. Culture ofa transtracheal wash yields Pasteurella sensitive to all the antimicrobials listed below. Which of these would be the drug ofchoice in this case?
a. gentamicin b. polymyxin B c. chloramphenicol d. cephalexin e. sulfadimethoxine
c. reduced minimum inhibitory concentration (MIC) for an organism after prolonged therapy d. development of bacterial resistance after administration e. persistent antimicrobial effect after drug concentrations have declined below the MIC for the bacteria being treated 197. Which antimicrobial is most nephrotoxic?
193. Which antimicrobial would be optimal for initial treatment ofa urinary tract infection in a cae? Assume that the bacteria involved are equally sensitive to all the drugs listed.
a. b. c. d. e.
177
a. b. c. d. e.
ampicillin erythromycin doxycycline gentamicin enrofloxacin
198. What is the bacterial site ofaction for penicillin antibiotics?
ampicillin doxycycline erythromycin enrofloxacin lincomycin
194. Which antimicrobial undergoes enterohepatic recycling?
a. b. c. c. e.
30S ribosomal subunit 50S ribosomal subunit DNA gyrase e nzyme dihydrofolate reductase enzyme transpeptidation enzymes
199. What is the most common adverse effect of penicillin antibiotics?
a. b. c. d.
carbenicillin oxytetracycline colistin cephalexin e. penicillin G /95. Which faeror is least important when selecting
a. b. c. d. e.
drug allergy kidney damage liver damage bone marrow suppression inner ear damage
an antimicrobial for clinical use? 191. After administering penicillin G for 5 days CO an anjmal with a Staphylococcus infection, the clinical condition has not improved. Which drug is now indicated?
a. b. c. d. e.
ampicillin amoxicillin cloxacillin phenoxymethyl penicillin pe nicillin V
a. doubled b. halved
a. b. c. d.
spectrum of antimicrobial activity safety index (toxic versus effective dose) potency presence of tissue residues after use (food a nimals) e. cost J96. What is the "postantibiotic effect" in
antimicrobiallherapy?
a. persistent tissue residues after administration b. reduced drug clearance with each successive dose
© 1998 Mosby.- Year Book, Illc. Photocopying is prohibited by law.
200. When selecting a generic drug to substitute for an original proprietary (brand name) drug that you have used for years, what is the most important factor to ensure that the generic drug will be as effective as the original drug?
a. dosage forms available in same sizes as the proprietary drug b. of comparable cost to ensure comparable quality c. same drug concentrations in various dosage forms to ensure equal potency d. dosage forms certified as bioequivalent to the proprietary drug e. manufactured under sanitary conditions
Correct answers are on pages 178-184.
178
SECTI ON 7
www.vet4arab.co.cc
Answers I. d UnJike a full ago nis t, a partial agonist
produces less than a maximal response, even when all the drug receptors are occupied. 2. a The lower the Ko• the higher a drug's affinity fo r a given receptor. 3. e Nonionized drugs morc easily cross the bloodbrain barrier; therefore this acidic drug will be morc nonionized at a more acidic pH and the dose must be reduced to avoid overdosage. 4. e The weak base, erythromycin, is more ionized and the refore ionically trapped in milk, which is more acidic than plasma. 5. e Five half·lives would pass after 7.5 hours, so peak drug levels would be halved five times. 6. c Body clearance is di recdy proportional to volume of distribution; therefore drug X has a smaller volume of distribution than drug Y. 7. d The glucuronide synthetic pathway of cats functions much more slowly than that of other species. 8. c The parasympathetic nervous system favors digestion. Therefore a drug that mimics it effects will increase gu t motility and decrease sphincter tone. 9. e Norepinephrine has the least affinity for 132adrenergic receptors. 10. d AI: 1000 solution is equivalent to I mg/ml. The refore 0.25 011 of th is solution will deliver 0.25 mg. 11. a The vasoconstricting effects of epinephrine. through its ("I-adrenergic action. prolong the duration of local anesthesia by preventing absorption of the drug. 12. b Neostigmine is an acetylcholine esterase inhibitor that allows accumulation of acetylcholine at the neuromuscular junction. This potentiates the action of succinylcholine, which is a depolarizing neuromuscular-blocking agent. 13. d Intravenous acetylcholine reduces blood pressure through muscarinic receptors on the vascular endot helium. Atropine is a muscarinic antagonist. 14. a In small doses, dopamine is specific for dopamine receptors. In larger doses. it binds to 0;- and p-adre nergic receptors. IS. c Fluorine metabolites impair the kidney's ability to concenlrate urine. 16. d Animals subjected to light planes of halothane anesthesia are more prone to spontaneous arrhythmias.
17. a The bond for fluorine is much stronger than for bromine or chlorine; thus little fluorine is released, as with methoxyflurane. IS. e Halothane has both direct and indirect effects on he patic and renal function. It also increases cere bral blood flow and intracranial pressure. 19. b Nitrous oxide can accumulate in any aircontaining space in the body. 20. c Propofo! decreases cerebral perfusion, resulting in decreased intracranial and intraocular pressures. It is a GABA agonist. lacks anticonvulsant properties, causes short periods of apnea. and is metabolized extrahepatically. 21. c Barbiturates bind to the barbiturate portion of the GAllA receptor complex, increasing chloride conductance in ne urons. 22. e The high lipid solubili ty ofthiobarbiturates results in rapid d istribution into the brain and redistribution to othe r tissues. Fat animals are quick to recover. whereas lean animals experience prolonged recovery. 23. b Tachycardia is a common finding of barbiturate anesthesia, as well as ventricular fibrillation when the animal is hypothermic. Barbiturates potentiate catecholamine- induced arrhythmias. Mean arterial pressure and cardiac output decrease. 24. c The therapeutic index of ketamine is wider in most species than that of barbiturates. It produces a cataleptic anesthetic state in which periphe ral reflexes are maintained. Ketamine is a poor visceral analgesic. 25 . e Ketamine is highly metabolized in the liver and excreted in the urine. Its use is contraindicated in animals with he patic or renal disease. 26. b Phenobarbital is metabolized by hepalic microsomal enzymes. It is a potent inducer of these enzymes; continued administration increases the rate of metabolism. Metabolites and unchanged drug are excreted in the urine. 27. a Primidone is metabolized to phenobarbital and phenylethylmalonamide in the liver. Because phenobarbital has the longer half-life, it tends to accumulate and blood levels are used as a guide to design dosage regimens. 2S. c Phenytoin , like some other anticonvulsants. is a potent inducer of microsomal enzymes. This drug has a very short half-life in dogs, and continued use further shortens its elimination half-life via enzyme induction.
@ 1998Mosby-YearBook,Inc.Photocopyingisprohibitedbylaw.
Pharmacology
29. b Benzodiazepines exert their anticonvulsant activity by enhancing the inhibitory effect of GAllA on excitatory neurons. 30. a As an anticonvulsant, bromide hyperpolarizes neurons by replacing intracellular chloride. Its elimination half-life is approximately 1 month; therefore, loading doses are recommended to achieve steady-state levels within I week after beginning therapy. 3 1. e Newer HI-specific antihistamines are less lipophilic and therefore poorly cross the blood brain barrier. 32. c Patients with mast-ceil tumors have high endogenous histamine levels and are prone to gastric hyperacidity and gastric ulcers. An appropriate presurgical drug is an Hz blocker, such as cimetidine. 33. d Omeprazole blocks the parietal cell pump in gastric oxyntiC glands. 34. a Glucocorticoids bind to cytosolic and nuclear glucocorticoid receptors. where they induce cellular Iipocortin synthesis. lipoconin interferes with phospholipase activation, reducing both leukotriene and prostaglandin synthesis. 35. a Aspirin is metabolized to salicylic acid, which in turn blocks the cyclooxygenase inflammatory pathway, leading to reduced prostaglandin syntheSis. 36. d Because cats cannot metabolize and excrete aspirin as well as other species, aspirin has the longest half-life in cats. 37. e Unlike other nonsteroidal antiinflammatory drugs, ketoprofen appears to inhibit both the lipoxygenase and cyclooxygenase pathways. 3S. b Vitamin K] is the treatment of choice in coumarin intoxication. 39. b For full anticoagulant activity, heparin requires antithrombin III. 40. a Heparin is the most rapid-acting anticoagulant. Coumarins have a lag pe riod because they exert their effect by inhibiting synthesis of endogenous clotting factors. 41. d Cardiac glycosides bind to and inhibit the sodium-potassium ATPase pump. 42. c Cardiac glycosides increase cardiac output. withdraw sympathetic tone, slow the hean rate. and reduce myocardial O2 consumption. 43. d Cardiac glycosides increase urine output in patients with heart failwe by improving renal perfusion secondary to improved hemodynamics. Diuresis is not caused by a direct effect on the kidney.
179
44 . b Cardiac glycosides do not convert atrial fibrillation to normal sinus rhythm. Rather, they slow the ventricular rate by decreasing atrioventric ular conduction. 45. b Because cardiac glycosides decrease atrioventricular conduction. they are contraindicated in first- or second-degree atrioventricular block. 46. c Prazosin is a prererential ai-adrenergic receptor antagonist. 47. b 22 lb -;- 2.2 Ib/kg = 10 kg. 10 kg X 5 JJ.g/kg/min = 50 JJ.g/min. 50 ....g/min x I ml/35 JJ.g = 1.43 m1/min. 1.43 m1/min x 15 drops/ml = 21 drops/min. 4S. b Blood volume decreases after therapy with ACE inhibitors because of natriuresis secondary to decreased aldosterone secretion. Natriuresis is followed by decreased ADH secretion and decreased renal water resorption. 49. d Angiotensin II causes constriction of efferent glomerular arterioles. ACE inhibitors prevent this constriction by decreasing angiotensin II levels. thereby reducing the glomerular filtration rate. 50. e An swers a though d tend to lower blood volume and result in prerenal azotemia. Volume contraction predisposes animals to ACE inhibitor- induced renal failure. 5 1. d Because enalapril has a slower onset of action but a longer duration of action than captopril, once-daily dosing is possible. 52. c Calcium channel blockers slow conduction through the atrioventricular node and are therefore indicated in treatment of supraventricular tachycardia. 53. d The predominant mechanism of action of all c1ass- l antiarrhythmics is blocking of sodium channels. 54. c The predominant mechanism of action of all class-3 antiarrhythmics is blocldng of potassium channels. 55. a Quinine is a c1ass-IA antiarrhythmic that blocks sodium channels. This blockade decreases the rate of phase-O depolarization in atrial and ventricular muscle. 56. b Mannitol, as do all other osmotic diuretics, acutely expands extracellular fluid volume. Such an expanded volume would exacerbate intracranial hemorrhage. 57. c Chloride dissociates from the ammonium chloride molecule in the blood, increasing the chloride load delivered to the kidney.
180
SECTION 7
58. e In part, sodium is absorbed in the renal tubule in exchange with protons. When protons are not available through inhibition of carbonic anhydrase, sodium is not resorbed. 59. b Chloride ions are co-secreted with protons in the coUecting duct. When protons are not available through inhibition of carbonic anhydrase, chloride ions accumulate in the blood and the urine becomes alkaline. 60. b Thiazide diuretics are secreted into the proximal tubule by an organic acid transport system, where they inhibit both sodium and chloride resorption in the distal tubule. 6 1. c Spironolactone is a competitive inhibitor of aldosterone receptors. 62. d Furosemide is actively secreted into the proximal tubule and inhibits chloride resorption in the thick ascending loop of Henle. 63. d Metoclopramide is not associated with altered colonic motility. 64. d Oral sodium bicarbonate results in liberation of CO 2 , which can result in gastric rupture, particularly at the site of gastric ulceration. 65. a Apomorphine is a pure dopamine receptor agonist, and its effects are not countered by opiate antagonists. 66. e Anticholinergic drugs, such as aminopentamide, prevent vomiting and decrease intestinal secretions and segmentaJ and propulsive intestinal movements. Neostigmine is a parasympathomimetic, and its effects are opposed by anticholinergic drugs. 67. b Narcotic cough suppressants work through depression oCthe medullary and pontine centers, where the cough reflex is controlled. 68. c Sulfonamides are structural analogs of PABA and prevent conversion of PABA to dihydrofolate. 69. a Trimethoprim and ormetoprim are competitive inhibitors of dihydrofolate reductase. Their combination with sulfonamides renders them bactericidal. 70. d Tetracyclines, not sulfonamides, arc associated with enamel staining. 71. c Penicillins are bactericidal because they disrupt bacterial cell wall syntllesis. 72. c Penicillin G cannot be given orally because of rapid degradation by gastric acid. 73. c CephaJosporins arc less susceptible to 13lactamases than are penicillins.
www.vet4arab.co.cc
74 . c Tetracyclines interfere with protein synthesis through binding to the bacterial 305 ribosomal subunit, resulting in interference with aminoacyl (RNA binding lO the acceptor (A) site. 75. e Pancytopenia has not been associated with tetracycline therapy. 76. a Chloramphenicol inhibits peptidyltransferase activity at the bacteriaJ 50S ribosomaJ subunit and inhibits elongation of the growing peptide chain. 77. e Cholestasis and arthropathy have not been associated with chloramphenicol toxicity. 78. b Erythromycin prevents translocation of amino acids to the growing peptide chain by reversibly binding to the bacterial 50S ribosomaJ subunit. 79. b Erythromycin is well distributed to most tissues, with the exception of the brain and spinal corel. 80. d Cardiovascular toxicity has been observed in primates, swine, and horses. 81. c Serious or fatal diarrhea has been associated with lincomycin use in horses, humans, and rabbits. 82. e Unlike other classes of antibiotics, fluoroquinolones work through interfering with bacterial DNA synthesis. 83. a Fluoroquinolone use is associated with arthropathy in young animals. 84. c Aminoglycosides are highly polar compounds and consequently are poorly absorbed orally and poorly cross biologic membranes. 85. d Aminoglycosides result in production of nonsense proteins through interference with reading the mRNA transcript. 86. e Aminoglycosides inhibit prejunctional acetylcholine release; paraJysis of the diaphragm has been associated \vith intrapleural injections. Neomycin is most likely to cause neuromuscular blockade; calcium infusion can reverse the neuromuscular blockade. Aminoglycosides have no effect on nicotinic receptors. 87. c Neomycin is the most nephrotoxic aminoglycoside and consequently is not recommended for parenteral use. 88. b Vancomycin is a complex peptide antibiotic. It is bactericidal, poorly absorbed orally, potentially nephrotoxic, and eliminated in the urine. Its use should he limited to penicilIinresistant Staphylococcus infections.
© 1998 Mosby-Year Book, Inc. Pilotocopying is pro/zibiced by law.
Pharmacology
89. a Rifampin is a highly lipid-soluble, acid-stable antibiotic that penetrates neutrophils and macrophages. Its spectrum of activity includes gram-positive bacteria and mycoplasmas. It has been used to treat gram-positive infections in horses. 90. c Metronidazole penetrates the central nervous system well and is indicated in treatment of anaerobic bacterial meningitis. 91. d Synthetic host-cell protein receptor anaJogs attach to the viral coat and prevent viral attachment to host-cell receptors. 92. a The antiviral activity of amantadine and rimantadine involves inhibition of late-stage assembly of virus. 93 . b These compounds must be converted to the triphosphate form to compete for incorporation into growing DNA strands. 94. b The major toxicity associated with amphotericin B therapy is dose-dependent nephrotoxicity. 95. a Ketoconazole has been successfully used in dogs to treat Cushing's disease caused by inhibition of corticosteroid synthesis and, consequently, cortisol+lowering properties. 96. b The nitrogen mustard, cyclophosphamide, is non-cell cycle specific and can be administered orally or intravenously. It must undergo hepatic activation. Its major toxicity is sterile hemorrhagiC cystitis. 97. d Busulfan is particularly effective for chronic granulocytiC leukemia but is not effective in acute leukemia. 98. e The high lipid solubility of carmustine aids in its transport into the central nervous system, making it effective for treating brain and meningeaJ tumors. 99. e In addition to cardiotoxicity, doxorubicin can cause acute tOxicity associated with mast-cell degranulation. 100. b The major toxicity of doxorubicin is acute cardiac toxicity or cardiomyopathy associated with cumulative doses. 101. b Corticosteroids cause hepatomegaly and elevated serum alkaline phosphatase activity. 102. c Such drugs have a large renal clearance limited primarily by renal blood flow. 103. c Local anesthetics may cause convulsions. The other answers are unrelated to dibucaine. 104. a Antihistaminics (e.g., diphenhydramine) are much more effective for prevention of motion sickness.
181
105. d Alteration of urinary pH has a negligible
effect on the pH of extracellular fluid. 106. d Morphine arrests peristalsis by its spasmodic action on smooth muscle in the gut 107. d 40z X 30 mlloz =120 ml total, comprised of 5 parts 70% ethanol and 2 parts 7% tincture. 108. e Amitriptyline appears to diminish apprehension and alter the related behavioral response. 109. e Alfalfa meal is an important dietary source of vitamin K. 1l0. e Some collies develop blindness, coma, and death in response to ivermectin. IlL d None of the other choices is accurate. 112. c Barbiturates cause induction of hepatic microsomal enzymes. This is an imporlant mechanism of drug interactions. 113. b Propantheline is a quaternary ammonium antimuscarinic compound. This confers appreciable activity at ganglionic and muscarinic sites. 114. b Oicumarol is a vitamin K antagonist within the liver. The other compounds listed are unsuitable for such usc. llS. c Stimulation of the carotid sinus by increased blood pressure slows the heart rate. 116. d Thromboxane is synthesized in platelets and is a powerful inducer of platelet aggregation. 117. d This solution of aluminum subacetate is used to treat acute, oozing skin lesions. 118. b Glucocorticoids increase protein catabolism. 119. d Pharmacologically active components of the other four compounds are excreted in milk and may cause diarrhea in the pigs. 120. a Some sulfonamides calise lower-nephron nephrosis by forming crystals in the renal wbules. 12 1. e The principal action of aspirin is to decrease synthesis of prostaglandins by inhibiting cyclooxygenase. 122. b Halothane depresses the myocardium and blunts the baroreceptor-mediated tachycardia in response to hypotension. 123. b Hyperkinesis is paradoxically improved by treatment with a central nervous system stimulant. 124. c Vitamin 0 3 is converted to 25hydroxycholecalciferol in the liver and is hydroxylated at the one-position by renaJ mitochondria.
184
SECTION 7
183. e Increasing the fraction of drug bound to
184.
185.
186.
187.
proteins would reduce the free concentration of drug. b A drug"accumulates in (he body unless the dosage interval is significantly longer than the half-life. a The dose directly determines the magnitude of the steady-slate concentration seen after drug admin istration. e The time it takes to reach the steady-state concenlration is solely a function of the drug's hatf-life. d Jf you double the dose, after one half-life the drug remaining in the animal is approximately the same as if the original dose were given . Thlls the withdrawal time need only be
increased by one half-life. 188. a In cont rast, iflhe half-life is doubled, it will now lake twice as long to eli minate the drug and
(he wilhdrawal time should now be doubled. 189. c Both these drugs have an identical mechanism of action. For optimal amibacterial efficacy. drugs used together should have diffe rent mechanisms of action. 190. d A prophylactic antibiotic should be administered according to this protocol so that maximum concentrations are present at the surgical site when organisms are introduced into tissues by surgical manipulation. The drug should not be present before or after this "window of opportunity. ~
www.vet4arab.co.cc
SECT ION
191. c Gram-positive staphylococci should be
susceptible to any lactam antimicrobial unless the bacteria produce lactamase. This is the only drug listed that is resistam to inactivation by lactamase. 192. e This sulfonamide has the optimal pharmacologic profile for treating this condition. The other compounds are too toxic, are not approved for use in food animals, or would produce persistent residues. 193. a This broad-spectrum antimicrobial is excreted by the kidney into the urine. 194. b This tetracycline is the onl y drug listed that is excreted into the bile, a prerequisite for enterohepatic recycling. All the others are excreted into the urine. 195. e Drug potency can be improved by form ulat ion and thus is not a limiting factor. The other factors are inherent properties of the drug and cannot be manipulated. Economics should always be a factor in selecting therapy. 196. e If a drug has a significant postantibiotic effect, therapeutic effects may persist even after drug concentrations fal l below minimum inhibitory concentrations. 197. d Gentamicin is an aminoglycoside antibiotic that can cause nephrotoxicity. 198. e Penicillins inhibit bacterial cell wall synthesis. 199. a Penicillins have minimal adverse effects, except for sensitization. 200. d By law, bioequivaJent drugs have the same therapeutic efficacy and thus can be used interchangeably.
NOTES
8 Principles of Surgery T.P. Colville
Recommended Reading Auer JA: Equine surgery, Philadelphia, 1992, WB Saunders. BOjrab MJ: Current techniques in small animal surgery, ed 4, Baltimore, 1998, Williams &Wilkins. Colahan PI el al: Equine medicine and surgery, ed 5, St. Louis, 1998, Mosby. Fossum TE: Small animal surgery, 51. Louis, 1997, Mosby. Jennings PB: The pracrice of large animal surgery, Philadelphia, 1984, we Saunders. Slatte r DH: Textbook of small animal mrgery, ed 2, Philadel phia, 1993, WB Saunders. Swaim SA, Henderson RA: Small animal wound managemellt, Philadelphia, 1990, Lea & r-ebiger.
..._ _-,p~,,~ctice answer sheet is on page 269.
Questions 1. Wllich surgical instrument is priman'ly used to hold orgam and tissues out ofrhe way to facilitate exposure of the operative field?
a. b. c. d. e.
elevator forceps retractor rongeur hemostat
2. Wllich orthopedic instrument has sharp, opposing. cup-sl/aped jaws used to shape bone by Mcliewing" out small pieces?
a. b. c. d. e.
e 1998 Mosby-Year Book, Inc. Photocopying is pro/libited by law.
chisel curette osteotome rongeur trephine
3. Wllich of the following is the correct surgical term for declawing?
a. b. c. d. e.
celiotomy cystotomy onychectomy hysterectomy colpotomy
4. Which type of needle is most appropriate for suturing intemal organs, such as gastrointestinal structures?
a. blunt b. cutting c. reverse cutting d. tapered
e. trocar
Correct answers are on pages 195- 198.
185
182
www.vet4arab.co.cc
SECTION 7
125. b Iverrnectin is approved in the United States for prevention ofhearrworm infections in dogs. 126. a Agonists interact wilh the ceU receptor to elicit a response. 127. c Benzalkonium chloride is the only cationic detergent in the group lis ted. 128. d Muscarinic siles are blocked by atropine. 129. a The OM is a division of the Department of Justice. 130. a Heat production is a resuh of metabolic activity, which is not affected by eicosanoids. 131. c Whitfield's ointment contains benzoic and salicylic acids. These compounds are fungistatic and keratolytic. 132. a Induction of microsomal enzymes increases the rate of biotransformation of the inducing agent and lo r other drugs. 133. a This combination of actions is unique to metoclopramide and prevents esophageal reflux of gastric fluid. 134. c This action results in increased excretion of sodium, chloride, and water. 135. c Leucovorin is a reduced, functional folate coenzyme. 136. c Urochloralic acid gives a false-positive result in tests for reducing sugars. 137. a These effects are associated most commonly \vith the phenothiazines but could also be seen with the butyrophcnoncs, because these effects are associated \vith dopaminergic receptors. 138. e Virtually all the drug would be eliminated from the body after five half-lives have elapsed. 139. c Pilocarpine is well absorbed from ophthalmic solUlions and ointments and causes contraction of the pupillary sphincter. 140. a The half-life for salicylate is only 30 minutes in cattle. 14 [. a Irritating drug solutions are infused slowly into a vein to allow for dilution. 142. c Use of chloramphenicol in these animals is prohibited because of the fear that residues in food products might cause aplastic anemia in susceptible people. 143. c Equilibration occurs more rapidly when the agent is not soluble in blood. 144. c Motor activity of the internal urethral sphincte r is mediated by sympathetic nerves. Receptors are of the a-adrenergic type. 145. e Procainamide is the only drug listed that is effective for treatment of s upraventricular arrhythmias.
146. c Most drugs are relatively small molecules that diffuse across membranes in response to concentration gradients. 147. b Corticosteroids inhibit healing, and this may lead to corneal perforation and loss of aqueous humor. 148. b Anthracycline antibiotics, such as doxorubicin, cause cumulative dose-related cardiomyopathy. 149. c Vasopressin tan.nate is more slowly absorbed than the other preparations listed. 150. a Insulin and glucose d rive potassium intracellularly, causing hypokalemia. 15 1. a Extra-label use involves use of an approved drug in a manner not consistent with the drug container label or package insert 152. c There are 13.4 milli equivalents (mEq) in I g (1000 mg) of potassium chloride. The equation is as follows:
mEq
=___---'m""' -_ __ Molccular weightlValence
153. b First divide the initial amount (5 million units) by the final amount (250,000 units/ ml). 5,000,000 + 250,000 = 20 m!. Then subtract the volume of diluent added (18 ml). 20 - 18 = 2 ml of powder originally in the vial. 154. a All inseClicides s hould be dispensed in the original container, with all EPA in formation on the label and in a safety-cap container or spray container. 155. b An isotonic solution has an osmolarity of300 mOsm/L, which is the same osmotic p ressure as for body fluid s. A solution with an osmolarity less th an 300 mOsm/L is hypotonic and could cause lysis of red blood ceUs. A solution with an osmolarity greater than 300 mOsm /L is hyperlonic. 156. a The safest way to monitor patients receiving aminoglycosides is to obtain serum samples and measure peak and trough levels to make sure drug levels are within the safe, therapeutic range. 157. e Ampicillin is elim inated primarily by the kidneys and normally administered every 6 hours for 7 to 10 days. If an animal has a serum creatinine value of3 mg/ d1 (normal, I to 2 mg/dl), this indicates reduced renal function, so the drug should be given less frequently because it is not being eliminated at a normal rate. A good rule of thumb is as rouows: Serum creatinine X Normal dosage interval = Adjusted dosage interval. 3 X 6 =18 hours.
C 1998 Mosby-Year Book, Inc. Photocopying is prollibited by law.
Pharmacology
158. c Only fluids that are sterile and pyrogen free should be given intravenously. 159. d Lactated Ringer's USP contains calcium chloride. Mixing with sodium bicarbonate forms an insoluble p recipitate, calcium carbonate. 160. e Detomidine and xylazine are chemically related. Animals that have an adverse reaction to xylazine may also react to detomidine. 161. c A 5% dextrose-5% guaifenesin solution contains 5 g of dextrose and 5 g of guaifenesin per 100 ml, which is equal to 50 g of each per 1000 ml. 162. d Cimetidine and ranitidine are H2 antagonists that block gastric acid secretion, which aids treatmen t of ulcers. Because the re are no H2 antagonists approved for veterinary use, these drugs can only be used in accordance with extralabellaws. 163. b Oxytetracycline chelates (binds) calcium. The vehicles used in some oxytetracycline products, such as propylene glycol, can cause anaphylaxis if these drugs are given by rapid intravenous injection. Because of their pH or vehicles, some products cannot be given intramuscularly. 164. d Aminophylline (a bronchodilator), prednisone (an antiinflammatory glucocorticoid), and terbutaline (a selective P2adrenergic agonis t) can be given in combination to treat chronic obstructive pulmonary disease. 16S. c Whole blood, plasma, and blood substitutes should not be used as diluents of any drugs, and they can be infused only by piggyback with sodium chloride injection 0.9% USP. 166. c Hetacillin potassium is ch emically related to ampici lli n. Some of the drug is metabolized to ampicillin, and some rem ains as hetacillin. 167. d Strong tincture of iodine USP is a 7% tincture, which has an alcohol base. It is commonly used to dip the umbilical cord of newborn calves. Mild tincture of iodine is a 2% solution and can be used to dip the umbilical cord of foals. 168. a Epinephrine is usually given by intravenous bolus injection. If it must be diluted, it should be added to dextrose 5% injection USP. 169. e A veterinarian registered with the DEA and prescribing controUed substances should keep accurate records. 170. e A veterinarian is not permitted to dispense any drugs to people. Severe penallies are associated with this offense.
183
171. d Sucralfate is an alkaline aluminum complex of sucrose sulfate that acts locally rather than systemically. It fonus a barrier at the ulcer site and protects the ulcer from further damage caused by pepsin, acid, or bile. 172. a Diethylcarbamazine citrate, used for heartworm prevention in dogs, decomposes when exposed to temperatures over 300 C (86F) and when exposed to light. Always dispense this drug in an amber bottle. 173. c The major toxicity associated with phenylbutazone is gastric ulcers. The dosage s hould be tapered to small amounts or the drug should be used for short periods only, because it accumulates. 174 . e Acctated Ringer's injection is the fluid of cho ice. Hypertonic dextrose solutions would act as an osmotic diuretic and further dehydrate the animal. 175. c Therapeutic drug monitoring involves measuring the amount of drug in the serum or plasma of the diseased animal to achieve the best therapeutic dosage, with the least toxicity. 176. a This is the only pharmacokinetic process listed that increases drug concentrations in the central companment. The others remove drug from this compartment and thus result in decreas ing plasma concentrations. 177. c The p rimary determinant of dis tribution is the drug's ability to cross biologic membranes. 178. d The bulk of parenchymal [issue of bone is calcified and not perfused, thereby restricting the distribution of drug. 179. a Clearance of drugs with a high hepatic extraction ratio is limited by the rate of hepatic blood flow. 180. e The initial amount of drug in the animal is 50 mg (5 mg/kg X 10 kg). After one half-life, 50% is eliminated, leaving 25 mg. After two halflives, 12.5 mg remains; after three half-lives, 6.25 mg remains; after four half-lives, 3.12 mg remain; after five half-lives, 1.6 mg remains; and after six half-lives. 0.8 mg remains. Six half-lives equal 180 minutes. 181. c Half-life is proportional to the clearance divided by the volume of distribution. If the clearance is one quarter of normal but the volume of distribution is doubled, the resulting half-life is twice normal, or 120 minutes. 182. c Clearance is independent of the volume of distribution.
www.vet4arab.co.cc
SECTION 8
186
5. Wlliell type of needle is mast commonly Ilsed for sltCllring in general surgery?
a. straight
10. Whic/I oftllefollowing best describes the location ofan incision extendingfrom the xiphoid process to tile umbiliCllS ofan animal?
a. b. c. d. e.
b. " circle c. ~ circle d ~ circle e. ~ curved
6. WI,;ell of the following is "ot considered an elective surgical procedure? a. correction of a an eye with proptosis b. dew claw removal in a field trial dog
c. mastectomy to remove a benign tumor d. ovariohysterectomy in a healthy 6-momh-old cat e. tail docldng in 3-day-old boxer puppies
7. Healing ofa properly sutured surgical wound is most appropriately termed:
n. first-intention healing
dorsal midline flank paracostal paramedian ventral midline
8. Wllieh of tile following lias tile poorest potential for healing and relllm (0 normal jimccioll after damage and effective surgical repair?
a. autoclave b. boiling c. dry heat d. ethylene oxide gas e. liquid chemical disinfectant
a. bone
microorganisms into an incision during a surgical procedure?
a. b. c. d.
contamination debridement dehiscence infection c. septicemia 17. Whidl of the following items does not have to be
driflto be Wied in an ort/lOpedic surgical procedure 12. Most appropriate for sterjfjzation of a needle holder to be used in a surgical procedure
13. Most appropriatefor sterilization of dissecting scissors to be Wied in a surgical procedure
c. silk
23. Wllich type ofsurgical gut is absorbed m ost rapidly from tissues after surgery?
18. Wilich o/the following is tile agem for
sterilization byautoclaving?
a. b. c. d. e.
chemical disinfectant solution dry heat ethylene oxide gas ionizing radiation steam
a. #10 b. #20
b. 121" F for 15 minutes
c. #30
c. 250 F for 5 minutes
d. #40 e. #50
0
d. 2500 C for 20 minutes e. 250 0 F for 20 minutes
immediate indicator that the conditions for sterilizao·on have been met in an autoclaved surgical pack?
a b. c. d. e.
appearance of the instruments autoclave tape chemical indicator culture results melting pellet
<0 1998 Mosby-Year Book. Inc. Photocopying is prohibited by law.
a. 0 b. 0000 c. 2-0 d. .2 e. 3-0
extrachromic heavy chromic medium chromic mild chromic plain gllt
a. b. c. d. e.
crushing curettage electrocoagulation ligation pressure
25. Wilich a/the/allowing is nota likely cause of
dehiscence ofan abdominal incision ?
20. Which suture size is smnlkst in diameter? J5. Mtich of the following is tile most effective and
a. b. c. d. e.
24. Wllich of tile following is not an effective/arm 0/ surgical hemostasis?
commonly used/or clipping the hair from a surgical site?
d. nervous tissue e. uterus
0
b. nylon
d. stainless-steel wire e. chromic catgut
19. Which size ofelectrical clipper blade is most 14. Which of the following describes the minimal
materials except:
a. drapes b. gloves c. gown d. mask e. suture material
a. 121 C for 15 minutes
a. debridement b. inspissation c. lithotripsy d. retraction e. baJlo(tcment
conductiOIl offluid bycapi/laryaction?
a. braided colton b. braided polyglycolic acid c. braided silk d. monofilament nylon e. chromic catgut
a. conan
c. liver
9. Wl!ich term describes removal of necrotic tissue from a wound?
21. Which suture material shows the lease
sterile during a surgical procedure involving aseptic technique?
11 . Most appropriate for sterilization ofan electric
exposure rime and temperature for aucoclaving a surgical pack?
b. intestine
16. What is the proper term for entrance of
187
22. All tile followillg are lIonabsorbabfe suture
For Questions 11 through 13, select the correct answer from the five choices below.
b. granulation
c. secondary union d. second-intention healing c. wound contraction
Principles a/Surgery
a. h. c. d. e.
chronic vomiting excessive physical activity s tormy recovery from anesthesia surgical wound infection suture maleriallarger than needed
Correct answers are on pages 195-198.
26. Concerning aseptic surgical technique. which statemem is U!astaccllrate?
a. A sterile item touched by a nonsterile item becomes nonsterile. b. Irlhe sterilify of an item is in doubt, consider it sterile. c. Nonscrubbed personnel can touch only nonslerile items. d. Only sterile items can contact exposed patient tissue. c. Only sierile items can contact other sterile items. 27. WI/iell a/the/allowing is an everting suture pattem rltat should only be used to close skin
incisions? a. b. c. d.
Cushing Lembert Parker-Kerr simple interrupted
28. Which D/the/allowing is an inverting suture pattem used mailily tosulure hollow imernai organs?
horizonlal maUress Lembert purse string simple interrupted or continuous vertical mattress
29. Which of tile following is most effective in minimizing the scarringfrom skin sutures and achieving good wolmd healing?
a. b. c. d. e.
c. granulation tissue d. wound contraction e. wound flushing 3/. \!VI/jcll oftlte following is nor a characteristic of first-jntemionlvowld Ileating?
a. b. c. d. e.
leaving sutures permanently in place removing the sutures 2 days after insertion removing the sutures 7 days after insertion using large-diameter suture material using suture material that produces significant inflammation
minimal contamination minimal tissue damage minimal role of wound contraction wound edges not approximated wound edges sutured
32. Which illcision is most appropriate for exploratory surgery i1l a dog's abdomen, in which the precise location of the problem if not known?
a. b. c. d. e.
c. vertical mattress
a. h. c. d. e.
www.vet4arab.co.cc
SECTION 8
188
dorsal midline flank paracostal paramedian ventral midline
33. \W,ich ofthefollowing is 1I0t an early sigtl of wOlllld dehiscence tll~ring tlte first 24 hours after abdominal surgery?
a. b. c. d. e.
body temperalUre elevation of]" to 2~ F change in texture of the wound edges serosanguineous discharge from the incision swollen incision very warm incision
34. Ti,e main goal ofaseptic surgical technique is to prevem cOlllamillatioll of
a. h. c. d. e.
operative personnel sterile fields slCrile zones surgical instruments surgical wounds
PrincIples o/Surgery
35. Wlt;ch factor related to infection ofa surgical wound is most sigtlificantlyaffected by aseptic technique?
a. number of microorganisms entering the wound b. pathogenicity of microorganisms entering the wound c. species of microorganisms entering the wound d. route of exposure to infectious microorganisms e. susceptibility of the patient 36. The effectiveness ofa bactericidal surgical scrub of ones hands and arms depends on tlte:
a. combination of contact time and scrubbing action b. le ngth of time the soap is in contact with the ski n c. pH ofthe skin surface d. scrubbing action of the brush e. temperature ofthe water 37. Wltich ofthe following, when used alone as a surgical scrub soap,forms a bacteriostatic film over the skin?
a. chlorhexidine b. chlorpheniramine c. hexadimethrine
a. d~bridement h. exuberant granulation tissue
1.0 1998 Mosby-Year Book, Inc. Photocopying is prollibited by law.
d. orthopedic equipment e. surgical drapes 40. TIte lime necessary to achieve disinfection of surgical instruments with liquid chemicals can be shortened by:
a. agitating the solution b. cooling the solution c. using a lower concentration than recommended d. using a higher concentration than reco mmended e. warming the solution 41. Which surgical wire size is smallest in diameter?
a. 40 gauge b. 10 gauge c. 26 gauge d. 32 gauge c. 20 gauge 42. Wlliclt surgical drape material prevents passage of bacteria through the drape to tlte patient's skin by capillary action when rite top surface of the drape becomes lvet?
a. cloth b. fenestrated paper c. musl in
d. hexachlorophene
d. paper
e. povidone-iodine
e. plastic
38. Which of the followillg tloes not normally have to be sterilized as part ofgootl aseptic surgical technique?
43. Which suture size is smaller in diameter than 3-D?
a. 2-0
a. cap
b #1
b. drapes
c. 113
c. gloves d. gown e. scrub brush
e. 4-0
d.1I4
44. WIljel, suture material is absorbable? 39. Liquid chemical sterilization is used primarily for:
30. Which of tile fol/owing does not enhance healing ofall open wound?
a. electrical equipment b. hemostatic forceps c. instruments with sharp edges
189
a. cotton b. nylon c. polypropylene d. silk
e. chromic catgut
Correct answers are on pages 195-/98.
www.vet4arab.co.cc
SECT ION 8
190 45. Wlzieh surgical instrument should not be routinely steam sterilized?
d. both nonscrubbed and scrubbed personnel e. neither nonscrubbed nor scrubbed personnel
a. Backhaus lowel clamp h. Halsted mosquito forceps c. Mayo-Hegar needle holder d. Mctzenbaum scissors
e. Bard-Parker scalpel handle 46. Castration o/a healtlly6-month-old cat is an exampleo/:
a. cosmetic surgery b. elective surgery c. emergency surgery d. exploratory surgery e. first-i ntention surgery
47. WllI'ell solution callses the least tissue damage and is most oppropriate[or woulldfIushing?
a. hydrogen peroxide b. isotonic sal ine c. povidone- iodine scrub d. povidone- iodine solution
e. tap water 48. Which incisioll provides the best ouerall exposure of the abdominal cavit),?
a. dorsal midline b. flank
c. paracosIaI d. paramedian e. vent ral midline
a. b. c. d. e.
acute e mergency cosmetic proble m only minor significance no Significance serious but not an acute e mergency
52. Why is a rece"t surgical wound usually slightly warmer lllan the su rrounding normal tissues?
a contamination b. debridement c. infecl"ion d. inflammation e. septicemia 53. How 10llgafter sllrgerydoes a sutured surgical wound hegillto gaill significant strellgrhjrom production ofcoUagen strands, so that tile wound edges are begi"ning to be held together by tissue as well as sutures?
a. b. c. d.
6 to 8 hours 4 to 6 days 12 to 14 days 24 to 26 days e. 28 to 30 days 54. WOllnd cOrUrac(iofl is produced hy:
is (lie most appropriate suture pattern to close an incision ill all allimal's stomach?
49. What
•.
5/. Wllat is tile significance ofdehiscellce of the muscle, subcutaneous tissue, alld skin la}'ers ill a ventral midlille surgical wou"d?
horizontal mattress
b. Lembert c. purse-suing d . simple interrupted
a. b. c. d. e.
movement of only the dermis movement of only the epidermis movement of a ll layers of the skin reproduction of epidermal cells reproduction of all skin cells
Principles o/Surgery
56. Whic/l suture size is 1n.rger ill diameter than size 112? •. 0 b. #1
c. 2-0
d. no significance e. serious but not an acute emergency 62. Which suture pattern
is neilJuran inl!erring nor
an el!erting pattern?
d. #3 e. 3-0
..
57. Wllicll suture material is synthetic? chromic catgut b. cotton c. nylon d. plain catgut e. silk
a. b. c. d. e.
Cushing horizontaJ mattress Lembert simple interru pted or continuous vertical mattress
63. Willa! is the healing potential ofa fractured bone that is properlya/igned and kept immobile?
58. Which of the following is not a typical sign of hemorrhagic shock in a postsurgical patient?
a. deep, slow breathing b. c. d. e.
191
pale mucous membranes slow capillary refill tachycardia weakness
a. excellent b. good c. fair d. poor e. very poor 64. Whidl ofthefollowingindicates thebest blood sllpplyto theedgesofa U/Owld in unpigmenU!d skin?
59. What is the most appropriate suture pattern to
use in closing an incision of the urinary bladder?
•.
Cushing b. horizontal mattress c. s imple continuous d. simple interrupted e. vertical mattress
a. b. c. d. e.
black wound edges bluish-purple wound edges gray wound edges pink wound edges white wound edges
65. What portiol! ofa surgical gown is considered sterile during stlrgery?
60. Suture material used to close a surgicaluJOurld represents what kind of irritant to hody tissues?
a. chemical b. infectious c. photic d. phys ical e. thermal
a. e ntire o utside of the gown b. front and sides of the gown, from the neck to the bottom, including the arms c. fron t of the gown, from the neck to the bottom, including the arms d. front and sides of the gown, from the neck to the waist e. front of the gown, from the waist up, including the arms
55. As a part ofeffective aseptic technique, surgical
e. vertical manress
gowns:
50. Wllich operating room persOlmel should try to face alOOY from sterile fields durillg a surgical procedure?
a. all personnel b. nonscrubbed personnel only c. scrubbed personnel only
a. b. c. d. e.
are commonly made of cloth or paper a re put on by touching only the outside are routinely sterilized by ethylene oxide gas do not need to be sterile, only clean protect against contamination from the waist down
e 1998 Mosby-Year Book, Inc. Photocopying is prohihited hy law.
61. Mlal is the usual sigttificance ofa small seroma deep to (beneath) the skin suture line after aseptic surgery?
a. acute emergency b. cosmetic problem only c. minor significance
66. Which bacterial form is most easily destroyed by commoll sterilization methods?
a. b. c. d. e.
spores of aerobes hyphated form dormant form spores or anaerobes vegetative form
Correct answers are on pages 195-198.
192 67. What is theftrsl phase a/wound healing? a. b. c. d. e.
epithelial phase fibrob last phase innammalory phase malUration phase scarring phase
c. gastroscopy d. gastrostomy e. gastrotomy 73. W11a1 is tile correct surgical tenn for creation ofa pemJQnellt arrificial opening illto the esophagus?
68. In theftrsl 24 hOllrs o/primary union wound healing, most of the resistance co opening a/the sutured WQund is provided by:
a. b. c. d. c.
www.vet4arab.co.cc
SECT ION 8
collagen strands fib rin strands fibroblasts granulation tissue sutures
a. b. c. d. c.
esophagectomy esopbagopexy esophagoscopy esophagostomy esopbagotomy
74. With which type a/abdominal incision can the abdominal wall be most effectively closed using a single layer a/sutures?
69. If there are 110 complications, how long after surgery should skin sutures generally be removed?
a.2to3days b. 4 10 5 days c. 71010 days d. 1510 17 days e. 18102 1 days
a. b. c. d. e.
high flank low flank paracostal paramedian ventral midline
75. Scrubbed surgical personnel become contaminated iftlley touch:
70. What is the correct surgical term for incision of the urinary bladder?
a. cystectomy b. cystopexy
a. b. c. d. e.
objects in sterile fie lds objects outside the sterile zone properly stcrilized s urgical instruments sterile objects fresh ly exposed tissues of the patient
c. cyslOscopy 76. NOr/scrubbed surgical personnel may properly touch anything that is:
d. cystostomy e. cystoto my
71. What is ti,e correCl surgical term for removal of
the kidney? a. nephrectomy b. nephropexy
a. b. c. d. e.
contaminated inside the patient inside the stcrilc zone part of a sterile field sterile
c. nephroscopy
77. How should scrubbed personnel pass each other in the operating room?
d. nephrostomy e. nephrotomy 72. What is the correct surgical tenn for sUlUring the stomach to the body wall to fIX the stomach in place?
a. gastrectomy b. gastropexy
@ 1998
a. b. c. d. e.
any way that is convenient back to back back to fronl front to back front to front
Mosby-Year Book, Inc. Pllotocopying is prohibited by law.
Principles of Sll rgery
78. When not otherwise occupied, scrubbed surgical personnel should stand with their:
a. arms folded across the chest b. hands held apart and above shoulder level c. hands clasped between waist and shoulder level d. hands held down and to each side e. hands on the surgery table 79. During surgery, when is it permissible for nOllScrllbbed surgical personnel to pass between scrubbed personnel and the patient?
a. b. c. d. e.
at any convenient time never when opening suture material when adjusting the anesthesia machine when adjusting the intravenous drip
80. Wllell aseptically opening a sterile surgical pack 0" an instrument stand, it is not proper for 1I0nscmbbed surgical personnel to toucll tile:
a. b. c. d. e.
autoclave tape conte nts of the pack corners of the wrap instrument stand outside of the wrap
81. Wlljcll characteristic applies to ethylene oxide gas?
a. b. c. d. e.
flammable Exposure is not considered a health hazard. noncombustible nontoxic to tissues safe to breathe
82. WI/icll type of needle is most appropriate for suturing muscle?
a. blunt b. cutting
c. reverse cutting d. tapered
193
83. Wllicll type of needle is most appropriate for suturing a ligament?
a. blunt b. cutting c. reverse cuning d. tapered e. trocar 84. Wlliell type of needle is most appropriate/or suturing tile uterus?
a. blunt b. cuning c. reverse cutting d. tapered c. trocar 85. Surgical removal o/a ruptured spleen is an example of
a. cosmetic surgery b. elective surgery c. eme rgency surgery d. exploratory surgery e. first- intention surgery 86. Removal ofa large skin tumor has left a large skill defect to be closed. Which suture pattern is least likely to cause skin tearing when large wounds are closed under tension?
a. Cushi ng b. horizontal mattress c. Lembert d. purse-string e. simple interrupted 87. Wllicll of tile following is lIot a likely contributor to dehiscellceof an abdominal surgical incision?
a. b. c. d. e.
chronic vomiting internal su ture ends cut too short infection skin sutures left in place too long suture material of too-small diameter
e. trOcar
OJrrect answers are on pages 195-198.
194
88. Which type ofdressing, when removed, provides tile least traumatic and least irritating means of debriding a wound with extensive tissue damage?
a. b. c. d. e.
dry gauze dry nonadhesive pad
gauze dressin g with an oily antiseptic gauze dressi ng with a water-soluble an tiseptic wet saline dressing
89. Most o/che clinical signs seen in animals in shock related to excessive blood loss are attributable to:
a. acidosis b. alkalosis
c. cell dealh d. redistribution of blood flow e. tissue hyperoxia 90. W1lich ollhe/allowing is a noncapillary suture material suitable for skin closure?
a. h. c. d. e.
braided conon braided potyglycolic acid braided silk monofilament stainless steel monofilament coUon
91 . The main goal of surgery to remove a pus-filled uterus (pyometra) is to:
a. b. c. d.
prevent subsequent pregnancy alter the behavior of the animal make a diagnosis restore the animal to a normal reproductive state e. restore health despite loss of normal reproductive funct ion 92. With what kind oJknol should sutures be routinely tied?
a. b. c. d. e.
www.vet4arab.co.cc
SECTI ON 8
bowline granny knot square knot slip knot half hitch
93. Concerning the principles ofcryosurgery, which statement is least accurate?
a. b. c. d. e.
Frozen tissues should be thawed slowly. Uttle aftercare is required. Multiple freeze-thaw cycles should be applied. Tissues should be frozen to - 25 0 C. Tissues should be frozen rapidly.
Principles o/Surgery
98. The use afextreme cold to destroy unwanted tissue is termed:
a. b. c. d. e.
cosmetic surgery cryosurgery elective surgery orthopedic surgery prophylactic surgery
195
c. nylon d. polypropylene e. s ilk 100. To be classified as nonabsorbable, suture material must maimain its tensile strength in tissue for longer than.'
a. 30 days 94. How long after surgery does the strength ofa sutured surgical skin wound return to its original preoperative strength?
a. b. c. d. e.
7 to 10 days 2 1 to 28 days 60 days 2 to 3 years never
95. In second-imention healing, which of the following must he present before wound contraction orepirheUal regeneration can occur?
a. b. c. d. e.
collagen fibers exudative tissue fi brin clot granulation tissue scar tissue
96. When putting on sterifeglovesJor aseptic surgery, which of the fallowing is not permitted?
a. touching one gloved thumb with the other gloved thumb b. touching the outside of the glove with scrubbed fingers c. touching the outside of the gown cuff with the inside of the glove cuff d. touching the outside of one glove with the outside of the other glove e. touching the ins ide of the glove cuff with scrubbed fingers 97. How should packs be placed in an autoclave for sterilization?
a. b. c. d. e.
diagonally horizontally tightly packed unwrapped vertically
© 1998 Mosby-Year Book. Inc. Ph otocopying is prohibited by law.
99. Which suture material is badly damaged by steam sterilization?
b. 60 days
c. 90 days d. 120 days
a. polyglycolic acid b. polyester
e. IBO days
Answers I. c The instrument is named for its fun ction of refracting organs and tissues oul of the way. 2. d None ofthe other instruments listed has opposing jaws. 3. c Declawing involves removal (-ectomy) ofthe nails or claws (oflych -). 4. d A tapered-po int needle easily passes through soft organs, making a tunnel through which the suture material is drawn. 5. c Th e ~- circle needle offers the best compromise of shape. allowing use in both shallow and deep incisions. 6. a An eye with proptosis must be returned to the eye socket quickly to prevent permanent damage to the eye. 7. a The basic requirements for healing by first intention are minimal tissue damage and apposition of the edges of the wound. usually with sutures. B. d The basic functional unit of the nervous system. the neuron. is incapable of reproduction. so damage to the nervous system is often repaired by scar tissue. The other organs and tissues listed have excellent healing pOlential. 9. a Debridement of a wound facilitates healing by minimizing the amount of inflammation necessary before filling of the defect can begin. 10. e The xiphoid process and umbilicus are both on the animal's ventral midline, 1 L d An electric drill would be damaged or inadequately sterilized by any of the other methods.
12. a Steam sterilization in an autoclave is most commonJy used for instruments and equipment not damaged by moisture or heal. 13. e The sharp edges of scissors are dulled by steam in an a utoclave. Boiling and dry heat are not sufficiently effective. The expense and hazards of ethylene oxide are not warranted. 14. a The minimal standard for sterilization of surgical instruments in an autoclave is 121 0 C (250° F) for at least 15 minutes. 15. c Chemical autoclave ind icators are the only type listed that can give inunediate information on all three basic criteria for autoclave sterilization (presence of steam at the proper combination of exposure time and temperature). 16. a Microorganisms in a wound during surgery are considered contaminants until, or unless, they multiply and cause damage. 17. d A surgical mask does not come in contact with anything sterile during a surgical procedure, so it need onJy be clean. lB. e An autoclave sterilizes by exposing packs to steam under pressure. 19. d A #40 clipper blade is a "surgical" blade. It clips the hair off at the skin surface. 20. b Numbered suture sizes (e.g., #2) decrease in size as the number gets smaller. down to size O. From that p oint on the sizes get smaller as the number of Os (or the number in front.ofthe 0) increases. 21. d All the other suture materials listed are either braided or twisted and have the potential for considerable capillary action.
196
SECTION 8
22. e Surgical gut is absorbed by the body. 23. e Treatment with chromic acid delays absorption of surgical gut by the body. 24. b Curettage involves the scraping of a tissue or cavity. 25. e Use of overly large suture material would not cause wound dehiscence. It would actually prOvide greater holding power than smaller suture material. 26. b Contaminated items appear identical to sterile items. If there is any doubt about the sterility of an item, it must be considered contaminated. 27. e All the other patterns are inverting or appositional. 28. b The other patterns are inverting (mattress patterns), appositional (simple pattern), or used only to close an orifice (purse-string). 29. c All the other choices promote increased scarring or early disruption of the wound. 30. b Exuberant granulation tissue (proud flesh) acts to block wound healing and epithelial regeneration. The other choices would likely enhance wound healing. 31. d One of the most important characteristics of wound healing by first intention is approximation of the wound edges. 32. e The ventral m idline approach to the abdomen gives the most extensive access to the abdominal cavity. 33. a Slight elevation of body temperature for 1 or 2 days is nonnal after major surgery. The other choices are all early indicators of wound dehiscence. 34. e Prevention of surgical wound contamination is the whole purpose of aseptic technique in the operating room. 35. a This is the only choice that can be influenced by aseptic technique. The others are inhe rent to the patient, the surgical procedure being performed, or the microorganisms in the environment. 36. a The antimicrobial effect of a surgical scrub depends on sufficient exposure of the skin to the soap, as well as the scrubbing action that loosens dead skin and debris and works the soap down into the cracks and crevices of the skin. 37. d Hexachlorophene fonns a bacteriostatic film on the skin if used exclusively to wash the hands and anns. Other soaps remove the protective film.
www.vet4arab.co.cc
38. a The surgical cap does not come in contact
with tissues of the patient direcdy or indirectly, so it need only be clean, not sterile. 39. c Uquid chemical sterilization does not dull sharp edges. 40. e Warming the solulion accelerates the chemical reactions necessary to kill microorganisms. 41. a The relative size of the wire, as measured by gauge, is inversely proportional to the gauge number. For example, 40-gauge wire is smaller than 32-gauge wire. 42. e Cloth and paper drapes are subject to capillary action. Plastic drapes are not. 43. e From largest to smallest, these sizes are ranked as follows: #4, H3, HI, 2-0, and 4-0. 3-0 is m idway in size between 2-0 and 4-0. 44. e All the other su ture materials listed are nonabsorbable. 45. d Steam dulls the sharp edges of scissors. 46. b Elective surge ry is done by choice, so it can be performed when condjtions are most appropriate. 47. b The other listed solutions are irritating to the tissues or are not isotonic with the patient's tissue fluid s. 48. e The ventral midJine approach provides the most elCtensive access to the abdominal cavity. 49. b Incisions in viscera are best closed with inverting suture patterns. The other listed patterns are everting, appositional or inappropriate for wound closure. 50. e All personnel in the operating room should face toward sterile fields so they are aware of their relationship to them. 5 1. a Dehiscence of all layers of the body wall exposes abdominal viscera. Repair must be immediate to prevent serious damage to abdominal structures. 52. d Inflammation results from any insult to the body, whether intentional (surgical) or unintentional {traumatic, infectious}. Increased blood supply to an inflamed area produces the increased warmth of the area. Good surgical technique minimizes inflammation but does not eliminate it. 53. b It takes 4 to 6 days for production of collagen strands in a wound to reach a significant level. Until that time, tlle wound is held together by sutures.
e 1998 Mosby-Year Book, Inc. Photocopying is prollibited by law.
Principles ofSllrgery
54. c Wound contraction represents movement of the entire thickness of the skin toward the center of the wound. 55. a All other choices are incorrect. 56. d All other choices are smaller. 57. c All other choices are from natural sources. 58. a A patient in shock would show rapid, shallow breathi ng in an effort to oxygenate the blood as rapidJy as possible. 59. a The Cushing pattern is the only one listed that is an inverting pattern appropriate for closure of a hollow organ. 60. d Suture material aClS as a physical irritant until it is absorbed, removed, or encapsulated wilh scar tissue. 61. b Unless they are very large or ruptured, postoperative seromas are unsightly but of little other importance to the animal's health. 62. d The simple pattern is an appositional pattern. It brings the incision edges together without inverting or everting them. 63. a Bone has excellent healing capadties, provided the fracture fragments are properly aligned and movement is kept 10 a minimum . 64. b Bluish-purple wound edges indicate that blood vessels in and under the skin are congested with blood. 65. e This is the only portion of a surgical gown that is considered sterile during surgery. 66. e The vegetative bacterial form is the actively feeding, growing, reproducing form. It is most easily destroyed by common sterilization and disinfection methods. 67. c Inflammation is the first step in wound healing. It "cleans up" the damage so the defect can be repaired by the balance of the healing process. 68. e Other than su tures, a surgical wound has no appreciable strength until significant numbers of collagen fibers arc produced at 4 to 6 days. 69. c Before 7 days, the wound may not have enough strength to resist separation. After 10 days, inflammatory reaction to the sUlUre material may cause significant scarring. 70. e The suffix -oromy means to male an incision into something. 71. a The suffix -ectomy means to s urgically remove something. 72. b The suffix -pexy means to fix something in place.
197
73. d The suffix -ostomy means to create an artificial opening in an organ or tissue. 74. e The linea alba, on the ventral midline afme abdominal muscle wall, is me tendinous attachment of the ventral abdominal muscles. One layer of sutures in this area effectively closes the whole thickness of the abdominal wall, after which the skin is closed. 75. b Anything outside the s terile zone in an operating room is considered contaminated. 76. a Nonscrubbed personnel should only touch things thai are not sterile. 77. b Passi ng back to back prevents accidental contamination oflhe front, sterile portions of their gown. 78. c The bands of scrubbed personnel should always be held between waist level and shoulder level to help prevent inadvertent contamination. Clasping the hands, when not mheIWise occupied, helps prevent fatigue from compromising the position of the hands and arms. 79. b Nonscrubbed personnel should never violate the sterile zone in which scrubbed personnel are working. 80. b The sterility of the pack contents would be des troyed if touched by a nonscrubbed person . 81. a Ethylene oxide gas is very flammab le. 82. d A tapered-point needle easily passes through muscle, making a tunnel through which the suture material is drawn. 83. e A reverse-culting needle cuts a tunnel through the tough tissue of a ligament that is less likely to tear through than the tunnel created by a standard (inside-curve) cutting needle. The other needle points would not easily pass through this tough tissue. 84. d A tapered-paint needle easily passes through muscle, making a tunnel through which the suture material is drawn. 85. e Splenic rupture is a potentially life-threatening condition. If surgery is indicated, it must be performed immediately. 86. b Mattress suture patterns spread the tension created by each suture over a broad area and are less likely to tear out because of tension on the suture line. 87. d Leaving skin sutures in place too long increases scarring but does nOl directly contribute to breakdown of the surgical wound.
198
SECTION 8
88. e Wet saJine dressings are useful to help debride wounds with extensive tissue damage. They absorb and remove inflammatory products from the wound. 89. d Redistribution of blood flow results in the pale mucous membranes, poor capiUary refill, and cold extremities seen in shock. 90. d The other suture materials listed are braided or twisted and can conduct fluid and microorganisms by capillary action from the surface of the sldn to the deeper layers. Cotton (answer choice e) is not available as monofilament. 91. e Removal ofthe ut erus may restore health, but it rusa precludes future breeding. 92. c A square knot is conve niently tied and
provides a very secure knOl. 93. b The principal action of cryosurgery is destruction of unwa nted tissue by freezing. This leaves dead tissue that must be liquefied and removed by inflammation. Such areas must be monitored closely and kept clean, and frequently they require bandaging.
www.vet4arab.co.cc
94. e The strength of the scar that res ults from heaJing of a surgicaJ skin wound never reaches that of the normal skin around it. 95. d After dead and damaged tissue has been removed from a wound by inflammation, a bed of granulation tissue, consisting primarily of collagen fibers and capillaries, must form on the floor of the wound so that the processes that reduce the size of the wound can begin. 96. b If the outside of the glove is touched by anything that is not sterile, including freshly scrubbed fingers, it becomes contaminated and must not be used for surgery. 97. e Packs placed verticaJly in the autoclave receive the best circulation of steam around their contents. 98. b The prefix cryo- m eans cold. 99. a Polyglycolic acid should never be sterilized in an autoclave. lOll b Suture material is considered nonabsorbable if it retains its tensile strength for at least 60 days after implantation in tissue.
NOTES
SECTION
9 Theriogenology W.F. Braun, Jr., T.J. Burke, l.A. Dierauf, M. Drost, B.E. Eilts, F.l. Frye, T. Gemeinhardt, S. Hudelson, D.G. Huff, J.D. Letcher, S. Mobini, D.H. Nielsen, M.B. Paster, D.O. Schaeffer, R.C. Tubbs
Recommended Reading Altman RB et al: Avian medicine and surgery, Philadelphia. 1997. WB Saunders. Feldman EC, Nelson RW: Canine and feline endocrinology and reproduction . cd 2. Philadelphia, 1996. WB Saunders. Frye FI.: Biomedical and surgical aspects o/captive reptile lIusbandry, ed 2, Melbourne. Fla. 1991, Krieger Publishing. Hafez ESE: Reproduction in /ann animals. ed 6, Baltimore, 1993, Williams & \Vilkins. Johnston SD. Olsen PN: Canine&/eline reproduction, Philadelphia. 1997. \VB Saunders. Leman AD et a1: Diseaseso/swine. ed 7. Ames, Iowa, 1992. Iowa State University Press. Mader DR: Reptile medicine and surgery, Philadelphia. 1996. WB Saunders. McKinnon A, Voss JI.: £quille reproduction. Baltimore, 1992, Williams & WLikins. Morrow DA: Current tllerapy in theriogenology, cd 2. Philadelphia, 1986, WB Saunders. Rosskopr n, Woerpcl R: Diseases 0/ cage and aviary birds. cd 3. Baltimore, 1996. Williams & Wilkins. Varner DD et a1: Manual 0/ equine reproduction, St. Louis. 1998, Mosby. PetcrsAR, Ball PJ: Reproduction in cattle. Ames, Iowa, 1995. Iowa State University Press. Radostits OM et a1: Veterinary medicine, ed 8, Philadelphia. 1994. WB Saunders. Youngquist RS: Current therapy in large animal theriogello/Qgy, Philadelphia. 1997. WB Saunders. Practice answer sheets are on pages 271 ·272.
DOGS AND CATS T.J . Burke
Questions J. The estrous cycle oftile auerage bitch classified as:
a. seasonally polyeslfous b. seasonaJly monestrous
10 1998 Mosby-Year Book, Inc. Photocopying is prolJibited by law.
is best
c. nonseasonally polyestrous d. nonseasonally monestrous e. a blend of seasonally and nonseasonally polyestrous
Correct answers are on pIlges 234-244.
199
2. The estrolts cycle of the average queen exposed to natural daylight is best cllillified as:
a. seasonally polyestrous
b. seasonally monestrous c. nonseasonally polyestrous d. nonseasonaJly monestrous e. a blend of seasonally and nonseasonally polyestrous 3. Mll'e/l dog breed normally exhibits only one estrus per year?
7. In tile hitch, which hormone is present during pregnancy but Itot during pseudopregnancy?
a. estradiol·1 7j3, b . relaxin c. prostaglandin Fz.. d. progesterone e. luteinizing hormone 8. Among bitches ofall breeds, including mongrels, which stage ofrheestrous cycle is tile most variable in lellgtM
a. proestrus
a. German shepherd b. minia ture poodle
b. estrus c. metestrus d. diestrus e. anestrus
c. basset hound d. basenji
e. great Dane 4. Wlljeh of the following is a reliable indication of ovulation ill the birch?
a. drop in rectal temperature below lOlO F b. change in color of the sanguineous vulvar discharge
c. glucose in Ihe vaginal [Juid d . rise in serum progesterone above baseline levels e. standing for mounting by the male 5. Which of the following isa reliable indication that wllelpirlg will ensue within 24 hours?
a. b. c. d. e.
www.vet4arab.co.cc
SECTION 9
200
rise in rectal tcmperature above 102.5 F drop in rectal temperature to 100.50 F drop in serum progesterone to baseline levels clear mucoid vulvar discharge food refusal 0
9. Which of rile following is least likely co terminate
standing heat;1I a queell?
a. mating with a fCltile tom b. mating widl an infertile tom c. injection of 0.25 mg of es tradiol cypionate d. oral administration of 5 mg of megestrol acetate daily for 10 days e. obtaining a deep vaginal sample fo r culture, using a sterile swab 10. Wllieh oftllefollowillg is approved in the United
States for prevelltion ofestrus ill the bitch, once proestrlls has begun?
a. m ibolerone b. megestrol ace1"ate c. estradiol cypionate d. methyltcstostcrone e. prostaglandin F2"
6. Which of the following is mosllikely to detect
pregllallcyat 18 days in the bitc1t?
a. b. c. d. e.
B-mode ultrasonography Doppler ultrasonography routine radiography palpation serum progesterone determination
11. Wh ich of tile following is approved ill the United
States for prolonged lise (lip to 2 years) in preventioll ofestr·us ;n the biecl/?
a. b. c. d. e.
mibolerone megestrol acetate estTadiol cypionate methyltestosterone diethylstilbestrol
0 1998 Mosby-Year Book, Inc. Photocopyillg is pro/dbited by Inw.
12. Which o/the/ollowing is approved in tile United States for prevention 0/estrus in cats?
a. b. c. d. e.
mibolerone megestrol acetate diedlylstilbestrol methyltes tosterone None of the drugs listed have been approved for this purpose in cats.
13. In dogs, final maturation o/spermatozoa occurs ill the:
a. seminiferous tubule
d. vas deferens
e. seminal vesicles 14. Theoptimllm time to inseminate a bitch is:
2 to 3 days before ovulation 4 to 6 days before ovulation I to 2 days after ovulation 4 to 6 days after ovulation 8 to 9 days after ovulation
para basal epithelial cell uncornified epithelial cell s uperficiaJ epithelial cell erythrocyte polymorphonuclear cell
18. Estrogenic hormones are frequently used in treatmem ofmismating in the bitch. Concerning such treatment, which statement is least accurate?
following has shown the best results in emptying the uterus o/pllrulent contents?
ejaculate containing the grea:est Ilumberofspeml?
miniature schnauzer German shepherd basenji great Dane Sperm numbers are about the same in all these breeds.
16. A normal stlld dog o/whicll breed produces all ejaculate concaining tile greatest concentration sperm?
a. b. c. d. e.
during standing heat (estrus) in the bitch?
a. b. c. d. e.
19. III medical treatment of pyometra, which of the
15. A normal stud dog of which breed produces all
a. b. c. d. e.
17. In vaginal cytology, which cell predominates
a. Bitches under 4 years of age are less likely to su ffe r bone marrow toxiciry. b. This treatment increases the incidence of uterine infection by about 30%. c. Treated bitches remain in heat longer than normal. d. The treatment causes no adverse side effects. e. The success rate is directly related to dle time between mismating and the onset of therapy.
b. rete testis
c. epididymis
a. b. c. d. e.
201
Tlleriogeno logy
a. b. c. d. e.
estradiol-17j3 progesterone oxytocin prostaglandin Fz.. calcium gluconate
20. Wh ich of the following is not a usual/eature of Brucella canis infection ill dogs?
0/
miniature schnauzer German shepherd basenji great Dane Sperm concentration is about the same in all these breeds.
a. b. c. d. e.
feve r prolonged bacteremia epididymitis abortion mild generalized lymphadenopathy
Conect answers are on pages 234-244.
21. Which of the following, used alorle, can confirm a
diagnosis a/brucellosis in dogs? a. history of abortion during (he last trimester of pregnancy h. blood culture c. tube agglutination test d. tube agglutination test using 2-mercaptoethanol e. rapid slide agglutination test 22. Concerning brucellosis in dogs. which statement is most accurate?
a. A combination of tetracycline hydrochloride and dihydrostreptomycin will cure an infected dog. b. Vaccination of at least 85% of the dogs in a kennel will halt an outbreak. c. All dogs exposed to the organism become infected. d. Infected dogs may show no clinical signs. e. Blood cultures within 2 weeks of infection will detect infected dogs. 23. In assessing sperm morphology, which of the following is not considered a primary defect?
a. b. c. d. e.
tightly coiled tail double head balloon head double midpiece detached head
c. a freemartin d . a monorchid e. infertile 26. Cryptorchidism in dogs is:
a. b. c. d. e.
often a cause of colic in affected neonates a recessive genetic defect unrelated to other health problems later in life less common than in cats readily diagnosed at birth in large breeds
27. A l-year-oldAKC-registered male German shepherd is presented to you. On physical examination, you can locate only one scrotal testis. The dog has had no previous surgery. The most appropriate recommendation to the owner is to:
a. b. c. d.
have the scrotal testis removed have the cryptorchid testis removed have both testes removed replace the cryptorchid testis to a scrotal position by orchiopexy e. replace the cryptorchid testis to a scrotal position with gonadotropin therapy
28. A 2-year-old miniature schnauzer has an empty
scrotum and an underdeveloped prepuce. During exploratory celiotomy, you discover a uterus and two gonads that, on histologic examination, are identified as ovaries. Tllis dog is most appropriately classified as a:
24. fnthe bitch. protrusion ofthefloor of the caudlll vagina and vestibule through the labia during proestrus/estrus:
a. b. c. d. e.
www.vet4arab.co.cc
SECTION 9
202
is not unus ual and is clinically normal indicates the bitch is ready to breed will not regress during metestrus/diestrus indicates uterine prolapse indicates vaginal hyperplasia
25. A lO-month-old male toy poodle with only one palpable testis in the scrotum is best described as:
a. normal b. a unilateral cryptorchid
a. b. c. d. e.
female pseudohermaphrodite male pseudohermaphrodite bilateral cryptorchid hermaphrodite semihermaphrodite
29. The most common endocrine cause of subfertility and infertility in the birch is:
a. b. c. d. e.
hypogonadism hyperadrenocorticism hypothyroidism hyperthyroidism cystic ovaries
© 1998 Mosby-Year Book, Inc. Photocopying is prohibited by law.
30. The ownerofa 3-year-old masti/fbitch says, "The dog has been in heat for the last 9 weeks." The dog has a slightly edematous vulva. Vaginal cytologic examination reveals no erythrocytes or leukocytes, and the epithelial cells are all intermediate to superficial cells. The most likely cause of these findings is:
a. b. c. d. e.
normal estrus normal metestrus pituitary adenoma vaginitis cystic ovaries
d. testicular Jamshidi needle biopsy e. testicular Franklin-modified Vim-Silverman needle biopsy 35. Radiography may help determine the viability of late-term fetuses. Which of the following is not a radiographic sign offetal death?
a. b. c. d. e.
31. Which of the following is not a common cause of
abortion, stillbirth, or fetal mummification in the queen?
a. b. c. d. e.
203
Theriogenoiogy
feline calicivirus infection feline herpesvirus infection feline parvovirus infection feline leukemia virus infection feline corona virus (nonemeric) infection
32. Concerning uterine subinvolution, which statement is most accurate?
a. It is effectively treated with several progesterone injections. b. It is common in primiparous queens. c. H can be prevented by injection of oxytOCin \vithin 24 hours after parturition. d. It is common in primiparous bitches. e. It is the first sign of pyometra. 33. Bacterial prostatitis:
a. is uncommon in older male cats b. produces azoospermia c. produces oligospermia d. interfe res with copulation e. has little effect on fertility 34. Which technique yields the best results in determining the cause ofazoospermia in a dog with bilateral testicular atrophy?
a. testicular fine -needle aspirate b. testicular wedge biopsy c. testicular Vim-Silverman needle biopsy
intrauterine gas absence of molar teeth in the fetal jaws malalignment of cranial bones, with overlap fetal subcutaneous gas hyperextension of the fetal bodies
36. A 2B-momh-old beagle bitch has had three normal heats. At the last two heats, the owner attempted to mate her with a proven male on the thirteenth and fifteenth days, respectively, after he first noticed blood drippingfrom the vulva. When the male mounted the bitch, she yelped, bit the male, and ran behind the owner. The most likely cause of these nonproductive breedings is:
a. b. c. d. e.
inexperience of the male preputial adhesions uterine hypoplasia vaginal stricture breeding too early in the cycle
37. Transmissible venereal CIImors ofdogs:
a. b. c. d. e.
are caused by a virus are nearly always malignant are transmitted only during coitus sometimes spontaneously regress do not respond to radiation therapy
3B. An B-year-old cryptorchid standard schnauzer has had no previous surgery other than standard puppy cosmetic procedures. The dog has shown abdominal discomfort for "about a dlly." Physical examination reveals a firm, nonpainful abdominal mass about 6 cm in diameter and a softer B-cm mass near the pelvic inlet. You suspect neoplasia involving a retained testis. Which tumor is most likely to befound in this dog?
a. h. c. d. e.
Sertoli-cell tumor interstitial-cell tumor Leydig-cell tumor granulosa-cell tumor seminoma
Correct answers are on pages 234-244.
204 39. Which tumor is most likely to produce bone marrow dysplasia? a. b. c. d. e.
Sertati-cell tumor interstitial-cell tumor Leydig-cell tumor granulosa-cell tumor seminoma
40. Routine aerobic cultures of swab samples taken from the caudal vagina ofa normal bitch are least likely to reveal:
a. Escherichio. coli h. Streptococcus [ecaUs c. j3-hemolytic Streptococcus d. Mycoplasma
e. Proteus vulgaris
41. Which afthe/allowing is not a sign afmale feminizing syndrome? a. pendulous prepuce b. gynecomastia
c. hypopigmentation of the skin d. fem ale posture (squatting) when urinating e. symmetric alopecia 42. Which of the following is a cricerion indicating
dystocia? a. 1 hour since birth of the previous fetus b. brachycephalic breed c. 72 days since the last breeding, with pregnancy confirmed d . strong co ntractions for 15 minutes e. male was larger than the female 43. A dog with bilateral aplasia of the vasa deferentia is likely to have: a. b. c. d. e.
www.vet4arab.co.cc
SECTION 9
azoospermia oligospermia necrozoospennia low volume of ejaculates decreased libido
44. On microscopic examiflation of vaginal smears from a birch, erythrocytes: a. b. c. d. e.
are present only during proestrus are no longer seen after ovulation occurs are present throughout the cycle do not indicate the stage of the cycle are nucleated until ovulation
45. You spay a 6-month-old mixed-breed bitch. and 4U years lacer tile angry owner presents the dogfor examination. The owner insists the dog had a heat cycle about J month previously and may have had another heat during the past year. Assuming the owner is correct, which stateme1lt is most accurate? a. The patient is likely to have an elevated serum estradiol level. b. The patient is likely to have an elevated serum progesterone level. c. The patient is likely to have hyperadrenocorticism. d. Vaginal cytologic examination is likely to show a preponderance of supe rficial cells. e. Ultrasonography is likely to reveal a cystic ova'Y. 46. A bitch that was presumably spayed but rlOW shows signs of heat mostUkely has: a. b. c. d. e.
hyperadrenocorticism a pituitary adenoma aright ·sided ovarian remnant a left-sided ovarian remnant an ectopic ovary
47. Which of the following will riot reliably termiflate pregflancy in the bitch?
a. progesterone
Theriogenology
48. Properly frozerl canine semen can still fertilize ova after storage in liquid nitrogen for: a. b. c. d. e.
up to 6 mo nths up to 1 year up to 3 years up to 6 years longer than 10 years
205
c. 48 hours d. 96hours e. I week 50. Concerning cats with a tortoiseshell hair-coat color pattern. which statement is least accurate? a. Most of them are females. b. Most males with a tortoiseshell coat are fertil e. c. Orange and black colors are related to alleles found only on X chromosomes. d . Ma1es with a tortoiseshell coat are usually XXY. e. Chimeric males may be fertile.
49. Sperm remain viable in fresh chilled-extended canine semen for approximately: a. 6 hours b. 24 hours
CATTLE M. Drost For Questions 51 through 55, select the correct answer from the five choices below.
a. anestrous
57. Thecircufating level of which hormone rises acutely during each estrous period? a. luteinizing hormone b. estrogen c. prostaglandin F2" d. progesterone e. fo llicle-stimulating hormone
b. estrous
c. metestrous d. m onestrous e. polyestrous
51. Having a single estrus during a breeding season 52. Not cycling 53. Having recurring periods ofestrus during a breeding season
54. Recently in heat 55. In heat
58. An advamage of nonsurgical collection of bovine embryos oller surgical collection of embryos is that nOrlSllrgical collection: a. requires little skill b. can be done repeatedly in the same donor, without complications c. can be done earlier after the onset of estrus in the do nor d. requires a smaller volume of flushing medium e. permits accurate counting of corpora lutea per rectum
b. ovariectomy
c. prostaglandin Fz., d. dexameth asone e. hysterectomy
© 1998 Mosby-Year Book. Inc. Photocopying is prohibited by law.
56. Estrogen is the predominant hormone during: a. pregnancy b. anestrus c. diestrus d. me testrus e. estrus
59. Progesterone is the predominant hormone during:
a. anestrus b. estrus
c. proestrus d. diestrus e. metestrus
Correct answers are on pages 234·244.
206
www.vet4arab.co.cc
SECT ION 9
6G. During prolonged dystocia j" a cow, tile calfs pelvis becomes lodged in file dam's birt" canal
(hip lock), and the cal/dies. Tile most appropriate
courseofactioll is to: a. perform a cesarean section b. repel the calf and then then rotate its hips for extraction c. use the calf puller to forcibly extract the dead calf d. pe rform a pubic symphysiotomy on the dam e. using a felotome. remove the craniaJ tw'Othirds ofthe calf, and then divide the calf's pelvis 61. A farmer has a Hereford cow tllat cycles regularly. His cows rim witll a bull at all times. 011 rectal
palpation to check tile cow for pregnancy, you are III/able {O retract the Ilterus caudally into the pelvic catwl. The most likely explanation is that the cow:
a. is 5 mon ths pregnant b. has pyometra c. has adhesions that are preventing movement of the uterus d. is carrying a mummified fetus e. is carrying a macerated fetus
64. As a species, cattle are:
a. monestrous b. polytocous c. uniparous d. primiparous e. multiparous 65. Hydramllios differs from lIydrallamois in tlJQI witli liydramnios:
a. there is more severe uterine and abdominaJ distention b. fetaJ deglutition or renal dysgenesis is the underlying cause c. adventitious pl acentation or placental dysfun ction is the underlying cause d. excessive fetal fluid is quickly replaced after percutaneous am niocentesis e. circulatory collapse is a major risk on cesarean section 66. A deji"itiue sign ofestrus in cows is:
a. b. c. d. e.
mucous vulvar discharge ruffled hair on the tail head a large follkle on the ovary vulvar swelling standing to be mounted
69. Prostaglandin Fa. induces estrus in cows Dilly
a. anestrus b. diestrus c. the immediate postpartum period d. metestrus e. proestrus 70. During dystocia in a Holstein heifer, the delay if! spomafleous deliueryofa large calfis least likely to be attributable to: b. c. d. e.
the horizontaJ d iameter of the bony pelvis incomplete dilation ofthe cervix a co nstricted vagina a constricted vestibulovaginal junction constricted labia
71. Euen practitioners experienced in bovine embryo
transfer Iwuedifficulty with:
a. achieving high fertilization rates b. achievi ng a consistent superovulatory response c. recovering at least 60% of the embryos d. cryopreservarion of quality embryos e. synchronizing the estrous cycles of donor and recipient cows
62. [n commercial esrablisl""eflts, embryos are
routinely recovered by nonsurgical methods. After tlleOllse! ofestrus ill tlledollor. tile embryos are recovered on day:
67. Teaser bulls witll surgically created lateral
deviation of the penis are least likely co transmit:
a.
a. II b. 7 c. 3 d.5 e. 9
b.
c. d. e.
63. WlIa! is tile shortest pllase of tile estrous cycle
campylobacteriosis leptospirosis infectious bovine rhinouacheitis tuberculosis bovine virus diarrhea
72. The most common cause of poor reproductive
performance in cows on large dairy farms is:
a. anestrus b. cystic fo llicles c. improper semen handling and poor artificial insemination technique d. poor estrus detection e. uterine infeC(ions
68. Wlla! is tile least developed accessory sex gland of
ofcows?
bulls?
a. estrus b. metestrus c. diestrus d. anestrus e. proestrus
a. prostate gland b. bulbourethral gland c. seminal vesicular gland d. ampulla e. body of the epididymis
10 1998 Mosby-Year Book, Inc. Photocopying is prohibited by law.
c. check the dam for trauma to the birth canal d. treat the calf's umbilical cord with tincture of iodine e. check all four teats o n the dam for normal flow of colostrum
wilen givell during:
•.
207
Tll eriogenology
73. After assisting a heifer in delivery of a live calf, tile
most appropriate next step is to:
a. examine the dam for another calf b. make sure the calf is breathing
74.
111ere is 110 commercially auailable vaccine for prevefl tiof! ofabortion caused by:
a. brucellosis b. bovine virus diarrhea C. infectious bovine rhinotracheitis d. listeriosis c. trichomoniasis 75. Postcoital pyometra
..
is a virtually patllognomolll'c
sigl/ ofil/fectioll with: Escherichia coli
b. Actillomyces pyogenes
c. Listeria monocytogenes d. Campylabacter fetus var. fetus c. Trirrichomonasferus 76. 011 rectal examination ofa grade Jersey cow that has been lactaringfor 280 days, you detect a
mummified fews. T1le most appropriate course of action is to:
a. infuse the uterus with mineraJ oil b. administer 50 IU ofoxylocin intramuscularly twice daily for 3 days c. administer 25 mg of prostaglandin F2"o intramuscularly d. send the cow to slaughter c. re move the fetus by hysterotomy 77. A cow in late gestation has a 360-degree uterine torsio". The most appropriate treatment is:
a. b. c. d. e.
cesarean section fetotomy rolling the cow (Schafer method) detorsio n with a detorsion rod uterine relaxant therapy (clenbuterol)
Correct answers are on pages 234-244.
206 78. There is an 80% chance that a cow is pregnant when progesterone levels in the milk are:
a. high at the time of artificial insemination and high 21 days later b. low at the time of artificial insemination and low 21 days later c. high at lhe time of artificial insemination and low 22 days later d. low at the time of artificial insemination and h igh 22 days later e. low at the time of artificial insemination and low 30 days later
79. What is the incitienceoffreemarrinism in heifer callies born as a twirl with a bull calf?
a.
25%
b. 40%
c. 55% d.70% e.85%
80. Scrotal circumference is an indicator of
a. b. c. d.
serving capacity of the bull fertility level of lhe bun early onset of puberty in female offspring libido a f the bull
e. temperature of the testicles 81. Wllat is the characteristic location ofrupture and hematoma ollhe bovine pen is?
a. in the glans penis b. dorsal to the attachment of the retractor penis muscles c. just proximal to the sigmoid flexu re d . in the perineal region , dorsal to the scrotum e. at the preputial reflection 82. Wllich disease causes abortion primarily from felal disease ralher than placental disease?
a. h. c. d. e.
www.vet4arab.co.cc
SECTION 9
brucellosis campylobacteriosis aspergillosis infectious bovine rhinotracheitis trichomoniasis
83. Wllat is tile most dependable indication of pregnancy 011 rectal examination ofa cow 75 days after artificial insemination?
a. b. c. d. e.
gross asymmetry of the uterine horns palpation of a fetus palpation of an amniotic vesicle fremitus in the uterine artery fluctuation in the uterine hom ipsilateral to the corpus luteum
84. To ensure maximal profit for tile producer, dairy
heifers sllould be bred:
a. to calve at 24 months of age b. at 24 momhs of age c. during the second or third heat after the onset of puberly d. to Angus bulls to minimize the incidence of dystocia e. no later than 9 months of age 85. Aggressive rectal palpation can result in abortion by:
a. causing one or more placentomes to hemorrhage b. slipping the chorioallantoic membrane c. damaging the amniotic vesicle d. traumatizing the young fetus e. triggering regression of the corpus luteum 86. In cows a dominant follicle:
a. b. c. d. e.
may fo rm on any day of the estrous cycle ovulates when the corpus luteum regresses interferes with successful superovulation is refractory to luteinizing hormone produces extra fo llicle-stimulating hormone
87. Wllal is tile longest pllase oftlte bovine esrrous cycle?
a. b. c. d. e.
proestrus metestrus estrus diestrus anestrus
C 1998 Mosby-Year Book, Inc. Photocopying is prohibited by law.
88. Tile corpus hemorrhagicum ofcows is:
a. b. c. d. e.
209
TlJeriogeno logy
small and of a soft consistency large and of a firm consistency small and of a firm consistency large and of a soft consistency larger than a corpus luteum
89. On any givell day, approximately how many heifers ill a group of 100 Ilormaliycyclillg 15month·old heifers are ill heat? a. 5 b. 15
c. 25 d.35
93. fertilization takes place in the:
a. b. c. d. e.
uterine body cervix oviduct tip of the uterine horn tubo-uterine junction
94. The adverse effecls ofsummer heat alld humidity Oil
fertility in dairy cattle are most deleterious:
a. b. c. d. e.
during the first week ofgestation at the time of fertilization during the second trimester of gestation during the last 2 weeks of gestation in 14- to 17-month-old heifers
e. 45 95. A cow witl! uterus unicorn is:
90. On reclal palpation the uterus of a cow pregnant for 2 or 3 months call bedifferemiacedfrom a uterus of comparable size in a cow Wifh pyometra by: a. slipping the chorioallantoic membrane b. asymmetry of the uterine horns c. the presence ofa large corpus luteum d. fluctuation of the larger uterine hom e. the absence of follicles or a corpus luteum on the ovaries 91. Schistosomus refle.:r.us is characterized by:
a. a split cranium and torticollis b. a duplicate caudal vertebral column, with bent rear legs c. cleft palate and a laterally deviated head d. an underdeveloped spine and severely contracted tendons e. a ventral abdominal defect and a ventrodorsally curved spine 92. Prolapse ofthe uterus is frequently associated with:
a. b. c. d. e.
hypocalcemia twin pregnancy an emphysematous fetus prolonged gestation hydrops allantois
a. b. c. d.
is sterile has long periods of anestrus also bas only one ovary has an insufficient number of carunc1es to sustain a pregnancy e. is likely to develop hydrallantois 96. I" dairy cows the onset ofestrus:
a. usually occurs between midnight and 6 AM b. usually occurs between 6 PM and midnight c. occurs at any time of day or night d. usually occurs between noon and 6 PM e. usually occurs between 6 AM and noon 97. llllhefirst postpartum insemination ofa cow, optimal fertility is achieved by depositing tile semen ill tile: a. cervix b. cranial vagina c. uterine horn contralateral to the graafian follicle d. uterine horn ipsilateral to the graafian follicle e. body of the uterus
Correct answers are on pages 234·244.
www.vet4arab.co.cc
SECTION 9
210
c. 8 months of gestation d. 6.5 months of gestation e. 5 months of gestation
98. TIle cliallce for pregnancy in rile near future is best for a cow:
a. carrying an emphysematous fetus b. a boning a 7-month-old fetus because of leptospirosis c. carryi ng a macerated fetus d. aborting a 6-month-old fetus because of
100. Mlat is the mQst appropriate specimen to examine to confirm suspected mycoricabortion ill a cow?
aspergillosis
a. b. c. d.
e. carrying a mummified fetus 99. Most mummified bovine fetuses Ilaue died at about:
fetal lung fetal abomasal contents cotyledon fetal spleen
e. fecal liver
a. 3.5 months of gestation h. 2 months of gestation
Theriogenology
105. III the Northern Hemisphere during the late Jaff, a mare may show signs ofestrus Jar as iong as 10 to 14 days when leased bya stallion. TIlis type of estrual behavior is:
a. prolonged but probably normal during the transition from cycling to anestrus b. normal for mares in the summ e r, but prolonged for mares in the fall c. prolonged and abnormal, and should be treated with 100 JJ.g of gonadotropinreleasing hormo ne d. prolonged and probably the result of cystic ovarian degeneration e. normal 106. Ovulation in mares occurs:
HORSES B.E. Eilts 101. Mares are best described as:
a. b. c. d.
nonseasonally monestrous seasonally monestrous nonseasonaUy polyestrous seasonally polyestrous c. nonseasonally induced ovulators
d. exposing the mares to 16 hours of light and 8 hours of darkness about 2 months beforc the estrous cycles are scheduled 10 commence e. exposing the mares to 8 hours of light and 16 hours of darkness a bout 2 months before the estrous cycles are scheduled to commence 104. Itl the Northern Hemisphere during the month of
102. In tile Northern Hemisphere. mares normally
have estrous cycles:
•.
in the spring
b. in the fall
c. in the winter d. at any time of the year
e. in the fall and winte r 103. The most effective method to induce estrous
cycles ill anestrous mares is by:
a. administering altrenogest atO.044 mg/ kg PO fo r 14 days b. administering 3000 IU of human chorionic go nadotropin 1M once c. adminis tcring 100 JJ.g of gonadotropinreleasing hormone 1M once
February. a mare may show signs of estrns (teasing, winkillg, squatting) for as long as 10 to 14 days when teased bya stallion. This duration ofestrual behavior is:
a. normal b. prolonged but probably normal during the transition from anestrus to cycling c. prolonged and abnormal and should be trea ted wilh 10,000 IU of human chorionic gonadotropin d. prolonged and proba bly the result of cystic ovarian degeneration e. normal for marcs in the s umme r but prolonged for mares in the spring or late winter
C 1998 Mosby-Year Book, Inc. Photocopying is prollibired by law.
a. b. c. d. e.
18 hours after the end of standing heat 2 days before the end of estrus on the first day of estrus the day after estrus ends shortly after coitus because the mare is an induced ovulator
107. To establish the approximate time ofoVlIlation by rectal palpation, a mare must be palpated:
a. 2 days after coitus because me mare is an induced ovulator b. every 48 hours c. every 72 hours d. every 12 hours e. I day after coitus because the mare is an induced ovulator J08. The drug oJcllOice to induceovuiation ill an
estrual mare with a 35-mm Jollicle on the ovary is:
a. progesterone h. prostaglandin c. equine chorionic gonadotropin (formerly pregnant mare serum gonadotropin) d. human chorionic gonadotropin c. estradiol
211
J09. In mares theovur1I:
a. enters the uterus within 20 minutes after fe rtilization b. enters the uterus about 5 days after ovulation c. undergoes a mandatory embryonic d ia pause before entering the uterus 14 days after ovulation d. enters the uterus abOlIl 10 days after ovulation e. remai ns in me oviduct until me pregnancy signal is received from the u lerus at 14 days after ovulation 1 JO. Mu ltipleovu/ario/lS in mares:
a. are extremely rare b. occur in about 75% of all mares and are generally dual ovulations c. are generally triple ovulations occurring 1 day apart d. are generally triple ovulations occurring within 1 102 hours e. occur in about 20% of mares and are generally dual ovulations For Questions III through 115, select the correct answer from the five choices below.
a. prostaglandin b. progesterone c. equine chorionic gonadotropin (formerly p regnant mare serum gonadotropin) d. estrogen e. human chorionic gonadotropin 111. Produced by the endometrium and ca uses lYSis
of tile corpus luteum 112. Produced by the corpus luteum
1J3. Produced by the endometrial cups
1/4. Has luteinizing ho rmone activity 115. Produced by the granulosa cells in theJollicle
Correct answers are on pages 234-244.
116. 71,e mobility pllase of the equine embryo lasts:
a. h. c. d. e.
I day 4 days 16days 20days 24 days
117. After the mobility phase. a s;ngfe equine embryo usually becomes attached:
a. ipsilateral to the corpus luteum to prevent the local countercurrent exchange of prostaglandin b. contralateral to the corpus ]uleum to be as
far away from the local countercurrent exchange of prostaglandin as possible c. o nly in the body of the uterus d. in either horn or the body of the uterus,
regardless or tile location ohile corpus
iuteum
e. only until the endometrial cups are formed J 18.
www.vet4arab.co.cc
SECTION 9
212
If twin embryos are diagnosed by ultrasollogmphyat /4 days after ovulation and are in tile same uterine horn, the most appropriate course ofactiotl is to:
a. move one of the embryos to the opposite uterine horn and crush it b. crush one of the embryos without moving either one c. wait until day 20 before moving one of the embryos to the opposite uterine horn and crushing it d. wait unlil day 20 and crush one of the embryos without moving either one c. reduce the feed intake of the mare so one of the embryos starves J 19. TI,e endometrial cups in a mare's uterus are of:
3. maternal origin and secrete equine chorionic gonadotropin b. maternal origin and secrete prostaglandin c. fetal origin and secrete prostaglandin d. fetal origin and secrete progesterone e. fetal origin and secrete equine chorionic gonadotropin
120. To maintain pregnancy in mares, the ovaries:
a. are needed throughout pregnancy b. are needed untiJ about 35 days of gestation, when the placenta begins producing progestogens c. are needed until about 120 days of gestation, when the placenta begins producing progestogens d. are needed until about 120 days of gestation, when the uterus begins producing progestogens e. are not needed after ovulation because the uterus produces progestogens throughout gestation 121. Rectal palpation of a mare ill diestrus
is most
likely to reveal:
a. b. c. d. e.
a fl accid uterus and a cervix with tone a uterus with tone and a flaccid cervix a flaccid uterus and a flaccid cervix a uterus with tone and a cervix with tone an edematous uterus and a flaccid cervix
122. Rectal palpation ofa mare in estrus is most
likely to reveal:
a. b. c. d. e.
a flaccid uterus and a cervix \vith tone a uterus with tone and a flaccid cervix a flaccid uterus and a fla ccid cervix a uterus with tone and a cervix with tone an edematous uterus and a cervix with tone
123. Qmcemingdetection of the corpus luteurn in a mare by a single rectal palpation, which sta/emelll is most accurate?
a. The corpus lute um is not readily identifiable by a single palpation. b. The corpus luteum is easily palpable, primarily because of the "crown" that extends beyond the ovarian surface. c. The corpus luteum is easily palpable, primarily because of the line of demarcation berween luteal tissue and ovarian stroma. d. The corpus luteum is palpable on the ovary contralateral to the ovulatory follicle. e. The corpus luteum is palpable, primarily because of the way it changes the shape of the ovary.
e 1998 Mosb~Year Book, Inc. Plzotocopying is prolzibited by law.
TheriogenoJogy
124. After parturition and placental expulsion in a
mare, YO" examine the placenta and find red, velvety tissue rhat comprises one wall of the placerua. Tllis red, velvety layer is the:
a. amnion b. uterine side of the alIantochorion c. fetal side of the allantochorion d. allantois e. umbilical stalk. 125. After parturitioll in mares, the placenta is
normally expelled within:
213
c. does not occur in mares because of the normalluteolytic process in nonpregnant mares d. occurs when the endometrial cups produce equine chorionic gonadotropin (pregnant mare serum gonadotropin) e. occurs when the mare retains the corpus luteum, does not return to estrus, and maintains tone in the uterus and cervix despite not being pregnant 129. In mares, stage /I of parturition normally lasts:
a. 30 minutes b. 8 hours c. 12 hours
a. 10 minutes b. 90 minutes c. 24 hours d. 48 hours e. 60 hours
d. 16 hours
e. 24 hours
J26. Aside from milk calcium levels, whaltliree
criteria are used to determine readiness for induction 0fparturition in mares?
a. at least 300 days of gestation. cervical relaxation, and correct positioning of the foal b. at least 300 days of gestation, cervical relaxation, and "waxing" of the teats c. at least 330 days of gestation, cervical relaxation, and "waxing" ofthe teats d. at least 310 days of gestation, correct pOSitioning of the foal, and ~waxing" of the teats e. at least 330 days of gestation, "waxing" of the teats, and correct positioning of the foal 127. The dmgofclloice to induce parturition in
mares is: a. prostaglandin Fz« b. oxytocin c. dexamethasone d. estradiol e. bromocriptine
130. To induce luteolysis in a mare, rile best metllod is to administer:
a. prostaglandin at least 5 days after the end of estrus b. prostaglandin 1 to 4 days after the end of estrus c. human chorionic gonadotropin at least 5 days after the end of estrus d. human chorionic gonadotropin 1 to 4 days after the end of estrus e. ahrenogest at least 5 days after the end of estrus 131. Superovulatio n in mares is:
a. best attained using follicle-stimulating hormone b. best attained using equine chorionic gonadotropin (formerly pregnant mare serum gonadotropin) c. best attained using altrenogest d. best attained using human chorionic gonadotropin e. not readily attainable
128. Pseudopregnancy ill mares:
a. occurs only when the mare retains the corpus luteum because of early embryonic death b. occurs when the corpus luteum persists for about the duration of gestation because the mare is not pregnant Correct answers are on pages 234-244.
214
www.vet4arab.co.cc
SECTION 9
132. Embryo collection for embryo transfer in mares is routinely performed: a. 3 days after ovulation by placing a catheter in the uterine body and flushing both horns simultaneously b. 7 days after ovulation by surgically flushing the oviduct ipsilateral to the ovulatory follicle c. 7 days after ovulation by placing a catheter in the Ulerine body and flushing hoth horns
simultaneously d. 7 days after the end of estrus by placing a catheter in the uterine body and flushing both horns simultaneously e. 14 days after the end of estrus by placing a catheter in the ulerine body and flush ing both horns simultaneously 133. In trails/erring an equine embryo into a recipient mare, it is 1Iecessary to:
a. place the embryo into the uterine body h. place the embryo as far as possible into the horn ipsilateral to the ovulating ovary c. place the embryo as far as possible into the horn contralateral to the ovulating ovary d. surgically implant the embryo in the horn ipsilateral to the ovulating ovary e. surgically implant the embryo in the hom contralateral to the ovulating ovary 134. Ifl mares. unfertilized oua are:
a. commonly found in the uterine hom ipsilateral to the ovulating ovary b. commonly found in the uterine horn contralateral to the ovulating ovary c. expelled from the cervix I to 2 days after ovulation d. retained in the uterus for months e. retained for months in the oviduct ipsilateral to the ovulating ovary
d. 38 to 40 days e. 55 to 60 days 136. During natural breedillg.lhe staffion ejaculates
into tlte mare's: a. b. c. d. e.
vestibule caudal vagina cranial vagina oviduct uterus
137. 111ecorrect method to artificially inseminate a
mare is to: a. place one hand in the rectum, insert the insemination pipette into the cranial vagina, and deposit the semen there b. place one hand in the rectum, insert the insemination pipette into the cervix, and deposit the semen there c. place one hand in the rectum, insert the insemination pipette into the uterine body, and deposit the semen there d. place one hand in the vagina, carry the insemination pipette into the cranial vagina. and deposit the semen there e. place one hand in the vagina, insert the inseminatjon pipette through the cervix, and deposit the semen in the uterus 138. Mlat is the minimum dose of normal, motile
sperm cells {or successful artificial insemilUltiOIl ofa mare?
a. 500 million b. 1 billion c. 10 billion d. 50 billion e. 100 billion 139. Stallion spermatozoa cafl survive ifl the mare's
135. After parturition, mares normally resume fertile
estrous cycles within approximately: a. 5 to 10 days b. 18 to 21 days c. 26 to 30 days
@
utermfor up to: a. b. c. d. e.
20 minutes 2 hours 1 day 2 days Sdays
1998 Mosby-Year Book. Inc. Photocopying is prohibited by law.
140. 711e stallion's penis is best c11aracterized as a:
a. b. c. d. e.
215
Tlleriogenology
high-volume. low-pressure erectile system high-volume, high-pressure erectile system low-volume. low-pressure erectile system low-volume. high-pressure erectile system high-volume. high-pressure fibroe lastic system
145. Tlze stallion's peflis is best described as:
a. b. c. d. e.
musculocavernous fibroelastic fibrocavernous musculoelastic musculofibrous
146. Spermatogenesis in stallioTlS requires: 141. What are tile primary stimuli that cause a
stallion to ejaculate?
a. Sight and smell b. s mell and temperature c. temperature and sight d. pressure and smell e. pressure and temperature
147. As compared with ejaculates during periods of long day length (16 hOllrs light. 8 hours dark).
142. In tile stallion testis. the ifllerstitial cells of
Leydig produce primarily: a. b. c. d. e.
follicle-stimulating honnone luteinizing hormone estradiol testosterone progesterone
during periods o{shon day length (8 hours light, 16 hours dark) the ejaculate of stallioTlS lias: a. b. c. d. e.
143. YOIl palpate the scrotum and testes ofa stallion
with colic to determine i{there is torsion of the spermatic cord. The normal oriefltatiOIl of the epididymis is witll the: a. b. c. d. e.
a. 12 days b. 24 days c. 55 days d. 75 days e. 100 days
head cranial. tail caudal, and body ventral head caudal. tail cranial. and body ventral head caudal, tail cranial. and body dorsal bead cranial, tail caudal, and body dorsal head cf<mial, tail caudal, and body lateral
144. TlleSertoli cells o/the testis are thollght to
more volume and more cells more volume and fewer cells less volume and fewer cells less volume and more cells an equal volume and equal number of cells
148. Concerning the accessory sex glaruls of stallioTlS.
wlzich statemellt is most accurate? a. The stallion has bulbourethral and vesicular glands but not a prostate gland. b. The stallion has a prostate gland but no bulbourerluai or vesicular glands. c. The stallion has prostate and bulbourethral glands but no vesicular glands. d. The stallion has prostate, bulbourethral, and vesicular glands. e. The stallion has no accessory sex glands.
produce: a. b. c_ d. e.
testosterone estradiol progesterone follicle-stimu lating hormone luteinizing hormone
149. Daily sperm output can be accurately
determifled in a stallion by collecting and analyzing semen once daily for at least: a. b. c. d. e.
3 days 7 days 21 days 28days 35days
Correct answers are on pages 234-244.
216
www.vet4arab.co.cc
SECTION 9
150. You collect semen [rom a stallion once and then again J IlOllr later. As compared with thefirsc ejaculate, the second ejaculate is likely to have:
c. the same volume and twice the number of spermatozoa d. half the volume and half the number of
spermatozoa e. the same volume and the same number of spermatozoa
a. half the volume and the same number of spermatozoa b. the same volume and ha1f as many spermatozoa
P IGS R.C. Tubbs 151. What is the most importam [acror influencing tile time ofollset ofpuberty ill gilts?
a. environment b. housing c. nutrition at the time of expected onset of puberty d. transport e. exposure to a mature boar near the time of expected onset of puberty J52. Amplitude-depth (A-mode or pulse-echo)
ultrasonography for pregnancy detection in sows:
a. is 100% accu rate between days 30 and 75 of gestation b. detects a fluid-filled object and emits an audible signal, visible light, or both c. converts reflected uhrasonic beams into twodimensional images of the interior cross section of the animal d. is o nly applicable after calcification of fetal skeletons e. detects fetal heart movements and movement of fluid lhro ugh umbilical vessels or uterine arteries 153. Lactation lengths of less than 21 days in sows have been associated with:
a. cystic ovaries in 25% of affected sows b. more rapid return to estrus and improved reproductive performance c. persistent anestrus in up to 40% ofsows d. increased litter size and weaning weights in the next liuer e. prolonged weaning-to-service intervals and decreased size of the next litter
154. Mummifiedfetuses:
a. are usually caused by in utero viral infectio ns that occur after day 35 a nd before day 70 of gestation b. are associated wllh a speCific n utritional defiCiency c. occur primarily in litters from sows with more than five parhies d. occur because of prolonged duration of farrowing e. may result from a high level of carbon monoxide in lhe farrowing house 155. Tile regimen for inducing parturition in sows iTivolves useot
a. 20 mg of dexamethasone on day 112 of gestation; parturition occurs in 2 days b. 20 mg of dexamethasone on day 112, followed by 20 IV of oxytocin on day 113; parturition occurs in 24 hours c. 10 mg of prostaglandin F2Q on day 112, fo Uowed by 5 IU of oxytocin 24 hours la ter; parturition begins 24 to 30 hours after prostaglandin injection d. 20 mg of estradiol cypionate on day 112, followed by 20 IU of oxytocin on day 113; parturition occurs on day 114 e. a combination of equine chorionic gonadotropin (pregnant mare serum gonadotropin) and human chorionic gonadotropin on day 113 of gestation; parturition occurs 24 hours later
C 1998 Mosby-Year Book, Inc. Photocopying is pro/libited by law.
156. Stiffbirths in swine:
a. usually are caused by physical abnormalities ofthe sow's reproductive tract b. are primarily caused by infectious agents that kill the fetuses before panurition begins c. are difficult to differentiate from early postpartum dealhs d. are primarily from noninfectious causes, and primarily occur during farrowing e. occur more often in middle-parity sows (three to five parities) lhan in older sows (more than six parities) 157. COllcerning abortion ill swine, which statemem is most accllrate? u. The cause is easily di agnosed by serologic
b. c. d. e.
217
Therlogeno logy
exam ination of m aternal serum. About 60% to 70% of a bortions have noninfectious causes. About 60% to 70% of abortions have infectious causes. The cause is almost never diagnosed. Abortions occur in about 5% of all p regnancies in most swine herds.
158. Seasonal infertility in swine:
a. is primarily caused by heat stress b. occurs primarily after June to September matings c. affects older sows more severely than younger sows d. is less severe in outdoor breeding herds than in indoor breeding herds e. is less severe when sows are housed in groups rather than in crates 159. Lordosis ill swine is characterized by:
a. rolling, high-pitched squealing, and frequent defecation b. frequent urination, squatting, and polydipsia c. erect hair, frequent rubbing, and rooting d. swelling and reddening of the vulva, immobile stance, arched back, and erect ears e. a cool nose, straight tail, and frequent sweating
160. What is the most accurate method to detect pregnancy in swine?
a. measurement of estrone sulfate levels in maternal serum 25 to 30 days after breeding b. using a mature boar and an experienced breeding manager to check for estrus 18 to 24 days after breeding c. measurement of progesterone levels in maternal serum 17 to 20 days after breeding d. measurement of prostaglandin Flo levels in maternal seru m 12 to 15 days after breed ing e. amplitude-depth ultrasonographic examination 30 to 45 days after breeding 161. 71Je normal sequence of endocrinologic ellellts
tllat occurs at tile onset of puberty in gilts is:
a. increased pulsatile release of gonadotropins leads to progressive development of ovarian follicles. increased levels of estradiol-I 7[3, the preovulatory luteinizing hormone surge, and ovulation b. increased pulsatile release of progesterone causes release of prostaglandin Flo. from the uterine endometrium, stimuJating lysis and ovulation c. high levels of luteinizing hormone cause pulsatile release of oxytocin, leading to ovulation d. high levels of progesterone cause release of relaxin, which triggers lhe preovulatory luteinizing hormone surge and ovulation e. cystic ovarian follicles are sensitive to the effects of pulsati le releases of gonadotropin. resulting in ovulation 162. What is the optimum daily caloric iTJIake (megacalories of metabolizable energy per day) thal results in the shortest mean weanillg-toservice interval?
a. b. c. d.
20
16 12
6
e. not known
Correct answers are on pages 234-244.
www.vet4arab.co.cc
SECTION 9
218
163. Altering tile suckling pattern aftlle litter by weaning IIa/flitters 2 days early decreases the weaning- fo-service intenlDf:
a. in high-producing sows but not in lowproducing sows b. in sows of some breeds but not in others c. in any sow under all circumstances d. only whe n the weaning-to-service interval is already prolonged e. whe n the weaning-ta-service interval is normal 164. Optimal fertility is attained wlien theftrsl postweallillg service occllrs:
a. b. c. d.
12 to 14 days after weaning 7 to 10 days after weaning 3 to 6 days after weaning 3 to 6 days after weaning in summer months and 6 to 10 days after weaning in winter months e. at any time; weaning-ta-service interval has no influence on fertility
/ 65. Norma l mating bellavior ill swine is controlled by pituiraryand o/Jarian hormones. During normal lactation, serum levels offolliclestimulating hormone, luteinizing honnone, and progesterone are low. TIle suckling reflex blocks lllleinizing 1I0rmone secretion, and inhibin blocks fofficle-stimllfating hormone secretion. What sequence ofevents normally occurs on remollal of tile sucklillg reflex and removal of negative f eedback by illliibill ?
a. A 30-day postlactational anestrus occurs in most sows. b. Incrensed pulsat ile secretion of luteinizing hormone, increased follicle -stimulating hormone levels, and increased follicular development lead to production of ovarian estrogens, the preovulatory luteinizing hormone surge, and onset of estrus within 5 to 15 days in most sows. c. Increased pulsatile secretion of progesterone and increased ovarian estrogens inhibit secretion offollicle-stimulating hormone and luteinizing hormone by the hypothalamohypophyseal-ovarian axis and prevent estrus in most sows fo r a minimum of 15 days.
d. Increased pulsatile secretion of ovarian estrogens stimulates the preovulatory progesterone surge and onset of estrus within 5 to 15 days in most sows. e. Increased pulsatile secretion of adrenal steroids is followed by ovarian hypertrophy, estrogen and progesterone secretion, increased prostaglandin F:z., levels, estrus, and ovulation. 166. A producer wil/I a 100-sow farrow-to-feeder pig operation caflsyou because hefound some aborted fetuses ill tile gestation unit 011 hisfarm. YOIl measure tile crowll -to-rump lellgth of the fetuses and determine that they are about 45 days'gestatiollal age. W1lich sample would be most IIseful in determining the cause ofabortion when submitted to a diagllostic laboratory? a. b. c. d.
maternal serum feed fetal flu ids and tissue culture swabs from the sow's reproductive tract e. seme n samples from boars 167. In fa te summerulId early fall, tllere is:
a. improved re productive performance because of decreased stress during lactation b. a decreased number of stillborn pigs and mummified fetuses c. a decreased number of sows that remain anestrous for 30 days or longer d. a decreased percemage of sows that return to estrus within 7 days of weaning e. increased expression of libido by mature boars but not by immature boars 168. In sows, ovulation occurs:
a. at the time estrus begins b. 12 hours after estrus ends c. 36 to 40 hours after the onset of standing estrus d. 6 hours befote estrus begins e. 18 to 24 hours after the onset of standing estrus
10 1998 Mosby-Year Book, Inc. Pholocopying is prohibited by law.
169. Prostaglandins are not usedforestrus synchronization in swine because:
a. prostaglandins are too expensive b. the porcine corpus luteum is unresponsive to the luteolytic effects of single doses of prostaglandins until day 12 of the cycle, and luteolysis begins normally on day 15 of the cycle; therefore the time fram e when it is effective is too narrow to be of practical use c. the porcine corpus luteum is not affected by prostaglandins d . pigs have 12-day cycles. so synchronization is not needed e. the porcine corpus luteum secretes high levels of estrogens, which negate the effects of prostaglandins 170. Wllat is tile normal sequence ofendocrinologic events tllat occur on days 13 to 15 ofa normal (nonpregna nt) estrous cycle;lI sows?
a. Levels of prostaglandin secreted by the uterine endomenium increase, causing lysis of the corpus luteum; ovarian progesterone levels decrease; luteinizing hormone and fo llicle-stimulating hormone increase; estrus and ovulation follow 5 to 8 days later. b. Progesterone increases, estrogen decreases. and ovulation occurs. c, Progesterone secreted by the ovary decreases prostaglandin levels, causing uterine involution, estrus, and ovulation. d. Progesterone causes the uterine endometrium to contract, releasing prostaglandin, which stimuJates oxytocin release and the onset of estrus. e. Follicular estrogens stimulate lysis of the corp us lute um and release of prostaglandins from the ovary, which causes the onset of estrus and ovulation. 171. Pregnancy is signaled by maternal recogllition of fetal:
a. prostaglandin b. estrogen c. progesterone
219
Th erlogenology
d. testosterone e. adre nocorticoids 172. For pregnancy to be maintained, the minimum number ofembryos necessary iutlle uterus all day 12 after matillg is:
a. b.3
c. 4 d. 6
e. B J 73. Wlljcll set of boar sem en characteristics is
lIormal?
a. volume= 100 ml; morphology = 87% abnormal; concentration = 2.2 million sperm cells/ mi b. volume = 50 mI ; morphology = 9% abnormal; co ncentration = 26 million sperm cells/ Illl c. volume = 100 mI; morphology = 3% abnormal; concentration = 13.8 million sperm cells/ mJ d. volume =100 mI: morphology = 3 1% abnormal; concentration = 3.3 million sperm ce ll s/ mI
e. volume = 250 mI; morphology = 12% abnormal ; concentration::: 55 million sperm cells/ml 174. W1lat is the theoretic adlJantage ofmating sows two or more times durillgone estrous period, as opposed to breedillg a single time during an estrous period?
a. The increased physical contact induces more ovulations. b. More matings tend to settle sows, so they are less likely to fight and disrupt pregnancy. c. The likelihood of mating at the optimum lime relative to ovulation is increased. d. There are no advantages to mating twice versus mating once. e. More matings keep boars in better physical condition, with bette r semen quality.
Correct answers are on pages 234-244.
www.vet4arab.co.cc
SECTION 9
220
175. The most common recommendatiofls[or using mature (older than 12 months) and immature boars (younger than 12 months) are to: 3. use mature boars for 5 to 7 matings per week; use immature boars for 3 to 5 matings per week
b. use mature boars fo r 3 to 5 matings per week; use immature boars for 1 or 2 matings per week
c. lise mature boars for 10 to 12 matings per week; use immature boars for 12 to 14 matings per week d. use mature boars for 1 or 2 matings per week; use immature boars for 1 or 2 matings per
month c. use mature boars without restriction, since they cannot be overused; use immature boars for 6 matings per week
176. Mummifiedfetuses usually indicate that infection occurred at which stage a/pregnancy and witl! which agelll?
a. before day 35 with Leptospira b. before day 35 with pseudorabies virus c. after day 35 with ActinobacjlJus pleuropneumoniae d. after day 35 with parvovirus e. after day 75 with the agent of porcine reproductive and respiratory syndrome 177. What approximate percentage ofmalllre
(multiparous) sows are normally in estrus within 7 days after weaningfollowingan approximate 21-day lactation? a. 75% to 80% b. 65% to 70% c. 35% to 40% d.l00% e. 85% to 90% 178. Sexual maturity ofboars is indicated by age and:
a. b. c. d. e.
testicular circumference carcass composition carcass length testicular weight back-fat level
179. An owner of a 450-sow feeder pig herd calls yort in January because his farrowing rate has fallen to 68% and preweaning mortality has risen from 16% to 20% in the past 2 months. During the initial visit you observe that tile next group of pigs due to be weaned has 15% mortality (normal for the herd is 15%) and that the farrowing rate in the next 2 groups is projected at 78% (normal for the herd is 80%). W1Jat is the most likelycallse of the problem?
a. porcine reproductive and respiratory syndrome b. pseudorabies c. seasonal fluctuations in fertil ity and preweaning mortality d. transmissible gastroenteritis e. subclinical infections with multiple pathogens 180. Infectious late-term abortions in swine herds are usually associated with infection by:
a. b. c. d. e.
parvovirus Streptococcus species encephalomyocarditis virus Leptospira species pseudorabies virus
181 . A producer has an average swine herd. One group of breeding-age gilts is seronegativefor porcine parvollirus. With regard to the seronegative gilts, what is the most appropriate advice for tile producer?
a. Because these gilts have not been ex-posed to the virus, they are unlikely to show clinical signs of the disease. b. Because these gilts a re unprotected, they should be exposed [Q the field virus or vaccinated before breeding, to protect the fetuses from parvovira! infection. c. These gilts should be bred only to seronegative boars. d. These gilts should be bred only to seropositive boars. e. These gilts should be culled.
© 1998 Mosby-Year Book, Inc. Photocopying is prohibited by law.
Theriogenology
J 82. What is the most common cause ofdystocia in sows?
a . eclampsia
186. What is the best biologic measurement of sow herd efficiency?
a. b. c. d. e.
b. cranial fetal presentation
c. caudal fetal presentation d. breech fetal presentation e. uterine inertia 183. A producer calls you because a group of4-monthold gilts on the finishingfioor have red, swollen vulvas and developing mammary glands. What is the most likely cause of these sigru?
a. The p roducer has developed a new line of early-maturing gilts. b. exposure to zearalenone c. exposure to aflatoxin d. exposure to a mature boar e. pseudopregnancy 184. By definition, "nonproductive days" are days on which sows:
a. b. c. d.
do not eat are not in the farrowing house are not gestaling or lactating are being held to ~dry up" after lactation before they are sold as culls e. are in the breeding area but not in estrus 185. Development oflean replacement breeding
females that reach puberty at a young age has been associated with:
a. small litter s ize in gilt litters, reduced secondlitter size, prolonged weaning-to-estrus intervals after the first lactation, and high subsequent culling rates b. large litter size a nd superior rebreeding perfonnance c. higher lifetime productivity d. bigger appetites, higher feed intake, and better milking performance e. higher prevalence of diarrhea and slow growth of offspring
221
pigs weaned per farrowing crate per year pigs weaned per inventoried sow per year cost to produce a weaned pig feed cost per pound of pork produced feed cost per ton of sow feed
187. W11ich Leptospira serovar is most commonly identified in serologic surveys of swine in the midwestern United States?
a. Lpomona b. L bratislava c. L IIardjo d. L icterohaemorrhagiae e. L. canicola 188. W1Jat are tlJe most common infectious reproductive disorders in swine herds?
a. porcine reproductive and respiratory syndrome, eperythrozoonosis, and erysipelas b. toxoplasmosis, streptococcal infection, and staphylococcal infection c. brucellosis, pseudorabies, and Eubacterium sllis infection d. leptospirosis, parvoviral infection, and pseudorabies e. leptospirosis, parvovirai infection, and campylobacteriosis 189. What is the normal duration of ejaculation in boars?
a. 18 seconds b. I to 2 minutes c.
4 to 6 minutes
d. 10 minutes
e. 11.5 minutes
Correct answers are on pages 234-244.
www.vet4arab.co.cc
SECTION 9
222
190. The primary purpose of medicated early weaning and modified medicated early weaning technologies is to:
a. h. c. d. e.
improve sow reproductive performance produce weaned pigs free of specific diseases eliminate all disease from a swine herd decrease the cost to produce a weaned pig decrease feed costs associated with producing a weaned pig through increased use of medication
191. A client witll 300 sows historically has low farrowing rates in December. A review afhis records
reveals that December farrowing rates for {he past 5 years ranged from 5 7% to 68%. He has 30 farrowing crates available per sow group. To ensure that farrowiflg crates are kept fILII in December, IlOw many sows must be mtlted per group? a. 53 sows in July matings
b. c. d. e.
53 sows in August matings 30 sows in all groups 44 sows in August matings 44 sows in September matings
192. The head of the epididymis in boars is located:
a. b. c. d. e.
at the dorsal pole of the testis at the ventral pole of the testis craniomedial to the testis within the parenchyma of the testis within the inguinal canal
193. The boar penis is:
a. hemodynamic b. cartilaginous, with a flared glans peniS c. fibroelastic, with the distal end coiled in a counterclockwise direction d. about 20 cm long and 5 cm in diameter e. retractable even though lacking a sigmoid flexure 194. What is theduratiOll of spermatogenesis in boars?
a. b. c. d. e.
20 days 18 days 45 days 35 days 60 days
195. In what part afthe sow's reproductive tract does fertilization occur? a. uterus b. vestibule c. uterotubular junction d. isthmus e. oviduct
Theriogenology
199. What is the typical sow culling rate in wellmanaged, intensive confinement swine production units?
200. What is the mean duration of second-stage labor and the average interval between individual births, respectively, during normal parturition in sows?
a. 10% to 11% b. 15% to 18%
a. b. c. d. e.
c. 20% to 30% d.40%t055%
e. 65% to 80% 196. What is the most common physical injury to the reproduClive tract of boars?
a. b. c. d. e.
testicular frostbite laceration of the distal end of the penis testicular trauma frostbite of the distal end of the penis laceration of the prepuce
a. b. c. d. e.
placental neonatal embryonic pre-attachment fetal
198. A cliem calls you about an award-winninggilt
Ile recently bought at the state fair. The gilt farrowed yesterday and only had five pigs. He wants to know what to do with tile animal after the litter is weaned. Your advice is to:
a. cull the animal after weaning, because litter size is unlikely to increase in subsequent matings b. breed the animal after weaning and hope for a larger litter, because there is little relationship between first-litter size and subsequent litter sizes c. use gonadotropin-releasing hormone to superovulate before the next breeding d. cross-foster the pigs to another sow and sell the gilt immediately e. perform a full serologiC profile and uterine culture and breed the gilt after weaning if the results of the diagnostic tests are negative
© 1998 Mosby-Year Book, Inc. Pllotocopying is prohibited by law.
2 to 3 hours; 15 to 16 minutes 6 to 8 hours; 8 to 10 minutes 2 to 3 hours; 20 to 35 minutes 2 to 6 hours; 35 to 40 minutes I hour; 10 minutes
SHEEP S. Mobini 201. How long is the estrous cycle ofsheep?
a. 16 days 197. During which developmental period is the porcine fetus most susceptible to the effects of teratogenic agents?
223
b. 19 days c. 21 days d. 28 days e. It varies because sheep are seasonal breeders. 202. Ewes are seasonally polyestrous. In the Northern Hemisphere they normally cycle:
a. b. c. d. e.
in the fall in the spring in the winter in the fall and winter the year around
progesterone estrogen luteinizing hormone follicle-stimulating hormone testosterone
a. b. c. d. e.
cotyledonary discoid zonary diffuse chorioallantoic
a. b. c. d. e.
toxoplasmosis chlamydiosis Q fever campylobacteriosis brucellosis
207. Which method is most widely usedfor estrus synchronization in ewes during the breeding season?
204. Administration of prostaglandin F2tJ. induces estrus in ewes only when given:
a. outside the breeding season b. during the breeding season
205. What type of placentation do sheep have?
206. What is the most significant cause ofabortion in sheep in the United States?
203. What is the predominant hormone during diestrus?
a. h. c. d. e.
c. in the summer d. during estrus e. immediately after lambing
a. introduction of a ram to the flock b. prostaglandin F20. administration c. use of progestogen-impregnated vaginal pessaries d. use of controlled intravaginal drug-releasing devices (CIDRs) e. adding progestogens to feed
Correct answers are on pages 234-244.
224
www.vet4arab.co.cc
SECTION 9
208. What is the most common and predictable way to induce and synchroni2.e estrus in mature ewes
duringanesrrus? 3.
prostaglandin Fla administration
b. use of progestogen-impregnated vaginal pessaries for 12 to 14 days c. artificial lighting programs d. progestogen administration for 12 to 14 days, followed by 400 to 500 IU of pregnant mare
serum gonadotropin e. melatonin adm inistration 209. What is the normal length afgestation in ewes?
a. 58 to 62 days b. 14510155 days c. 278 to 290 days
a. mucous discharge from the vulva b. vulvar swelling c. bleating and tail wagging d. "winking" of the vul va
e. standing to be mounted 214. Examination ofa ewe with dystocia reveals a live fetlls in anterior presefltotioll, with one forelimb retained within the birth canal. This [erus is most appropriately delivered by:
a. cesarean section b. pubic symphysiotomy of the dam c. felolo my d. judicious traction on the head and exposed forelimb e. repulsion, followed by repositioning of the retained forelimb
d. 11 2 toi l Sdays
e. 335 to 345 days 210. TIle optimum time for insemination ofa ewe is:
a. 9 hours after the onset of estrus, to the end of estrus b. 15 to 30 hours after the onset of estrus c. 12 10 IS hours after the onset of estrus d. every other day, beginning on the second day of eslrus e. 6 to 8 hours after the end of estrus 211. What is the average volume of semen ejaculated by rams?
a. I to 1.5 ml b. 5 to 10 rnl c. 30to 100 mi d. 150 to 300 ml e. 300 to 400 ml 2 J2. A major consideration in an artifICial insemination program is semen collection. While still maintaining adequate spenn concentrations, semen can be collected from rams:
a. b. c. d. e.
213. What is a sign ofestrus in ewes?
3 times daily, 2 days per week fo r 1 week once daily, every other day for 2 weeks many times per day for severa) weeks many times per day for I week twice daily, 3 days per week fo r 6 weeks
@
215. TIle penis oftlJe ram is most accurately
described as:
a. b. c. d. e.
fibroelastic, with a corkscrew shape fibroelastic, with a filiform process fibroelastic, with an upturned tip musculocavernous, with a bell-shaped glans muscular, with a flared glans
216. What is tile major cause ofmortality in neonatal lambs?
a. b. c. d. e.
hypothermia and hypoglycemia birth inj ury infec tions predation congenital defects
217. What is tile most common defect of the penis and prepuce oframs?
a. b. c. d. e.
TlleriogerlOlogy
218. What is tlTe major cause ofepididymitis in rams?
a. b. c. d. e.
brucellosis actinomycosis campylobacteriosis chlamydiosis salmoneUosis
225
222. In ewes, parturition can be induced in tlTe last few days ofpregnancy by administration ot
a. dexamethasone b. prostaglandin Fz.:. c. p rogesterone d. ovariectomy e. oxytocin
219. COllceming artificial insemination ofewes,
which statemem is most accurate?
a. It is simple because the structure of the cervix alIO\'IS easy passage of the pipette. b. Correct p laceme nt of the piperte is very d ifficult because of the structure of the cervix. c. It is complicated because a spiral p ipette must be carefully manipulated through the cervix. d. It is easily done transvaginally using a speCUlum. e. It is diffic ult but can be learned quickly by owners and handlers 220. W.hen embryos are surgically recovered from
donor ewes, on which day after ovulatioTl should embryos be recovered?
a. b. c. d. e.
day 1 day3 day6 day9 day II
a. toxoplasmosis b. brucellosis c. Akabane virus disease d. border disease e. enzootic abortion of ewes 224. Rams are evaluated for breeding sOlllldness by:
a. a thorough physical examination, \vith no other examin ations or tests b. semen analysis, with no other examinations or tests c. a thorough physical examination, with emphasis on the reproductive organs, and seme n analysis d. measurement of scrotal circumference, with no other examinations or tests e. test matings 225. Scrotal circumference in rams is directly correlated witli:
221. What is the most accurate means ofdetecting early pregnancy ill ewes?
a. b. c. d. e.
223. ArtlJrogryposis and hydranencepltaly in fetal lambs are caused by:
Doppler ultrasonography radiography palpation of the abdomen B-mode ultrasonography serum progesterone determination
a. b. c. d. e.
economic value libido testicular texture serving capacity sperm production
spiral deviation of the penis ventral deviation of the penis corpus cavernosal shunt failure of the retractor penis muscle to relax balanoposthitis
J 998 Mosby-Year Book, Inc. Pliotocopying is prollibited by law.
Correct answers are 011 pages 234·244.
226
www.vet4arab.co.cc
SECTION 9
233. To optimize pregnancy rates in caprine embryo transfer, how many embryos must be transferred to the recipient?
GOATS W.F. Braun, Jr. 226. Wlwt is the best method to detect esrrus in the doe?
a. observing standing and mounting behavior among does b. observing a reddened, swollen vulva, with vaginal mucus secretion c. use of a teaser or breeding buck d. use of a "huck rag" impregnated with the
a. Chlamydia psirtaci. Brucella melitensis. and Campylabaeter jejun i b. Chlamydia psircaci, Coxiella burnetii, and
Brucella melitcnsis c. Ccunpyfobacrer jejuni, Coxiella burnerii, and
scent of a buck
c. serum progesterone assay 227. Assays of wlliell of lhe following is not used for pregnancy detection in the doe?
a. pregnancy-specific protein B b. chorionic gonadotropin c. estrone sulfate d. placental lactogen e. progesterone 228. The microscopic structure of the cnprine placenta is classified as:
a. b. c. d. e.
230. m/iell organisms are the most common infectious causes ofcaprine abortion in the United States?
hemochorial syndesmochorial endotheliochorial epitheiiochorial discoidal
Toxoplasma gOlldi;
d. C/Ilamydia psittaci, Toxoplasma gondU, and Coxiella bumeti;
c. Cflfamydia psirraci, Campylabaeter jejuni, and Brucella abortlls 23 1. What is tile SOl/rce ofprogesterolle that maifltains pregnancy in goats?
a. corpus luteum until day 55 of gestation, then the placenta b. primary corpus luteum , roUowed by accessory corpora lutea, then the placenta c. placenta only d. placenta and adre nal glands e. corpus luteum only 232. Omcemingabortion induced with prostaglandin administration, which statement is most accurate?
229. Artificial insemination ill the doe is commonly performed by:
a. transcervical insemination using a vaginal speculum b. transcervical insemination via rectal manipulation c. transabdominal insemination via a laparoscope d. cranial vaginal deposition of semen e. transabdominal insemination using a syri nge and 4-inch needle
a. It can be performed anytime during gestation. b. Prostaglandin administratio n does not induce abortion in goats. c. It is usually performed between day 7 and the end of gestation. d. It is usually performed after day 55 of gestation. e. It is usually performed from days 1 to 35 of gestation.
C 1998 Mosby-Year Book, Inc. Photocopying is prollibited by law.
a. b. c. d. e.
one two three four five
240. Retained fetal membranes (placenta) in goats are associated with all the following conditions except:
a, b. c. d, e.
For Questions 234 through 238, select the correct answer from the five choices below. a. b. c. d. e.
227
Therlogenology
habitual abortion campylobacteriosis toxoplasmosis listeriosis chlamydiosis
234. Abortions caused by this disease may be diagnosed on the farm by the presence of macroscopic, calcified foci of necrosis jn the p/acemal cotyledons. 235. Abortions caused by this disease may be preceded by reduced appetite and milk production and signs ofsepticemia. 236. Abortions caused by this disease are rare in the United States; the organism is ingested and about one third of infected fetuses have grossly visible fiver necrosis. 237. This is the most common cause of infectious caprineabortion ill the United States.
selenium deficiency dystocia mannosidos is abortion metritis
241. Pregnancy ketosis in the doe is the result ofan energy deficiency during ltlte gestation. although the exact etiology is unknown, which of the following does "ot predispose to the condition?
a. b. c. d. e.
mu ltiple fetuses fatty liver overconditioned doe suboptimal feed intake high dietary energy levels in early lactation
242. Pseudopregnancy is commol! in goats and is characterized bya persistent corpus [uteum, anestrus, and a variableaccumultltiOIl offluid ill tile uterus. Concerning pseudopregnancy ill goats, which statemem is least accurate?
a. Does may develop an enlarged udder and abdomen. b. The treatment of choice is a luteolytic dose of prostaglandin. c. Diagnosis is based on ultrasonographic examination of the uterus. d. Diagnosis is based on radiographic examination of the uterus. e. Does may deliver a "cloudburst" of fluid at the end of a period equal to normal gestation.
238. This is most often seen in older Angora does. 239. W1lat is the most common cause ofdystocia in the doe?
a. fetomaternal disproportion b. failure of the cervix to dilate c. attempted expulsion of more than one fetus at the same time d . posterior presentation of the fetus e. uterine inertia
243. W1,at is the normal duration ofgestation in goats?
a. b. c. d.
3 months 4 months 5 months 6months e. 7 months
Correct answers are on pages 234-244.
www.vet4arab.co.cc
SECTION 9
228
244. Concerning urolithiasis (urinary calculi) in male goats. which statemelll is least accurate?
a. It is an important problem in Pygmy goat wethers. b. Management factors associated with the condition are prepubertal castration, highconcentrate diets, and insufficient water intake during very cold weather. c. Common sites of obstruction are the base of the urethral process and the distal bend of the sigmoid flexure. d. Catheterization of the urinary bladder is an imponant pan of therapy. e. Ultrasonography may be used to confirm the presence of bladder lesions and calculi in the urethra. 245. During the hreeding season, ovulation in does
occurs: a. 18 hours after the end of standing heat h. 12 to 36 hours after the onset of standing heat
c. 24 hours after intromission d. 18 hours before the onset of stand ing heat e. 24 to 36 hours after the end of standing heat 246. What is tile duration ofesrrus in goats?
a. b. c. d. e.
21 days variable because of induced ovulation J2 to 24 hours 8 to 12 hours 36 to 48 hours
247. Which afthefollowing most accurately describes buck ejaculates?
a. volume of 0.5 to 2 ml, concentration of 1.5 to 4 X 109 sperm/ml b. volume of5 to 10 ml, concentration of l. 5 to 4 X 109 sperm/ml c. volume of 0.5 to 2 mI, concentration of l.5 to 4 X 106 sperm/ml
Theriogenology
d. volume of 5 to 10 ml, concentration of 1.5 to 4 X 106 sperm/ml e. volume of25 to 50 ml, concentration of 1.5 to 4 X 106 sperm/mI
BIRDS M.B. Paster 251. Follicle-stimulating hormone induces all ofrhe
248. What are the accessory sex glands ofbucks?
a. ampulla, seminal vesicles, disseminated prostate, and bulbourethral glands b. ampulla, seminal vesicles, compact prostate, and bulbourethral glands c. ampulla and seminal vesicles only d. seminal vesicles, disseminated prostate, and bulbourethral glands only c. compact prostate only 249. Concerning the intersex condition in goats, which statement is least accurate?
a. An intersex is an animal that externally shows both male and female characteristics. b. It is more prevalent in dairy goats than in any other domestic species. c. The gonads (testes or ovotestes) may be scrotal, inguinal, or abdominal in location. d. Most polled intersexes are karyotypically male (XY). e. It is more prevalent in dairy goats that are polled (hornless) than in horned goats. 250. Which of tile following most accurately describes the caprine estrous cycle?
a. seasonally polyestrous, cycling induced by a decreaSing photoperiod b. seasonally polyestrous, cycling induced by an increasing photoperiod c. nonseasonal polyestrous, cycles 21 days long d. monestrous, cycling induced by a decreasing photoperiod e. monestrous, cycling induced by an increasing photoperiod
following except:
a. b. c. d. e.
development of ovarian follicles secretion of estrogen by the ovaries feather growth spermatogenesis tubular growth of the testes
252. The serum level of which hormone is generally higher in male tllan infemale birds during tile breeding season?
a. b. c. d. e.
adrenocorticotropic hormone luteinizing hormone triiodothyronine mesotodn tetraiodothyronine
253. Prolactin induces all of the following except:
a. h. c. d. e.
nesting behavior incubation cessation of egg laying caring for the young increased production of luteinizing hormone and follicle-stimulating hormone in both sexes
254. The offspring ofaltricial species are characterized by all tile following except:
a. hatch naked h. hatch blind c. are exothermic, with a metabolic rate lower than that of adults d. hatch with thermoregulatory ability e. hatch with a low metabolic rate that increases during development and usually exceeds that of adults by fledgling time 255. Tile offspring of precocial species are characterized by all of the following except:
a. hatch with their eyes open b. covered with down when hatched
© 1998 MosUy-Year Book. Inc. Photocopying is prohibited by law.
229
c. alert when hatched d. hatch with thermoregulatory ability e. are exothermic, with a metabolic rate lower than that of adults 256. Which hormone induces the preovulatory surge of luteinizing hormone?
a. b. c. d. e.
triiodothyronine progesterone estradiol prolactin arginine vasotocin
257. Which hormone induces expulsion of tile egg from the oviduct?
a. b. c. d. e.
thyroxine estradiol arginine vasotocin prolactin progesterone
258. Which hormone stimulates development of "crop milk" in pigeons and doves?
a. tetraiodothyronine b. estradiol c. prolactin d. progesterone e. arginine vasotocin 259. The testes have all thefollowingfunctions except:
a. production of male gametes b. secretion of steroid hormones thai control development c. secretion of steroid hormones that control function of accessory sex organs d. secretion of steroid hormones that suppress song behavior e. secretion of steroid hormones that influence secondary sex characteristics
Correce answers are on pages 234-244.
230
www.vet4arab.co.cc
SECTION 9
260. 771e bursa of Fabricius is associated with all the /ollow;ngexcept:
a. adrenal gland development b. development of immunity through production of humoral factors c. production of ted blood cells d. testicular growth e. development of immunity through production of cellular units
261 . Conceming the relations11ip between molting and reproduction, which stalemeflt is least
d. The ovaries periodically hypertrophy and atrophy on alternate sides throughout life. e. The ovaries become especially active during molting. 264. Which hormone regulates I/le pattern ofgrowth and maturation ofovarian follicles?
a. b. c. d. e.
prolactin foUicle-stimulating hormone estradiol progesterone luteinizing hormone
accurate? a. Androgens and estrogens may delay or inhibit molting. b. The molting cycle is often associated with a marked increase in serum levels of triiodothyronine and thyroxine. c. Sex hormones have a protective effect on mature feathers. d. Molting is associated with decreased levels of testosterone and estrogen. e. ThyrOid hormones stimulate early development of new feathers. 262. In male birds, wllicll cells secrete testosterone?
a. b. c. d. e.
13 cells of the islels of Langerhans ScrtoHcells chromaffin cells argentaffin cells Leydig cells
263. Concerning tile reproductive tract offemale birds, which statement is most accurate?
a. The left ovary is more well developed than the right ovary. b. The right ovary is more well developed than the left ovary. c. The ovaries are equally well developed throughout life.
265. Mlich hormone decreases production offolliclestimulating hormone and luteinizing hormone and stimulates brooding behavior?
a. b. c. d. e.
prolactin estradiol progesterone cholecalciferol mibolerone
268. Concerning the blood chemistry of laying birds,
wllich statement is least accurate?
a. Serum levels of phospholipids, fatty acids, and neutral fats increase by 3- to IS-fold in birds laying eggs. b. Ovarian hormones increase serum levels offalS. c. The blood glucose level in layers is double that of nonlayers or males. d. The blood calcium level is doubled in laying birds. e. Plasma cholesterol levels are increased 3-fold in layers. 269. Concerning hormone interactions during the ovulatory cycle, which statement is least accurate?
a. Estrogen never stimulates release of luteinizing hormone. b. Testosterone is rarely found in the plasma of laying hens.
a polyovous layer imennittently polyovous a determinate layer seasonally fertile seasonally subfertile
267. A hen tllat con tinues laying eggs until a specific number afeggs has accumulated in the nest and then ceases laying eggs is best described as:
a. b. c. d. e.
a determinate layer a clutch-limited layer a polyovous layer an indetermina te layer an environmental layer
C 1998 Mosby-Year Book, Inc. PhOTocopying is prollihired by law.
c. Progesterone stimulates secretion of luteiniZing hormone. d. Progesterone is the principal hormone exerting a positive-feedback effect on release of luteinizing hormone in hens. e. Estradiol, produced by ovarian follicles, induces the physiologic changes associated with egg laying. 270. Reproductivefailure in captive birds is usually associated willi:
a. failure of the ovary to develop follicles that ovulate b. failure of males to develop normal testes c. failure of the pituitary gland to respond to hypotllalamic stimulation d . failure of the ovary to respond to gonadotropin stimulation e. stress, which causes release of adrenocorticotropic hormone
ZOO AND EXOTIC ANIMALS
266. A hen thaI lays a cerlain number ofeggs during a specified period and then ceases laying eggs is best described as:
a. b. c. d. e.
231
Tileriogenology
L.A. Dierau!, F.L. Frye, T. Gemeinhardt, S. Hudelson, D. Huff, J.D. Letcher, D.H. Nielsen, D.O. Schaeffer 271. Wllich stage is nol included in thedevelopmem ofreptiles?
a. b. c. d. e.
larva juvenile adult embryo fetus
a. b. c. d. e.
272. The mealls by which an all-female line of reptiles call develop is called:
a. b. c. d. e.
androgenesis misanthropism parthenogenesis gynogenesis he rmaphroditism
273. Reptile species that [ayeggs that subsequently hatch outside the body are described as:
nulliparous oviparous viviparous polyploid monoecious
274. A hermaphrodite:
a. is sexually attracted to members of the same sex b. is sexually attracted to members of the opposite sex c. has only one type of gonad, either ovaries or testicles d . has both testicular and ovarian tissue e. lacks gonadal tissue altogether
Correct answers are on pages 234-244.
www.vet4arab.co.cc
SECTION 9
232
275. Difficulty in fayingeggs or giving birth is called:
a. b. c. d. e.
dyspareunia dysecdysis dyspepsia dystocia dysautonomia
276. Sexual precocity refers to:
a. delayed sexual maturation b. accelerated sexual maturation c. failure to deposit ova or deliver young on schedule d. premature ejaculation e. inability to attain or maintain an erection 277. In reptiles, fracwre of an intraoviductal egg, with leakage a/tile contents into the body cavity, is:
a. harmless under most circumstances and can be ignored h. an emergency that warrants immediate surgical treatment c. a consequence of inbreeding or line breeding d. normal in many snakes, lizards, and tortoises e. usually caused by vitamin B6 deficiency 278. Mastitis in tortoises is:
a. usually related to breeding of a very immature animal b. usually seen only in older tortoises that have had many litters of young c. not a clinical entity in this species d. caused by chilling during hibernation e. thought to be related to infection with soil fungi 279. In most female reptiles, menopause:
a. occurs at about mid-life b. occurs only after an animal bas failed to give birth or lay eggs c. occurs in the fifth or sixth decade of life
d. occurs when the animal reaches about 45 years of age e. does not occur 280. Rattlesnakes, pygmy rattlesnakes, copperheads, and cottonmouth water moccasins giue birth to liuing young, whereas coral snakes lay eggs. Which statement best describes the young of these snakes?
a. Alhough the young of these snakes are born with fangs, they do not have poison (venom) uncil they have eaten several meals and have shed at least twice. b. Baby snakes, even venomous ones, must learn from their mother how to bite to protect themselves. c. Baby venomous snakes can deliver serious bites even before they emerge from the egg or protective fetal membranes. d. Baby venomous snakes do not develop fangs until they have shed their skin at least twice. e. Because these baby snakes are small, they can be handled without fear of envenomation. 281. The egg shells of some lizards can be oueriy soft when they are deposited. The most likely cause of this softness is:
a. b. c. d.
phosphorous deficiency andlor renal disease calcium deficiency andlor renal disease dietary protein deficiency dietary manganese and magnesium deficiency e. chronic organophosphate toxicity 282. Reptiles haue a cloaca through which feces, urinary wastes, and reproductive products are passed. The eggs or liuing youngoffemale reptiles pass out of the female's body through the:
a. b. c. d. e.
urodeum urethra coprodeum vestibuJe proctodeum
© 1998 Mosby-Year Book, Inc. Photocopying is prohibited by law.
283. Amphigonia retardata refers to:
a. retention of eggs in the oviduct for a prolonged period b. retention of spermatozoa for a prolonged period afler mating c. migration of eggs from one oviduct into the opposite one d. slowed sexual maturity caused by subnormal environmental temperatures e. prolonged penile erection 284. The eggs ofmost reptiles resemble those ot
a. b. c. d. e.
233
Theriogenology
amphibians primates insects birds fish
288. III most reptiles the erectile portion of the penis or hemipenis is composed of
a. b. c. d. e.
mucous glandular tissue fibrovascular tissue fibrocollagenous connective tissue serous glandular tissue bone
289. Spermatozoa can remain viable in the reprodllctiue tract of some mated female reptiles for:
a. 2 to 4 weeks b. 2 to 4 monrhs c. 2 to 4 years d. 2 to 4 hours e. 2 to 4 minutes 290. Examples of inherited disorders ill reptiles are:
285. Which pathogen can be transmitted from a female ouiparolls reptile to her eggs?
a. Pasteurella b. Pseudomonas c. Salmonella d. Aeromonas e. Morganella
a. b. c. d.
dicephaly and spina bifida albinism and melanism cleft palate and microphthalmia molluscum contagiosum and diabetes mellitus e. Canfield's syndrome and amyloidOSiS 291. Polyspermia refers to:
286. Why are adult male tortoises separated from females carrying shelled eggs?
a. Mismating and confusion over parentage of the offspring may occur. b. Intraoviductal eggs might become damaged if mating is repeated. c. Some eggs could be doubly fertilized if mating is repeated. d. Developing embryos within the eggs might develop into hybrids. e. The female might attack and injure the male. 287. In reptiles, fertilization occurs:
a. after the eggs have been deposited by the female in water b. shortly after the egg shells have been deposited around the developing embryo and yolk mass c. in the oviduct d. within the terminal end of the epididymis e. within the terminal end of the vasa deferentia
a. deposition of more than one male's spermatozoa in a female's reproductive tract b. penetration of more than a single spermatozoon into an egg c. spermatozoa attached to each other by their acrosomes d . defective spermatozoa characterized by multiple tails e. storage of excess spermatozoa after a mating 292. Whieh species has testicles located in asemtal sac?
a. Atlantic white-sided dolphin (Lage1lorhynchus acutus) b. common dolphin (Delphinus delphis) c. northern fur seal (Callorhinus ursi1lus) d. Pacific harbor seal (Phoca uitulina) e. Atlantic bottle-nosed dolphin (Tursiops truncatus)
Correct mlSluers are Of! pages 234·244.
234
www.vet4arab.co.cc
SECTION 9
c. mink d. llama e. bear
293. Although somewhat variable. gestation in llamas normally lasts: a. h. c. d. e.
113 to 125 days 140 to 155 days 260 to 280 days 330 to 360 days 390 to 450 days
297. WllQt is the gestation length ofguinea pigs?
294. Ferrets have an average litter sizeo[B (range, 2 to 17). Tile deaf, blilld infants are born after an average gestation of a. 21 days b. 32 days
a. 21 days b. 28days c. 42 days d. 45 days e. 63 days 298. In which species is dystocia a major problem?
a. rot h. hamster
c. 42 days d. 50 days e. 64 days
c. gerbil d. rabbit
295. Overpopulation is a cOrlStant problem ;nfreeranging and captive feral horses. A contraceptive that has been used in female feral horses, as well as in dogs, can be administered remotely, does not illferJere with pregnancies already in progress, has no effect on social behavior; and has effects tltat are reversible. This contraceptive is:
a. porcine zona pellucida b. diethylstilbestrol c. testosterone propionate d. ethinylestradiol homogeneous Silastic implants e. northisterone
e. guinea pig 299. In which species is it absoilltely necessary to remove the male before parturition?
a. b. c. d. e.
rat mouse gerbil hamster guinea pig
300. Which species bears young tllat are precocial (fully fllrred, eyes open)?
296. Wl!icll species apparently is not all induced ovulator?
a. Bactrian camel b. ferret
a. b. c. d. e.
mouse rat hamster gerbil guinea pig
Answers 1. d Bitches have one cycle at a time and, if they live long enough. usually have at least one cycle in each season of the year. The average interestrus period for all breeds is 7.4 months. 2. a Queens have several cycles within a breeding season. Breeding seasons are determined in large part by light cycles.
3. d Basenjis are one of the few breeds that truly cycle once per year. The Tibetan mastiff (rarely seen as a pet) is another. 4. d Although the signs listed have been said to indicate ovulation. only answer d stands up to scientific scrutiny.
© 1998 Mosby-Year Book, Inc. Photocopying is prohibited by law.
Tlleriogenology
5. c High serum progesterone levels are necessary to maintain pregnancy, and decreased serum progesterone levels indicate impending parturition. A drop in rectal temperature to 100S F is not low enough. A rectal temperature below 99 F accompanies decreased serum progesterone levels (prostaglandin effect). 6. a None of the other answers can detect pregnancy as early as 18 days. 7. b Relaxin is a product ofplacentation. 8. e All the other stages listed have a relatively narrow range of duration. A bitch cycling every 4 months will have a short anestrus compared with one that cycles every 9 months, but both are normal. 9. c Answers a, b, and e will probably induce ovulation. Answer d wiU pharmacologically overcome estrogen-expressed signs of heat. Answer c will prolong the signs of heat. 10. b Megestrol has been approved for estrus prevention once proestrus has begun. 11. a Mibolerone can be used over the long term for estrus prevention. 12. e None of these drugs has been approved for use in cats. 13. c Sperm mature in the epididymis. 14. d Following ovulation, a requisite period of maturation before fertilization makes this the ideal time to inseminate. Mature canine ova are relatively short lived . 15. d Sperm nwnbers per ejaculate are a function of testicular size, which is a function of body size. Thus big dogs produce more sperm than little dogs. 16. e Sperm concentration is about the same for all normal males. The total number of sperm per ejaculate and total volume of the ejaculate increase with body size, but sperm concentration varies only slightly. 17. c Parabasal and uncornified epithelial cells are synonymous. Their numbers decrease as estrogen levels increase. Polymorphonuclear cells disappear, and red blood cell numbers are highly variable. 18. d Although not totally immune to the serious side effects of estrogen, young bitches are more tolerant. Answers b, c, and e are true and should be made known to the owner when assessing the risk/benefit ratio before therapy is begun. 19. d This drug has a direct effect, causing myometrial contraction . The effect of oxytocin is negligible in clinical cases. 0
235
20. a Fever is not a feature of this disease. 21. b The other answers listed are ancillary
diagnostic methods. Confirmation requires isolation of the organism, which is best recovered from blood. 22 . d There is no reliable medical treatment for brucellosis. Blood cultures usually are not positive until at least 30 days after infection. 23. e Separation of the head from the midpiece and tail is often an artifact resulting from poor sample handling, such as cold shock. The other defects listed arise during spermiogenesis in the testes. 24. e Vaginal hyperplasia occurs during the follicular (estrogen) phase and regresses spontaneously. Uterine prolapse does not regress. 25. b At this age, both testes should be scrotal. This dog may be fertile. Monorchidism refers to agenesis of one testis and is very rare or nonexistent in dogs. 26. b Cryptorchidism is uncommon in cats. In dogs, retained testes have a higher incidence of neoplasia than do scrotal testes. A diagnosis is not possible until scrotal development occurs, and not at birth in any breed. It is not a dominant trait. 27. c Because of an increased prevalence of neoplasia in retained testes and possibly also in scrotal testes and because of the hereditary nacure of the condition (unilateral cryptorchids are often fertile), bilateral caslration is recommended. This dog is too old for medical therapy. Orchiopexy is considered unethical. 28. a Hermaphroditism is diagnosed by the histologic appearance of the gonads, not by the gross appearance of the external genitalia. 29. c Hypothyroidism is commonly associated with subfertility or infertility. 30. e This bitch is showing signs of estrogen stimulation. Once treated, this dog will cycle normally. Pituitary adenoma is not a common cause. Nine weeks is far longer than the normal duration of estrus. 31. a All the other infections listed can cause these problems. Calicivirus infection has not been incriminated, although it may cause kitten mortality. 32. d Subinvolution of placental sites is most commonly a one-time occurrence in young, firstlitter bitches. It cannot be prevented and, by itself, is not harmful. The only sure immediate cure is hysterectomy. Pyometra frequently occurs without subinvolution.
236
SECTION 9
33. a Prostatic diseases of all kinds are very uncommon in cats. in dogs, prostati tis frequcndy causes necrozoospermia (dead sperm). However, it does not usually affect spe rm numbers. 34. b Needle biopsies do not yield a specimen with retained tissue architecture sufficient for diagnosis. Because cytologic diagnosis is usually not possible, a surgical biopsy is preferred. 35. b Absence of molar teeth is a function offetal age. The other choices listed are signs of fetal
death. 36. d Pain at intromission usually indicates an anatomic problem in the bitch; vaginal strictures ure the most common cause of such pain. 37. d A vi ral etiology has not been proven. Tnlllsmission by licking is common. Many
38.
39.
40.
41 . 42.
43.
44.
transmissible venereal tumors are benign and regress spontaneously. Most respond well to radiation therapy. In some parts of the world, malignancy is more common than in the United Stales. a Granulosa-cell tumors occur in females. Only Sertoli-cell tumors usually reach such size. The second (soft) mass probably represents a cystic prostate gland or uterus masculinus as a result of the esnogens produced by the Sertoli-cell (Umor. a Of tile tumors listed. a Scrtoli-celltumor is most likely to produce e nough estrogen [Q affect the bone marrow. d All the other organisms listed may be isolated on aerobic cultures, but mycoplasms require special techniques for isolation. c Feminizing syndrome usually produces hyperpigme ntation. c Answers band e do not automatically cause dystocia; answers a and d are normal. Answer c definitely indicates prolonged gestation and most probably represents a maternal dystocia, although, with lack of owner observation, it could be felal. a There will be no sperm in this dog's ejaculate (answer a), rather than few (answer b) or dead (answer c) sperm . Because most of the ejaculate is comprised of prostatic fluid , ejaculate volume is little affected. Ubido is likewise unaffected because the testes are normal. d Erythrocytes may be seen before proestrus and well into diestrus/metestrus. They are unreliable indicators of the stage of the cycle and may also be seen in s mears from dogs with neoplasia or trauma.
www.vet4arab.co.cc
45. b TIle o nly reliable indicator of a functional ovarian remna nt is an elevated serum progesterone level. II is too lale for an elevated serum estrad iol level. Spayed bitches with hyperadrenocorticism do not cycle. Answer d would not be present. Parabasal cells would predominate. Ultrasonography is totally unreliable in this circumstance. 46. c An ovarian rem nant is most likely. Most of these are 011 the right side because the right ovary is more difficult to exteriorize during routine ovariohysterectomy. Ectopic ovaries are very rare. 47. a Progesterone prolongs gestation. The other answers remove the source of progesterone or the fetuses themselves. 48. e The duration ofviabiJi ty of frozen cani ne semen has not yet been dctermined, but it is well in excess of 10 years. 49. c Forty-eight hours is a reasonable maximum, using current semen extenders. 50. b All females are XX and most ~torti es~ are fema le. Those few males arising from chimerism are usually XXV (infertile) , but a few have been ferti le. Thus fertil e male tortoiseshell cats are very rare though not necessarily valuable. 51. d By definition, monestrous females have a single heat during a breeding season. 52. a Anestrous means not cycling. 53. e Bydefinition, polyestrous females have multiple heats during a breeding season . 54. c Females rC(:emly in heat are considered to be in metestrus. 55. b Estrus is the period of heal. 56. e Estrogcn is produced by the ovulatory follicle during estrus. 57. a Ci rculating levels of luteinizing hormone rise to peak level at the onset of estrus, triggering ovulation. 58. b Unlike surgical embryo collection, nonsurgical collection does not cause adhesion formation and thus can be done repeatedly. 59. d Progesterone is produced by the corpus IUleum, which is maximally developed during diestrus. 60. e ~ Hip-Iocked ~ calves cannot be repelled, because they arc firm ly wedged in the birth canal. Partial fetolomy would be least traumatic 10 the dam. 61. c Adhesions are the o llly answer Listed that is compatible with regular cycling.
e 1998 Mosby-Year Book, 1nc. Photocopying is prollibited by law.
T/leriogellology
62. b On day 3 the embryo is still in the oviduct and on day 5 it still may be in the oviduct and so cannot be rC(:overed. By day 9 the embryo has lost its zona pellucida and is much more difficult to identify. 63. a Estrus las ts only 12 to 18 hours, versus more than 2 days for all the other phases. 64. c Catlie normally give birth to a Single offspring, as opposed to a litter (multiparous). 65. b Hydramnios occurs as a result ofa defective fetus. 66. e This is the only indisputable sign of estrus. 67. a Campylobacteriosis is a venereal disease. 68. a The prostate gland of bulls is poorly developed. 69. b Diestrus is the only time during which a mature corpus luteum is present, with a sufficient number of receptors. 70. c As compared with the other s ites listed, the vagina is relatively pliable and elastic. 71. b The response to superovulation is not very controllable because of variations in drug dosages, day of the cycle that treatment is initiated, and the animal 's age, lactational status, breed and nutritional s tatus. 72. d Poor estrus detection is a never-ending manageme nt problem. 73. b Respiration must be initiated within 5 minutes, o r the calf will die. 74. d The prevalence of listeriosis in pregnant cows is too low to warrant development of a commercial vaccine. 75. e Pyometra is almost always seen postpartum, except when caused by venereal transmission of trichomoniasis. 76. d For economic reasons, slaughtering the cow is the most a ppropriate action. It would take at least 10 more months for this cow already in late lactation to begin "earning her keep" with a new lactation. 77. a In this situation the fetus is not accessible for fetotomy or through use of a detorsion rod. There are no relaxant drugs approved for use in food animals. The degree of torsion is too great to correct by rolling. 78. d There should be no corpus luteum at the time of artificial insemination, so progesterone levels should be low. If the cow is pregnant at the time of the next anticipated estrus (22 days later), there should be a corpus luteum, and progesterone levels should be high.
237
79. e About 85% of heifer calves born twin to a bull calf are freemartins. 80. c Scrotal circumference is also correlated with sperm output. 81. b This s ite is most likely to sustain damage when the erect penis is accidentally bent during a thrust against an immovable object. 82. d All the other diseases listed primarily affect the placenta. 83. b Asymmetry and fluctuation of the uterine horn could be caused by pyometra. The amniotic vesicle is not palpable after 60 days. The uterine artery does not develop fremitus until 3.5 to 4 months of gestation. 84. a Heifers are first bred to calve at 24 months of age. 85. c The delicate vascular aflachments of the amniotic vesicle are vulnerable to damage during palpation. 86. b The decrease in circulating progesterone concentration permits final maturation of the ovulatory follicle. 87. d Diestrus lasts 13 or 14 days. 88. a The bovine corpus hemorrhagicum is small and soft. 89. a Heifers generally have a 20-day cycle. Therefore the chances are I in 20 (5%) that a heifer will be in heat on any given day. With 100 heifers, that means 5 would be expected to be in heat. 90. a Slipping the chorioallantoic membrane is the only positive sign of pregnancy. 91. e Affected bovine fetuses have a ventral abdom inal defect and a ventrodorsally curved spine. 92. a Hypocalcemia results in a flaccid myometrium. 93. c Bovine ova are fertilized in the oviduct. 94. a Excessive heat and humidity retard early cell division, and the embryo dies before day 16. 95. b The unaccompanied corpus luteum persists for long periods through lack of a luteolytic signaJ because there is no adjacent uterine horn. 96. a Estrus usually begins between midnight and 6 AM in dairy cows. 97. e Semen deposition at this site is unlikely to damage the endometrium. 98. e The endometrium in this cow is unlikely to be damaged and is free from infection. and the caruncles are already involuted. 99. a This is the time when the fetus is most susceptible to peracute, fatal viral infections.
opposite horn and crushed. Waiting may increase the chance for spontaneous reduction, but if
100. e An impressio n smear of a cotyledon should 101.
102.
103.
104.
105.
106.
107.
106.
109. 110.
Ill.
lI Z. 113.
11 4. 115. ll6. 11 7. Il6.
www.vet4arab.co.cc
SECTI ON 9
238
be stained and examined for fungal hyphae. d Mares have multiple estrous cycles during the spring and so are considered seasonally polyestrous. a Mares normally cycle during periods of longer day length, which, in the Northern Hemisphere, is in the spring (March to July). d Aflificially increasing day length to 16 hours of light, with 6 hours of darkness, induces cycling in about 2 months. b During late winter and early spring, it is normal for a mare to show prolonged signs of estrus because of anovulatory follicles. a During the fall it is normal for mares to show prolonged signs of estrus because of anovulatory follicles. b Mares normally go out of estrus 2 days after ovulation, so the bes t description is that they ovulate 2 days before the end of estrus. d The ovulatory follicle can quickly fill with blood after ovulatio n. To determine when the follicle collapses, the ovary must be palpated every 12 hours or the blood that fills the follicle can be mistaken for the follicle itself. d Human chorionic gonadotropin has luteinizing action and can induce ovulation in mares. b The fertilized ovum remains in the oviduct fo r about 5 days before entering the uterus. e Approximately 20% of all mares have multiple ovulations, most of which are double ovulations within 24 hours of each other. a Prostaglandin is produced by the endometrium and causes luteolysis. b Progesterone is produced by the corpus luteum. c The endometrial cups produce equine chorionic gonadotropin (formerly pregnant mare serum gonadotropin). e Human chorionic gonadotropin has luteinizing activity. d The granulosa cells produce estrogen. c The equine embryo normally moves for 16 days before fixation. d The equine embryo attaches anywhere in the uterus, regardless of the side of ovulation. a At 14 days postovulation the remaining embryo is most likely to survive if one twin is moved to the
~
reduction does not occur, chances of me noncrushed embryo surviving are reduced. 119. e Endometrial cups are of fe tal origin and
120.
12 1.
122.
123. 124. IZ5.
126.
127.
126.
lZ9. 130.
131. 132.
133.
p roduce equine chorionic gonadotropin (pregnant mare gonadotropin). e The mare can be ovariectomized after 120 days of gestation and still maintain pregnancy with placental progestogens. d Tn diestrus the mare has a IOnic uterus and tonic cervix because of the influence of progesterone. c In estrus the mare has a flaccid uterus and flaccid cervix because of the influence of estrogen. a The corpus luteum is not readily palpable in mares because it is embedded within the ovary. b The red velvety tissue is the maternal or uterine side of the allantochorion. b Stage III , or placental expulsion, should occur within 90 minutes after parturition, or the placenta is considered retained. c Mares should be at 330 days of gestation and have cetvical relaxation and some ~waxi ng" of the teats before parturition can be induced with a good success rate. b Parturition is best induced using oxytocin. Prostagland in Fla tends to cause premature placen tal separation. e Pseudopregnancy is described as a period of retained corpus luteum (regardless of whether the mare is bred) , which induces tone in the uterus and cervix. a Fetal expulsion (stage II) normally lasts only 30 minu tes in mares. a Prostaglandin is luteolytic only when a mature corpus luleu m exists, which is 5 days after the end of estrus. e Superovulation is not readily attainable in mares. c To collect embryos from a mare for embryo transfer, it is best to flu sh both uterine horns simultaneously 7 days after ovulation. a To transfer an embryo into a mare, it is only necessary to place the embryo in the uterine body. The location {right or left} of the corpus luteum makes no difference because the early embryo is mobile.
1998 Mosby-Year Book, Inc. Phorocopyillg is prohibited by law.
Tl,erioge'lology
134. e It is generally assu med that unfertilized ova are retained in the oviduct by the ovulating
ovary fo r many months before being released into the uterus. 135. a Mares normally have a fertile "foal heat" within 9 days after parturition. 136. e ~Be llin g" of the glans penis during ejaculation causes the jets of semen to directly enter the uterus. 137. e Artificial insemination is performed by carrying the pipette into the vagina and then placing the tip in the uterus, where the semen is deposited. 136. a At least 500 million normal motile sperm cells are required if an aliquot of semen is to attain acceptable fertili ty using artificial insem inatio n in the mare. 139. e Stallion sperm can survive for as long as 5 days in the mare's uterus. 140. a The stallion's penis is a high-volume, lowpressure, musculocavernous rype of erectile tissue. 141. e Both pressu re and temperature must be optimal for the stallion to ejaculate. 142. d IllIerstitial cells of Leydig in the testis prod uce testosterone. 143. d The epididymis is oriented on the testis with the head cranial. the body dorsal and the tail caudal. 144. b Estradiol is thought to be produced by the Sertoli cells and involved in the feedback system of follicle-stimulating hormone production . 145. a The stallion's penis is of the musculocavernous type. 146. c It takes 55 days after the first divisio n of the primary spermatocyte until mature spermatozoa arc released for ejaculation. 147. e During the nonbreeding season (short day length}, the stallion's ejaculate has less volume and fewer spermatozoa than during the breed ing season. 148. d Accessory sex glands of stallions include the vesicular glands (seminal vesicles), pros tate gland, and bulbourethral gland. 149. b After 7 consecutive days of collection, daily sperm output stabilizes. 150. b A second ejaculate, collected 1 hour after the first, should have the same volume and half as many spermatozoa. If this is not the case, one of the ejaculates is not representative.
239
151. e Although the other factors listed may
influence the time of onset ofpubcrty, exposure of gilts of the proper age to a mature boar is the most consistent faclOr influencing the onset of
puberty. 152. b This accurately describes the way A-mode ultrasonography works. 153. e Shorter lactation lengths are associated with longer weaning-to-service intervals and smaller subsequent liner sizes. 154. a All other answers refer primarily to sliUbinhs. 155. e None of the other regimens works well in sows. 156. d Answers a and b may occur but are less common than answer d. Answers c and e are untrue statemelllS. 157. b Abortions are primarily the result of noninfectious causes. Diagnosed cases lend to be th ose with infectious causes. 156. b Pigs apparently are seasonal breeders. The effects of season are more profound and last longer than the effects of heat stress. Younger sows, outdoor breeding herds, and grouphoused sows are more severely affected. 159. d This is the only answer that accurately describes lordosis in swine. 160. b An experienced manager working with a mature boar is 96% accurate in detecting pregnant sows. 16 1. a This is the only accurate description of the events. 162. e The optim um is not known ; 12 appears to be the minimum. 163. d This practice is only effective if the weaningto -service interval is already too long. 164. c Services 3 to 6 days after weaning result in higher farrowing rates and litter sizes. A ~tro u gh" occurs as a result of setvices 7 to 14 days after weaning. 165. b This is the only answer that accurately describes the sequence of endocrinologic events. 166. c Fetal fluids and tissues are more useful than any of the other selections. The other answers might yield results in individual cases, but in general they are extremely fru strating d iagnostic techniques. 167. d Manifestations of seasonal infertility include an increased number of sows with delayed return to estrus after weaning. 168. c Ovulation occurs 36 to 40 hours after the onset of estrus in most sows.
240
\69. b The porcine corpus luteum is unresponsive to prostaglandins until day 12 of the cycle. It is
thought that a lack of or unresponsiveness of receptor sites may be involved. Recent studies
170. 171.
172.
173.
174.
\75.
176.
177-
\7S.
179.
180.
lSI.
www.vet4arab.co.cc
SECTION 9
have shown that the porcine corpus luleum is responsive to multiple doses of prostaglandins, beginning at about day 5, but this is still impractical for estrus synchronization on farms. a This is the only physiologically accurate description of the events. b Estrogens produced by the fetuses themselves around day 12 after mating signal the sow that pregnancy has occurred. c At least four embryos must be present in the uterus on day 12 to produce the amount offetal estrogens necessary for maternal recognition of pregnancy. e All other selections have too little volume, too many abnormalities, or low sperm concentration. c Mating twice increases the likelihood of a mating at the optimal time relative to ovuJation. although recent studies suggest single mating may be adequate for sows in estrus 3 to 6 days after weaning. a Mature boars can be mated 5 to 7 times per week, whereas immature boars should be used 3 to 5 times per week. d Fetal mummification is primarily caused by viral infections between day 35 and day 70 of gestation. e Approximately 85% to 90% of multiparous sows should be in estrus within 7 days of weaning following a 21-day lactation. A shoner lactation decreases the percentage of sows in estrus within 7 days. d Age and testicular weight reflect sexual maturity in boars. Testicular circumference is difficult to measure in boars. The other measurements listed have no relationship to sexual maturity. c Seasona1 fluctu ations are the likely cause, considering the percentage of changes described and the time of year given. d Any of the other organisms listed could potentially cause late-term abortions in individual sows. However, leptospirosis is most common and may occur as a herd problem. b Gilts seronegative at the time of breeding are at risk of contracting lhe virus during early gestation. with subsequent early embryonic
182.
183. 184. 185.
186.
187.
188.
189. 190.
191.
192.
death or fetal mummification. They require exposure before breeding so that they can develop immunity before early to midgestation. e Cranial and caudal fetal presentations are normal in pigs. Breech presentations usually do not cause dystocia. Eclampsia is relatively uncommon in sows. b Exposure to zearalenone is the most likely cause, considering the signs described. d Nonproductive days are defined as days on which sows are neither gestating nor lactating. a Answers b. c. and d describe higher levels of reproductive performance and are all incorrect. With answer e, although slower growth may be associated with leaner pigs, higher incidences of diarrhea are not associated. b The best biologic measure of sow herd efficiency is pigs per sow. The other measures may be useful in other areas of herd efficiency. Pigs weaned per farrowing crate may be the most useful measure of facility use. b Leptospira bratislava has been commonly identified in serologic surveys in the midwestern United States, although culture results and definitive diagnoses have not often documented its association \'lith reproductive problems. Leptospira pomona is most frequently documented as a cause of late-term abortions. d The other answers contain at least one organism that is not an important common herd pathogen. c Boars ejaculate over a period of 4 to 6 minutes. b The main use is production of weaned pigs free of specific diseases. Sow reproductive performance may in fact be worse (answer a). It is unlikely that all diseases can be eliminated by any procedure (answer c). The cost to produce a wcaned pig may actual ly increase because of increased medication costs and decreased sow reproductive performance (answer d). The feed cost to produce a weaned pig probably will not change. Feed utilization improvements as a resuh of the technique are gained primarily after weaning. b To ensure that crates are kept full in December, the number of marings that occur about 4 months earlier (August) should be calculated using the lower historica1 farrowing rate (57%): 53 x 0.57 = 30 sows. b In boars the head of the epididymis lies at the ventral pole of tile testis.
01998 Mosby-Year Book, Inc. PllOtocOpyillg is prollibited by law.
Tllulogenology
\93. c Boars have a fibroelastic, corkscrew-shaped
penis. 194. d In boars a cycle of spermatogenesis takes 35 days. 195. e Ova arc fertilized in the oviduct. 196. b Laceration of the distal end of the penis, caused by masturbation. homosexual activity. and biting and fighting associated with penmating, is the most common physical injury to the boar's reproductive tract. 197. c l11e embryon ic period is the period of greatest susceptibility to teratogens. During the preattachment and fetal periods. susceptibility is fairly low. Placental and neonatal periods are not recognized phases of fetal development. 19B. b Although the first litter was small. this animal could have more pigs in subsequent liuers. 199. d Despite our desire for lower cuUing rates, most herds have a culling rate of 40% to 55%. and the rate is higher in many herds. 200. a The second stage of labor lasts 2 to 3 hours. Pigs are born about 15 minutes apart. 201. a Ewes cycle every 16 days. 202. d Ewes normally cycle during periods of shorter day length, which. in the United States and Canada. are in the fal l and winter (August through March). 203. a Progesterone is produced by the corpus luteum during diestrus. 204. b A mature corpus luteum (CL) must be presen t if prostaglandin F.... is to induce estrus. A mature CL is present omy during breeding season. The CL is not mature during estrus and immediately after lambing. 205. a Placentation is cotyledonary in sheep. diffuse in swine and horses. zonary in dogs and cats. and discoid in primates. 206. d Campylobacteriosis has a wide geographic distribution and causes high rates of abortion. The disease often causes severe financial losses. 207. c Progestagen-impregnated vaginal pessaries are widely used for estrus synchronization. 208. d Estrus can be induced and synchronized during anestrus by administration of progesterone for 12 to 14 days. followed by administration of pregnant mare serum gonodotropin (PMSG) at the time of or 24 hours before progestagen withdrawal. 209. d The normal gestation length in ewes is about 150 days. with a range among breeds of 145 to 155 days.
241
210. c Estrus averages about 30 hours in ewes. Ovulation occurs in the last part of estrus. The
optimum time for insemination is 12 to 15 hours after the onset of estrus. 211. a Of livestock species, sheep and goats have the smallest volume of ejaculates. The average ram ejaculates I to 1.5 ml of semen. containing J to 3 billion sperm. 212. c Semen can be collected from rams many times each day for several weeks before depleting epid idymal reserves of sperm because each ejaculate is of small volume. In contrast. boars and stallions quickly deplete their epididymal reserve because they expel large numbers of sperm in each ejaculate. 213. c Bleating and tail wagging indicale estrus in ewes. Answers a, b. and e are observed in cows; answer d is a sign in mares. 214. d Lambs in anterior presentation. with one forelimb retained. can usually be delivered by judicious traction alone. 215. b The penis of rams is fibroelastic, with a filiform process at the end of the glans penis. 216. a Hypothermia and hypoglycemia from starvation (mismothering-exposure complex) accounts for most neonatal losses in a sheep flock. 217. e "Pizzle rot" is the most common condition affecting the penis and prepuce of rams. The disease is caused by Corynebacterium nma/e, which grows in the alkaline urine of sheep fed a high-protein diet. 21B. a Brucella avis is the major cause of epididymitis in rams. 219. b Artificial insemination of ewes is difficuh because of the anatomy of the ovine cervix. Deposition of semen cranial to I cm into the cervical canal is impossible. Laparoscopy is most common ly used for artificial insemination. 220. c Embryos are best collected on day 6 after ovulation. At days 1 and 3 the embryos are too immature. By day 9 the embryos have lost their zona pellucida and are difficult to identify. The embryos are too malUre by day II. 221. dB-mode ultrasound provides accurate information regardi ng pregnancy status. number of fetuses. fetal defects. and stage of gestation. Doppler ultrasonography haS been used for pregnancy diagnosis but cannot provide detailed information.
242
www.vet4arab.co.cc
SECTION 9
222. a In ewes the corpus IOleum of pregnancy initially secretes progesterone: the developing placenta assumes this func tion later in gestation. After about day 5, pregnancy cannot be terminated by ovariectomy or administration of luteolytic doses of prostaglandins. Panurition can be induced with gluoocorticoids or estrogens. 223. c Arthrogryposis and hydranencephalyare caused by Akabane virus disease. 224. c Breeding soundness evaluation of rams involves a thorough physical examination, with empbasis on the reproductive organs (palpation, scrotal Circumference), and semen analysis (sperm motility and morphology). 225. c Scroml circumference is correlated with sperm production. 226. c The buck is always the best detector of estrus in the doe. Only a small percentage of does show mounting behavior or have a swollen vulva. The "buck rag" is only a substitution for the real thing (the buck). Although a low serum progesterone level may indicate estrus, it could also be an indication of anestrus. 227. b The goat has no known chorionic gonadotropin. A1llhe other substances listed have been used for pregnancy determination. 228. d Goats have a cOl)'ledonary placenta that is microscopically classified as epitheliochorial at the level of the placentome. 229. a Artificial insemination has successfully been performed in goats for years via transcervical insemination using a vaginal speculum to locate and stabilize the cervix. 230. d Although all the listed organisms may cause abortions in goats, chlamydiosis, tOxoplasmosis, and Q fever are the most common causes of infectious abortion in the Un ited States. In theo ry, Brucella melitensis is no longer present in the United States. Campylobacrer jejllni and Bmcella abortllS only rarely cause abortion. 231. e To maintain pregnancy, goats totally depend on progesterone produced by the corpus luteum throughout gestation. 232. c Prostaglandin Flo is luteolytic in goats and can terminate pregnancy at any stage. The corpus luteum mllst be 5 to 7 days old before it will respond to the effects of prostaglandin. 233. b Freemartins are extremely rare in goats, so twins from embryo transfer are not a problem.
234.
235.
236.
237. 238. 239.
240.
241. 242.
243. 244.
245. 246. 247. 248.
249. 250.
Tranferring fewer or more than two embryos is not as successful as transferring two embryos in establishing and maintaining pregnancy. c Abortions caused by toxoplasmosis are characterized by calcified foci of cotyledon necrosis. d Usteriosis is one of the few organisms causing abortions that also causes the dam to become sick:. It may also result in macerated fetuses. b Rarely reported in the United States, campylobacteriosis is acquired by ingestion in small rum inan ts and not by intromission, as in cattle. e Chlamydiosis is widespread throughout the United States. a Older Angora does may habitually abort. c Any of the answers may caLise dystocia, but the multiple felUses of goats resull in dystocia from simultaneous presentation in the birth canal. c Mannosidosis is a storage abnormality affecting the central nervous system. The other answers have been associated with retained fetal membranes in goats. e Dietary energy levels during early lactation play no role in pregnancy toxemia. d Ultrasonography is the definitive diagnostic procedure for diagnosing pseudopregnancy. Radiography would only give an indication of an enlarged uterus, which could be aruibutable to pseudopregnancy, pyometra, or early pregnancy. c Gestation in goats normally lasts 147 to 153 days. d Catheterization of the buck's bladder is very difficult or impossible because of a diverticulum in the dorsum of the urethra at the level of the ischial arch. b Goats ovulate during the latter part of standing estrus. c Estrus lasts 12 to 24 hours in most does, but it may last up to 48 hours in some. a Buck: ejaculates are of very low volume but contain a very high concentration of sperm. a As a ruminant, the buck: has all fo ur accessory sex glands, with the prostate being disseminated along th e pelvic urethra. d Most polled goats are genotypic females (XX). a Goats are seasonally polyestrous, responding to decreasing photoperiod. In the northern latitud es, the breeding season is in the late summer and fall (short-day breeders).
C 1998 Mosby- Year Book, Inc. Photocopying is fJfo11ibited by law.
TlleriogetlOlogy
251. c Follicle-stimulating hormone stimulates development of ovarian foUicles, secretion of estrogen by the ovaries. and tubular growth and spermatogenesis in the testes. 252. b Plasma levels of luteinizing hormone are generally higher in males than in females during the breeding season. 253. e Prolactin depresses production of luteinizing hormone and follicle-stimulating homlOne in both sexes. 254. d Altricial young hatch \'lith a low metabolic rate. Body heat increases during development. 255. e The young ofp recocial species hatch with the ability to thermoregulate; this capability develops during incubation. 256. b Progesterone causes a preovulatory surge of luteinizing hormone. 257. c Arginine vasotocin causes oviposition (expulsion of the egg from the oviduct). 258. c Prolactin induces production of "crop milk" in passerines. 259. d The testes secrete testosterone. which increases the size of the brain nuclei associated with song behavior. 260. c The bursa of Fabricius has no role in red blood cell production. 261. d The molting cycle is associated with increased testosterone and estrogen levels. 262. e Interstitial glandular Leydig ceUs are the testicular source of testosterone. 263. a The left ovary is more well developed than the right ovary. 264. b Follicle-stimulating hormone influences follicular development and maturation . 265. a Prolactin depresses production of folliclestimulating hormone and luteiniZing hormone and initiates brooding behavior. 266. c Such hens are described as determinate layers. 267. d Such hens are described as indeterminate layers. 268. e Plasma cholesterol levels do not increase in the blood of laying birds. 269. b Testosterone is found in the plasma oflaying hens. Levels peak about 8 hours before ovulation. 270. a Reproductive failure in captive birds usually associated with failure to develop foUicies that ovuJate.
243
271. a Reptiles do not have a larval stage. 272. c Parthenogenesis results in all-female progeny
in reptiles. 273. b Species that lay eggs that later hatch are termed oviparous.
274. d Hermaphrodites have both testicular and ovarian tissllo. 275. d Difficulty in giving birth or laying eggs is termed dystocia. 276. b Sexually precocious animals reach puberty early. 277. b This can lead to peritonitis and death. 278. c Mastitis cannot occur in a tortoise because they have no mammary glands. 279. e Menopa use does not occur in reptiles. 280. c The bite of baby venomous snakes can be quite dangerous. 281. b Renal disease or calcium deficien cy can cause soft egg shells. 282. e Eggs or living young are delivered through the proctodeum imo and through the cloaca of reptiles. 283. b Amphigonia retardata refers to retention of viable sperm in the reproductive tract of previously mated female animals. 284. d The eggs of reptiles most closely resemble those of birds, except that they do not have an air chamber. 285. c Salmonella is easily transferred from an infected female reptile to her ova, similar to infection in avian species. 286. b Sexually aggressive males are separated from gravid females because of the risk of damage to intraoviductal eggs by the male's erect penis. 287. c Fertilization occurs in the oviduct. 288. b The erectile portion of the penis or hemipenis of most reptiles is fibrovascular. 289. c So me mated female reptiles can store viable sperm in their reproductive tract for as long as 2 to 4 or even more years. 290. b Albinism and melanism are genetically linked conditions, but the other conditions listed are not. 291. b With polyspermia, more than one spermatozoon penetrates the ovum. 292. cOnly otariids (sea lions and fur seals) have testicles located in a scrotal sac. All the other species have testicles located under the skin in the inguinal area.
SECTION 9
244 293. d Old World camels have a longer gestation (approximately 390 days). 294. c Gestation in ferrets lasts 42 days. 295. a Porcine zona pcllucida is an excellent
contraceptive in feral horses. Diethylstilbestrol causes fetal loss in pregnant mares. Testosterone
propionate is used in s tallion~ Silastic implants do not lend themselves to remote delivery.
Northistcrone appears to increase fertility. 296. e Most bears apparently exhibit nonna! ovarian cycles and noninduced ovulation. Implantation is delayed in many bear species. 297. e This is the same gestation length as in dogs and cats.
www.vet4arab.co.cc
SECTION
298. e If a guinea pig is bred for the first time after 8
10
months of age, the pelvic bones cannot separate adequately at parturition and dystocia is likely. 299. d Female hamsters should be housed alone after breeding because they may attack and Jdll the male. They should not be disturbed for at least I week following parturition, because any disturbance could cause them to caImibalize Iheiryoung. In aUother species listed, the male helps care for the young. 300. e Guinea pigs are born fully furred and with open eyes.
NOTES
Toxicology GoO. Osweiler
Recommended Reading BeasleyVR et a1: A systems affected approach to small animal toxicology, Urbana, Ill, 1994. University of Illinois, CoUege of Veterinary Medicine. Howard JL: Current veterinary therapy: food animal praclice, ed 3, Philadelphia. 1992. WB Saunders. Bonagura JO: Kirk's current veterinary therapy XlI: small animal practice, Philad elphia. 1995. WB Saunders. Morgan RB: Handbook of small animal pracrice, ed 2. Philadelphia, 1992. WB Saunders. Osweiler GO: Toxicology. Baltimore. 1996. Williams & Wilkins.
Practice answer sheet is on page 273.
Questions 1. A concentration of0.01 % is equivalent to how many parts per million (ppm)?
a. I ppm b. 10 ppm
c. 100 ppm d. 1000 ppm e. 10,000 ppm
2. A blood lead concentration reported as 80 ..,.gldl is the same as:
3.
if the toxic level ofa drug infeed is 100 ppm fora 20-kg pig, what is the estimated roxicityofthe drug on a milligram per kilogram ofbody weight basis? Assume the feed is air dried and the pig eats feed at the rate of6% of irs body weight daily. a. 2 mg/kg b. 4 mg/kg c. Gmg/kg d. 8 mg/kg e. 10 mg/kg
a. 0.08ppm b. 0.8 ppm c. 8ppro d.80ppm e. 800 ppm
Ii)
/998 Mosby-Year Book, Inc. Photocopying is prollibired by!nw.
Correct answers are orr pages 252-253.
245
www.vet4arab.co.cc
SECTION 10
246
4. Induction ofemesis is recommended as a detoxifica tion procedure in dogs ingesting any of the /offow;ng except:
a. h. c. d.
antifreeze (ethylene glycol) acetaminophen gasoline liquid aspirin
e. chocolate 5. TIle antidotal agent N-aceryicysreine is indicated for treatment of poisoning with:
a. b. c. d.
cholecalciferol rodenticides acetaminophen brodifacoum chlorpyrifos
e. copper
Questions 9 through 11 A 9-mOflth-old male Doberman pinscher puppy is seen because of weigilt loss and periodic vomiting. witl! intermitrent periods ofdepressiofl alternating with seizures or hysteria during the past 6 weeks. All vaccinations are current, parasites are not a problem, a nd there is flO history of injury. Ati abdominal radiograph reveals two large radiodense objects in the stomach. A complete blood count reveals normal leukocyte numbers, mild anemia, and an excessive number of nucleated red blood cells.
9. Your temative diagnoses should itlelude: a. b. c. d. e.
lead or arsenic toxicosis lead or zinc toxicosis zinc or copper toxicosis iron or zinc toxicosis iron or cadmium toxicosis
6. [norganicarsenic toxicosis is manifested clinically
as, a. b. c. d. e.
10. W1w t is the best single sample to analyze f or
icterus, anemia, and hemoglobinuria amaurosis, incoordination, and constipation cardiomyopathy, hydrothorax, and ascites photosensitization, dermatitis, and hair loss vomhing, gastroenteritis, diarrhea, and dehydration
7. Which combination a/mineral additives is most useful in preventing chron ic copper toxicosis in slleep?
a. b. c. d. e.
selenium and molybdenum selenium and sulfate zine and molybdenum sulfate and molybdenum arsenic and sulfate
8. Chronic diarrhea, achromotrichia , emaciation, and lameness in cattle suggest toxicosis involving:
a. b. c. d. e.
arsenic lead mercury molybdenum selenium
confirmation ofrhe suspected toxicosis?
a. b. c. d. e.
whole blood serum urine stomach contents hair
11. Based on the history and clinical signs, wlmt is
the most likely cause of tile problem?
a. b. c. d. e.
cadmium toxicosis copper toxicosis iron toxicosis lead toxicosis zinc toxicosis
12. Deficiency of what element in tile sow predisposes bally pigs to toxicosis lly injectable iron preparatiotlS?
a. b.
c. d.
e.
copper chromium magnesium selenium zinc
0 1998 Mosby-Year Book, Inc. Photocopying is prohibited by la w.
13. Selenium absorption lly crop plants is favored by
soil tllat is:
a. b. c. d. e.
247
Toxicology
acidic, wet, and poorly drained acidic, semi-arid, and weU drained alkaline, well aerated, and well drained alkaline, wet, and poorly aerated acidic or alkaline, wet, and poorly aerated
18. In cattle. chronicfluoride toxicosis causes:
a. diarrhea. pale hair coat, lameness. and hoof overgrov.rth b. icterus, hemoglobinuria. and photosensitization c. emaciation, hair loss, and lameness d. rumen stasis, nephrosis, and constipation e. lameness, exostosis, and excessive dental wear
14. [n callie, which ofthefoUowing is least likely ro
be confused w ith chronic selenium toxicosis?
a. b. c. d. e.
chronic ergotism molybdenosis flu oride poisoning laminitis associated with grain overload mercury poison ing
15. In horses. wllat is the most likely cause ofacute allorexia, sweating, colic, stiffness. progressive ataxia, paresis, tachycardia, and hypote1lsion?
a. b. c. d. e.
19. Wllicll allribacterial has been associated with acute seizures in calves and "ro und heart disease" cardiomyopathy in pollitry? a. b. c. d. e.
arsanilic acid furazolidone lincomycin gentamicin chloramphenicol
20. Which organic symhetic herbicide is often
considered dangerous because it induces accumulation ofnirrites in some weed species?
arsenic poisoning antibiotic toxicosis mone ns in toxicosis poisoning by red maple leaves organophosphate poisoning
16. In rumina nts. urea toxicosis is characterized by:
a. rumina! alkalOSis, systemic acidosis, and elevated blood ammonia levels b. ruminal acidosis, systemic aIlcaIosis, and elevated blood ammonia levels c. ruminal alkalosis, systemic alkalos is, and elevated blood ammonia levels d. ruminal acidosis, systemic alkalosis, and decreased blood ammonia levels e. rumina! alkalosis, systemic alkalosis. and elevated blood urea nitrogen levels
a. h. c. d. e.
paraquat glyphosate lindane 2,4 -dichlorophenoxy acetic acid pentachlorophenol
21. Which category of insecticidal compollnds
presents a problem of persistent residues in fatty tissuesofanimals?
a. b. c. d. e.
carbamates organochlorines organophosphates pyrethrins juvenile hormones
22. If acute organophosphate insecticide poisoning is
17. [11 swine tissue sections. microscopic lesions of eosinophilic m eningoencephalitis are characteristic of:
a. b. c. d. e.
antifreeze ingestion aminoglycoside toxicosis water deprivation mercury toxicos is monensin toxicosis
suspected. wllat is the best initial sample to obtain from a liveanimalfor itlitial diagnostic testing?
a. b. c. d. e.
serum whole blood urine stomach conte nts fat biopsy
Correct answers are on pages 252·253.
www.vet4arab.co.cc
SECTION 10
250
54. CAuses tremors and myocardial damage in grazing horses; passed in the milk oflnctating animals 55. Its seeds cOfltamjflare small grains; causes acute
to chronic liver and pulmonary damage. especially in swine and poultry
For Questions 56 through 60, select the correct answer from the five choices below.
62. A cat develops incoordination,followed ill 12 hOllrs by vomiting, weakness, tachypnea, oliguria, hypothermia, and marked metabolicaddosis. 71lOracic radiographs reveal 110 abnormalities. Mlat is the most likely calise of these signs?
a. b. c. d. c.
arsenic poisoning caffeine toxicosis djnitrophenoJ (ONP) poisoning ethylene glycol poisoning organophosphate poisoning
a. nitrite 63. lvermectin toxicosis ill collies is characterized by:
b. cyanide
c. oxalate d. tannin e. pyrrolizidinc alkaloid
56. Causes methemoglobinemia 57. Inhibits cytochrome oxidase and prevents miUzation of oxygenated blood
a. excitation, profuse salivation, tremors, and vomiting b. vomiting and hyperesthesia c. ataxia, incoordination, weakness, and depression d. blindness, colic, and seizures e. dyspnea, polypnea, and tachycardia 64. Fluorescence ofblood on exposure to ultraviolet
58. [merJems witlt rumen junction. precipitates proteins, and causes renal damage if! cattle
59. Reduces available serum calcium and produces crystals demonstrable in the vascular walls. kidney. and urine 60. Produces chronic emo.ciatiOIl. hepatoloxicosis. and sometimes hepatic encep/lalopathy in horses
61. Which of tile following is a major western toxic piant associated with acwe to SIlbacute neurologic sigrlS ill caule, vacuolacion in cerebellar neurons, and congenital skeletal malformations? a. b. c. d. e.
Senecio Halogeton Delphinium
light is IIseful in diagnosing:
a. b. c. d. e.
aflatoxicosis of swine ethylene glycol poisoning of cats fluoride poisoning of canJe lead poisoning of waterfowl selenium toxicosis of horses
65. A group of swine sllOws auuia, paralysis, hoof
(corollary band) and hair lesions, and lesions of focal symmetric pofiomyeiomalacia. The most likely cause of these signs is toxicosis involving:
a. arsenic b. copper c. lead d. selenium e. zinc
66. [n dogs, such neurologic signs as disorientation, delirium, agitation, excitement, and incoordination WQuld be expected after ingestion ofany of the following except:
a. b. c. d. e.
Datura seeds morning glory seeds marijuana Easter lily leaves ergot alkaloids
67. Laminitis may develop in horses housed in a stall whose floor is covered with:
a. black cherry shavings
a. serum amylase activity b. serum creatine phosphokinase activity c. serum -y-glutamylnansferase activity d. platelet count e. prothrombin time 72. The avian toxicant 4-aminopyridine is toxic to
dogs and causes cUnical effects similar to those of
c. oak shavings d. cedar shavings e. pentachlorophenol-treated shavings
a. b. c. d. e.
68. Which toxic principle does not occur in the seeds of the following respective pkmts?
a. aesculin in Ohio buckeye b. carboxyatractyloside in cocklebw c. coniine in poison hemlock d. hydrogen cyanide in wild cherry e. nitrate in Sorghum 69. Which protein supplement contains a toxic
prinCiple causing congestive heart failure and liver necrosis and is well tolerated by marure cattle but relatively toxic to swine and immature (nonruminating) calves?
a. alfalfa meal b. canola meal c. cottonseed meal d. linseed meal e. soybean meal
b. dogs c. gerbils
@1998Mosby-YearBook, Inc. PllOtocopying is prohibited by law.
71. Which clinicopatllOlogic value is least likely to be
b. black walnut shavings
a. cats
Astragalus
d. parakeets e. reptiles
abnormai}210 24 hours aftera dog is bitten bya rattlesnake?
arsenic ethylene glycol lead organophosphates strychnine
73. The toxic principles in chocolate that affect dogs are:
a. anagyrine alkaloids b. cyclic polypeptides
c. L-CYSleine and tryptophan d. methylxanthine alkaloids e. solanidine and ergonovine 74. Diarrhea in swine and polioencephalomalacia in cattle have been observed when livestock consume looter containing high concentrations of:
a. arsenic b. fluoride
c. nitrate d. sulfate
e. zinc
70. Overheated Teflon-coated frying pans release IKlPOrs that are especially toxic to;
Cemallrea
251
Toxicology
75. In cats, ingestion of Easter lily leaves causes fatal toxicosis affecting primarily the:
a. nervous system b. lungs c. liver d. kidneys e. gasnointestinai tract
Correct answers are on pages 252-253.
252
SECTION 10
www.vet4arab.co.cc
Answers I. c Convert % to ppm by moving the decimal point fo ur places to the right. 2. b 80 p.g/dl = 800 IJ-g/ L = 0.8 mg/L = 0.8 ppm. 3. c 20 kg X 0.06 :: 1.2 kg feed/day. 1.2 kg X 100 mg/kg =120 mg total intake. 120 mg + 20 kg = 6 mg/kg. 4. c Ingestion of gasoline. a volatile hydrocarbon, may lead to inhalation during emesis, with subsequcm pneumonia.
5. b N-acetylcysteine helps replenish glutathione. 6. e The principal effects of arsenic are gastrointestinal, espccially those involving the gut mucosa. 7. d Sulfate Is converted to sulfide and forms insoluble copper sulfide in the rumen. Molybdenum forms a thiomolybda te complex that promotes copper excretion. 8. d Molybdenum p romotes copper excretion, resulting in copper deficiency. 9. b Qmy lead and zinc affect the hemogram in a simila r manner. 10. a Lead toxicosis is best reflected in red blood ceUs. nO! in serum or urine. 11 . d The combination of gastroi ntestinal, ne urologic, and hematologic changes suggests only lead toxicosis. 12. d Several studies have documented that selenium deficiency in the sow predisposes piglets to iron lOxicosis. 13. c Alkaline conditions a nd well ~drained soil favor selenium absorption. 14. e Mercury causes central nervous syste m signs, bl indness. and h igh mortality. The other choices listed are mainly chronic, peripheral effects. 15. c Monensin affect's m itochondrial function and ion transport, resulting in metabolic myodegeneratio n. 16. a Ammonia produces alkaline conditions in the rumen. A high blood ammon ia level inllibits the tricarboxylic acid cycle, causing me ta bolic acidosis. 17. c This is tlle classic microscopic lesion of wate r deprivation (salt poisoning) in swine. 18. e Fluorides affe ct enamel formation and bone remodeling. 19. b Nitrofura ns, such as furazoli done, interfere with carbohydrate metabolism in L~e central nervous system and cardiac muscle.
20. d 2,4-Dichlorophenoxy acetic acid is a pla nt gro\vth hormone that alters nitrogen metabolism, especially in plants with a tendency to accumulate nitrate. 21. b Chlorinated hydrocarbon insecticides are highly lipophilic. 22. b Cholinesterase activity is most concentrated in red blood cells. 23. d Some visual disturbance may accompany miosis, bUI true blindness is not a feature. 24 . d The increased stability of this bond makes therapy more difficult as time passes. 25. c Pers iste nce in tissue m ay necessitate therapy fo r 3 to 4 weeks. 26. e Fluoroacel'ate is more toxic to dogs than to rodents. 27. b Signs are caused by suppression ofinhibitOiY lnte m e urons in the spinal reflex arc. 28. d Bromethalin affects Na ~ K ATPase. leading to loss of cellular flu id control. 29. a Clinical signs do not a ppear until coagulation factors a re depleted. 30. c Acute tissue damage is followed by mineralization when vitamin 0 rodenticides are consumed. 31. c Carboxyhemoglobin is red to pink and causes maternal ~ fetal hypoxia. 32. b Carbon dioxide causes loss of consciousness before respiratory failure. 33. d Sulfides are irritating and rapidly produce loss of consciousness. 34. e Oxides of nitrogen often form in silage containing high levels of nitrates. 35. a Ammonia is a major toxicant or pollutant formed from animal wastes. 36. e Amphetamines are commonly used in people. At high dosages, nervous signs predomina te. 37. a These effects are especially predominant in cats. 38. b Aspirin should be used a t the recommended dosages in animals, especially in cats. 39. d Methylxanthine alkaloids (caffeine. theobromine) are hazardous to dogs. 40. c Large doses of nonsteroidal a ntiinflammatory drugs may induce toxicos is afte r several day-s' exposure.
Toxicology
41. c Oicumarol inhibits reactivation of vitamin K, which is necessary for coagulation factor synthesis. 42. b Vomitoxin causes nausea and induces a learned aversion to the contaminated feed. 43. d Ergot alkaloids inhibit prolactin in many species. 44. e Zearalenone is a physiologic estrogen with estrogenic effects. 45. a Aflatoxins are regulated by the Food a nd Drug Administration because of the ir toxic and carcinogenic nature. 46. d Aspergillus ochraceus produces ochratoxin, a potent nephrotoxin. 47. c Aspergillus fIavus is a common species producing aflatoxin. 48. a Acremonium coenopliialum is a n endophyte fungus of fescue, associated with ~ fescue foot. ~ 49. e Fusarium roseum prod uces zearalenone, an estrogen. 50. b Fusarium moniliforme produces fumonisins. which cause malacia of cerebral white m a tter. 51. a Japanese yew is a common cause of toxicosis in horses and cattle and is also toxic to dogs. 52. c The toxic prinCiple causing nephrosis has not been defined, but lesions are s imilar to those seen in oak (acorn) toxicosis. 53. b It causes early oxidant damage and Heinz body fo rmation in red blood cells. 54. d This plant is hazardous to both adult animals and nursing foals or calves. 55. e It contains high concentratio ns of pyrrolizidine alkalOids, which cause liver damage. 56. a Iron is oxidized to the ferric state by the n itrite ion. 57. b Affected cells cannot accept electrons and hence cannot bind oxygen. 58. d Tannin has a potent effect on proteins. It is the toxic principle of acorns or oak buds. 59. c High oxalate levels are present iJl several plants prevalent in the midwestern and western United States, 60. e Alkaloids induce hepatomegalocytosis and nuclear dam age and sometimes are carcinogenic. Chronic liver damage leads to encephalopathy.
253
61. e Swainsonine alkalOids in Astragalus (locoweed) inhibit a~mannosidase, causing vacuolated neurons; arthrogryposis is another sequela. 62. d Lack of fever differentiates ethyle ne glycol from dinitrophenol toxicosis. 63. c Ivermectin causes depression, with lillie or no tendency toward excitement or se izures. 64. d Porphyrin metabolism is blocked. The accumulated porphyrins are de tected by their fluorescence on exposure to ultraviolet light. 65. d Selenium affects the skin, hair, and hoof through inte rference with sulfur amino acids. It is the only metal of those listed that causes poliomyelomalacia. 66. d Easter lily contains a nephrotoxicant and is of apparent low toxicity in dogs. AJI the othe rs listed a re neurotoxicants that could cause the signs described. 67. b An unknown toxic principle is present in black walnut. Only horses manifest this reaction. 68. e Nitrate accumula tes only in vegetative plant tissues, not in seeds. 69. c Cottonseed meal contains a phenolic principle, gossypol, tha t is poorly tolerated by nonruminan ts, especially swine. 70. d Teflon fum es sensitize the budgerigar's heart to epinephrine and endogenous catecholamines. 71. a Amylase is a pancreatic enzyme. AJI the other values may be affected by the necrotizing and cytotoxic effects of Crotalidae venoms. 72. e HyperestheSia, tremors, and seizures result when dogs ingest aminopyridine. 73. d Toxic principles in chocolate include the stimulants caffeine, theophylline, and theobrom ine. 74. d Sulfates induce osmotic diarrhea that is usually transient. in cattle, sulfates may be reduced to sulfides and interfere with thiam in synthesis or utilization. '75. d Cats a re very sensitive to the nephrotoxicant in Easter lily.
www.vet4arab.co.cc
SECTION 10
248
23. Cholinesterase inhibitor pesticides typically cause all a/the following except:
a. salivation b. miosis
26. No longer legal to use because of probability of secondary or relay toxicosis; Iligllly toxic to dogs 27. Om calise acute tetanic seizures, with tmlrked hyperesthesia, hyperreflexia, and death from respiratory muscle failure
c. dyspnea d. blindness e. bradycardia 24. When applied to organophosphate insecticide poisoning, the term aging refers to:
a. loss of insecticidal activity with time b. isomerization afthe organophosphate to a
28. Can cause acute to chronic neurologic signs by altering cen tral nervous system/luid balance, leading to cerebral edema 29. Clinical signs may be delayed for 1 or 2days after exposure, regardless ofdosage
more toxic chemical form
c. hydrolysis afthe choli nesteraseorganophosphate bond induced by oxime drugs d. a chemical change that increases the stability of the organophosphate-cholinesterase bond e. altered toxicity of organophosphates from
spontaneous hydrolysis of ester groups 25. Newera nricoagulant rodenticides, also known as second-generation anticoagulants, are important in veterinary medicine because they:
a. have been developed to be toxic in rats but not in other classes of m ammals b. have effeCls that are readily treated by synthetiC vitamin K injection c. a re more potent or longer acting than firstgeneration a nticoagulants, requiring prolonged therapy d. are more readily detected by chemical analysiS than first-generation rodenticides e. do not interact with other drugs or chemicals For Questions 26 through 30, select the correct answer from the five choices below. a. b. c. d. e.
anticoagulant rodentkides strychnine cholecalciferol bromethaJin sodium fluoroacetate
30. Ca n cause acute renal damage, hypercalcemia,
For Questions 36 through 40, select the correct answer from the five choices below. a. b. c. d. e.
acetaminophen aspirin ibuprofen caffeine amphetamine
a. b. c. d. e.
ammonia carbon dioxide carbon monoxide hydrogen sulfide nilfogen dioxide
31. Causes cherry- red blood and may induce premature delivery of stillborn piglets 32. Low levels stimulnte respimrion, but high levels calise loss ofconsciousness, followed by depression of respiratory centers
36. Central nervous system stimulanlused for appetite suppression and mood elevation
37. One or two 325-mg tablets can cause methemoglobinemia, facial edema, and hepatic damage
33. Highly toxic gas released wilen maT/ure pies are agitated; causes mild respiratory irritation, loss of olfactory function, and rapid loss ofconsciousness; dea th is from roxic depression of respiratory centers
38. Chronic use or excessive short-term IISeCallSes gaseric II lcers, vomiting, depression, anorexia, and toxic hepatitis; more toxic to cats than dogs
39. Can cause acute central nervous system stimlllation, with Ilyperrefiexia, tonic seizures, cach}'cardia, polypnea, and hyperthermia 40. Acute toxicosis causes oliguria, renal papillary necrosis, and llremia;chronic olJemse may cause gastric ulcers in dogs
For Questions 41 through 45, select the correct answer from the five c hoices below. a. b. c. d. e.
aflatoxin deoxynivalenol (Vornitoxin) dicumarol ergot zearalenonc
41. Produced in moldy sweet clover; resulting in
34. Hea vier than air;formed in silos IInder some conditions; causes direct irritation of respiratory traer and lesions in the lllngs 35. Stro ng upper respiratory irritant formed by decomposition ofhigil-nitrogen swine and poultry wastes; most clinical effects are from local irritation
C 1998 Mosby-Year Booi.:, Inc. Photocopying is prohibited by law.
45. Chronic hepatotoxill alld carcinogell tllat can suppress immullefunctioll
For Questions 46 through 50, select the correct answer from the five choices below.
and tissue mineralization
For Questions 31 through 35, select the correct answer from the five choices below.
249
Tox icology
vitamin K-responsive coagulopatlly 42. Principal effect is dose-relntedfeed refusal 43. Causes agalactia in periparturient animals by inhibition ofproiactin release 44. In p igs, causes prolonged anestrus or pseudopregnancy, as well as early embryonic death
a. Acremonium coenophialum b. Fllsarium moniliforme c. Aspergillus/lavllS d. Aspergillus ochraceus e. Fllsarium roseum 46. Causes chronic rellal failure in swine, characterized by polyuria, polydipsia, and emaciation 47. Causes chronic filler damage wilh bile duct hyperplasia 48. Causes peripheral gangrene, sloughing ofhooves, and intolerance to hot weather in cattle. and placental lesions and abortion in horses 49. Causes hyperestrogenism and vulvovaginitis in prepubertal gilts 50. Causes lellkoellcephalomalacia in horses
For Questions 5 1 through 55, select the correct answer from the five choices below. a. b. c. d.
Japa nese yew red maple redroat p igweed white snakeroot e. Crotalaria
51. Omamelllal evergreen shmb witll all acute cardiotoxic prillciple that causes sudden death in horses and cattle 52. Nirrate-accumularing plnnt common in gardens, crop fields, andfeedloes; causes acllte toxic tubular nephrosis with perirenal edema and ascites ill swine and cattle 53. Causes acute hemolytic anemia in horses
Correct atlSweTS are on pages 252-253.
www.vet4arab.co.cc
ISBN 0-8151 -7462-4
9
28273